All Krok Pdfs

You might also like

Download as pdf or txt
Download as pdf or txt
You are on page 1of 329

Krok 1 Medicine 2005 1

1. A patient with hypochromic anemia of while color and 5-10 mm long were
has splitting and loss of hair, increased found on the underwear. On microscopy
nail brittling and taste alteration. What of the scrape from the perianal folds
is the mechanism of the symptoms achromic ovums of unsymmetrical form
development? were revealed. Indicate what helminth is
parasiting on the child?
A. Deficiency of iron-containing enzymes
B. Deficiency of vitamin 12 A. Enterobins vermicularis
C. Decreased production of parathyrin B. Ancylostoma duodenalis
D. Deficiency of vitamin А C. Trichuris
E. Decreased production of thyroid D. Trichina
hormones E. Ascaris lumbricoides
2. A 27- year-old woman has used peni- 6. A patient has been brought to the
cillin containing eye drops. In a few mi- hospital with the complaints of headache,
nutes itching, skin burning, lips and eyeli- pain in left hypochondrium. He has been
ds edema, whistling cough, decreasing BP ill for 1,5 weeks. The sudden illness began
appeared. What antibodies can lead to with the increase of body temperature up
this allergic reaction? to 39, 90C. In 3 hours the temperature
decreased and hydropoiesis began. The
A. IgE and IgG attacks repeat rhythmically in 48 hours.
B. IgM and IgG The patient had visited one an Afri-
C. IgA and IgM can country. The doctors have suspected
D. IgM and IgD malaria. What method of laboratory di-
E. IgG and IgD agnostics is necessary to use?
3. A patient with hypersecretion of the A. Blood examination
gastric juices was recomended to exclude B. Immunological tests
concentrated bouillons and vegetable C. Stool examination
decoctions from the diet because of their D. Examination of vaginal and urethral
stimulation of gastric secretion. What is discharge
dominating mechanism of stimulation of E. Urine examination
secretion in this case?
7. Slime, blood and protozoa 30-200 mi-
A. Stimulation of gastrin production by crons of length have been revealed in a
G-cells man’s feces. The body is covered with ci-
B. Irritation of taste receptors lias and has correct oval form with a li-
C. Irritation of mechanoreceptors of the ttle bit narrowed forward and wide round
oral cavity shaped back end. On the forward end a
D. Irritation of mechanoreceptors of the mouth is visible. In cytoplasm there are
stomach two nucleuses and two short vacuoles. For
E. Stimulation of excretion of secretin in whom are the described attributes typi-
the duodenum cal?
4. A journalist’s body temperature has A. Balantidium
sharply increased in the morning three B. Lamblia
weeks after his mission in India, it C. Dysenteric amoeba
was accompanied with shivering and D. Trichomonas
bad headache. A few hours later the E. Intestinal amoeba
temperature decreased. The attacks began
to repeat in a day. He was diagnosed 8. Galactosemia has been revealed in a
with tropical malaria. What stage of child. Concentration of glucose in the
development of Plasmodium is infective blood has not considerably changed. What
for anopheles-female? enzyme deficiency caused this illness?
A. Gametocytes A. Galactose-1-phosphate uri-
B. Shizontes dyltransferase
C. Merozoites B. Amylo-1,6-glucosidase
D. Microgamete C. Phosphoglucomutase
E. Sporozoites D. Galactokinase
E. Hexokinase
5. A 10-year-old child complains of
weakness, nausea, irritability. Helminthes 9. Synthesis of phospholipids is disordered
Krok 1 Medicine 2005 2

under the liver fat infiltration. Indicate A. Tromboembolism


which of the following substances can B. Thrombosis
enhance the process of methylation duri- C. Tissue embolism
ng phospholipids synthesis? D. Embolism with foreign body
E. Fat embolism
A. Methionine
B. Ascorbic acid 14. Purulent endometritis developed in a
C. Glucose woman after delivery. Treating with anti-
D. Glycerin biotics inhibitors of murein synthesis was
E. Citrate ineffective. Wide spectrum bactericidal
antibiotic was administered to her. In 6
10. An individual is characterized by hours temperature rapidly increased up
rounded face, broad forehead, a mongoli- to 400 C with shiver. Muscle pains have
an type of eyelid fold, flattened nasal bri- appeared. BP dropped down to 70/40
dge, permanently open mouth, projecting mmHg. Oligura has developed. What is
lower lip, protruding tongue, short neck, the main reason for the development of
flat hands, and stubby fingers. What di- this condition?
agnosis can be put to the patient?
A. Endotoxic shock
A. Down’s syndrome B. Toxic effect of preparation
B. Klinefelter’s syndrome C. Internal bleeding
C. Alkaptonuria D. Anaphylactic shock
D. Supermales E. Bacteremia
E. Turner’s syndrome
15. Decreased blood supply to the organs
11. A patient in three weeks after acute causes hypoxia that activates fibroblasts
myocardial infarction has pain in the heart function. Volume of what elements is
and joints and pneumonia. What is the increased in this case?
main mechanism of development of post-
infarction Dressler’s syndrome? A. Intercellular substance
B. Vessels of microcircular stream
A. Autoimmune inflammation C. Nerve elements
B. Ischemia of myocardium D. Parenchymatous elements of the organ
C. Resorption of enzymes from necrotized E. Lymphatic vessels
area of myocardium
D. Secondary infection 16. Methotrexate (structural analogue
E. Vessels ’ thrombosis of the folic acid which is competitive
inhibitor of the dihydrofolatreductase)
12. A patient visited a dentist with is prescribed for treatment of the mali-
complaints of redness and edema of his gnant tumour. On which level does
mouth mucous membrane in a month methotrexate hinder synthesis of the
after dental prosthesis. The patient was nucleic acids?
diagnosed with allergic stomatitis. What
type of allergic reaction by Gell and A. Mononucleotide synthesis
Cumbs underlies this disease? B. Replication
C. Transcription
A. Delayed type hypersensitivity D. Reparation
B. Cytotoxic E. Processing
C. Immunocomplex
D. Anaphylactic 17. A 2-year-old child experienced
E. Stimulating convulsions because of lowering calcium
ions concentration in the blood plasma.
13. A patient suffering from trombophlebi- Function of what structure is decreased?
tis of deep veins suddenly died. The
autopsy has shown freely lying red friable A. Parathyroid glands
masses with dim crimped surface in the B. Hypophysis
trunk and bifurcation of the pulmonary C. Adrenal cortex
artery. What pathologic process was D. Pineal gland
revealed by the morbid anatomist? E. Thymus
18. From the nasopharynx of a 5-year-old
child a microorganism was excreted which
is identical to Corynebacterium diphtheri-
Krok 1 Medicine 2005 3

ae dose according to morphological and A. Teratoma


biochemical signs. Microorganism does B. Squamous cell carcinoma of ovary
not produce exotoxin. As a result of what C. Melanoma
process can this microorganism become D. Sarcoma of ovary
toxigenic? E. Metastase of cervical carcinoma
A. Phage conversion 22. A man died 8 days after the beginni-
B. Cultivation in the telluric environment ng of the disease. He was diagnosed with
C. Passing through the organism of the dysentery. At the autopsy it was found out
sensative animals a thickened wall of the sigma and rectum,
D. Growing with antitoxic serum fibrinous membrane on the surface of
E. Chromosome mutation mucous membrane. Histologically: there
is a deep necrosis of mucous membrane
19. A patient with encephalopathy was with infiltration of necrotic masses with fi-
admitted to neurological department. brin. What kind of colitis does correspond
Correlation of increasing encephalopathy to the changes?
and substances absorbed by the
bloodstream from the intestines was A. Diphtheritic
revealed. What substances created in the B. Catarrhal
intestines can cause endotoxemia? C. Ulcerative
D. Chronic
A. Indole E. Gangrenous
B. Butyrate
C. Acetacetate 23. A patient has undergone an amputati-
D. Biotin on of lower extremity. Some time later
E. Ornithine painful nodules appeared in a stump.
Amputatious neuromas were found out
20. A 50-year-old man has felt vague at the microscopic examination. To what
abdominal discomfort within past 4 pathological processes do those formati-
months. Physical examination revealed ons relate?
no lymphadenopathy, and no abdomi-
nal masses or organomegaly at palpation. A. Regeneration
Bowel sounds are heard. An abdominal B. Dystrophy
CT scan shows a 20 cm retroperitoneal C. Inflammation
soft tissue mass obscuring the left psoas D. Hyperemia
muscle. A stool specimen tested for occult E. Metaplasia
blood is negative. Which of the following
neoplasms is this man most likely to have? 24. During the fetal period of the
development in the vascular system of
A. Lipoma the fetus a large arterial (Botallo’s)
B. Melanoma duct is functioning which converts into
C. Hamartoma lig.arteriosum after birth. What anatomi-
D. Adenocarcinoma cal formations does this duct connect?
E. Lymphoma
A. Pulmonary trunk and aorta
21. A 40-year-old woman has had a feeli- B. Right and left auricle
ng of abdominal discomfort for the past C. Aorta and inferior vena cava
8 months. On pelvic examination, there is D. Pulmonary trunk and superior vena
the right adnexal mass. Abdominal CT cava
scan demonstrates a 7 cm cystic mass E. Aorta and superior vena cava
involving the right ovary with small areas
of calcification. The uterus is normal in 25. A 22-year-old patient was admitted
size. The right fallopian tube and ovary to the hospital with complaints of heavy
have been removed surgically. Grossly, nasal breathing. During the examination
the mass on sectioning is filled with of her nasal cavity the doctors found thi-
abundant hair and sebum. Microscopi- ckened mucous membrane, a lot of mucus
cally, the mass has glandular spaces li- and nodular infiltrates without erosions
ned by columnar epithelium, squamous in the nose.The nasal rhinoscleroma was
epithelium with hair follicles, cartilage, diagnosed. The biopsy was taken. What
and dense connective tissue. What type typical morphological changes may be
of tumour is it? found?
Krok 1 Medicine 2005 4

A. Granulomas with Mikulicz’s cells back during active tightening on the hori-
B. Granulomas with Virchow’s cells zontal bar. Objectively: pain while movi-
C. Granulomas with Langhan’s cells ng upper extremity, limited pronation
D. Granulomas with foreign body cells and adduction functions. Sprain of what
E. Interstitial inflammation muscle can be observed here?
26. A woman suffering from dysfuncti- A. М.latissimus dorsi
onal metrorrhagia was made a diagnostic B. М.levator scapulae
abortion. Histologically in the scrape C. М.romboideus major
there were a lot of small stamped D. М.trapezius
glandulars covered with multirowed epi- E. М.subscapularis
thelium. The lumens of some glandulars
were cystically extended. Choose the vari- 31. The alternate usage of dichloti-
ant of general pathologic process in the azide, etacrin acid and lasex did not
endometrium. cause marked diuretic effect in the
patient with marked peripheral edema.
A. Glandular-cystic hyperplasia of The aldosterone level in the blood is
endometrium increased. Indicate which medicine should
B. Atrophy of endometrium be prescribed:
C. Metaplasia of endometrium
D. Neoplasm of endometrium A. Spironolacton
E. Hypertrophic growth B. Mannit
C. Clopamid
27. A 60-year-old patient was hospitali- D. Urea
sed to the surgical department because E. Amilorid
of infection caused by blue pus baci-
llus (Pseudomonas aeruginosa) which is 32. A 46 year-old man complains of di-
sensative to penicillin antibiotics. Indicate fficult nose breathing. Mikulich cells,
which of the given penicillins has marked storage of epithelioid cells, plasmocytes,
activity to the Pseudomonas aeruginosa? lymphocytes, hyaline balls are discovered
in the biopsy material of the nose thi-
A. Carbenicillin disodium ckening. What is the most likely di-
B. Benzylpenicillin agnosis?
C. Phenoxymethylpenicillin
D. Oxacillin A. Scleroma
E. Methicillin B. Virus rhinitis
C. Allergic rhinitis
28. A 45-year-old woman suffers from D. Rhinovirus infection
allergic seasonal coryza caused by the E. Meningococcal nasopharyngitis
ambrosia blossoming. What adipose cells
group stabilizer medicine can be used for 33. A 56-year-old patient complaining
prevention of this disease? of thirst and frequent urination was di-
agnosed with diabete mellitus. Butamin
A. Ketotifen was prescribed. How does the medicine
B. Diazoline act?
C. Phencarol
D. Tavegyl A. It stimulates β-cells of Langergans’
E. Dimedrol islets
B. It helps to absorb the glucose by the
29. A lung of a premature infant cells of the organism tissues
is presented on electronic photomi- C. It relieves transport of glucose through
crography of biopsy material. Collapse of the cells’ membranes
the alveolar wall caused by the deficiency D. It inhibits α-cells of Langergans’ islets
of surfactant was revealed. Disfunction of E. It inhibits absorption of glucose in the
what cells of the alveolar wall caused it? intestines
A. Alveocytes type II 34. During the breakout of acute respi-
B. Alveocytes type I ratory infection in order to diagnose
C. Alveolar macrophages influenza the express-diagnosis, based
D. Secretory cells on revealing of specific viral antigen in
E. Fibroblasts the examined material (nasopharyngial
lavage), is carried out. Which reaction is
30. A young man felt sharp pain in the used for this?
Krok 1 Medicine 2005 5

membrane of what part of the stomach


A. Immunofluorescence was studied?
B. Complement binding
C. Agglutination A. Pyloric part
D. Precipitation B. Fundus of stomach
E. Opsonization C. Cardia
D. Body of stomach
35. A patient with clinical signs E. -
of immunodeficiency has unchanged
number and functional activity of 40. Live vaccine is injected into the human
T and B lymphocytes. Dysfunction’s body. Increasing activity of what cells of
defect of antigen-presentation to the connective tissue can be expected?
immunocompetent cells was found duri-
ng investigation on the molecule level. A. Plasmocytes and lymphocytes
Defect of what cells is the most probable B. Macrophages and fibroblasts
here? C. Pigmentocytes and pericytes
D. Adipocytes and adventitious cells
A. Macrophages, monocytes E. Fibroblasts and labrocytes
B. Т-lymphocytes, В-lymphocytes
C. NK-cells 41. When a patient with traumatic impai-
D. Fibroblasts, Т-lymphocytes, В- rment of the brain was examined, it was
lymphocytes discovered that he had stopped to di-
E. 0-lymphocytes stinguish displacement of an object on
the skin. What part of the brain was
36. The action of electric current on the damaged?
exitable cell caused depolarization of
its membrane. Movement of what ions A. Posterior central gurus
through the membrane caused depolari- B. Occipital zone of the cortex
sation? C. Parietal zone of the cortex
D. Frontal central gurus
A. Na+ E. Frontal zone
B. −
3
C. 2+ 42. In the blood of a 26-year-old man
D. l− 18% of erythrocytes of the spherical, ball-
E. + shaped, flat and thorn-like shape have
been revealed. Other eritrocytes were in
37. Electrocardiogram of a 45-year-old the form of the concavo-concave disks.
man showed absence of P-wave in all the How is this phenomenon called?
leads. What part of the conducting system
is blocked? A. Physiological poikilocytosis
B. Pathological poikilocytosis
A. Sinu-atrial node C. Physiological anisocytosis
B. Atrioventricular node D. Pathological anisocytosis
C. Common branch of the bundle of His E. Erytrocytosis
D. Branches of the bundle of His
E. Purkinje’s fibres 43. In case of enterobiasis acrihine - the
structural analogue of vitamin B2 - is
38. The alveolar ventilation of the pati- administered. The synthesis disorder of
ent is 5 L/min, the breath frequency is 10 which enzymes does this medicine cause
per/min, and the tidal volume is 700 ml. in microorganisms?
What is the patient’s dead space ventilati-
on? A. FAD-dependent dehydrogenases
B. Cytochromeoxidases
A. 2,0 L/min C. Peptidases
B. 0,7 L/min D. NAD-dependet dehydrogenases
C. 1,0 L/min E. Aminotransferases
D. 4,3 L/min
E. - 44. When the pH level of the stomach
lumen decreases to less than 3, the antrum
39. During histological examination of of the stomach releases peptide that acts
the stomach it was found out that glands in paracrine fashion to inhibit gastrin
contained very small amount of pariental release. This peptide is:
cells or they were totally absent. Mucose
Krok 1 Medicine 2005 6

A. GIF The patient has acidosis. Point substances,


B. Acetylcholine which accumulation in the blood results in
C. Gastrin-releasing peptide (GRP) these manifestations:
D. Somatostatin
E. Vasoactive intestinal peptide (VIP) A. Ketone bodies
B. Amino acids
45. Different functional groups can be C. Monosaccharides
presented in the structure of L-amino aci- D. High fatty acids
d’s radicals. Identify the group that is able E. Cholesterol esters
to form ester bond:
51. Only one factor can influence the
A. -OH charge of amino acid radicals in the active
B. -SH centre of enzyme. Name this factor:
C. -CONH2
D. -CH3 A. pH medium
E. -NH2 B. Pressure
C. Temperature
46. The conjugated protein necessarily D. The presence of a competitive inhibitor
contains special component as a non- E. The surplus of a product
protein part. Choose the substance that
can’t carry out this function: 52. The high level of Lactate
Dehydrogenase (LDH) isozymes
A. HNO3 concentration showed the increase of
B. АТP LDH-1 and LDH-2 in a patient’s blood
C. Thiamine pyrophosphate plasma. Point out the most probable di-
D. AMP agnosis:
E. Glucose
A. Myocardial infarction
47. A denaturation of proteins can be B. Skeletal muscle dystrophy
found in some substances. Specify the C. Diabetes mellitus
substance that is used for the incomplete D. Viral hepatitis
denaturation of hemoglobin: E. Acute pancreatitis
A. Urea 53. Succinate dehydrogenase catalyses the
B. Toluene dehydrogenation of succinate. Malonic
C. Sulfuric acid acid HOOC − CH2 − COOH is used
D. Nitric acid to interrupt the action of this enzyme.
E. Sodium hydroxide Choose the inhibition type:
48. Moving of the daughter chromatids to A. Competitive
the poles of the cell is observed in the mi- B. Allosteric
totically dividing cell. On what stage of C. Non-competitive
the mitotic cycle is this cell? D. Limited proteolysis
E. Dephosphorylation
A. Anaphase
B. Metaphase 54. Pyruvate concentration in the patient’s
C. Telophase urine has increased 10 times from normal
D. Prophase amount. What vitamin deficiency can be
E. Interfase the reason of this change:
49. Nowadays about 50 minor bases have A. Vitamin B1
been found in the t-RNA structure besi- B. Vitamin C
des the main four nitrogenous bases. C. Vitamin A
Choose the minor nitrogenous base: D. Vitamin E
E. Vitamin B6
A. Dihydrouracil
B. Uracil 55. Hydroxylation of endogenous
C. Cysteine substrates and xenobiotics requires a
D. Adenine donor of protons. Which of the followi-
E. Cytosine ng vitamins can play this role?
50. The patient with diabetes mellitus has
been delivered in hospital in the state of
unconsciousness. Arterial pressure is low.
Krok 1 Medicine 2005 7

A. Vitamin C 61. During surgical operation a blood


B. Vitamin P transfusion was made. The blood must be
C. Vitamin B6 checked to find antigens of some disease.
D. Vitamin E What disease is expected to be found?
E. Vitamin A
A. Virus of hepatitis B
56. The gluconeogenesis is activated in B. Virus of hepatitis A
the liver after intensive physical trai- C. Adenovirus
nings .What substance is utilized in D. Enterovirus
gluconeogenesis first of all in this case: E. Virus of hepatitis E
A. Lactate 62. A person has steady HR not exceedi-
B. Pyruvate ng 40 bpm. What is the pacemaker of the
C. Glucose heart rhythm in this person?
D. Glutamate
E. Alanine A. Atrioventricular node
B. Sinoatrial node
57. The formation of a secondary mediator C. His’ bundle
is obligatory in membrane-intracellular D. Branches of His’ bundle
mechanism of hormone action. Point E. Purkinye’ fibers
out the substance that is unable to be a
secondary mediator: 63. The sterile Petri dishes and pipettes
are necessary to prepare for microbi-
A. Glycerol ological tests in bacteriological laboratory.
B. Diacylglycerol What way of sterilization should be appli-
C. Inositol-3,4,5-triphosphate ed in this case?
D. CAMP
E. Ca2+ A. Dry-heat sterilization
B. Tyndallization
58. There is only one hormone among the C. Pasteurization
neurohormones which refers to the deri- D. Steam sterilization in autoclave
vatives of amino acids according to classi- E. Boiling
fication. Point it out:
64. A 50-year-old male farm worker has
A. Melatonin been brought to the emergency room. He
B. Thyroliberin was found confused in the orchard and si-
C. Vasopressin nce then has remained unconscious. His
D. Oxytocin heart rate is 45 and his blood pressure is
E. Somatotropin 80/40 mm Hg. He is sweating and sali-
vating profusely. Which of the following
59. A patient with complaints of 3-day- should be prescribed?
long fever, general weakness, loss of
appetite came to visit the infectionist. A. Atropine
The doctor suspected enteric fever. Which B. Norepinephrine
method of laboratory diagnosis is the best C. Proserine
to confirm the diagnosis? D. Physostigmine
E. Pentamine
A. Detachment of blood culture
B. Detachment of myeloculture 65. A 58-year-old female has undergone
C. Detachment of feces culture surgery for necrotic bowel. Despi-
D. Detachment of urine culture te having been treated with antibi-
E. Detachment of pure culture otics, on postoperative day 5, she
develops symptoms (fever, hypotension,
60. A consumptive patient has an open tachycardia, declining urine output, and
pulmonary form of disease. Choose what
sputum staining should be selected for fi- confusion) consistent with septic shock.
What hemodynamic support would be
nding out the tubercle (Koch’s) bacillus? helpful?
A. Method of Ziel-Neelsen
B. Method of Romanowsky-Giemsa
C. Method of Gram
D. Method of Neisser
E. Method of Burry-Gins
Krok 1 Medicine 2005 8

A. Fluids and Dobutamine infusion A. (R)CFT- Reiter’s complement fixation


B. Dobutamine infusion test
C. Antibiotic administration B. IFA - Immunofluorescence assay
D. Fluid administration C. Immunoblot analysis
E. Atropine administration D. RDHA - Reverse direct hemaggluti-
nation assay
66. A 42-year-old man who has been E. RIHA - Reverse indirect hemaggluti-
injured in a car accident is brought into nation assay
the emergency room. His blood alcohol
level on admission is 250 mg/dL. Hospital 70. Some diseases reveal symptoms of
records show a prior hospitalization for aldosteronism with hypertension and
alcohol related seizures. His wife confi- edema due to sodium retention in the
rms that he has been drinking heavily organism. What organ of the internal
for 3 weeks. What treatment should be secretion is affected on aldosteronism?
provided to the patient if he goes into wi-
thdrawal? A. Adrenal glands
B. Testicle
A. Diazepam C. Ovaries
B. Phenobarbital D. Pancreas
C. Pentobarbital E. Hypophysis
D. Phenytoin
E. None 71. A patient had been taking glucocorti-
coids for a long time. When the
67. A 13-year-old girl with history of preparation was withdrawn he developed
asthma complained of cough, dyspnea the symptoms of disease aggravation,
and wheezing. Her symptoms became decreased blood pressure and weakness.
so severe that her parents brought her What is the reason of this condition?
to the emergency room. Physical exami-
nation revealed diaphoresis, dyspnea, A. Appearance of adrenal insufficiency
tachycardia and tachypnea. Her respi- B. Hyperproduction of ACTH
ratory rate was 42/min, pulse rate was 110 C. Sensibilization
beats per minute, and blood pressure was D. Habituation
130/70 mm Hg. Choose from the following E. Cumulation
list the most appropriate drug to reverse
the bronchoconstriction rapidly: 72. A patient after hypertension stroke
does not have voluntary movements in
A. Salbutamol his right arm and leg with the increased
B. Cromolyn muscle tone in these extremites. What
C. Beclomethasone type of disfunction of nervous system is
D. Methylprednidsolone it?
E. Ipratropium
A. Central paralysis
68. A doctor administered Allopurinol B. Peripheral paralysis
to a 26-year-old young man with the C. Peripheral paresis
symptoms of gout. What pharmacological D. Reflex paresis
action of Allopurinol ensures therapeuti- E. Central paresis
cal effect?
73. A patient, who suffers from congeni-
A. By inhibiting uric acid synthesis tal erythropoietic porphyria, has skin
B. By increasing uric acid excretion photosensitivity. The accumulation of
C. By inhibiting leucocyte migration into what compound in the skin can cause it?
the joint
D. By general anti-inflammatory effect A. Uroporphyrinogen 1
E. By general analgetic effect B. Protoporphyrin
C. Uroporphyrinogen 2
69. A patient who came to the doctor D. Coproporphyrinogen 3
because of his infertility was admini- E. Heme
stered to make tests for toxoplasmosis
and chronic gonorrhoea. Which reacti- 74. A patient has a malignisation
on should be performed to reveal latent of thoracic part of esophagus. What
toxoplasmosis and chronic gonorrhoea of lymphatic nodes are regional for this
the patient? organ?
Krok 1 Medicine 2005 9

A. Anulus lymphaticus cardiae A. К


B. Nodi lymphatici paratrachealis B. А
C. Nodi lymphatici prevertebralis C. D
D. Nodi lymphatici pericardiales laterales D. Е
E. Nodi lymphatici mediastinales posteri- E. Carotene
ores
80. Blood analysis of a patient showed si-
75. A sick man with high temperature and gns of HIV infection (human immunodefi-
a lot of tiny wounds on the body has been ciency virus). Which cells does HIV-virus
admitted to the hospital. Lice have been primarily affect?
found in the folds of his clothing. What
disease can be suspected in the patient? A. Cells that contain receptor T4 (T-
helpers)
A. Epidemic typhus B. Cells that contain receptor IgM (B-
B. Tularemia
C. Scabies lymphocytes)
D. Malaria C. Specialized nervous cells (neurons)
E. Plague D. Mast cells
E. Proliferating cells (stem hematoplastic
76. A patient with suspicion on epidemic cells)
typhus was admitted to the hospital. Some
arachnids and insects have been found in 81. The preventive radioprotector was
his flat. Which of them may be a carrier of given to a worker of a nuclear power
the pathogen of epidemic typhus? station. What mechanism from the below
mentioned is considered to be the main
A. Lice mechanism of radioprotection?
B. Spiders
C. Bed-bugs A. Inhibition of free radicals formation
D. Cockroaches B. Prevention of tissue’s hypoxia
E. Houseflies C. Activation of oxidation reactions
D. Increasing of tissue blood supply
77. A businessman came to India from E. Increasing of respiration
South America. On examination the
physician found that the patient was 82. The pulmonalis embolism has
suffering from sleeping-sickness. What suddenly developed in a 40 year-old
was the way of invasion? patient with opened fracture of the hip.
Choose the possible kind of embolism.
A. As a result of bug’s bites
B. As a result of mosquito’s bites A. Fat
C. With contaminated fruits and vegetables B. Thrombus-embolus
D. Through dirty hands C. Air
E. After contact with a sick dogs D. Tissue
E. Foreign body
78. The patient has come to the hospital
from the smelting workshop in the condi- 83. A 52 year-old patient with bronchi-
tion of hyperthermia. What is the direct al asthma was treated with glucocorticoi-
cause of loss of consciousness at the heat ds. Fever reaction appeared as a result
stroke? of postinjective abscess. The patient
had subfebrile temperature, which didn’t
A. Decreased brain blood supply correspond to latitude and severity of
B. Arterial pressure drop inflammatory process. Why did patient
C. Increased water loss through sweating have low fever reaction?
D. Decrease of heart output
E. Dilatation of peripheral vessels A. Inhibited endogen pyrogens production
B. Violation of heat loss through lungs
79. There is an inhibited coagulation in C. Inflammatory barrier formation in
the patients with bile ducts obstruction, injection place
bleeding due to the low level of absorbti- D. Violation of heat-producing mechani-
on of a vitamin. What vitamin is in defici- sms
ency? E. Thermoregulation center inhibition
84. Patient 54 year-old, 5th day
after surgical operation. Blood count:
Krok 1 Medicine 2005 10

Erythrocytes 3, 6 ∗ 1012 /l, Hemoglobin A. Decreased hydrostatic pressure in


95 g/l, Erythrocyte’s hemoglobin content glomerular capillaries
(color index) 0,78; Leukocytes 16 ∗ 109 /l, B. Increased osmotic pressure in
Platelets 450 ∗ 109 /l Blood picture: ani- glomerular capillaries
zocytosis, poikilocytosis, reticulocytes- C. Increased pressure in Bowman’s capsule
3,8%. What anemia does this patient D. Increased vasopressin blood
have? concentration
E. Trauma of the urinary bladder
A. Acute posthemorragic anemia
B. Acquired hemolytic anemia 89. A 46 year-old patient has complai-
C. Anemia from iron deficiency ned of headache, fatigue, thirst, pains
D. Hypoplastic anemia in the spine and joints for the last 2
E. Chronic posthemorragic anemia years. Clinically observed disproportional
enlargement of hands, feet, nose, superci-
85. A patient’s blood was analyzed and liary arches. He notes that he needed
the decreased erythrocyte’s sedimentation to buy bigger shoes three times. What
rate (ESR) was discovered. What disease is the main reason of such disproporti-
from the listed below is accompanied with onal enlargement of different parts of the
decreased ESR? body?

A. Polycytemia A. Cartilaginous tissue proliferation under


B. Hepatitis growth hormone influence
C. Splenomegaly B. Increased sensitivity of the tissues to
D. Vitamin B deficiency growth hormone
E. Myocardial infarction C. Joints dystrophy development
D. Increased sensitivity of the tissues to
86. X-ray examination discovered lungs insulin
emphysema in the patient. What is the E. Joints chronic inflammation
reason of short breath development in this development
case?
90. A 55-year-old patien was hospitali-
A. Decreased lungs elasticity zed in result of the trauma of the medial
B. Increased lungs elasticity group of femoral muscles. What kind of
C. Inhibition of respiratory center movements is the patient unable to do?
D. Excitation of respiratory center
E. Decreasing of alveoli receptors sensiti- A. Adduction of femur
vity B. Abduction of femur
C. Flexion of femur
87. The patient with pneumonia was D. Extension of femur
treated with antibiotics for a long peri- E. Suppination of femur
od. After treatment patient complains of
frequent and watery stool, abdomenal 91. A mother of a newborn complai-
pain. What is the reason of intestine ns of her baby’s constant belching with
function disorder? undigested milk. Which developmental
anomaly is it an evidence of?
A. Intestinal disbacteriosis development
B. Antibiotics toxic influence on the GIT A. Esophageal atresia
C. Autoimmune reaction development B. Labium leporium
D. Bacteria toxins influence C. Faux lupinum
E. Hereditary enzyme defect D. Anal atresia
E. Esophageal fistula
88. A 16 year-old patient got numerous
traumas in automobile accident. Now the 92. During the endoscopy the inflammati-
patient is haning a shock. АP - 80/60 mm on of a major papilla of the duodenum
Hg. daily urine volume 60-80 ml. What and the disturbances of bile secretion
pathogenic mechanism leads to kidneys were found. In which part of duodenum
function violation? were the problems found?
Krok 1 Medicine 2005 11

A. Descendent part bleeding?


B. Ascendant part
C. Bulb A. The mandible’s edge
D. Upper horizontal part B. The mental process
E. Lower horizontal part C. The mandible’s branch
D. The nose’s back
93. A 18-year-old patient came to the out- E. The molar bone
patient department with the complaints
of bleeding trauma in the vestibule of 98. A 19 year-old patient was diagnosed
his nose. On examination: the mechanical with appendicitis and was hospitali-
injure of the mucous layer of the vesti- zed. The surgical operation on ablating
bule without continuation into nasal cavi- appendix vermiformis is to be performed.
ty proper. What is the boundary between What artery must be fixed to stop bleedi-
the vestibule and nasal cavity proper? ng during the surgical operation?
A. Nasal limen A. The ileocolic artery
B. Nasal roller B. The colica dextra
C. Nasal septa C. The colica media
D. Choanes D. The colica sinistra
E. Nostrils E. The iliac
94. A 32-year-old patient has been di- 99. An autopsy has revealed that kidneys
agnosed with bartholinitis (inflammation are enlarged, surface is large-granular
of Bartholin’s glands ). In what part of the because of multiple cavities with smooth
female urogenital system are the Bartholi- wall, which are filled with clear fluid. What
n’s glands located? kidney disease did the patient have?

A. The labia major A. Polycystic kidney


B. The labia minor B. Necrotic nephrosis
C. The clitoris C. Pyelonephritis
D. The vagina D. Glomerulonephritis
E. The uterus E. Infarction

95. A 50 year-old patien was injured on 100. An old woman was hospitalized with
the occipital region of the head. The acute pain, edema in the right hip joint;
closed skull’s trauma was diagnosed. She the movements in the joint are limited.
was taken to the hospital. The medical Which bone or part of it was broken?
examination: deregulation of walking and
balance, trembling of arms. What part of A. The neck of the thigh
brain was injured? B. The body of the thigh bone
C. Condyle of the thigh
A. The cerebellum D. Pubic bone
B. The medulla oblongata E. Ischial bone
C. The mind-brain
D. The inter-brain 101. A 45-year-old man fell on the ri-
E. The spinal cord ght knee and felt the acute pain in the
joint. On examination: severe edema on
96. A 50 year-old patient had hemorrhage the anterior surface of the knee joint.
of the brain and was taken to the hospital. Crunching sounds are heard while moving
The place of hemorrhage was revealed on the joint. Which bone is destroyed?
the lateral hemispheres surfaces during
the medical examination. What artety was A. Knee-cap
injured ? B. Neck of the thigh bone
C. Left epicondyle of the thigh
A. The middle cerebral artery D. Right epicondyle of the thigh
B. The anterior cerebral artery E. Head of the thigh bone
C. The posterior cerebral artery
D. The anterior communicating artery 102. During the operation on the hip joi-
E. The posterior communicating artery nt of a 5-year-old child her ligament was
damaged which caused bleeding.What li-
97. A 30-year-old patient was hospitalized gament was damaged?
due to bleeding of the facial artery . What
place on the face has to be pressed to stop
Krok 1 Medicine 2005 12

A. The head of the thigh 107. A 35-year-old man under the


B. Perpendicular of the acetabule treatment for pulmonary tuberculosis has
C. Iliofemoral acute-onset of right big toe pain, swelling,
D. Pubofemoral and low-grade fever. The gouty arthritis
E. Ischiofemoral was diagnosed and high serum uric acid
level was found. Which of the followi-
103. A 6-year-old child fell on the cutti- ng antituberculosis drugs are known for
ng object and traumatized soft tissues causing high uric acid levels?
between tibia and fibula . What kind of
bone connection was injured? A. Pyrazinamide
B. Cycloserine
A. Membrane C. Thiacetazone
B. Suture D. Rifampicin
C. Ligament E. Aminosalicylic acid
D. Fontanel
E. Gomphosis 108. A person was selling "homemade
pork"sausages on the market. State sani-
104. An autopsy revealed: soft arachnoid tary inspector suspected falcification of
membrane of the upper parts of cerebral the sausages. What serological immune
hemisphere is plethoric, of yellowish- reaction can identifiy food substance?
green color, soaked with purulent and
fibrose exudate, it lookes like a cap. A. Precipitation test
What disease is characterised by these B. Indirect hemagglutination test
symtoms? C. Agglutination test
D. Immunofluorescence test
A. Meningococcal meningitis E. Complement- fixation test
B. Tuberculous meningitis
C. Influenza meningitis 109. A 25-year-old woman with red and
D. Meningitis at anthrax itchy eczematoid dermatitis visits your
E. Meningitis at typhus office. She had a dental procedure one
day earlier with administration of a local
105. Usually the intravenous injection is anesthetic. There were no other findings,
done into median cubital vein because it although she indicated that she had a hi-
is slightly movable due to fixation by the story of allergic reactions. Which of the
soft tissues. What does it fix in the cubital following drugs is most likely involved?
fossa?
A. Procaine
A. Aponeurosis of biceps muscle B. Cocaine
B. Tendon of the triceps muscle C. Lidocaine
C. Brachial muscle D. Bupivacaine
D. Brachioradial muscle E. Etidocaine
E. Anconeus muscle
110. The CNS stimulation produced by
106. A 55-year-old patient with continuing methylxanthines, such as caffeine, is most
ventricular arrhythmias was admitted to likely due to the antagonism of one of the
the hospital. The patient is taking timolol following receptors:
drops for glaucoma, daily insulin injecti-
ons for diabetes mellitus, and an ACE A. Adenosine receptors
inhibitor for hypertension. You have deci- B. Glycine receptors
ded to use phenytoin instead of procai- C. Glutamate receptors
namide. What is the reason? D. GABA receptors
E. Cholinergic muscarinic receptors
A. The anticholinergic effect of procai-
namide would aggravate glaucoma 111. A patient has elbow joint trauma
B. The local anesthetic effect of procai- with avulsion of medial epicondyle of
namide would potentiate diabetes humerus. What nerve can be damaged in
C. The hypertensive effects of procainami- this trauma?
de would aggravate the hypertension
D. The local anesthetic effect of procai- A. Ulnar
namide would aggravate the hypertension B. Radial
E. The cholinergic effects of procainamide C. Musculocutaneous nerve
would aggravate the diabetes D. Cardiac cutaneous nerve
E. Medial cutaneous nerve of forearm
Krok 1 Medicine 2005 13

112. A student is thoroughly summarisi- A. Lactate


ng a lecture. When his groupmates begin B. Pyruvate
talking the quality of the summarising C. 1,3-bisphosphoglycerate
worsens greatly. What type of inhibition D. α-ketoglutarate
in the cerebral cortex is the cause of it? E. 3-phosphoglycerate
A. External 117. Substitution of the glutamic acid on
B. Protective valine was revealed while examining initi-
C. Dying al molecular structure. For what inherited
D. Differential pathology is this symptom typical?
E. Delayed
A. Sickle-cell anemia
113. While enrolling a child to school B. Thalassemia
Mantu’s test was made to define whether C. Minkowsky-Shauffard disease
revaccination was needed. The test result D. Favism
is negative. What does this test result E. Hemoglobinosis
mean?
118. Inflamation is characterised by
A. Absence of cell immunity to the increasing penetration of vessels of mi-
tuberculosis crocirculation stream, increasing of their
B. Presence of cell immunity to the fluid dynamic blood pressure. Increasing
tuberculosis of the osmotic concentration and dispersi-
C. Absence of antibodies for tubercle ty of protein structures can be found in the
bacillus intercellular fluid. What kind of edema are
D. Absence of antitoxic immunity to the to be observed in this case?
tuberculosis
E. Presence of antibodies for tubercle A. Mixed
bacillus B. Hydrodynamic
C. Colloid-osmotic
114. The donor who had not donated the D. Lymphogenic
blood for a long time was examined with E. Membranogenic
IFA method. Anti-HBs antibodies were
revealed. What does positive result of IFA 119. Scraps of the mycelium of a fungus,
in this case mean? spores, air bubbles and fat drops were di-
scovered on microscopy of the patient’s
A. Previous hepatitis B hair excluded from the infected areas.
B. Acute hepatitis B What fungus disease is characterised by
C. Acute hepatitis C this microscopic picture?
D. Chronic hepatitis В
E. Chronic hepatitis С A. Favus
B. Microspory
115. Inhibition of alpha-motoneuron of C. Trichophytosis
the extensor muscles was noticed after sti- D. Epidermophytosis
mulation of α-motoneuron of the flexor E. Sporotrichosis
muscles during the experiment on the spi-
nal column. What type of inhibition can 120. Autopsy of the 58-year-old man hads
be caused by this process? revealed that mitral valve is deformed, thi-
ckened, does not totally close. Microscopi-
A. Reciprocal cally: centers of collagen fibers are eosi-
B. Presynaptic nophilic, have positive fibrin reaction. The
C. Depolarizational most probable diagnosis is:
D. Recurrent
E. Lateral A. Fibrinoid swelling
B. Fibrinoid inflammation
116. A worker has decreased buffer C. Mucoid swelling
capacity of blood due to exhausting D. Hyalinosis
muscular work The influx of what acid E. Amyloidosis
substance in the blood can cause this
symptom? 121. A highly injured person has gradually
died. Please choose the indicator of bi-
ological death:
Krok 1 Medicine 2005 14

A. Autolysis and decay in the cells it?


B. Disarray of chemical processes
C. Loss of consciousness A. Sinusoidal
D. Absence of palpitation and breathing B. Fenestrational
E. Absence of movements C. Somatical
D. Visceral
122. A 37-year-old man was admitted E. Lymphatic
to the surgical department with the
symptoms of acute pancreatitis: vomiti- 127. A 57-year-old patient was admitted
ng, diarrhea, bradycardia, hypotention, to the gastroenterological department wi-
weakness, dehydration of the organism. th suspicion of Zollinger-Ellison syndrom
What medicine should be used first of all? because of rapid increase of gastrin level
in the blood serum. What the most
A. Contrycal probable disorder of the secretory functi-
B. No-spa on of the stomach here?
C. Platyphylline
D. Etaperazine A. Hyperacidity hypersecretion
E. Ephedrine B. Hyperacidity hyposecretion
C. Achylia
123. On autopsy a 35-year-old man the D. Hypoacidity hyposecretion
focus of carnification 5 cm in diametre E. Hypoacidity hypersecretion
enclosed in a thin capsule was revealed in
the second segment of the right lung . The 128. After breathing with poisonous
focus consists of a tough dry friable tissue steams there is an increased quantity of
with a dim surface. For what disease are slime in respiratory passages of a chemi-
these morphological changes typical? cal production worker. What of respi-
ratory tract epithelial cells participate in
A. Tuberculoma mucousa moistening?
B. Lung cancer
C. Chondroma A. Goblet cells
D. Tumorous form of silicosis B. Fibroblasts
E. Postinflammatory pneumosclerosis C. Endocrine cells
D. Langergans cells
124. A 62-year-old patient was admi- E. Intercalated cells
tted to the neurological department due
to cerebral haemorrage. His conditi- 129. A patient suffering from thyrotoxi-
on is grave. There is evident progressi- cosis symptoms of vegetoasthenic
on of deep and frequent breath that syndrome was revealed. What of the
turnes into reduction to apnoea and the following would show the histological
cycle repeates. What respiration type has appearance of a thyroid gland being sti-
developed in the patient? mulated by thyroid-stimulating hormone
(TSH)?
A. Cheyne-Stockes respiration
B. Kussmaul respiration A. Columnar-shaped follicular cells
C. Biot’s respiration B. Decreased numbers of follicular cells
D. Gasping respiration C. Increased numbers of parafollicular
E. Apneustic respiration cells
D. An abundance of colloid in the lumen
125. Punctata hemorrhage was found out of the follicle
in the patient after application of a tourni- E. Decreased numbers of parafollicular
quet. With disfunction of what blood cells capillaries
is it connected?
130. For a long time a 49-year-old woman
A. Platelets had suffered from glomerulonephri-
B. Eosinophiles tis which caused death. The autopsy
C. Monocytes revealed that the size of her kidneys
D. Lymphocytes was 7х3х2,5 sm, weight 65,0 g, they were
E. Neutrophiles dense and small-grained. Microscopi-
cally: fibrinogenous inflammation of
126. In the microspecimen of red bone serous and mucous capsules, dystrophic
marrow multiple capillares were revealed changes of parenchymatous organs, brain
through the walls of which mature blood edema. What complication can cause such
cells penetrated. What type of capillares is changes of serous capsules and inner
Krok 1 Medicine 2005 15

organs? woman is healthy and there were no cases


of hemophilia in her family. What is the
A. Uraemia risk of having a sick child in this family?
B. Anemia
C. Sepsis A. 0
D. DIC-syndrome B. 100%
E. Thrombopenia C. 75%
D. 50%
131. The reason of occurrence of some di- E. 25%
seases of an oral cavity is connected wi-
th structural peculiarities of its mucous 136. Oval and round organelles with
membrane. What morphological attri- double wall are seen at the electron mi-
butes characterize these features? crograph. The outer membrane is smooth,
the inner membrane folded into cristae
A. No muscularis mucosa, stratified contain enzyme ATPase synthetase. These
squamous epithelium are:
B. Transitional epithelium, no submucosa
C. Simple columnar ciliated epithelium A. Mitochondria
D. Well developed muscularis, no B. Golgi complex
submucosa C. Lysosomes
E. Transitional epithelium, no muscularis D. Centrioles
mucosa E. Ribosomes
132. There is the change of teeth at 137. A tissue sample of benign tumor was
the 6-8-year-old children: deciduous are studied under the electron microscope.
replaced by permanent. What embrionic A lot of small (15-20 nm) spherical bodi-
tissues are the sources of formation of es, consisting of 2 unequal subunits were
permanent teeth tissues? detected. These are:
A. Ectodermal epithelium of a tooth plate A. Ribosomes
and mesenhime B. Golgi complex
B. Entodermal epithelium of a tooth plate C. Smooth endoplasmic reticulum
and mesenhime D. Microtubules
C. Mesodermal epithelium and mesenhime E. Mitochondria
D. I, II brachial arches
E. Entodermal epithelium and mesoderm 138. A woman who was sick with rubella
during the pregnancy gave birth to a deaf
133. The B cells of endocrine portion child with hare lip and cleft palate. This
of pancreas are selectively damaged by congenital defect is an example of:
alloxan poisoning. How will it be reflected
in blood plasma? A. Phenocopy
B. Edward’s syndrome
A. The content of sugar increases C. Genocopy
B. The content of fibrinogen decrease D. Patau’s syndrome
C. The level of sugar decreases E. Down’s syndrome
D. The content of globulins decreases
E. The content of albumins decreases 139. A woman who was infected with
toxoplasmosis during the pregnancy has a
134. A healthy woman has three sons child with multiple congenital defects.This
affected by color blindness who were born is a result of:
after her two marriages. Children both of
her husbands are healthy. What is the most A. Teratogenesis
possible pattern of inheritance of this di- B. Cancerogenesis
sease? C. Biological mutogenesis
D. Chemical mutogenesis
A. X-linked recessive E. Recombination
B. Y-linked
C. Autosomal recessive 140. At the aboratory experiment
D. Autosomal dominant the eukocyte culture was mixed with
E. X-linked dominant staphylococci. Neutrophile leukocytes
engulfed and digested bacterial cells. This
135. A couple came for medical genetic processes are termed:
counseling. The man has hemophilia, the
Krok 1 Medicine 2005 16

A. Phagocytosis
B. Pinocytosis A. Thyroxine
C. Diffusion B. Cortisol
D. Facilitated diffusion C. Mineralocorticoids
E. Osmosis D. ACTH
E. Insulin
141. Marked increase of activity of МВ-
forms of CPK (creatinephosphokinase) 146. The concentration of albumins
and LDH-1 was revealed by examination in human blood sample is lower than
of the patient’s blood. What is the most normal. This leads to edema of tissues.
probable pathology? What blood function is damaged?
A. Miocardial infarction A. Maintaining the oncotic blood pressure
B. Hepatitis B. Maintaining the Ph level
C. Rheumatism C. Maintaining the body temperature
D. Pancreatitis D. Maintaining the blood sedimentation
E. Cholecystitis system
E. All answers are correct
142. An isolated muscle of a frog
is rhythmically irritated with electric 147. A patient with tissue trauma was
impulses. Every next impulse is in a period taken a blood sample for the determinati-
of relaxation from the previus contraction. on of blood clotting parameters. Specify
What contraction of the muscle occurs? the right sequence of extrinsic pathway
activation.
A. Waved tetanus
B. Single A. III – VIIa – Xa
C. Asynchronous B. III – IV – Xa
D. Continuous (smooth) tetanus C. IV – VIII: TF – Xa
E. Tonic D. IV – VIIa – Xa
E. III – VIII: TF – Xa
143. M-r S presents all signs of the
hepatic coma: loss of consciousness, 148. A patient has been taking a mi-
absence of reflexes, cramps, convulsion, xture prescribed by neuropathologist for
disorder of heart activity, recurrent (peri- neurasthenia for two weeks. The patient
odical) respiration. What are cerebrotoxi- feels better but has developed coryza,
cal substances which accumulate in blood conjunctivitis, rash, inertia, decrease of
under hepar insufficiency? memory. She is diagnosed with bromizm.
What should be prescribed to decrease the
A. Ammonia symptoms?
B. IL-1
C. Autoantibody A. Natrium chloride
D. Necrosogenic substances B. Glucose solution 5%
E. Ketonic body C. Asparcam
D. Polyglucin
144. A 16-year-old boy was performed an E. -
appendectomy. He has been hospitalized
for right lower quadrant abdominal pain 149. On autopsy of a still-born infant
within 18 hours. The surgical specimen is abnormalities have been revealed: ventri-
edematous and erythematous. Infiltration cles are not separated, a single arterial
by what of the following cells is the most trunk originates from the right part. For
typical for the process occuring here? what class of vertebrates is such heart
construction characteristic?
A. Neutrophils
B. Eosinophils A. Amphibian
C. Basophils B. Fishes
D. Limphocytes C. Reptiles
E. Monocytes D. Mammals
E. Birds
145. A 19-year-old female suffers from
tachycardia in rest condition, weight loss, 150. If strong oxidizers get into the
excessive sweating, exophtalmos and irri- bloodstream, a methemoglobin is formed.
tability. What hormone would you expect It is a compound, where iron (II) becomes
to find elevated in her serum? iron (III). What has to be done to save the
Krok 1 Medicine 2005 17

patient? A. The rejection of food


B. Aggressive behaviour
A. Interchangeable hemotransfusion has C. Depression
to be done D. Thirst
B. Patient has to be exposed to the fresh E. Unsatisfied hunger
air
C. He has to be calmed down and put to 155. During the experiment on the
bed influence of chemical substances in the
D. He has to be given pure oxygen muscles the reaction of Ca2+ -pump is
E. Respiratory centers have to be sti- weakened. Which phenomenum will be
mulated observed?
151. A 10-year-old child complains of A. Prolonged relaxation
weakness, nausea, irritability. Helminthes B. Prolonged duration of the AP
of white color and 5-10 mm long have C. Decreased AP
been found on the underwear. On mi- D. Activation of the sodium-potassium
croscopy of the scrape from the perianal pump
folds achromic ova of the unsymmetrical E. Decreased velocity of the AP distributi-
form have been revealed. Which helminth on
is in the organism of the child?
156. The process of heart transplantati-
A. Enterobins vermicularis on determined the viability of myocardi-
B. Ascaris lumbricoides al cells. The determination of what
C. Ancylostoma duodenalis myocardium parameter is the most
D. Trichina important?
E. Trichuris
A. Rest potential of cardiomyocytes
152. The low specific gravity of the B. Heart temperature
secondary urine (1002) was found out in C. Concentration of oxygen in heart vessels
the sick person. Wat is the most distant D. Concentration of calcium-ions in myofi-
part of nephron where concentration of brils
secondary urine takes place? E. Concentration of Ca-ions in heart
vessels
A. In the collecting duck
B. In the nephron’s glomerulus 157. The calcium canals of cardiomyocytes
C. In proximal tubule of nephron have been blocked on an isolated rabbit’s
D. In ascending part of loop of Henle heart. What changes in the heart’s activity
E. In distal tubule of nephron can happen as a result?

153. A patient with the symptoms of acute A. Decreased rate and force of heart beat
alcoholic poisoning was brought to the B. Decreased heart beat rate
hospital. What carbohydrates metabolism C. Decreased force of the contraction
changes are typical for this condition? D. Heart stops in systole
E. Heart stops in diastole
A. The gluconeogenesis velocity in liver is
decreased 158. After the trauma, the patient’s right
B. The gluconeogenesis is increased in n.vagus was damaged. Which violation of
liver the cardiac activity is possible in this case?
C. The breakage of glycogen is increased
in liver A. Violation of the automatism of a Kiss-
D. The anaerobic glucose metabolism Fleck node
predominates in muscles B. Violation of the automatism of a atrio-
E. The anaerobic breakage of glucose is ventricular node
increased in muscles C. Violation of a conductivity in the right
auricle
154. A 60-year-old patient was diagnosed D. Block of a conductivity in the atrio-
with hypothalamic lateral nuclei stroke. ventricular node
What changes in patient’s behavior may E. Arrhythmia
be expected?
159. A 59-year-old man has symptoms
of parenchymatous jaundice and portal
hypertension. Histological examinati-
on of the puncture of the liver bi-
Krok 1 Medicine 2005 18

optate has revealed an affected beam- sm strengthens destructive effects in the


lobule structure, part of hepatocytes inflammation area?
has signs of fat dystrophy, port-portal
connective tissue septa with formation of A. Secondary alteration
pseudo-lobules, with periportal lympho- B. Primary alteration
macrophage infiltrations. What is the most C. Emigration of lymphocytes
probable diagnosis? D. Diapedesis of erythrocytes
E. Proliferation of fibroblasts
A. Liver cirrhosis
B. Alcohol hepatitis 164. Analeptical remedy of reflective type
C. Chronic hepatosis from the H-cholinomimetics group was
D. Viral hepatitis given to the patient for restoration of
E. Toxic dystrophy breathing after poisoning with carbon
monoxide. What medicine was prescribed
160. A microscopic examination of the to the patient?
enlarged neck gland of a 14-year-old
girl revealed destruction of the tissue A. Lobeline hydrochloride
structure of the node, absence of the B. Atropine sulphate
lymph follicles, sclerotic and necrosis C. Adrenalin hydrochloride
parts. Cell constitution of the node is D. Mesaton
polymorphous, lymphocites, eosinophi- E. Pentamin
les, atypical cells of the large size wi-
165. Intrapleural pressure is being
th multiple-lobule nuclei (Beresovsky-
measured in a person. In what phase does
Shternberg cells) and onenucleus large a person hold his breath if the pressure is
size cells are observed. What is the most - 25 cm H2 O?
likely diagnosis?
A. Forced inspiration
A. Lymphogranulomatous B. Quiet expiration
B. Acute lympholeucosis C. Quiet inspiration
C. Chronic lympholeucosis D. Forced expiration
D. Berkitt’s lymphoma E. -
E. Fungous mycosis
166. Blood sampling for bulk analysis
161. A damage of the atomic power is recommended to be performed on an
plant reactor resulted in the run- empty stomack and in the morning. What
out of radioelements. People in the changes in blood composition can occur
superstandard radiation zone were radi- if to perform blood sampling after food
ated with approximately 250-300 r. and intake?
were immediately hospitalized. What
changes in the blood count would be typi- A. Increased contents of leukocytes
cal? B. Increased contents of erythrocytes
C. Increased plasma proteins
A. Lymphopenia D. Reduced contents of thrombocytes
B. Leukopenia E. Reduced contents of erythrocytes
C. Anemia
D. Thrombopenia 167. Examination of a person revealed
E. Neutropenia that minute volume of heart is 3500 mL,
systolic volume is 50 mL. What is the
162. Obturative jaundice developed in frequency of cardiac contraction?
a 60-year-old patient because of mali-
gnant tumour of the big papillary of the A. 70 bpm
duodenal. Lumen of what anatomical B. 60 bpm
structure is squeezed with tumour? C. 50 bpm
D. 80 bpm
A. Hepatopancreatic ampulla E. 90 bpm
B. Cystic duct
C. Common hepatic duct 168. Glomerular filtration rate (GFR)
D. Right hepatic duct increased by 20% due to prolonged
E. Left hepatic duct starvation of the person. The most evi-
dent cause of filtration changes under this
163. Necrosis focus was observed in the conditions is:
area of hyperemia and skin edema in
a few hours after burn. What mechani-
Krok 1 Medicine 2005 19

A. Decrease of oncotic pressure of blood A. Inversion


plasma B. Deletion
B. Increase of systemic blood pressure C. Doubling
C. Increase of penetration of the renal D. Translocation
filter E. Replication
D. Increase of filtration coefficient
E. Increase of renal plasma stream 174. A histological spacemen presents
parenchymal organ, which has cortex
169. In the ovary specimen colored with and medulla. Cortex consists of epi-
hematoxylin-eosin, follicle is determined theliocytes bars with blood capillaries
where cubic-shaped follicle epithelium between them; the bars form three zones.
cells are placed in 1-2 layers, and scarlet Medulla consists of chromaffinocytes and
covering is seen around ovocyte. Name venous sinusoids. Which organ has these
this follicle: morphological features?
A. Primary A. Adrenal gland
B. Primordial B. Kidney
C. Secondary C. Lymph node
D. Mature D. Thymus
E. Atretic E. Thyroid
170. A patient complains of frequent and 175. A patient with bronchial asthma
difficult urination. Imperfection of what had been taking tablets which caused
formation can cause it? insomnia, headache, increased blood
pressure. What medecine can cause such
A. Prostate complications?
B. Testicles
C. Bulb-uretic glands A. Ephedrine
D. Testicle adnexa B. Adrenaline
E. Sperm bubbles C. Chromolin sodium
D. Euphyline
171. A 45-year-old man with domestic E. Izadrine
apper arm injuiry came to the trauma
unit. The objective data are: there are no 176. A patient died 3 days after the
extension, adduction or pronation functi- operation because of perforated colon
ons of the arm. What muscle damage with manifestations of diffuse purulent
caused this condition? peritonitis. The autopsy revealed: colon
mucos membrane was thickened and
A. Teres major covered with a fibrin film, isolated
B. Subscapular ulcers penetrated at different depth.
C. Teres minor The histology result: mucous membrane
D. Subspinous necrosis, leukocytes infiltration with
E. Supraspinous hemorrhages focuses. What disease
complication caused the patient’s death?
172. A 68-year-old woman can not move
her upper and lower right extremities A. Dysentery
after stroke. Muscle tone of these extremi- B. Typhoid
ties and reflexes are increased. There are C. Nonspecific ulcerative colitis
pathological reflexes. What form of the D. Crohn’s disease
paralysis is it? E. Amebiasis
A. Hemiplegia 177. Periodic renal colics attacks are
B. Paraplegia observed in a woman with primery
C. Tetraplegia hyperparathyroidizm. Ultrasonic exami-
D. Monoplegia nation revealed small stones in the ki-
E. Dissociation dneys. What is the most plausible reason
of the stones’s formation?
173. Part of the DNA chain turned about
180 degrees due to gamma radiation.
What type of mutation took place in the
DNA chain?
Krok 1 Medicine 2005 20

A. Hypercalcemia
B. Hyperphosphatemia A. 1,5 mmol/L
C. Hypercholesterinemia B. 8,0 mmol/L
D. Hyperuricemia C. 10,0 mmol/L
E. Hyperkalemia D. 3,3 mmol/L
E. 5,5 mmol/L
178. Ovarian tumour was diagnozed in
a woman. Surgery should be perfomed. 183. A 63-year-old woman developed
What ligament should be extracted by the symptoms of rheumatoid arthritis. Their
surgeon to disconnect the ovary and the increase of which blood values indicators
uterus? could be the most significant in proving
the diagnosis?
A. The ovarial ligament
B. Broad ligament of uterus A. Additive glycosaminoglycans
C. Lateral umbilical ligament B. Lipoproteids
D. Suspensory ligament of ovary C. Acid phosphatase
E. Round ligament of uterus D. General cholesterol
E. R-glycosidase
179. An intraoperational biopsy of
mammal gland has revealed the si- 184. An autopsy revealed large (1-2 cm)
gns of atypical tissue with disorder of brownish-red, easy crumbling formations
parenchyma stroma proportion with covering ulcerative defects on the external
domination of the last, gland structures of surface of the aortic valve. What is the
different size and shape, lined with single- most likely diagnosis?
layer proliferative epithelium. What is the
most probable diagnosis? A. Polypus-ulcerative endocarditis
B. Recurrent warty endocarditis
A. Fibroadenoma C. Acute warty endocarditis
B. Papilloma D. Fibroplastic endocarditis
C. Noninfiltrative cancer E. Diffusive endocarditis
D. Infiltrative cancer
E. Mastitis 185. Red colonies spread in the large
quantity in the Endo culture medium were
180. The increased intraocular tensi- revealed on bacteriological stool exami-
on is observed in a patient with nation of a 4-month-old baby with the
glaucoma.Secretion of aqueous humor symptoms of acute bowel infection. What
by the ciliar body is normal. Injury of microorganism can it be?
what structure of the eyeball caused the
disorder of flow-out from the anterior A. Escherichia
chamber? B. Salmonella
C. Staphylococcus
A. Venous sinus D. Streptococcus
B. Ciliar body E. Shigell
C. Choroid
D. Ciliary muscle 186. A patient with abscess of the cut
E. Back epithelium of cornea wound applied to the traumatological
department.The wound was washed with
181. Chronic glomerulonephritis was di- 3% hydrogen peroxide to be cleaned from
agnosed in a 34-year-old patient 3 years the pus. Foam was not observed. What
ago. Edema has developed within the last caused inefficiency of the drug?
6 monthes. What caused the edema?
A. Inherited insufficiency of catalase
A. Proteinuria B. Low concentration H2 O2
B. Hyperproduction of vasopressin C. Inherited insufficiency erythrocyte’s
C. Liver disfunction of protein formation phosphatdehydrogenase
D. Hyperosmolarity of plasma D. Shallow wound
E. Hyperaldosteronism E. Pus in the wound
182. Patient with diabetes mellitus experi- 187. A patient with injured muscles of
enced loss of consciousness and convulsi- the lower extremities was admitted to
ons after an injection of insulin. What the traumatological department. Due to
might be the result of biochemical blood what cells is reparative regeneration of
analysis for concentration of sugar? the muscle fibers and restoration of the
Krok 1 Medicine 2005 21

muscle function possible? A. Hyperpolarization


B. Depolarization
A. Satellite-cells C. Action potential
B. Myoblasts D. Local response
C. Myofibroblasts E. No changes
D. Fibroblasts
E. Myoepithelial cells 193. The electronic microphoto of kidney
fragment has exposed afferent glomerular
188. The study of the genealogy of a fami- arteriole, which has giant cells under
ly with hypertrichosis (helix excessive pi- its endothelium, containing secretory
losis) has demonstrated that this symptom granules. Name the type of these cells:
is manifested in all generations only in
men and is inherited by son from his A. Juxtaglomerular
father. What is the type of hypertrichosis B. Mesangial
inheritance? C. Smoothmuscular
D. Juxtavascular
A. Y-linked chromosome E. Interstitial
B. Autosome-recessive
C. Autosome-dominant 194. The energy inputs of a healthy man
D. X-linked recessive chromosome have been measured. In what position was
E. X-linked dominant chromosome the patient if his energy inputs were less
than the main exchange?
189. A 50-year-old patient with typhoid
fever was treated with Levomycetin, the A. Sleep
next day his condition became worse, B. Rest
temperature rose to 39, 60 . What caused C. Easy work
the complication? D. Nervous exertion
E. Calmness
A. The effect of endotoxin agent
B. Allergic reaction 195. A man after 1,5 litre blood loss has
C. Irresponsiveness of an agent to the suddenly reduced diuresis. The increased
levomycetin secretion of what hormone caused such
D. Secondary infection addition diuresis alteration?
E. Reinfection A. Vasopressin
190. A patient after pathological process B. Corticotropin
has a thickened alveolar membrane. The C. Natriuretic
direct consequence of the process will be D. Cortisol
the reduction of: E. Parathormone

A. Diffuse lung capacity 196. A 2-year-old child has got intesti-


B. Oxygen capacity of blood nal dysbacteriosis, which results in
C. Minute respiratory capacity hemorrhagic syndrome. What is the most
D. Alveolar lung ventilation likely cause of hemorrhage of the child?
E. Reserve expiratiory capacity A. Vitamin K insufficiency
191. Where should the cathetor for B. Activation of tissue thromboplastin
evacuation of the lymph from the thoracic C. PP hypovitaminosis
lymph duct be inserted? D. Fibrinogen deficiency
E. Hypocalcemia
A. To the left venous corner
B. To the right venous corner 197. To anaesthetize the manipulation
C. To the superior vena cava related to burn surface treatment, a pati-
D. To the inferior vena cava ent was intravenously injected a medi-
E. To the left inguinal vein cation for short-acting narcosis. 1 minute
later the patient being under anaesthesia
192. The penetration of the irritable cell had increased blood pressure, tachycardia,
membrane for potassium ions has been increased tone of sceletal muscles; reflexes
increased during an experiment. What remained. After awakening the patient
changes of membrane electric status can had desorientation and visual hallucinati-
occur? ons. What medication was the patient
injected?
Krok 1 Medicine 2005 22

A. Ketamine ltration. What is the most likely diagnosis?


B. Sombrevin
C. Diethyl ether A. Myocardial infarction
D. Thiopental sodium B. Myocardial ischemic dystrophy
E. Nitrous oxide C. Focal exudate myocarditis
D. Diffuse exudate myocarditis
198. During complicated labour the E. Productive myocarditis
symphysis pubis ruptured. What organ
can be damaged mostly? 200. A young man has an unpainfull
formation without marked borders in the
A. Urinary blader soft tissues of his thigh. On the tissue bi-
B. Rectum optate the formation lookes like flesh of
C. Ovaria fish and consists of immature fibroblast-
D. Uterine tubes like cells with multiple mitosis growing
E. Uterus through the muscles. What is the most li-
kely diagnosis?
199. A patient died from acute cardiac
insufficiency. The histological examinati- A. Fibrosarcoma
on of his heart revealed the necrotized B. Myosarcoma
section in myocardium of the left ventri- C. Fibroma
cle, which was separated from undamaged D. Cancer
tissue by the zone of hyperimic vessels, E. Myoma
small hemorrhages and leukocytic infi-
Krok 1 Medicine 2006 1

1. A patient with hypochromic anemia A. Galactose-1-phosphate uri-


has splitting hair and loss of hair, increased dyltransferase
nail brittling and taste alteration. What B. Amylo-1,6-glucosidase
is the mechanism of the development of C. Phosphoglucomutase
these symptoms? D. Galactokinase
E. Hexokinase
A. Deficiency of iron-containing enzymes
B. Deficiency of vitamin 12 6. Synthesis of phospholipids is disturbed
C. Decreased production of parathyrin as a result fatty infiltration of liver. Indi-
D. Deficiency of vitamin А cate which of the following substances can
E. Decreased production of thyroid enhance the process of methylation duri-
hormones ng phospholipids synthesis?
2. A 27-year-old woman has dropped A. Methionine
penicillin containing eye drops. In a few B. Ascorbic acid
minutes there appeared feeling of itchi- C. Glucose
ng, burning of the skin, lips and eyeli- D. Glycerin
ds edema, whistling cough, decrease of E. Citrate
BP. What imunoglobulins take part in the
development of this allergic reaction? 7. An individual is characterized by
rounded face, broad forehead, a mongoli-
A. IgE and IgG an type of eyelid fold, flattened nasal bri-
B. IgM and IgG dge, permanently open mouth, projecting
C. IgA and IgM lower lip, protruding tongue, short neck,
D. IgM and IgD flat hands, and stubby fingers. What di-
E. IgG and IgD agnosis can be put to the patient?
3. A person felt thirsty after staying under A. Down’s syndrome
the conditions of hot weather for a long B. Klinefelter’s syndrome
time. Signals of what receptors caused it C. Alkaptonuria
first of all? D. Supermales
E. Turner’s syndrome
A. Osmoreceptors of hypothalamus
B. Sodium receptors of hypothalamus 8. A patient visited a dentist with complai-
C. Osmoreceptors of liver nts of redness and edema of his mouth
D. Glucoreceptors of hypothalamus mucous membrane in a month after dental
E. Baroreceptors of aortic arch prosthesis. The patient was diagnosed wi-
th allergic stomatitis. What type of allergic
4. Slime, blood and protozoa 30-200 mi- reaction by Gell and Cumbs underlies this
crons long have been revealed in a man’s disease?
feces. The body is covered with cilias
and has correct oval form with a little A. Delayed type hypersensitivity
bit narrowed anterior and wide round B. Cytotoxic
shaped posterior end. At the anterior end C. Immunocomplex
a mouth is visible. In cytoplasm there are D. Anaphylactic
two nucleui and two short vacuoles. What E. Stimulating
are the described features typical for?
9. A patient suffering from trombophlebi-
A. Balantidium tis of the deep crural veins suddenly
B. Lamblia died. Autopsy has shown freely lying
C. Dysenteric amoeba red friable masses with dim crimped
D. Trichomonas surface in the trunk and bifurcation of
E. Intestinal amoeba the pulmonary artery. What pathologic
process was revealed by morbid anatomi-
5. A child has got galactosemia. st?
Concentration of glucose in blood has not
considerably changed. Deficiency of what A. Tromboembolism
enzyme caused this illness? B. Thrombosis
C. Tissue embolism
D. Embolism with foreign body
E. Fat embolism
10. Decreased blood supply to the organs
Krok 1 Medicine 2006 2

causes hypoxia that activates fibroblasts was admitted to the neurological in-
function. Volume of what elements is patient department. There was revealed
increased in this case? a correlation between increasing of
encephalopathy and substances absorbed
A. Intercellular substance by the bloodstream from the intestines.
B. Vessels of microcircular stream What substances that are formed in the
C. Nerve elements intestines can cause endotoxemia?
D. Parenchymatous elements of an organ
E. Lymphatic vessels A. Indole
B. Butyrate
11. RNA-polymerase B(II) is blocked due C. Acetacetate
to amanitine poisoning (poison of death- D. Biotin
cup). It disturbs: E. Ornithine
A. Synthesis of m-RNA 16. A 50-year-old man has felt vague
B. Synthesis of t-RNA abdominal discomfort within past 4
C. Reverse transcription months. Physical examination revealed
D. Primers synthesis no lymphadenopathy, and no abdomi-
E. Maturation of m-RNA nal masses or organomegaly at palpation.
Bowel sounds are heard. An abdominal
12. Secretion of what gastrointestinal CT scan shows a 20 cm retroperitoneal
hormones will be primarily decreased as a soft tissue mass obscuring the left psoas
result of iduodenum removal? muscle. A stool specimen tested for occult
blood is negative. Which of the following
A. Cholecystokinin and secretin neoplasms is this man most likely to have?
B. Gastrin
C. Histamine A. Lipoma
D. Gastrin and histamine B. Melanoma
E. Neurotensin C. Hamartoma
D. Adenocarcinoma
13. It is planned to use the territory of E. Lymphoma
an old cattle burial ground (which is
not used for more than 50 years) for 17. A patient has undergone an amputati-
building houses. But ground analysis on of lower extremity. Some time later
revealed presence of the pathogen of a painful nodules appeared in a stump.
very dangerous illness. Which of the indi- Amputatious neuromas were found out
cated microorgonisms is likely to remain at the microscopic examination. To what
in the ground for such a long time? pathological processes do those formati-
ons relate?
A. Bacillus anthracis
B. Francisella tularensis A. Regeneration
C. Brucella abortus B. Dystrophy
D. Yersinia pestis C. Inflammation
E. Mycobacterium bovis D. Hyperemia
E. Metaplasia
14. From the nasopharynx of a 5-
year-old child it was excreted a mi- 18. During the fetal period of the
croorganism which is identical to development in the vascular system of
Corynebacterium diphtheriae dose the fetus a large arterial (Botallo’s)
according to morphological and bi- duct is functioning which converts into
ochemical signs. But this microorganism lig.arteriosum after birth. What anatomi-
does not produce exotoxin. As a result cal formations does this duct connect?
of what process can this microorganism
become toxigenic? A. Pulmonary trunk and aorta
B. Right and left auricles
A. Phage conversion C. Aorta and inferior vena cava
B. Cultivation in the telluric environment D. Pulmonary trunk and superior vena
C. Passing through the organism of the cava
sensative animals E. Aorta and superior vena cava
D. Growing with antitoxic serum
E. Chromosome mutation 19. A 22-year-old patient was admitted
to the hospital with complaints of heavy
15. A patient with encephalopathy nasal breathing. During the examination
Krok 1 Medicine 2006 3

of her nasal cavity the doctors found thi- A. .latissimus dorsi


ckened mucous membrane, a lot of mucus B. .levator scapulae
and nodular infiltrates without erosions C. .romboideus major
in the nose.The nasal rhinoscleroma was D. .trapezius
diagnosed. The biopsy was taken. What E. .subscapularis
typical morphological changes may be
found? 24. A 52-year-old patient has the followi-
ng diagnosis: systemic amebiasis with
A. Granulomas with Mikulicz’s cells involvment of intestines, liver, lungs. What
B. Granulomas with Virchow’s cells drug should be prescribed?
C. Granulomas with Langhan’s cells
D. Granulomas with foreign body cells A. Metronidasol
E. Interstitial inflammation B. Quiniofone
C. Tetracycline
20. Intake of oral contraceptives containi- D. Quingamine
ng sex hormones inhibits secretion of the E. Enteroseptol
hypophysiae hormones. Secretion of whi-
ch of the indicated hormones is inhibited 25. A 38-year-old man who poisoned hi-
while taking oral contraceptives with sex mself with mercury dichloride was taken
hormones? to the admission room in grave conditi-
on. What antidote should be immediately
A. Follicle-stimulating introduced?
B. Vasopressin
C. Thyrotropic A. Unithiol
D. Somatotropic B. Dipiroxim
E. Oxytocin C. Atropine
D. Nalorphine
21. A 60-year-old patient was admi- E. Isonitrosine
tted to the surgical department because
of infection caused by blue pus baci- 26. Children often have heavy nasal
llus (Pseudomonas aeruginosa) which is breathing resulting from excessive
sensative to penicillin antibiotics. Indicate development of lymphoid tissue of
which of the given penicillins has marked pharyngeal mucous membrane. What
activity to the Pseudomonas aeruginosa? tonsils growth may cause this effect?

A. Carbenicillin disodium A. T onsilla pharyngea


B. Benzylpenicillin B. T onsilla palatina
C. Phenoxymethylpenicillin C. T onsilla lingualis
D. Oxacillin D. T onsilla tubaria
E. Methicillin E. All above mentioned tonsils

22. A 45-year-old woman suffers from 27. The alternate usage of dichlotiazide,
seasonal allergic rhinitis caused by the etacrin acid and lasex did not influence
ambrosia blossoming. What medicine diuretically upon the patient with marked
from the stabilizer of the adipose cells peripheral edemata. The aldosterone rate
group can be used for prevention of this in the blood is increased. Indicate which
disease? medicine should be prescribed:

A. Ketotifen A. Spironolacton
B. Diazoline B. Mannit
C. Phencarol C. Clopamid
D. Tavegyl D. Urea
E. Dimedrol E. Amilorid

23. A young man felt acute pain in the 28. A 46-year-old man complains of di-
back during active drawing up on the hori- fficulties with nasal breathing. Mikulicz’s
zontal bar. Objectively: pain while movi- cells, accumulation of epithelioid cells,
ng upper extremity, reduced pronation plasmocytes, lymphocytes, hyaline balls
and adduction functions. Sprain of what were discovered in the biopsy material of
muscle can be observed here? the thickened nasal mucosa. What is the
most likely diagnosis?
Krok 1 Medicine 2006 4

A. Scleroma A. Reduction of haematocrite rate


B. Virus rhinitis B. Increase of haematocrite rate
C. Allergic rhinitis C. Leukopenia
D. Rhinovirus infection D. Leucocytosis
E. Meningococcal nasopharyngitis E. Polycythemia
29. A 56-year-old patient complaining 33. A 60-year-old patient has reduced
of thirst and frequent urination was di- perception of high-frequency sounds.
agnosed with diabete mellitus. Butamin What structures’ disorder of auditory
was prescribed. How does the medicine analizer caused these changes?
act?
A. Main membrane of cochlea near the
A. It stimulates β-cells of Langergans’ oval window
islets B. Main membrane of cochlea near heli-
B. It helps to absorb the glucose by the cotrema
cells of the organism tissues C. Eustachian tube
C. It relieves transport of glucose through D. Muscles of middle ear
the cells’ membranes E. Tympanic membrane
D. It inhibits α-cells of Langergans’ islets
E. It inhibits absorption of glucose in the 34. The effect of electric current on the
intestines exitable cell caused depolarization of
its membrane. Movement of what ions
30. A man who was bitten by the unknown through the membrane caused depolari-
dog applied to the surgeon. Wide ragged sation?
woundes were localized on the face. What
curative-prophylactic aid should be given A. Na+
to prevent rabies? B. −
3
C. 2+
A. Start immunisation with rabies vaccine D. l−
B. Prescribe combined antibiotic therapy E. +
C. Immediate injection of
DPT(Diphtheria, Pertusis, Tetanus) 35. Live vaccine was injected into the
vaccine human body. Increasing activity of what
D. Hospitalize the patient and keep under connective tissue cells can be expected?
the doctor’s supervision
E. Immediately inject normal gamma A. Plasmocytes and lymphocytes
globulin B. Macrophages and fibroblasts
C. Pigmentocytes and pericytes
31. A patient with clinical signs of D. Adipocytes and adventitious cells
immunodeficiency has no changes of E. Fibroblasts and labrocytes
the number and functional activity of
T- and B- lymphocytes. Defect with di- 36. When a patient with traumatic impai-
sfunction of antigen-presentation to the rment of the brain was examined, it was
immunocompetent cells was found during discovered that he had stopped to di-
examinatio on the molecule level. Defect stinguish displacement of an object on
of what cells is the most probable? the skin. What part of the brain was
damaged?
A. Macrophages, monocytes
B. Т-lymphocytes, В-lymphocytes A. Posterior central gurus
C. NK-cells B. Occipital zone of the cortex
D. Fibroblasts, Т-lymphocytes, В- C. Parietal zone of the cortex
lymphocytes D. Frontal central gurus
E. 0-lymphocytes E. Frontal zone

32. A 38-year-old woman was admitted to 37. In the blood of a 26-year-old man
the admission-diagnostic department with 18% of erythrocytes of the spherical, ball-
uterine bleeding. What are the most likely shaped, flat and spinous shape have been
changes of blood? revealed. Other eritrocytes were in form
of the concavo-concave disks. How is this
phenomenon called?
Krok 1 Medicine 2006 5

A. Physiological poikilocytosis A. Ketone bodies


B. Pathological poikilocytosis B. Amino acids
C. Physiological anisocytosis C. Monosaccharides
D. Pathological anisocytosis D. High fatty acids
E. Erytrocytosis E. Cholesterol esters
38. A 10-year-old girl often experiences 43. A person has steady HR not exceedi-
acute respiratory infections with multi- ng 40 bpm. What is the pacemaker of this
ple punctate haemorrages in the places of person’s heart rhythm?
clothes friction. Hypovitaminosis of what
vitamin has the girl? A. Atrioventricular node
B. Sinoatrial node
A. C. His’ bundle
B. 6 D. Branches of His’ bundle
C. 1 E. Purkinje’s fibers
D.
E. 2
44. A doctor administered Allopurinol
to a 26-year-old young man with the
39. Different functional groups can be symptoms of gout. What pharmacological
presented in the structure of L-amino aci- action of Allopurinol ensures therapeuti-
d’s radicals. Identify the group that is able cal effect?
to form ester bond:
A. By inhibiting uric acid synthesis
A. −OH B. By increasing uric acid excretion
B. −SH C. By inhibiting leucocyte migration into
C. −CONH2 the joint
D. −CH3 D. By general anti-inflammatory effect
E. −NH2 E. By general analgetic effect
40. Moving of the daughter chromatids to 45. A patient who came to the doctor
the poles of the cell is observed in the mi- because of his infertility was admini-
totically dividing cell. At what stage of the stered to make tests for toxoplasmosis
mitotic cycle is this cell? and chronic gonorrhoea. Which reacti-
on should be performed to reveal latent
A. Anaphase toxoplasmosis and chronic gonorrhoea of
B. Metaphase the patient?
C. Telophase
D. Prophase A. (R)CFT- Reiter’s complement fixation
E. Interphase test
B. IFA - Immunofluorescence assay
41. During examination of a patient, C. Immunoblot analysis
there was found a neoplasm in the whi- D. RDHA - Reverse direct hemaggluti-
te substance of cerebral hemispheres with nation assay
localization in the knee and frontal part E. RIHA - Reverse indirect hemaggluti-
of posterior crus of internal capsule. Fi- nation assay
bres of what conductive tract of the brain
will be disrupted? 46. Under some diseases it is
observed aldosteronism accompanied by
A. T r. pyramidalis hypertension and edema due to sodium
B. T r. f rontothalamicus retention in the organism. What organ of
C. r. thalamocorticalis the internal secretion is affected under
D. T r. f rontopontinus aldosteronism?
E. T r. parietooccipitopontinus
A. Adrenal glands
42. A patient with diabetes mellitus has B. Testicle
been delivered in hospital in the state of C. Ovaries
unconsciousness. Arterial pressure is low. D. Pancreas
The patient has acidosis. Point substances, E. Hypophysis
which accumulation in the blood results in
these manifestations: 47. A patient had been taking glucocorti-
coids for a long time. When the
preparation was withdrawn he developed
the symptoms of disease aggravation,
Krok 1 Medicine 2006 6

decreased blood pressure and weakness.


What is the reason of this condition? A. Lice
B. Spiders
A. Appearance of adrenal insufficiency C. Bed-bugs
B. Hyperproduction of ACTH D. Cockroaches
C. Sensibilization E. Houseflies
D. Habituation
E. Cumulation 53. A businessman came to India from
South America. On examination the
48. A patient has tissue ischemia below physician found that the patient was
the knee joint accompanied with intermi- suffering from sleeping-sickness. What
ttent claudication. What artery occlusion was the way of invasion?
should be suspected?
A. As a result of bug’s bites
A. Popliteal artery B. As a result of mosquito’s bites
B. Peroneal artery C. With contaminated fruits and vegetables
C. Posterior tibial artery D. Through dirty hands
D. Anterior tibial artery E. After contact with sick dogs
E. Proximal part of femoral artery
54. Patients with bile ducts obstruction
49. An experiment proved that UV- suffer from inhibition of blood coagulati-
radiated cells of patients with xeroderma on, bleedings as a result of low level of
pigmentosum restore the native DNA vitamin assimilation. What vitamin is in
structure slower than cells of healthy deficiency?
individuals as a result of reparation
enzyme defection. What enzyme helps A. К
this process? B. А
C. D
A. Endonuclease D. Е
B. RNA ligase E. Carotene
C. Primase
D. DNA polymerase III 55. Blood analysis of a patient showed si-
E. DNA gyirase gns of HIV infection (human immunodefi-
ciency virus). Which cells does HIV-virus
50. A patient, who suffers from congeni- primarily affect?
tal erythropoietic porphyria, has skin
photosensitivity. The accumulation of A. Cells that contain receptor T4 (T-
what compound in the skin cells can cause helpers)
it? B. Cells that contain receptor IgM (B-
lymphocytes)
A. Uroporphyrinogen 1
B. Protoporphyrin C. Specialized nervous cells (neurons)
C. Uroporphyrinogen 2 D. Mast cells
D. Coproporphyrinogen 3 E. Proliferating cells (stem hematoplastic
E. Heme cells)

51. A sick man with high temperature and 56. The preventive radioprotector was
a lot of tiny wounds on the body has been given to a worker of a nuclear power
admitted to the hospital. Lice have been station. What mechanism from the below
found in the folds of his clothing. What mentioned is considered to be the main
disease can be suspected? mechanism of radioprotection?

A. Epidemic typhus A. Inhibition of free radicals formation


B. Tularemia B. Prevention of tissue’s hypoxia
C. Scabies C. Activation of oxidation reactions
D. Malaria D. Increasing of tissue blood supply
E. Plague E. Increasing of respiration

52. A patient with suspicion on epidemic 57. The pulmonalis embolism has
typhus was admitted to the hospital. Some suddenly developed in a 40-year-old pati-
arachnids and insects have been found in ent with opened fracture of the hip.
his flat. Which of them may be a carrier of Choose the possible kind of embolism:
the pathogen of epidemic typhus?
Krok 1 Medicine 2006 7

A. Fat out-patient department with the complai-


B. Thrombus-embolus nts of bleeding trauma in the vestibule of
C. Air his nose. On examination: the mechanical
D. Tissue injury of the mucous layer of the vesti-
E. Foreign body bule without continuation into nasal cavi-
ty proper. What is the boundary between
58. A patient’s blood was analyzed and the vestibule and nasal cavity proper?
the decreased erythrocyte’s sedimentation
rate (ESR) was discovered. What disease A. Nasal limen
from the listed below is accompanied with B. Nasal roller
decreased ESR? C. Nasal septa
D. Choanes
A. Polycytemia E. Nostrils
B. Hepatitis
C. Splenomegaly 63. A 50-year-old patient was injured
D. Vitamin B deficiency on the occipital region of the head. The
E. Myocardial infarction closed skull trauma was diagnosed. She
was taken to the hospital. The medical
59. A 46-year-old patient has complai- examination: deregulation of walking and
ned of headache, fatigue, thirst, pains balance, trembling of arms. What part of
in the spine and joints for the last 2 brain was injured?
years. Clinically observed disproportional
enlargement of hands, feet, nose, superci- A. The cerebellum
liary arches. He notes that he needed B. The medulla oblongata
to buy bigger shoes three times. What C. The mind-brain
is the main reason of such disproporti- D. The inter-brain
onal enlargement of different parts of the E. The spinal cord
body?
64. A patient with complaints of dryness
A. Cartilaginous tissue proliferation under in the mouth, photophobia and visi-
growth hormone influence on impairment was admitted to the
B. Increased sensitivity of the tissues to reception-room. Skin is hyperemic, dry,
growth hormone pupils are dilated, tachycardia. Poisoning
C. Joints dystrophy development with belladonna alkaloids was diagnosed
D. Increased sensitivity of the tissues to on further examination. What medicine
insulin should be prescribed?
E. Joints chronic inflammation
development A. Prozerin
B. Diazepam
60. A mother of a newborn complai- C. Pilocarpine
ns of her baby’s constant belching with D. Armine
undigested milk. Which developmental E. Dipiroxim
anomaly is it an evidence of?
65. Autopsy revealed that right lung is
A. Esophageal atresia enlarged, solid, there are fibrin layers on
B. Labium leporium the pleura. Lung tissue is light grey color
C. Faux lupinum on incision with muddy liqued exudates.
D. Anal atresia What lung disease are these symptoms
E. Esophageal fistula typical for?
61. During the endoscopy the inflammati- A. Croupous pneumonia
on of a major papilla of the duodenum B. Bronchopneumonia
and the disturbances of bile secretion C. Interstitial pneumonia
were found. In which part of duodenum D. Pulmonary gangrene
were the problems found? E. Fibrosing alveolitis
A. Descendent part 66. During the operation on the hip joi-
B. Ascendant part nt of a 5-year-old child her ligament was
C. Bulb damaged which caused bleeding.What li-
D. Upper horizontal part gament was damaged?
E. Lower horizontal part
62. An 18-year-old patient came to the
Krok 1 Medicine 2006 8

A. The head of the thigh A. Precipitation test


B. Perpendicular of the acetabule B. Indirect hemagglutination test
C. Iliofemoral C. Agglutination test
D. Pubofemoral D. Immunofluorescence test
E. Ischiofemoral E. Complement-fixation test
67. On autopsy it was revealed: pia mater 71. A patient with rheumatoid arthritis
of the upper parts of cerebral hemisphere who had been treated with indometacin
is plethoric, of yellowish-green color, has got signs of gastropathy. What acti-
soaked with purulent and fibrose exudate, vity of the drug can this complication be
looks like a cap. What disease is it typical connected with?
for?
A. Anticyclooxygenase
A. Meningococcal meningitis B. Antiserotonin
B. Tuberculous meningitis C. Antihistamine
C. Grippal meningitis D. Antikinine
D. Meningitis connected with anthrax E. Locally irritating
E. Meningitis connected with typhus
72. A 25-year-old woman with red and
68. A 55-year-old patient with continuing itchy eczematoid dermatitis visits your
ventricular arrhythmias was admitted to office. She had a dental procedure one
the hospital. The patient is taking timolol day earlier with administration of a local
drops for glaucoma, daily insulin injecti- anesthetic. There were no other findings,
ons for diabetes mellitus, and an ACE although she indicated that she had a hi-
inhibitor for hypertension. You have deci- story of allergic reactions. Which of the
ded to use phenytoin instead of procai- following drugs is most likely involved?
namide. What is the reason?
A. Procaine
A. The anticholinergic effect of procai- B. Cocaine
namide would aggravate glaucoma C. Lidocaine
B. The local anesthetic effect of procai- D. Bupivacaine
namide would potentiate diabetes E. Etidocaine
C. The hypertensive effects of procainami-
de would aggravate the hypertension 73. In course of metabolic process active
D. The local anesthetic effect of procai- forms of oxygen including superoxide ani-
namide would aggravate the hypertension on radical are formed in the human body.
E. The cholinergic effects of procainamide By means of what enzyme is this anion
would aggravate the diabetes inactivated?

69. A 35-year-old man under the A. Superoxide dismutase


treatment for pulmonary tuberculosis has B. Catalase
acute pain onset of right big toe, swelling, C. Peroxidase
and low-grade fever. The gouty arthritis D. Glutathioneperoxidase
was diagnosed and high serum uric acid E. Glutathionereductase
level was found. Which of the followi-
ng antituberculous drugs are known for 74. A patient has elbow joint trauma
causing high uric acid levels? with avulsion of medial epicondyle of
humerus. What nerve can be damaged in
A. Pyrazinamide this trauma?
B. Cycloserine
C. Thiacetazone A. Ulnar
D. Rifampicin B. Radial
E. Aminosalicylic acid C. Musculocutaneous nerve
D. Cardiac cutaneous nerve
70. A person was selling "homemade E. Medial cutaneous nerve of forearm
pork"sausages at the market. State sani-
tary inspector suspected falcification of 75. A student is writing a thorough
the sausages. What serological immune summary of a lecture. Quality of summari-
reaction can identifiy food substance? zing has considerably worsened when his
neighbours started talking. What kind of
inhibition in the cerebral cortex caused
this effect?
Krok 1 Medicine 2006 9

A. External cient. It was determined that it was equal


B. Protective 1. What have the patient been keeping to?
C. Fading
D. Differentiated A. With domination of carbohydrates
E. Delayed B. With domination of proteins and fat
C. With domination of fat and
76. While enrolling a child to school carbohydrates
Mantu’s test was made to define whether D. Mixed
revaccination was needed. The test result E. With domination of proteins and
is negative. What does this test result carbohydrates
mean?
81. Buffer capacity of a worker’s blood
A. Absence of cell immunity to the was decreased due to exhausting muscular
tuberculosis work. By coming of what acid substance
B. Presence of cell immunity to the in the blood can this state be explained?
tuberculosis
C. Absence of antibodies for tubercle A. Lactate
bacillus B. Pyruvate
D. Absence of antitoxic immunity to the C. 1,3-bisphosphoglycerate
tuberculosis D. α-ketoglutarate
E. Presence of antibodies for tubercle E. 3-phosphoglycerate
bacillus
82. Examination of initial molecular
77. The donor who didn’t donate the blood structure revealed substitution of the
for a long time was investigated with glutamic acid by valine. What inherited
IFA method. Anti-HBs antibodies were pathology is it typical for?
revealed. What does positive result of IFA
in this case mean? A. Sickle-cell anemia
B. Thalassemia
A. Previous hepatitis B C. Minkowsky-Shauffard disease
B. Acute hepatitis B D. Favism
C. Acute hepatitis C E. Hemoglobinosis
D. Chronic hepatitis В
E. Chronic hepatitis С 83. Inflamation is characterised by
increased permeability of vessels of mi-
78. Testosterone and it’s analogs increase crocirculation stream, increase of their
the mass of skeletal muscles that allows to hydrodynamic blood pressure. Increasing
use them for treatment of dystrophy. Due of the osmotic concentration and dispersi-
to interaction of the hormone with what ty of protein structures is present in the
cell substrate is this action caused? intercellular fluid. What kind of edema
will appear in this case?
A. Nuclear receptors
B. Membrane receptors A. Mixed
C. Ribosomes B. Hydrodynamic
D. Chromatin C. Colloid-osmotic
E. Proteins-activators of transcription D. Lymphogenic
E. Membranogenic
79. Inhibition of α-motoneuron of the
extensor muscles was noticed after sti- 84. Increased fragility of vessels, enamel
mulation of α-motoneuron of the flexor and dentine destruction resulting from
muscles during the experiment on the spi- scurvy are caused by disorder of collagen
nal cord. What type of inhibition is this maturation. What stage of procollagen
process based upon? modification is disturbed under this avi-
taminosis?
A. Reciprocal
B. Presynaptic A. Hydroxylation of proline
C. Depolarizational B. Formation of polypeptide chains
D. Recurrent C. Glycosylation of hydroxylysine residues
E. Lateral D. Removal of C-ended peptide from
procollagen
80. A patient who has been strictly keepi- E. Detaching of N-ended peptide
ng to a certain diet for 10 days went
through examination of respiratory coeffi- 85. Scraps of the mycelium of a fungus,
Krok 1 Medicine 2006 10

spores, air bubbles and fat drops were di- 90. A person has reduced di-
scovered on microscopy of the patient’s uresis, hypernatremia, hypokalemia.
hair excluded from the infected areas. Hypersecretion of what hormone can
What fungus disease is characterised by cause such changes?
this microscopic picture?
A. Aldosterone
A. Favus B. Vasopressin
B. Microspory C. Auricular sodiumuretic factor
C. Trichophytosis D. Adrenalin
D. Epidermophytosis E. Parathormone
E. Sporotrichosis
91. Microspecimen of red bone marrow
86. On autopsy of the 58-year-old man it contains multiple capillares through
was revealed: mitral valve is deformed, the walls of which mature blood cells
thickened, not totally closed. Microscopi- penetrate into the bloodstream. What
cally: foci of collagen fibers are eosinophi- type of capillares are these?
lic, have positive fibrin reaction. The most
likely it is: A. Sinusoidal
B. Fenestrational
A. Fibrinoid swelling C. Somatical
B. Fibrinoid inflammation D. Visceral
C. Mucoid swelling E. Lymphatic
D. Hyalinosis
E. Amyloidosis 92. A 57-year-old patient was admitted
to the gastroenterological department wi-
87. Hypertrychosis of auricles is caused by th suspicion of Zollinger-Ellison syndrom
a gene that is localized in Y-chromosome. because of rapid increase of gastrin level
Father has this feature. What is the in the blood serum. What is the most
probability to give birth to a boy with such probable disorder of the secretory functi-
anomaly? on of stomach?
A. 100% A. Hyperacidic hypersecretion
B. 0% B. Hyperacidic hyposecretion
C. 25% C. Achylia
D. 35% D. Hypoacidic hyposecretion
E. 75% E. Hypoacidic hypersecretion
88. A 54-year-old man was admitted to the 93. For a long time a 49-year-old woman
hospital with complaints of pain in the ri- had suffered from glomerulonephritis
ght subcostal region, vomiting with blood. which caused death. The autopsy revealed
Objectively: enlarged liver, varicose veins that the size of her kidneys was 7 х
in the stomach and esophagus. Disfuncti- 3 х 2,5 cm, weight 65,0 g, they were
on of what vessel is likely to have taken dense and small-grained. Microscopically:
place? fibrinogenous inflammation of serous
and mucous membranes, dystrophic
A. V ena porta changes of parenchymatous organs, brain
B. Aorta abdominalis edema. What complication can cause such
C. V ena hepatica changes of serous membranes and inner
D. V ena cava superior organs?
E. V ena cava inf erior
A. Uraemia
89. A 37-year-old man was admitted B. Anemia
to the surgical department with the C. Sepsis
symptoms of acute pancreatitis: vomiti- D. DIC-syndrome
ng, diarrhea, bradycardia, hypotention, E. Thrombopenia
weakness, dehydration of the organism.
What medicine should be used first of all? 94. Upper neck node of sympathetic
trunk was removed from the rabbit on
A. Contrycal experiment. Reddening and increased
B. No-spa temperature of the skin of head is
C. Platyphylline observed. What disorder of peripheral ci-
D. Etaperazine rculation of the blood has developed?
E. Ephedrine
Krok 1 Medicine 2006 11

A. Neuroparalytic arterial hyperemia A. Neutrophils


B. Neurotonic arterial hyperemia B. Eosinophils
C. Metabolic arterial hyperemia C. Basophils
D. Venous hyperemia D. Limphocytes
E. Stasis E. Monocytes
95. A tissue sample of benign tumor was 100. ATP synthesis is totaly blocked in a
studied under the electron microscope. cell. How will the value of membrane rest
A lot of small (15-20 nm) spherical bodi- potential change?
es, consisting of 2 unequal subunits were
detected. These are: A. It will disappear
B. It will be slightly increased
A. Ribosomes C. It will be considerably increased
B. Golgi complex D. First it will increase, then decrease
C. Smooth endoplasmic reticulum E. First it will decrease, then increase
D. Microtubules
E. Mitochondria 101. A patient who was previously ill with
mastectomy as a result of breast cancer
96. A woman who was infected with was prescribed radiation therapy. What
toxoplasmosis during the pregnancy has vitamin preparation has marked radi-
born a child with multiple congenital oprotective action caused by antioxidant
defects.This is a result of: activity?
A. Teratogenesis A. Tocopherol acetate
B. Cancerogenesis B. Ergocalciferol
C. Biological mutogenesis C. Thiamine chloride
D. Chemical mutogenesis D. Riboflavin
E. Recombination E. Folic acid
97. A considerable increase of activity of 102. A patient with tissue trauma was
МВ-forms of CPK (creatinephosphoki- taken a blood sample for the determinati-
nase) and LDH-1 was revealed on the on of blood clotting parameters. Specify
examination of patient’s blood. What is the right sequence of extrinsic pathway
the most likely pathology? activation:

A. Miocardial infarction A. III - VIIa - Xa


B. Hepatitis B. III - IV - Xa
C. Rheumatism C. IV - VIII: TF - Xa
D. Pancreatitis D. IV - VIIa - Xa
E. Cholecystitis E. III - VIII: TF - Xa

98. An isolated muscle of a frog 103. Autopsy of a man who died from
is rhythmically irritated with electric chronic cardiacvascular collapse revealed
impulses. Every next impulse is in a period "tiger heart". Sidewards of endocardium
of relaxation from the previus contraction. a yellowish-white banding can be seen;
What contraction of the muscle occurs? myocardium is dull, dark-yellow. What
process caused this pathology?
A. Waved tetanus
B. Single A. Fatty parenchymatous degeneration
C. Asynchronous B. Carbohydrate degeneration
D. Continuous (smooth) tetanus C. Hyaline degeneration
E. Tonic D. Fatty vascular-stromal degeneration
E. Amyloidosis
99. A 16-year-old boy was performed an
appendectomy. He has been hospitalized 104. Parents with an ill child consulted
for right lower quadrant abdominal pain an infectionist. They had been working in
within 18 hours. The surgical specimen is one of Asian countries for a long time.
edematous and erythematous. Infiltration The child has sallow skin, loss of appeti-
by what of the following cells is the most te, laxity, enlarged liver, spleen, peripheral
typical for the process occuring here? lymph nodes. What protozoal illness can
be suspected?
Krok 1 Medicine 2006 12

A. Visceral leishmaniasis A. Lymphogranulomatosis


B. Balantidiasis B. Acute lympholeucosis
C. Amebiasis C. Chronic lympholeucosis
D. Toxoplasmosis D. Berkitt’s lymphoma
E. Lambliasis E. Fungoid mycosis
105. Healthy parents have got a fair- 109. As a result of the damage of one of
haired, blue-eyed girl. Irritability, anxiety, the Atomic Power Plant reactors the run-
sleep and feeding disturbance developed out of radioelements took place. People
in the first months of the infant’s li- in the high-radiation area were radiated
fe. Neurological examination revealed with approximately 250-300 r. They were
developmental lag. What method of immediately hospitalized. What changes
genetic investigation should be used for in the blood count would be typical for
the exact diagnosis? the victims?
A. Biochemical A. Lymphopenia
B. Cytological B. Leukopenia
C. Gemellary C. Anemia
D. Genealogical D. Thrombopenia
E. Population-statistical E. Neutropenia
106. The process of heart transplantati- 110. Dystrophic alterations of heart are
on determined the viability of myocardi- accompanied with dilation of heart caviti-
al cells. The determination of what es, decreased force of heart contractions,
myocardium parameter is the most increased blood volume that remains duri-
important? ng systole in the heart cavity, vein overfill.
What heart condition is it typiccal for?
A. Rest potential of cardiomyocytes
B. Heart temperature A. Myogenic dilatation
C. Concentration of oxygen in heart vessels B. Tonogenic dilatation
D. Concentration of calcium-ions in myofi- C. Emergency stage of hyperfunction and
brils hypertrophy
E. Concentration of Ca-ions in heart D. Cardiosclerosis
vessels E. Cardiac tamponade
107. The calcium canals of cardiomyocytes 111. Nowadays about 50 minor bases
have been blocked on an isolated rabbit’s have been found in the t-RNA structure
heart. What changes in the heart’s activity besides the main four nitrogenous bases.
can result from it? Choose the minor nitrogenous base:
A. Decreased rate and force of heart beat A. Dihydrouracil
B. Decreased heart beat rate B. Uracil
C. Decreased force of the contraction C. Cysteine
D. Heart stops in systole D. Adenine
E. Heart stops in diastole E. Cytosine
108. Microscopic examination of the 112. A 60-year-old patient fell ill with
enlarged neck gland of a 14-year-old obturative jaundice as a result of mali-
girl revealed destruction of the ti- gnant tumour of the big papillary of the
ssue structure of the node, absence of duodenal. Lumen of what anatomical
the lymph follicles, sclerotic areas and structure is compressed by tumour?
necrosis foci, cell constitution of the node
is polymorphous, lymphocites, eosinophi- A. Hepatopancreatic ampulla
les, big atypical cells with multilobular B. Cystic duct
nuclei (Beresovsky-Sternberg cells) and C. Common hepatic duct
mononuclear cells of the large size are D. Right hepatic duct
present. What is the most likely diagnosis? E. Left hepatic duct
113. Thyrotoxicosis leads to increased
production of thyroidal hormones T3 and
T4 , weight loss, tachycardia, psychic exci-
tement and so on. How do thyroidal
hormones effect energy metabolism in the
Krok 1 Medicine 2006 13

mitochondrion of cells? 118. A patient has got a spasm of smooth


muscles of bronchi. Activators of what
A. Disconnect oxidation and oxidative membrane cytoreceptors are phisiologi-
phosphorylation cally reasoned to stop an attack?
B. Activate substrate phosphorylation
C. Stop substrate phosphorylation A. β-adrenoreceptors
D. Stop respiratory chain B. α-аdrenoreceptors
E. Activate oxidative phosphorylation C. α- and β-аdrenoreceptors
D. Н-cholinoreceptors
114. While shifting the gaze to the closely E. М-cholinoreceptors
situated object the refracting power of
eye’s optical mediums will increase by 10 119. Intrapleural pressure of an individual
diopters. It results from changing of such is being measured. In what phase did he
eye structure: hold his breath if the pressure is - 25 cm
H2 O?
A. Lens
B. Cornea A. Forced inspiration
C. Vitreous body B. Quiet expiration
D. Liquid of the anterior chamber of eye C. Quiet inspiration
E. Muscle that dilatates pupil D. Forced expiration
E. -
115. Necrosis focus appeared in the area
of hyperemia and skin edema in few hours 120. Blood sampling for the haematology
after burn. What mechanism strengthens is recommended to carry out on an
destructive effect in the inflammation empty stomach and in the morning. What
area? changes in blood formula are possible if
blood sampling was carried out after food
A. Secondary alteration intake?
B. Primary alteration
C. Emigration of lymphocytes A. Increase of leukocyte number
D. Diapedesis of erythrocytes B. Increase of erythrocyte number
E. Proliferation of fibroblasts C. Increase of plasm proteins
D. Decrease of thrombocyte number
116. Analeptical remedy of reflective type E. Decrease of erythrocyte number
from the H-cholinomimetics group was
given to the patient for restoration of 121. Glomerular filtration rate (GFR)
breathing after poisoning with carbon increased by 20% as a result of prolonged
monoxide. What medicine was prescribed starvation of an individual. The most evi-
to the patient? dent cause of filtration changes under this
conditions is:
A. Lobeline hydrochloride
B. Atropine sulphate A. Decrease of oncotic pressure of blood
C. Adrenalin hydrochloride plasma
D. Mesaton B. Increase of systemic blood pressure
E. Pentamin C. Increase of penetration of the renal
filter
117. A patient operated on complicated D. Increase of filtration coefficient
appendicitis has the following changes of E. Increase of renal plasma stream
blood count: erythrocytes - 4, 0 · 1012 /l,
Нb - 120 g/l, color index - 0,9, leukocytes 122. The ovary specimen colored with
- 18 · 109 /l, basophils - 0, eosinophi- hematoxylin-eosin contains a follicle, in
ls - 0, myelocytes - 0, juvenile - 0, which cubic-shaped follicle epithelium
stab neutrophils - 20, segmentonuclear cells are placed in 1-2 layers, and scarlet
neutrophils - 53, lymphocytes - 21, membrane is seen around the ovocyte.
monocytes - 5. How is such nuclear shi- Name this follicle:
ft of leukocytic formula called?
A. Primary
A. Degenerative left shift B. Primordial
B. Right shift C. Secondary
C. Regenerative left shift D. Mature
D. Hyperregenerative E. Atretic
E. Regeneratively-degenerative
123. A 42-year-old man suffering from
Krok 1 Medicine 2006 14

gout has increased level of urinary acid with acute attack of chronic tonsillitis.
in blood. Allopurinol was prescribed What action of glucocorticoids caused this
to decrease the level of urinary acid. complication?
Competitive inhibitor of what enzyme is
allopurinol? A. Immunosuppressive
B. Anti-inflammatory
A. Xanthine oxidase C. Antishock
B. Adenosine deaminase D. Antiallergic
C. Adenine phosphoribosiltransferase E. Antitoxic
D. Hypoxanthine phosphoribosi-
ltransferase 129. Autopsy of a 12-year-old gi-
E. Guanine deaminase rl revealed: multiple cutaneous
hemmorhages (mostly into the skin of
124. A child has inhaled a button. Where buttocks, lower extremities), serous and
is it likely to be? mucous memrane hemmorhages, cerebral
hemmorhages. Adrenal glands show focal
A. In the right main bronchus necrosis and massive hemmorhages; ki-
B. In the left main bronchus dneys show necrotic nephrosis, suppurati-
C. In the trachea ve arthritis, iridocyclitis, vasculitis. What
D. In the larynx is the most probable diagnosis?
E. In the esophagus
A. Meningococcemia
125. A patient with serious damage of B. Epidemic typhus
muscular tissue was admitted to the C. Periarteritis nodosa
traumatological department. What bi- D. Systemic lupus erythematosus
ochemical urine index will be increased E. Radiation sickness
in this case?
130. A 68-year-old woman can’t move her
A. Creatinine upper and lower right extremities due to
B. Common lipids the stroke. Muscle tone of these extremiti-
C. Glucose es and their reflexes are increased. There
D. Mineral salts are pathological reflexes. What form of
E. Uric acid paralysis is it?
126. Donor skin transplantation was A. Hemiplegia
performed to a patient with extensive B. Paraplegia
burns. On the 8-th day the graft became C. Tetraplegia
swollen and changed colour; on the 11-th D. Monoplegia
day graft rejection started. What cells take E. Dissociation
part in this process?
131. Most participants of Magellan
A. T-lymphocytes expedition to America died from avi-
B. Erythrocytes tominosis. This disease declared itself
C. Basophils by general weakness, subcutaneous
D. Eosinophils hemmorhages, falling of teeth, gingival
E. B-lymphocytes hemmorhages. What is the name of this
127. As a result of craniocerebral trauma avitiminosis?
a patient reveals the following symptoms: A. Scurvy
intention tremor, dysmetry, adiadochoki- B. Pellagra
nesis, dysarthria. What structure of the C. Rachitis
brain is injured? D. Polyneuritis (beriberi)
A. Cerebellum E. Biermer’s anemia
B. Striatum 132. A large-scale reaction with
C. Motor cortex parapertussis and pertussis diagnosticums
D. Pale sphere was made in order to make serologi-
E. Black substance cal diagnostics of the whooping cough.
128. A patient with infectious At the bottom of the test-tubes with di-
mononucleosis had been taking agnosticum of Bordetella parapertussis
glucocorticoids for two weeks. He was a granular sediment formed. What anti-
brought into remission, but he fell ill bodies did this reaction reveal?
Krok 1 Medicine 2006 15

A. Agglutinins thickened on palpation. In the area of


B. Precipitins genital mucous membrane there is a
C. Opsonins small-sized ulcer with thickened edges
D. Bacteriolysins and "laquer"bottom of greyish colour.
E. Antitoxins What is the most probable diagnosis?
133. Part of the DNA chain turned 180 A. Syphilis
degree as a result of gamma radiation. B. Tuberculosis
What type of mutation took place in the C. Lepra
DNA chain? D. Trophic ulcer
E. Gonorrhea
A. Inversion
B. Deletion 138. A 45-year-old man applied to the
C. Doubling trauma station because of domestic
D. Translocation shoulder trauma. Objectively: extension,
E. Replication reduction and pronation functions of the
shoulder are absent. What muscle was
134. While having the dinner the child injured?
choked and aspirated the food. Meavy
cough has started, skin and mucose are A. Teres major muscle
cyanotic, pulse is rapid, respiration is B. Subscapular muscle
infrequent, expiration is prolonged. What C. Teres minor muscle
disorder of the external respiration has D. Infraspinous muscle
the child? E. Supraspinous muscle
A. Stage of expiratory dyspnea on 139. Diagnostic scraping was performed
asphyxia to the woman with dysfunctional uteri-
B. Stage of inspiratory dyspnea on ne bleeding. Multiple convoluted glands,
asphyxia ganglially dilated lumens of some glands
C. Stenotic respiration were revealed histologically in the scrape.
D. Alternating respiration Name the type of general pathological
E. Biot’s respiration process in endometry:
135. A pregnant woman had been havi- A. Hyperplasia glandulocystica
ng toxicosis with severe repeated vomi- B. Atrophy
ting for 24 hours. In the end of the day C. Metaplasia
there appeared tetanic convulsions and D. Displasia
fluid loss. What shift of acid-base state E. Hypertrophic excrescence
caused these changes?
140. Tuberculine was introduced
A. Excretory alkalosis intracutaneously to the child for
B. Gaseous alkalosis tuberculin test. Marked hyperemia, ti-
C. Gaseous acidosis ssue infiltration developed on the place
D. Metabolic acidosis of injection in 24 hours. What mechanism
E. Excretory acidosis caused these modifications?
136. After a trauma a 44-year-old patient A. Cell cytotoxity
had a rupture of left palm muscle tendons B. Reagin type cytotoxity
and of the superficial blood vessels. After C. Antibody cytotoxity
operation and removal of the most part D. Granuloma formation
of the necrotically changed muscle ti- E. Immunocomplex cytotoxity
ssue the bloodstream was normalized.
What vessels have helped to restore the 141. An intraoperational biopsy of
bloodstream? mammal gland has revealed the signs
of atypical tissue presented by disorder
A. Arcus palmaris prof undus of parenchyma stroma proportion with
B. Arcus palmaris superf icialis domination of the last, gland structures of
C. Aa. digitales palmares communes different size and shape, lined with single-
D. Aa. metacarpeae palmares layered proliferative epithelium. What is
E. Aa. perf orantes the most probable diagnosis?
137. An 18-year-old patient has enlarged
inguinal lymphnodes, they are painless,
Krok 1 Medicine 2006 16

A. Fibroadenoma 147. A patient working at a chemical


B. Papilloma plant was admitted to the toxicologi-
C. Noninfiltrative cancer cal department with mercury poisoning.
D. Infiltrative cancer What medicine should be used?
E. Mastitis
A. Unithiol
142. A 34-year-old patient was diagnosed B. Isonitrozin
with chronic glomerulonephritis 3 years C. Naloxone
ago. Edemata have developed within the D. Activated carbon
last 6 monthes. What caused the edemata? E. Enterosorbent
A. Proteinuria 148. A 63-year-old woman shows
B. Hyperproduction of vasopressin symptoms of rheumatoid arthritis. The
C. Liver dysfunction of protein formation increase of what blood indices could be
D. Hyperosmolarity of plasma the most significant for proving the di-
E. Hyperaldosteronism agnosis?
143. An old woman was hospitalized with A. Additive glycosaminoglycans
acute pain, edema in the right hip joint; B. Lipoproteids
the movements in the joint are limited. C. Acid phosphatase
Which bone or part of it was broken? D. General cholesterol
E. R-glycosidase
A. The neck of the thigh
B. The body of the thigh bone 149. An autopsy revealed large (1-2 cm)
C. Condyle of the thigh brownish-red, easy crumbling formations
D. Pubic bone covering ulcerative defects on the external
E. Ischial bone surface of the aortic valve. What is the
most likely diagnosis?
144. Concentration of pyruvate is
increased in the patient’s blood, the most A. Polypous-ulcerative endocarditis
of which is excreted with urine. What avi- B. Recurrent warty endocarditis
taminosis has the patient? C. Acute verrucous endocarditis
D. Fibroplastic endocarditis
A. Avitaminosis 1 E. Diffusive endocarditis
B. Avitaminosis E
C. Avitaminosis 3 150. A 7-year-old girl has signs
D. Avitaminosis B6 of anemia. Laboratory examinati-
E. Avitaminosis 2 on revealed pyruvate kinase defici-
ency in erythrocytes. What process di-
145. A patient with suspected diphtheria sturbance plays the main role in anemia
went through bacterioscopic examinati- development?
on. Examination of throat swab revealed
rod-shaped bacteria with volutin granules. A. Anaerobic glycolysis
What etiotropic preparation should be B. Oxidative phosphorylation
chosen in this case? C. Tissue respiration
D. Peroxide decomposition
A. Antidiphtheric antitoxic serum E. Aminoacids desamination
B. Bacteriophage
C. Diphtheria antitoxin 151. A 5-year-old child who often fells ill
D. Eubiotic with respiratory diseases has eczematous
E. Interferon appearances after consumption of some
food products, tendency to prolonged
146. A patient with diabetes mellitus course of inflammatory processes. What
experienced loss of consciousness and kind of diathesis can be suspected in this
convulsions after injection of insulin. case?
What is the result of biochemical blood
analysis for concentration of the sugar? A. Exudative-catharral
B. Hemmorhagic
A. 1,5 mmol/L C. Arthritism
B. 8,0 mmol/L D. Lymphohypoplastic
C. 10,0 mmol/L E. Asthenic
D. 3,3 mmol/L
E. 5,5 mmol/L 152. Low level of albumins and fibri-
Krok 1 Medicine 2006 17

nogen was detected in the patient’s blood. sturbed metabolism of what aminoacid
Decreased activity of what organelle of underlies this phenomenon?
the liver hepatocytes can most probably
cause it? A. Phenilalanine
B. Methionine
A. Granular endoplasmatic reticulum C. Tryptophan
B. Agranular endoplasmatic reticulum D. Glutamic acid
C. Mitochondrions E. Histidine
D. Golgi complex
E. Lysosomes 157. A 25-year-old patient complained of
the decreased vision. Accommodation di-
153. A duodenal content smear of a pati- sorders, dilated pupil, lack of reaction for
ent with indigestion contains protosoa 10- the light were revealed on examination.
18 mcm large. They have piriform bodi- What muscles function is disturbed?
es, 4 pairs of filaments, two symmetrically
located nuclei in the broadened part of A. Pupil narrowing muscle, ciliary
body. What kind of the lowest organisms B. Pupil dilating muscle, ciliary
is it? C. Inferior oblique muscle, ciliary
D. Lateral rectus muscle, pupil narrowing
A. Lamblia E. Pupil narrowing and dilating muscles
B. Dysentery ameba
C. Trichomonas 158. A patient with continious
D. Intestinal ameba bronchopneumonia was admitted to
E. Balantidium the therapeutic department. Antibiotic
therapy didn’t give much effect. What
154. A 44-year-old woman complains of medication for improvement of immune
common weakness, heart pain, consi- state should be added to the complex
derable increase of body weigt. Objecti- treatment of this patient?
vely: moon-like face, hirsutism, AP-
165/100 mm Hg, height - 164 cm, wei- A. Timaline
ght - 103 kg; fat is mostly accumulated in B. Analgin
the region of neck, upper shoulder girdle, C. Sulfocamphocaine
stomach. What is the main pathogenetic D. Benadryl
mechanism of obesity? E. Paracetamol
A. Increased production of glucocorticoids 159. Patient with abscess of the cut
B. Decreased production of thyroidal wound applied to the traumatological
hormones department. In order to clean the wound
C. Increased production of insulin from the pus doctor washed it with 3%
D. Decreased production of glucagon hydrogen peroxide. Foam was absent.
E. Increased production of mineralocorti- What caused the absence of the drug acti-
coids vity?
155. After a serious viral infection a 3- A. Inherited insufficiency of catalase
year-old child has repeated vomiting, loss B. Low concentration H2 O2
of consciousness, convulsions. Examinati- C. Inherited insufficiency of erythrocyte
on revealed hyperammoniemia. What phosphatdehydrogenase
may have caused changes of biochemical D. Shallow wound
blood indices of this child? E. Pus in the wound
A. Disorder of ammonia neutralization in 160. Microspecimen of spinal cord contai-
ornithinic cycle ns a nucleus that should be analyzed.
B. Activated processes of aminoacids Its neurons form motor endings in the
decarboxylation skeletal muscles. What nucleus of spinal
C. Disorder of biogenic amines neutrali- cord is meant?
zation
D. Increased purtefaction of proteins in A. Proper nucleus of the anterior horn
intestines B. Thoracic nucleus
E. Inhibited activity of transamination C. Intermediate lateral nucleus
enzymes D. Proper nucleus of the posterior horn
E. Proper nucleus of gray substance
156. Albinos can’t stand sun impact - they
don’t aquire sun-tan but get sunburns. Di- 161. Patient with injured muscles of
Krok 1 Medicine 2006 18

the lower extremities was admitted to cardiac insufficiency has decreased


the traumatological department. Due to volume of daily urine - oliguria. What is
what cells is reparative regeneration of the mechanism of this phenomenon?
the muscle fibers and restoration of the
muscle function possible? A. Decreased glomerular filtration
B. Decreased number of functioning
A. Satellite-cells glomerules
B. Myoblasts C. Drop of oncotic blood pressure
C. Myofibroblasts D. Rise of hydrostatic blood pressure in
D. Fibroblasts capillars
E. Myoepithelial cells E. Reduced permeamility of renal filter
162. A 50-year-old patient with typhoid 167. A 60-year-old man felt asleep after
fever was treated with Levomycetin, cerebral hemorrhage for a long time.
next day his condition became worse, Damage of what structure caused this
temperature rose to 39, 60 . What caused state?
the complication?
A. Reticular formation
A. The effect of endotoxin agent B. Hippocampus
B. Allergic reaction C. Nuclears of the cerebral nerves
C. Irresponsiveness of an agent to the D. Cortex of the large hemispheres
levomycetin E. Black substance
D. Secondary infection addition
E. Reinfection 168. Tuberculosis can be treated by means
of combined chemotherapy that includes
163. In the specimen of one of the parts substances with different mechanisms of
of respiratory system a tubular organ action.What antituberculous medication
was found. It has low epithelium, well inhibits transcription of RNA into DNA
developed muscular tunic, glands and in mycobacteria?
cartilage are absent. Name this organ:
A. Rifampicin
A. Minor bronchs B. Isoniazid
B. Trachea C. Streptomycin
C. Larynx D. Ethionamide
D. Major bronchs E. Para-aminosalicylic acid
E. Median bronchs
169. A patient experienced a sudden
164. A physician examined a patient temperature rise up to 390 . After 6 hours
and found inguinal hernia. Through what the temperature normalized. On the 2-nd
anatomic formation does it penetrate into day the attack recurred: in the period of
the skin? paroxysm the temperature reached 410 ,
apyrexial period began after 8 hours.
A. Hiatus saphenus What type of temperature profile is it?
B. Anulus f emoralis
C. Canalis adductorius A. Intermitting
D. Lacuna musculorum B. Recurrent
E. Anulus inguinalis superf icialis C. Hectic
D. Septic
165. A 40-year-old patient complains of E. Continued
intensive heartbeats, sweating, nausea, vi-
sual impairment, arm tremor, hypertensi- 170. Cerebral trauma caused increase
on. From his anamnesis: 2 years ago he of ammonia formation. What aminoacid
was diagnosed with pheochromocytoma. takes part in removal of ammonia from
Hyperproduction of what hormones cerebral tissue?
causes the given pathology?
A. Glutamic
A. Catecholamines B. Tyrosine
B. Aldosterone C. Valine
C. Glucocorticoids D. Tryptophan
D. ACTH E. Lisine
E. Thyroidal hormones
171. Microscopic examination of the
166. A 58-year-old patient with acute sputum of a patient with pneumonia
Krok 1 Medicine 2006 19

occasionally revealed some larvae. Eosi- 176. A 17-year-old boy fell seriously ill,
nophiles were detected on blood exami- body temperature rose up to 38, 50 , there
nation. What helminthiasis can be di- is cough, rhinitis, lacrimation, nasal di-
agnosed? scharges. What kind of inflammation is
it?
A. Ascariasis
B. Enterobiosis A. Catarrhal inflammation
C. Trichocephaliasis B. Serous inflammation
D. Paragonimiasis C. Fibrinous inflammation
E. Opisthorchosis D. Suppurative inflammation
E. Hemorrhagic inflammation
172. A 40-year-old woman was admitted
to the infectious diseases department wi- 177. A genetics specialist analyzed the
th high body temperature. Objectively: genealogy of a family and found that both
marked meningeal symptoms. A spinal males and females may have the illness,
cord punction was made. What anatomic not across all the generations, and that
formation was puncturated? healthy parents may have ill children.
What is the type of illness inheritance?
A. Spatium subarachnoideum
B. Spatium subdurale A. Autosomal recessive
C. Spatium epidurale B. Autosomal dominant
D. Cavum trigeminale C. X-linked dominant
E. Cisterna cerebellomedullaris posterior D. X-linked recessive
E. Y-linked
173. The energy inputs of a healthy man
have been measured. In what state was 178. A patient who has been treated
the patient if his energy inputs were less with diazepam on account of neurosis
than the main exchange? complains of toothache. Doctor admi-
nistered him an analgetic, but its dose
A. Sleep was lower than average therapeutic dose.
B. Rest What phenomenon did the doctor take
C. Easy work into account while prescribing the patient
D. Nervous exertion an underdose?
E. Calmness
A. Potentiation
174. Examination of a newborn boy’s geni- B. Summation
tals revealed a cleft of urethra that opens C. Cumulation
on the inferior surface of his penis. What D. Drug dependence
developmental anomaly is meant? E. Tolerance
A. Hypospadia 179. Inflammatory process of modified
B. Hermaphroditism subserous layer around cervix of the
C. Epispadia uterus caused an intensive pain syndrome.
D. Monorchism In what region of genitals does the
E. Cryptorchism pathological process take place?
175. To prevent long-term effects of 4-day A. P arametrium
malaria a 42-year-old patient was prescri- B. Mesometrium
bed primaquine. On the 3-rd day from C. Myometrium
the begin of treatment there appeared D. Endometrium
stomach and heart pains, dyspepsia, E. P erimetrium
general cyanosis, hemoglobinuria. What
caused side effects of the preparation? 180. Analysis of amniotic fluid that was
obtained as a result of amniocentesis
A. Genetic insufficiency of glucose 6- (puncture of amniotic sac) revealed cells
phosphate dehydrogenase the nuclei of which contain sex chromatin
B. Cumulation of the preparation (Barr’s body). What can it be evidence of?
C. Decreased activity of microsomal liver
enzymes
D. Delayed urinary excretion of the
preparation
E. Drug potentiation by other preparations
Krok 1 Medicine 2006 20

A. Development of female fetus A. Ketamine


B. Development of male fetus B. Sombrevin
C. Genetic disorders of fetus development C. Diethyl ether
D. Trisomy D. Thiopental sodium
E. Polyploidy E. Nitrous oxide
181. A 2-year-old child has got intesti- 185. Desulfiram is widely used in medical
nal dysbacteriosis, which resultsed in practice to prevent alcocholism. It inhi-
hemorrhagic syndrome. What is the most bits aldehyde dehydrogenase. Increased
likely cause of hemorrhage of the child? level of what metabolite causes aversion
to alcochol?
A. Vitamin K deficiency
B. Activation of tissue thromboplastin A. Acetaldehyde
C. PP hypovitaminosis B. Ethanol
D. Fibrinogen deficiency C. Malonyl aldehyde
E. Hypocalcemia D. Propionic aldehyde
E. Methanol
182. Autopsy of a 46-year-old man
revealed multiple brown-and-green layers 186. A 1-year-old child with symptoms of
and hemmorhages on the mucous muscle involvement was admitted to the
membrane of rectum and sigmoid colon; hospital. Examination revealed carnitine
slime and some blood in colon lumen; hi- deficiency in his muscles. What process di-
stologically - fibrinous colitis. In course of sturbance is the biochemical basis of this
bacteriological analysis of colon contents pathology?
S.Sonne were found. What is the most
probable diagnosis? A. Transporting of fatty acids to mi-
tochodrions
A. Dysentery B. Regulation of Ca2+ level in mi-
B. Cholera tochondrions
C. Salmonellosis C. Substrate phosphorylation
D. Yersiniosis D. Lactic acid utilization
E. Crohn’s disease E. Actin and myosin synthesis
183. A patient had been ill with bronchi- 187. A patient died from acute cardiac
al asthma for many years and died from insufficiency. The histological examinati-
asthmatic fit. Histologic lung examinati- on of his heart revealed the necrotized
on revealed: lumen of bronchioles and section in myocardium of the left ventri-
small bronches contain a lot of mucus cle, which was separated from undamaged
with some eosinophils, there is sclerosis tissue by the zone of hyperimic vessels,
of alveolar septums, dilatation of alveole small hemorrhages and leukocytic infi-
lumen. What mechanism of development ltration. What is the most likely diagnosis?
of hypersensibility reaction took place?
A. Myocardial infarction
A. Reagin reaction B. Myocardial ischemic dystrophy
B. Cytotoxic reaction C. Focal exudate myocarditis
C. Immunocomplex reaction D. Diffuse exudate myocarditis
D. Cytolysis determined by lymphocytes E. Productive myocarditis
E. Granulomatosis
188. Kidneys of a man under examinati-
184. To anaesthetize the surgical on show increased resorbtion of calci-
treatment of burn surface, a patient was um ions and decreased resorbtion of
intravenously injected a medication for phosphate ions. What hormone causes
short-acting narcosis. 1 minute later the this phenomenon?
patient being under anaesthesia showed
increased blood pressure, tachycardia, A. Parathormone
increased tone of skeletal muscles; B. Thyrocalcitonin
reflexes remained. After recovering from C. Hormonal form D3
anaesthesia the patient had disorientation D. Aldosterone
and visual hallucinations. What medicati- E. Vasopressin
on was the patient injected?
189. An isolated cell of human heart
automatically generates excitation
impulses with frequency 60 times pro mi-
Krok 1 Medicine 2006 21

nute. What structure does this cell belong 194. Objective examination of a patient
to? revealed: slender figure, big skull, highly
developed frontal region of face, short
A. Sinoatrial node extremities. What constitutional type is
B. Atrium it characteristic for?
C. Ventricle
D. Atrioventricular node A. Respiratory
E. His’ bundle B. Muscular
C. Digestive
190. While preparing a patient to the D. Cerebral
operation the heart chambers’ pressure E. Mixed
was measured. In one of them the
pressure changed during one heart cycle 195. A 38-year-old patient died during
from 0 to 120 mm Hg. What chamber of intractable attack of bronchial asthma.
heart was it? Histologic examination revealed mucus
accumulation in bronchial lumen, a lot
A. Left ventricle of fat cells (labrocytes) in the wall of
B. Right ventricle bronches, many of them are in the state of
C. Right atrium degranulation, there are also a lot of eosi-
D. Left atrium nophils. What pathogenesis of bronchial
E. - changes is it?
191. Part of alveoles of a preterm infant A. Atopy
didn’t spread because of enhanced elastic B. Cytotoxic, cytolytic action of antibodies
recoil of lungs. How can this recoil be C. Immunocomplex mechanism
reduced? D. Cellular cytolysis
E. Granulomatosis
A. By surfactant introduction
B. By pure oxygene inhalation 196. A patient with thrombophlebitis
C. By artificial pulmonary ventilation of lower extremities had got chest pai-
D. By fluid suction from the respiratory ns, blood spitting, growing respiratory
tracts failure that caused his death. Autopsy
E. By glycose introduction revealed multiple pulmonary infarctions.
What is the most probable reason of their
192. A young man has a painlessl formati- development?
on without marked borders in the soft ti-
ssues of his thigh. On the tissue bioptate A. Pulmonary artery embolism
the formation looks like flesh of fish and B. Pulmonary artery thrombosis
consists of immature fibroblast-like cells C. Bronchial artery thrombosis
with multiple mitosis growing through D. Bronchial artery embolism
the muscles. What is the most likely di- E. Pulmonary venous thrombosis
agnosis?
197. Diabetes mellitus causes ketosis as a
A. Fibrosarcoma result of activated oxidation of fatty acids.
B. Myosarcoma What disorders of acid-base equilibrium
C. Fibroma may be caused by excessive accumulation
D. Cancer of ketone bodies in blood?
E. Myoma
A. Metabolic acidosis
193. In course of prophylactic medi- B. Metabolic alcalosis
cal examination a 7-year-old boy was C. Any changes woun’t happen
diagnosed to have daltonism. Parents D. Respiratory acidosis
are healthy, color vision is normal. But E. Respiratory alcalosis
grandfather from the mother’s side has
the same disorder. What is the type of 198. A woman with 0 (I) bllod group has
inheriting of this anomaly? born a child with AB blood group. This
woman’s husband has A blood group.
A. Recessive, sex-linked What genetic interaction explains this
B. Dominant, sex-linked phenomenon?
C. Incomplete domination
D. Autosomal-recessive
E. Autosomal-dominant
Krok 1 Medicine 2006 22

A. Recessive epistasis madescence in the region of mammilla, a


B. Codominance small ulcer with inflammatory hyperemia
C. Polymery and cutaneous edema. Histologic exami-
D. Incomplete dominance nation of tissue sampling from this area
E. Complementation revealed in the malpighian layer of thi-
ckened epidermis atypical cells with li-
199. A 30-year-old woman was diagnosed ght and optically empty cytoplasm, with
with insufficiency of exocrinous function no intracellular bridges. Such cells were
of pancreas. Hydrolisis of what nutrients also found in the orifice of big mammal
will be disturbed? gland ducts. What is the most probable di-
A. Proteins, fats, carbohydrates agnosis?
B. Proteins, fats A. Paget’s disease
C. Proteins, carbohydrates B. Intraductal cancer
D. Fats, carbohydrates C. Basal cell carcinoma
E. Proteins D. Epidermoid cancer
200. A 39-year-old woman has E. Melanocarcinoma
Krok 1 Medicine 2007 1

1. A patient had to go through an A. Antegastrial bursa


operation. Doctors introduced him di- B. Omental bursa
thylinum (listenone) and performed C. Hepatic bursa
intubation. After the end of operati- D. Left mesenteric sinus
on and cessation of anesthesia the E. Right mesenteric sinus
independent respiration wasn’t restored.
Which enzyme deficit prolongs the action 6. A 4 y.o. child with signs of durative
of muscle relaxant? proteinic starvation was admitted to the
hospital. The signs were as follows: growth
A. Pseudocholinesterase inhibition, anemia, edemata, mental
B. Succinate dehydrogenase deficiency. Choose a cause of edemata
C. Carbanhydrase development:
D. N-acetyltransferase
E. K-Na-adenosine triphosphatase A. Reduced synthesis of albumins
B. Reduced synthesis of globulins
2. A patient is ill with diabetes mellitus C. Reduced synthesis of hemoglobin
that is accompanied by hyperglycemia of D. Reduced synthesis of lipoproteins
over 7,2 millimole/l on an empty stomach. E. Reduced synthesis of glycoproteins
The level of what blood plasma protein
allows to estimate the glycemia rate 7. Autopsy of a patient who suffered
from croupous pneumonia and died from
retrospectively (4-8 weeks before exami- pneumococcal sepsis revealed 900 ml of
nation)? turbid greenish-yellow liquid in the ri-
A. Glycated hemoglobin ght pleural cavity. Pleural leaves are dull,
B. Albumin plephoric. Name the clinicopathological
C. Fibrinogen form of inflammation in the pleural cavity:
D. C-reactive protein A. Empyema
E. Ceruloplasmin B. Fibrinous inflammation
3. A 60 y.o. patient has a reduced percepti- C. Phlegmon
on of high-frequency sounds. What D. Chronic abscess
structures’ disorder of auditory analizer E. Acute abscess
caused these changes? 8. Examination of a patient revealed
A. Main membrane of cochlea near the reduced contents of magnesium ions that
oval window are necessary for attachment of ribosomes
B. Main membrane of cochlea near heli- to the granular endoplasmatic reticulum.
cotrema It is known that it causes disturbance
C. Eustachian tube of protein biosynthesis. What stage of
D. Muscles of middle ear protein biosynthesis will be disturbed?
E. Tympanic membrane A. Translation
4. A patient ill with diabetes mellitus B. Transcription
felt acute pain in his right foot. Objecti- C. Replication
vely: foot thumb is black, foot tissues are D. Aminoacid activation
edematous, there are foci of epidermis E. Termination
desquamation, stinking discharges. What 9. Analysis of blood serum of a
clinicopathological form of necrosis is it? patient revealed increase of alanine
A. Moist gangrene aminotransferase and aspartate ami-
B. Bedsore notransferase level. What cytological
C. Sequestrum changes can cause such a situation?
D. Dry gangrene A. Cellular breakdown
E. Infarction B. Disturbed function of energy supply of
5. A patient with a stab wound of the cells
anterior stomach wall is in surgical care. C. Disorder of enzyme systems of cells
What formation of abdominal cavity did D. Disturbance of genetic apparatus of
the stomach contents get into? cells
E. Disturbance of cellular interrelations
10. Power inputs of a boy increased from
500 to 2000 kJ pro hour. What can be the
Krok 1 Medicine 2007 2

cause of it? as a result of substitution of glutamin by


valine in the hemoglobin molecule. What
A. Physical exercise is the cause of this disease?
B. Raise of outer temperatute
C. Mental activity A. Gene mutation
D. Food intake B. Disturbance of mechanisms of genetic
E. Transition from sleep to wakefulness information realization
C. Crossingover
11. A patient presents high activity D. Genomic mutations
of LDH1,2 , aspartate aminotransferase, E. Transduction
creatine phosphokinase. In what organ
(organs) is the development of a 16. A patient died under conditions
pathological process the most probable? of cardiovascular insufficiency. Autopsy
results: postinfarction cardiosclerosis,
A. In the heart muscle (initial stage of myocardium hypertrophy and dilatati-
myocardium infarction) on of its cavities, especially of its right
B. In skeletal muscles (dystrophy, atrophy) ventricle. Liver is enlarged, its surface
C. In kidneys and adrenals is smooth, incision revealed that it was
D. In connective tissue plethoric, with dark-red specks against
E. In liver and kidneys the background of brownish tissue. Hi-
stologically: plethora of central parts of
12. According to the data of WHO, for lobules; peritheral parts around portal
about 250 mln of Earth population fall ill tracts contain hepatocytes in a state of
with malaria. This disease is mostly spread adipose degeneration. How are these li-
in tropical and subtropical regions. Range ver changes called?
of its spread falls into the areal of the
following mosquitoes: A. Nutmeg liver
B. Pseudonutmeg liver
A. Anopheles C. Amyloidosis
B. Culex D. Liver cirrhosis
C. Aedes E. Liver steatosis
D. Mansonia
E. Culiseta 17. Autopsy of a man who died from
chronic cardiovascular collapse revealed
13. Autopsy of a 58 y.o. man revealed that "tiger heart". Sidewards of endocardium
bicuspid valve was deformed, thickened a yellowish-white banding can be seen;
and unclosed. Microscopically: foci of myocardium is dull, dark-yellow. What
collagen fibrilla are eosinophilic, react process caused this pathology?
positively to fibrin. The most probably it
is: A. Fatty parenchymatous degeneration
B. Carbohydrate degeneration
A. Fibrinoid swelling C. Hyaline degeneration
B. Fibrinous inflammation D. Fatty vascular-stromal degeneration
C. Mucoid swelling E. Amyloidosis
D. Hyalinosis
E. Amyloidosis 18. In course of an experiment a skeletal
muscle is being stimulated by a series of
14. Labeled aminoacids alanine and electric impulses. What type of muscle
tryptophane were introducted to a mouse contraction will arise, if every subsequent
in order to study localization of protein impulse comes in the period of shortening
biosynthesis in its cells. Around what of the previous single muscle contraction?
organellas will the accumulation of
labeled aminoacids be observed? A. Holotetanus
B. Partial tetanus
A. Ribosomes C. Asynchronous tetanus
B. Agranular endoplasmic reticulum D. A series of single contractions
C. Cell centre E. Muscle contracture
D. Lysosomes
E. Golgi apparatus 19. Ammonia is a very toxic substance,
especially for nervous system. What
15. In some regions of South Africa there substance takes the most active part in
is a spread sickle-shaped cell anemia, in ammonia detoxication in brain tissues?
which erythrocytes have shape of a sickle
Krok 1 Medicine 2007 3

A. Glutamic acid
B. Lysine A. Collagen
C. Proline B. Mucin
D. Histidine C. Myosin
E. Alanine D. Ovoalbumin
E. Troponin
20. A man was admitted to the hospital
on the 5th day of disease that manifested 25. A patient with infectious mononucleosis
itself by jaundice, muscle aching, chill, has been taking glucocorticoids for two
nose bleedings. In course of laboratory di- weeks. He was brought into remission,
agnostics a bacteriologist performed dark- but he fell ill with acute attack of chronic
field microscopy of the patient’s blood tonsillitis. What action of glucocorticoids
drop. Name a causative agent of this di- caused this complication?
sease:
A. Immunosuppressive
A. Leptospira interrogans B. Anti-inflammatory
B. Borrelia dutlonii C. Antishock
C. Calymmatobacterium granulomatis D. Antiallergic
D. Bartonella bacillof ormis E. Antitoxic
E. Rickettsia mooseri
26. A peripheral segment of vagus nerve
21. A patient has a transverse disrupti- on a dog’s neck was being stimulated in
on of spinal cord below the IV thoracic course of an experiment. The followi-
segment. What changes of respiration will ng changes of cardiac activity could be
it cause? meanwhile observed:
A. Respiration will stay unchanged A. Heart rate fall
B. Respiration will stop B. Heart hurry
C. Respiration will become less frequent C. Enhancement of atrioventricular
D. Respiration will become deeper conduction
E. Respiration will become more frequent D. Heart rate and heart force amplification
E. Increased excitability of myocardium
22. 12 hours after an accute attack of
retrosternal pain a patient presented a 27. As a result of exhausting muscular
jump of aspartate aminotransferase activi- work a worker has largely reduced
ty in blood serum. What pathology is this buffer capacity of blood. What acidic
deviation typical for? substance that came to blood caused this
phenomenon?
A. Myocardium infarction
B. Viral hepatitis A. Lactate
C. Collagenosis B. Pyruvate
D. Diabetes mellitus C. 1,3-bisphosphoglycerate
E. Diabetes insipidus D. 3-phosphoglycerate
E. -
23. A lightly dressed man is standing in a
room, air temperature is +140 C, windows 28. A patient was delivered to the hospi-
and doors are closed. In what way does he tal by an emergency team. Objectively:
emit heat the most actively? grave condition, unconscious, adynamy.
Cutaneous surfaces are dry, eyes are
A. Heat radiation sunken, face is cyanotic. There is
B. Heat conduction tachycardia and smell of acetone from the
C. Convection mouth. Analysis results: blood glucose -
D. Evaporation 20,1 micromole/l (standard is 3,3-5,5 mi-
E. Perspiration cromole/l), urine glucose - 3,5% (standard
is - 0). What is the most probable di-
24. A 30 y.o. woman had been ill for a year agnosis?
when she felt pain in the area of joints
for the first time, they got swollen and A. Hyperglycemic coma
skin above them became reddened. Provi- B. Hypoglycemic coma
sional diagnosis is rheumatoid arthri- C. Acute heart failure
tis. One of the most probable causes of D. Acute alcoholic intoxication
this disease is a structure alteration of a E. Anaphylactic shock
connective tissue protein:
Krok 1 Medicine 2007 4

29. A 62 y.o. woman complains of frequent 34. Removal of gall bladder of a patient
pains in the area of her chest and has disturbed processes of Ca absorption
backbone, rib fractures. A doctor assumed through the intestinal wall. What vitamin
myelomatosis (plasmocytoma). What of will stimulate this process?
the following laboratory characteristi-
cs will be of the greatest diagnostical A. D3
importance? B. P P
C. C
A. Paraproteinemia D. B12
B. Hyperalbuminemia E. K
C. Proteinuria
D. Hypoglobulinemia 35. A hepatitis outbreak was registered in
E. Hypoproteinemia a settlement. This episode is connected wi-
th water factor. What hepatitis virus could
30. Nappies of a newborn have dark spots have caused the infective outbreak in this
that witness of formation of homogenti- settlement?
sic acid. Metabolic imbalance of which
substance is it connected with? A. E
B. C
A. Thyrosine C. D
B. Galactose D. G
C. Methionine E. B
D. Cholesterine
E. Tryptophane 36. Examination of a 43 y.o. patient
revealed that his stomach has difficulti-
31. Having helped to eliminate es with digestion of protein food. Gastric
consequences of a failure at a nuclear juice analysis revealed low acidity. Functi-
power plant, a worker got an irradiati- on of which gastric cells is disturbed in this
on doze of 500 roentgen. He complai- case?
ns of headache, nausea, dizziness. What
changes in leukocytes quantity can be A. Parietal exocrinocytes
expected 10 hours after irradiation? B. Main exocrinocytes
C. Mucous cells (mucocytes)
A. Neutrophilic leukocytosis D. Endocrinous cells
B. Lymphocytosis E. Cervical mucocytes
C. Leukopenia
D. Agranulocytosis 37. A 16 y.o. boy from a countryside
E. Leukemia entered an educational establishment.
Scheduled Manteux test revealed that the
32. A 12 y.o. boy who suffers from bronchi- boy had negative reaction. What are the
al asthma has an acute attack of asthma: most reasonable actions in this case?
evident expiratory dyspnea, skin pallor.
What type of alveolar ventilation di- A. To perform BCG vaccination
sturbance is it? B. To repeat the reaction in a month
C. To perform serodiagnostics of
A. Obstructive tuberculosis
B. Restrictive D. To isolate the boy temporarily from his
C. Throracodiaphragmatic mates
D. Central E. To perform rapid Price diagnostics
E. Neuromuscular
38. A patient complains of frequent di-
33. A woman who has been keepi- arrheas, especially after consumption
ng to a clean-rice diet for a long time of fattening food, and of body weight
was diagnosed with polyneuritis (beri- loss. Laboratory examination revealed
beri). What vitamin deficit results in steatorrhea; hypocholic feces. What can
development of this disease? be the cause of this condition?
A. Thiamine
B. Ascorbic acid
C. Pyridoxine
D. Folic acid
E. Riboflavin
Krok 1 Medicine 2007 5

A. Obturation of biliary tracts significant excess of glycogene. Glucose


B. Mucous membrane inflammation of concentration in the blood stream is below
small intestine normal. What is the cause of low glucose
C. Lack of pancreatic lipase concentration?
D. Lack of pancreatic phospholipase
E. Unbalanced diet A. Low (absent) activity of glycogene
phosphorylase in liver
39. A patient consulted a doctor about B. Low (absent) activity of hexokinase
bowels disfunction. The doctor establi- C. High activity of glycogen synthetase
shed symptoms of duodenitis and enteri- D. Low (absent) activity of glucose 6-
tis. Laboratory examination helped to phosphatase
make the following diagnosis: lambliosis. E. Deficit of a gene that is responsible
What medication should be administered? for synthesis of glucose 1-phosphaturidine
A. Metronidazole transferase
B. Erythromycin 44. A 63 y.o. man with collapse symptoms
C. Monomycin was delivered to the emergency hospital.
D. Chingamin A doctor chose noradrenaline in order to
E. Tetracycline prevent hypotension. What is the action
40. A group of mountain climbers went mechanism of this medication?
through the blood analysis at the hei- A. Activation of α1 -adrenoreceptors
ght of 3000 m. It revealed decrease of B. Activation of serotonin receptors
HCO3 to 15 micromole/l (standard is 22- C. Activation of β-adrenoreceptors
26 micromole/l). What is the mechanism D. Activation of dopamine receptors
of HCO3 decrease? E. Block of M-cholinoreceptors
A. Hyperventilation 45. From pharynx of a child with
B. Intensification of acidogenesis suspected diphtheria a pure culture
C. Hypoventilation of microorganisms was isolated. Their
D. Decrease of ammoniogenesis morphological, tinctorial, cultural and bi-
E. Decrease of bicarbonate reabsorption ochemical properties appeared to be typi-
in kidneys cal for diphtheria causative agents. What
study should be conducted in order to
41. Introduction of a pharmaceutical drow a conclusion that this is a pathogenic
substance to an experimental animal diphtheria bacillus?
resulted in reduction of salivation, pupil
mydriasis. Next intravenous introduction A. Estimation of toxigenic properties
of acetylcholine didn’t lead to any signi- B. Estimation of proteolytic properties
ficant changes of heart rate. Name this C. Estimation of urease activity
substance: D. Estimation of cystinous activity
E. Estimation of ability to decompose
A. Atropine starch
B. Adrenaline
C. Propranolol 46. Examination of a child revealed some
D. Proserin whitish spots looking like coagulated milk
E. Salbutamol on the mucous membrane of his cheeks
and tongue. Analysis of smears revealed
42. Continious taking of a drug can gram-positive oval yeast-like cells. What
result in osteoporosis, erosion of causative agents are they?
stomach mucous membrane, hypokali-
emia, retention of sodium and water, A. Candida
reduced content of corticotropin in blood. B. Staphylococci
Name this drug: C. Diphtheria bacillus
D. Actinomycetes
A. Prednisolone E. Fusobacteria
B. Hydrochlorothiazide
C. Digoxin 47. A 35 y.o. patient who often consumes
D. Indometacin alcohol was treated with diuretics. There
E. Reserpine appeared serious muscle and heart
weakness, vomiting, diarrhea, AP- 100/60
43. A child is languid, apathetic. Liver mm Hg, depression. This condition is
is enlarged and liver biopsy revealed a caused by intensified excretion with uri-
Krok 1 Medicine 2007 6

ne of: tamin absence caused disturbance of their


hydroxylation?
A. Potassium
B. Sodium A. Vitamin C
C. Chlorine B. Vitamin K
D. Calcium C. Vitamin A
E. Phosphates D. Thiamine
E. Vitamin E
48. After intake of rich food a patient
feels nausea and sluggishness; with ti- 53. A patient who suffers from pneumonia
me there appeared signs of steatorrhea. has high body temperature. What biologi-
Blood cholesterine concentration is 9,2 cally active substance plays the leading
micromole/l. This condition was caused part in origin of this phenomenon?
by lack of:
A. Interleukin-I
A. Bile acids B. Histamine
B. Triglycerides C. Bradykinin
C. Fatty acids D. Serotonin
D. Phospholipids E. Leukotrienes
E. Chylomicrons
54. A 27 y.o. patient put eye drops
49. Examination of a man who hadn’t that contain penicillin. After a few mi-
been consuming fats but had been getti- nutes she felt itching and burning of
ng enough carbohydrates and proteins her body, there appeared lip and eye-lid
for a long time revealed dermatitis, poor edemata; arterial pressure began to drop.
wound healing, vision impairment. What What immunoglobulins took part in the
is the probable cause of metabolic di- development of this allergic reaction?
sorder?
A. lgE and lgG
A. Lack of linoleic acid, vitamins A, D, E, B. IgM and IgG
K C. IgA and IgM
B. Lack of palmitic acid D. IgM and IgD
C. Lack of vitamins P P , H E. IgG and IgD
D. Low caloric value of diet
E. Lack of oleic acid 55. A patient suffers from hepatic ci-
rrhosis. Examination of which of the
50. An experimental animal has been gi- following substances excreted by urine
ven excessive amount of carbon-labeled can characterize the state of antitoxic
glucose for a week. What compound can function of liver?
the label be found in?
A. Hippuric acid
A. Palmitic acid B. Ammonium salts
B. Methionine C. Kreatinine
C. Vitamin A D. Uric acid
D. Choline E. Aminoacids
E. Arachidonic acid
56. A 2 y.o. child has convulsions as a
51. RNA that contains AIDS virus result of lowered concentration of calci-
penetrated into a leukocyte and by means um ions in blood plasma. It is caused by
of reverse transcriptase forced a cell to reduced function of:
synthetize a viral DNA. This process is
based upon: A. Parathyroid glands
B. Hypophysis
A. Reverse transcription C. Adrenal cortex
B. Operon repression D. Pineal gland
C. Reverse translation E. Thymus
D. Operon depression
E. Convariant replication 57. Inflammation of a patient’s eye was
accompanied by accumulation of turbid
52. Examination of a patient with frequent liquid with high protein at the bottom
hemorrhages from internals and mucous of anterior chamber that was called
membranes revealed proline and lysine hypopyon. What process underlies the
being a part of collagene fibers. What vi- changes under observation?
Krok 1 Medicine 2007 7

A. Disturbance of microcirculation A. HIV infection


B. Primary alteration B. Being ill with AIDS
C. Secondary alteration C. Being infected with HBV
D. Proliferation D. Having had AIDS recently
E. - E. HBV persistence
58. Heart rate of a man permanently 63. Parents of a 10 y.o. boy consulted a
equals 40 beats pro minute. What is the doctor about extension of hair-covering,
pacemaker? growth of beard and moustache, low voi-
ce. Intensified secretion of which hormone
A. Atriventricular node must be assumed?
B. Sinoatrial node
C. His’ bundle A. Of testosterone
D. His’ bundle branches B. Of somatotropin
E. Purkinje’s fibers C. Of oestrogen
D. Of progesterone
59. A 48 y.o. patient was admitted to the E. Of cortisol
hospital with complaints about weakness,
irritability, sleep disturbance. Objectively: 64. A patient who suffers from severe di-
skin and scleras are yellow. In blood: sorder of water-salt metabolism experi-
conjugated bilirubin, cholalemia. Feces enced cardiac arrest in diastole. What is
are acholic. Urine is of dark colour (bi- the most probable mechanism of cardiac
lirubin). What jaundice is it? arrest in diastole?
A. Mechanic A. Hyperkaliemia
B. Hemolytic B. Hypernatremia
C. Parenchymatous C. Organism dehydratation
D. Gilbert’s syndrome D. Hypokaliemia
E. Crigler-Najjar syndrome E. Hyponatremia
60. Bacteriological examination of a 65. Examination of coronary arteries
patient with food poisoning required revealed atherosclerotic calcific plaques
inoculation of a pure culture of bacteria that close vessel lumen by 1/3. The muscle
with the following properties: gram- has multiple whitish layers of connecti-
negative movable bacillus that grows in ve tissue. What process was revealed in
the Endo’s medium in form of colourless myocardium?
colonies. A representative of which speci-
es caused this disease? A. Diffuse cardiosclerosis
B. Tiger heart
A. Salmonella C. Postinfarction cardiosclerosis
B. Shigella D. Myocarditis
C. Iersinia E. Myocardium infarction
D. Esherichia
E. Citrobacter 66. Reaction of passive hemagglutinati-
on conducted with erythrocytic typhoid
61. A patient was diagnosed with active Vi-diagnosticum helped to reveal some
focal pulmonary tuberculosis. What drug antibodies in the dilution of the patient’s
should be prescribed in the first place? serum at a ratio of 1:80 that exceeds the
diagnostic titer. Such result witnesses of:
A. Isoniazid
B. Sulfalen A. Being a potential carrier of typhoid
C. Cyclocerine bacilli
D. Ethionamide B. Being ill with acute typhoid fever
E. Ethoxide C. Typhoid fever recurrence
D. Incubation period of typhoid fever
62. Examination of a young man in the E. Reconvalescence of a patient ill with
AIDS centre produced a positive result typhoid fever
of immune-enzyme assay with HIV anti-
gens. Patient’s complaints about state of 67. A teenager was irradiated with hi-
his health were absent. What can the posi- gh radiation dose that resulted in seri-
tive result of immune-enzyme assay be ous damages of lymphoid system, lysis of
evidence of? many lymphocytes. Restoration of normal
Krok 1 Medicine 2007 8

hemogram is possible due to the functi- an air volume that is meanwhile contained
oning of the following gland: in his lungs:
A. Thymus A. Functional residual capacity
B. Thyroid B. Residual volume
C. Liver C. Expiratory reserve volume
D. Pancreas D. Respiratory volume
E. Adrenal E. Vital lung capacity
68. A man with cut wound of his right 73. Examination of an isolated cardi-
foot sole was admitted to the hospital omyocyte revealed that it didn’t generate
ward. The patient has limited elevati- excitation impulses automatically. This
on of the lateral foot edge. In course cardiomyocyte was obtained from:
of wound management the injury of a
muscle tendon was revealed. What muscle A. Ventricles
is injured? B. Sinoatrial node
C. Atrioventricular node
A. Long peroneal D. His’ bundle
B. Anterior tibial E. Purkinje’s fibers
C. Long extensor muscle of toes
D. Triceps muscle of crus 74. Examination of a man established that
E. Short peroneal cardiac output equaled 3500 ml, systolic
output - 50 ml. What is the man’s heart
69. A patient who suffers from heart fai- rate pro minute?
lure has enlarged liver, edemata of lower
extremities, ascites. What is the leading A. 70
mechanism in the development of this B. 60
edema? C. 50
D. 80
A. Hydrodynamic E. 90
B. Colloid osmotic
C. Lymphogenous 75. A man who went for a ride on a
D. Membranogenic roundabout had amplification of heart
E. - rate, sweating and nausea. What receptors
stimulation is it primarily connected with?
70. A 32 y.o. man is tall, he has
gynecomastia, adult woman pattern of A. Vestibular
hair distribution, high voice, mental defici- B. Proprioceptors
ency, sterility. Provisional diagnosis is Kli- C. Tactors
nefelter’s syndrome. In order to specify D. Auditory
diagnosis it is necessary to analize: E. Visual

A. Caryotype 76. In order to estimate toxigenity of di-


B. Leukogram phtheria agents obtained from patients
C. Spermatogenesis the cultures were inoculated on Petri dish
D. Blood group with nutrient agar on either side of a filter
E. Genealogy paper strip that was put into the centre
and moistened with antidiphtheric anti-
71. Examination of a miner revealed toxic serum. After incubation of inoculati-
pulmonary fibrosis accompanied by di- ons in agar the strip-like areas of medium
sturbance of alveolar ventilation. What is turbidity were found between separate
the main mechanism of this disturbance? cultures and the strip of filter paper. What
immunological reaction was conducted?
A. Limitation of respiratory surface of
lungs A. Precipitation gel reaction
B. Constriction of superior respiratory B. Coomb’s test
tracts C. Agglutination reaction
C. Disturbance of neural respiration D. Rings precipitation reaction
control E. Opsonization reaction
D. Limitation of breast mobility
E. Bronchi spasm 77. A man’s intrapleural pressure is being
measured. In what phase did the man hold
72. A man took a quiet expiration. Name his breath, if his pressure is 7,5 cm Hg?
Krok 1 Medicine 2007 9

and painfulness. What type of allergic


A. Quiet inspiration reaction was developed?
B. Quiet expiration
C. Forced inspiration A. IV
D. Forced expiration B. I
E. - C. II
D. III
78. A 49 y.o. woman consulted a doctor E. V
about heightened fatigue and dyspnea
during physical activity. ECG: heart 83. Examination of a patient revealed
rate is 50/min, PQ is extended, QRS extremely myotic pupils, sleepiness,
is unchanged, P wave quanity exceeds infrequent Chain-Stoke’s respiration, uri-
quantity of QRS complexes. What type nary retention, slowing-down of heart
of arrhythmia does the patient have? rate, enhancement of spinal reflexes. What
substance caused the poisoning?
A. Atrioventricular block
B. Extrasystole A. Morphine
C. Sinus bradycardia B. Atropine
D. Ciliary arhythmia C. Phosphacole
E. Sinoatrial block D. Caffeine
E. Barbital
79. A patient with neuritis of femoral
nerve has disturbed flexion of thigh as well 84. A patient is followed up in an
as disturbed crus extension in the knee joi- endocrinological dispensary on account of
nt. What muscle’s function is disturbed? hyperthyreosis. Weight loss, tachycardia,
finger tremor are accompanied by hypoxia
A. Quadriceps muscle of thigh symptoms - headache, fatigue, eye flicker.
B. Biceps muscle of thigh What mechanism of thyroid hormones
C. Triceps muscle of thigh action underlies the development of
D. Semitendinous muscle hypoxia?
E. Semimembranous muscle
A. Disjunction, oxydation and phosphori-
80. Autopsy of a newborn boy revealed lation
polydactylia, microcephalia, cheiloschisis B. Inhibition of respiratory ferment
and uranoschisis as well as hypertrophy synthesis
of parenchimatous organs. These defects C. Competitive inhibition of respiratory
correspond with the description of Patau’s ferments
syndrome. What is the most probable D. Intensification of respiratory ferment
cause of this pathology? synthesis
E. Specific binding of active centres of
A. Trisomy of the 13th chromosome respiratory ferments
B. Trisomy of the 18th chromosome
C. Trisomy of the 21st chromosome 85. A doctor administered a patient with
D. Nondisjunction of sex chromosomes allergic dermatitis a H1 -histamine blocker
E. Partial monosomy as a part of complex treatment. Name this
medication:
81. An ovary specimen stained by
hematoxylin-eosin presents a follicle, A. Loratadine
where cells of follicular epithelium are B. Cromolyn sodium
placed in 1-2 layers and have cubic form, C. Prednisolone
there is a bright-red membrane around D. Adrenaline
the ovocyte. What follicle is it? E. Hydrocortisone
A. Primary 86. An injured man has bleeding
B. Primordial from branches of carotid artery. For
C. Secondary a temporary arrest of bleeding it is
D. Mature necessary to press the carotid artery to
E. Atretic the tubercle of a cervical vertebra. Which
vertebra is it?
82. A woman has been applying a
new cosmetic preparation for a week
that resulted in eye-lid inflammation
accompanied by hyperemia, infiltration
Krok 1 Medicine 2007 10

A. VI atypic light epithelial Langhans cells and


B. V giant cells of cyncytiotrophoblast in blood
C. IV lacunas. What tumour is it?
D. III
E. II A. Chorioepithelioma
B. Squamous cell nonkeratinous carci-
87. As a result of an accident a patient noma
has intense painfullness and edema of the C. Adenocarcinoma
anterior crus surface; dorsal flexion of foot D. Fibromyoma
is hindered. Function of which crus muscle E. Vesicular mole
is most likely to be disturbed?
92. A 40 y.o. patient complains of intensi-
A. M.tibialis anterior ve heartbeats, sweating, nausea, visi-
B. M.f lexor digitorum longus on impairment, arm tremor, hypertensi-
C. M.f lexor hallucis longus on. From his anamnesis: 2 years ago he
D. M.peroneus longus was diagnosed with pheochromocytoma.
E. M.peroneus brevis Hyperproduction of what hormones
causes the given pathology?
88. The first grade pupils were exami-
ned in order to sort out children for A. Catecholamines
tuberculosis revaccination. What test was B. Aldosterone
applied for this purpose? C. Glucocorticoids
D. ACTH
A. Mantoux test E. Thyroid hormones
B. Schick test
C. Supracutaneous tularin test 93. A 56 y.o. patient has been suffering
D. Burnet test from thyreotoxicosis for a long time. What
E. Anthraxine test type of hypoxia can be developed?
89. The permeability of the irritable cell A. Tissue
membrane has been increased for potassi- B. Hemic
um ions during an experiment. What C. Circulatory
changes of membrane electric status can D. Respiratory
occur? E. Mixed
A. Hyperpolarization 94. A sportsman was recommended to
B. Depolarization take a medication that contains carniti-
C. Action potential ne in order to improve his results. What
D. Local response process is activated by carnitine the most?
E. No changes
A. Fatty acids transport to mitochondrions
90. A patient with II stage hypertensi- B. Synthesis of steroid hormones
on has been taking one of hypotensive C. Synthesis of ketone bodies
medications for the purpose of treatment. D. Synyhesis of lipids
After a time arterial pressure decreased, E. Tissue respiration
but the patient started complaining of
flaccidity, sleepiness, indifference. A bit 95. A patient has symptoms of inflammati-
later he felt stomach pain. He was di- on of urogenital tracts. Examination of a
agnosed with ulcer. What hypotensive vaginal smear revealed big monocellular,
medication has the patient been taking? pear-shaped organisms with the pointed
spike at the posterior end of body, big
A. Reserpine nucleus and undulating membrane. What
B. Dibazole protozoa were found in the smear?
C. Furosemide
D. Verapamil A. T richomonas vaginalis
E. Captopril B. T richomonas hominis
C. T richomonas buccalis
91. 6 months after delivery a woman had D. T rypanosoma gambiense
uterine bleeding. Gynecological exami- E. Lamblia intestinalis
nation revealed in the uterine cavity a
dark-red tissue with multiple cavities 96. A patient has extrasystole. ECG shows
that resembled of "sponge". Microscopic no P wave, QRS complex is deformed,
examination of the tumour revealed some there is a full compensatory pause. What
Krok 1 Medicine 2007 11

extrasystoles are these? canned meat whether it contains botuli-


num toxin. For this purpose an extract of
A. Ventricular test specimen and antitoxic antibotulinic
B. Atrial serum of A, B, E types were introducted
C. Atrioventricular to a group of mice under examination; a
D. Sinus control group of mice got the extract wi-
E. - thout antibotulinic serum. What serologi-
cal reaction was applied?
97. A patient who suffers from cancer of
back of tongue has an intense bleeding A. Neutralization
as a result of affection of dorsal lingual B. Precipitation
artery by the tumour. What vessel should C. Complement binding
be ligated to stop bleeding? D. Opsono-phagocytic
E. Double immune diffusion
A. Lingual artery
B. Dorsal lingual artery 102. To prevent postoperative bleeding
C. Deep lingual artery a 6 y.o. child was administered vicasol
D. Facial artery that is a synthetic analogue of vitamin K.
E. Ascending pharyngeal artery Name post-translational changes of blood
coagulation factors that will be activated
98. In course of an experiment a big by vicasol:
number of stem cells of red bone marrow
was in some way destructed. Regenerati- A. Carboxylation of glutamin acid
on of which cell populations in the loose B. Phosphorylation of serine radicals
connective tissue will be inhibited? C. Partial proteolysis
D. Polymerization
A. Of macrophags E. Glycosylation
B. Of fibroblasts
C. Of pigment cells 103. As a result of spinal-cord trauma
D. Of lipocytes a 33 y.o. man has a disturbed pain and
E. Of pericytes temperature sensitivity that is caused by
damage of the following tract:
99. Histological examination of a 40
y.o. man’s thymus revealed decreased A. Spinothalamic
share of parenchymatous gland elements, B. Medial spinocortical
increased share of adipose and loose C. Posterior spinocerebellar
connective tissue, its enrichment with D. Lateral spinocortical
thymus bodies. The organ’s mass was E. Anterior spinocerebellar
unchanged. What phenomenon is it?
104. For the preparation of a patient’s
A. Age involution burn skin surface a certain medication was
B. Accidental involution used. Its antiseptic action is provided by
C. Hypotrophy free oxygen that segregates in presence
D. Dystrophy of organic substances. Choose the right
E. Atrophy answer:
100. Mucous membrane of the right A. Potassium permanganate
palatine tonsil has a painless ulcer wi- B. Furacilin
th smooth lacquer fundus and regular C. Chlorhexidine
cartilagenous edges. Microscopically: D. Boric acid
inflammatory infiltration that consists E. Sodium bicarbonate
of lymphocytes, plasmocytes, a small
number of neutrophils and epithelioid 105. After a 2 y.o. child has had flu, there
cells; endovasculitis and perivasculitis. appeared complaints about ear ache. A
What disease is it? doctor revealed hearing impairment and
inflammation of the middle ear. How did
A. Syphilis the infection penetrate into the middle
B. Actinomycosis ear?
C. Tuberculosis
D. Pharyngeal diphtheria
E. Ulcerous necrotic Vincent’s angina
101. Bacteriological laboratory examines
Krok 1 Medicine 2007 12

A. Through the auditory tube A. Disturbed neutralization of ammonia in


B. Through f oramen jugularis liver
C. Through canalis caroticus B. Disturbed neutralization of biogenic
D. Through atrium mastoideum amines
E. Through canalis nasolacrimalis C. Increased putrefaction of proteins in
bowels
106. A 50 y.o. patient with chronic cardi- D. Activation of aminoacid decarboxylati-
ac insufficiency and tachyarrythmia was on
prescribed a cardiotonic drug. What drug E. Inhibition of transamination enzyms
was prescribed?
110. Examination of a patient revealed
A. Digoxin hyperkaliemia and hyponatremia. Low
B. Dopamine secretion of which hormone may cause
C. Dobutamine such changes?
D. Amyodarone
E. Mildronate A. Aldosteron
B. Vasopressin
107. Microscopical examination of a C. Cortisol
removed appendix revealed an edema, di- D. Parathormone
ffuse neutrophilic infiltration of appendix E. Natriuretic
wall along with necrosis and defect of
mucous membrane with affection of its 111. In course of indirect histogenesis
muscle plate. What appendicitis form was of tubular bone tissue a plate is formed
developed? between epiphyseal and diaphyseal ossi-
fication centres that provides further
A. Ulcerophlegmonous lengthwise growth of bones. What
B. Phlegmonous structure is it?
C. Gangrenous
D. Superficial A. Metaphyseal plate
E. Apostematous B. Osseous cuff
C. Osseous plate
108. A 39 y.o. woman went through an D. Osteon
operation in course of which surgeons E. Layer of interior general plates
removed her uterine tube that was
enlarged and a part of an ovary with a big 112. A 40 y.o. woman was admitted to
cyst. Histological examination of a tube the infectious diseases department wi-
wall revealed decidual cells, chorion vi- th high body temperature. Objectively:
lli. What was the most probable diagnosis evident meningeal symptoms. A spinal
made after examination of the uterine cord punction was made. What anatomic
tube? formation was punctured?
A. Tubal pregnancy A. Spatium subarachnoideum
B. Placental polyp B. Spatium subdurale
C. Choriocarcinoma C. Spatium epidurale
D. Papyraceous fetus D. Cavum trigeminale
E. Lithopedion E. Cisterna cerebellomedullaris posterior
109. A 4 y.o. boy has had recently seri- 113. After resection of the middle third of
ous viral hepatitis. Now there are such femoral artery obliterated by a thromb
clinical presentations as vomiting, loss the lower extremity is supplied with blood
of consciousness, convulsions. Blood due to the surgical bypass. Name an artery
analysis revealed hyperammoniemia. Di- that plays the main role in reestablishment
sturbunce of which biochemical process of blood flow:
caused such pathological condition of the
patient? A. Deep femoral artery
B. Superficial circumflex artery of hip bone
C. Descending genicular artery
D. Superficial epigastric artery
E. Deep external pudendal artery
114. A patient’s knee joint doesn’t extend,
there is no knee-jerk reflex, skin sensi-
tivity of the anterior femoral surface
Krok 1 Medicine 2007 13

is disturbed. What nerve structures are test (Mantoux test) to a child a 10 mm


damaged? papule appeared on the spot of tuberculin
introduction. What hypersensitivity
A. Femoral nerve mechanism underlies these changes?
B. Superior gluteal nerve
C. Big fibular nerve A. Cellular cytotoxicity
D. Obturator nerve B. Anaphylaxis
E. Inferior gluteal nerve C. Antibody-dependent cytotoxicity
D. Immune complex cytotoxicity
115. An electron micrograph of a kidney E. Granulomatosis
fragment presents an afferent arteriole.
Under its endothelium some big cells can 120. Examination of a newborn boy’s
be seen that contain secretory granules. genitals revealed a cleft of urethra that
What type of cells is it? opens on the inferior surface of his penis.
What developmental anomaly is it?
A. Juxtaglomerular
B. Mesangeal A. Hypospadia
C. Smooth muscle cells B. Hermaphroditism
D. Juxtavascular C. Epispadia
E. Interstitial D. Monorchism
E. Cryptorchism
116. For the purpose of retrtospective di-
agnostics of recent bacterial dysentery it 121. A patient has a disturbed absorbtion
was decided to perform serological exami- of fat hydrolysates. It might have been
nation of blood serum in order to determi- caused by a deficit in the small intestine
ne antibody titer towards Shiga bacilli. cavity:
What of the following reactions should
be applied? A. Of bile acids
B. Of bile pigments
A. Passive hemagglutination C. Of lipolytic enzymes
B. Bordet-Gengou test D. Of sodium ions
C. Precipitation E. Of liposoluble vitamins
D. Hemolysis
E. Bacteriolysis 122. Inhabitants of territories with cold
climate have high content of an adapti-
117. Examination of a 43 y.o. anephric ve thermoregulatory hormone. What
patient revealed anemia symptoms. What hormone is meant?
is the cause of these symptoms?
A. Thyroxin
A. Reduced synthesis of erythropoietins B. Insulin
B. Enhanced destruction of erythrocytes C. Glucagon
C. Iron deficit D. Somatotropin
D. Vitamin B12 deficit E. Cortisol
E. Folic acid deficit
123. Violation of safety rules resulted
118. A 50 y.o. patient was admitted to in calomel intoxication. Two days later
the hospital with complaints about pain the daily diuresis was 620 ml. A pati-
behind his breastbone, asphyxia during ent experienced headache, vomiting,
physical activity. Angiography revealed convulsions, dyspnea, moist rales in lungs.
pathological changes in the posteri- What pathology is it?
or interventricular branch of the right
coronary artery. What heart parts are A. Acute renal insufficiency
affected? B. Chronic renal insufficiency
C. Uraemic coma
A. Posterior wall of the right and left D. Glomerulonephritis
ventricles E. Pyelonephritis
B. Left atrium
C. Anterior wall of the right and left 124. A newborn child with pylorostenosis
ventricles has often repeating vomiting accompani-
D. Right atrium ed by apathy, weakness, hypertonicity,
E. Right atrioventricular valve sometimes convulsions. What disorder
form of acid-base balance is it?
119. 48 hours after performing tuberculin
Krok 1 Medicine 2007 14

A. Nongaseous alkalosis of this opening. What anomaly of external


B. Gaseous alkalosis genitals development is the case?
C. Gaseous acidosis
D. Metabolic acidosis A. Epispadia
E. Excretory acidosis B. Phimosis
C. Hermaphroditism
125. Glutamate decarboxylation results D. Paraphimosis
in formation of inhibitory transmitter in E. Hypospadia
CNS. Name it:
131. A patient complains of pain in the
A. GABA area of his liver. Duodenal intubation
B. Glutathione revealed yellowish, oval, narrowed at the
C. Histamine poles eggs with an operculum at the end.
D. Serotonin Size of these eggs is the smallest among all
E. Asparagine helminth eggs. What is the most probable
diagnosis?
126. In course of histidine catabolism a bi-
ogenic amin is formed that has powerful A. Opisthorchosis
vasodilatating effect. Name it: B. Teniasis
C. Beef tapeworm infection
A. Histamine D. Echinococcosis
B. Serotonin E. Diphyllobothriasis
C. Dioxyphenylalanine
D. Noradrenalin 132. Colonoscopy of a patient ill wi-
E. Dopamine th dysentery revealed that mucous
membrane of his large intestine is
127. Utilization of arachidonic acid hyperemic, edematic, its surface was
via cyclooxigenase pathway results in covered with grey-and-green coats. Name
formation of some bioactive substances. the morphological form of dysenteric
Name them: collitis:
A. Prostaglandins A. Fibrinous
B. Thyroxine B. Catarrhal
C. Biogenic amins C. Ulcerous
D. Somatomedins D. Purulent
E. Insulin-like growth factors E. Necrotic
128. As a result of damage to certain 133. 24 hours after appendectomy
structures of brainstem an animal lost ori- blood of a patient presents neutrophi-
entation reflexes. What structures were lic leukocytosis with regenerative shift.
damaged? What is the most probable mechanism of
A. Quadritubercular bodies leukocytosis development?
B. Medial nuclei of reticular formation A. Amplification of leukopoiesis
C. Red nuclei B. Redistribution of leukocytes in the
D. Vestibular nuclei organism
E. Black substance C. Decelerated leukocyte destruction
129. Osmotic pressure of a man’s blood D. Deceleratied emigration of leukocytes
to the tissues
plasma is 350 mosmole/l (standard E. Amplification of leukopoiesis and
pressure is 300 mosmole/l). First of all it decelerated emigration of leukocytes to
will result in high secretion of the followi- the tissues
ng hormone:
134. Arterial pressure of a surgeon who
A. Vasopressin performed a long operation rised up to
B. Aldosteron 140/110 mm Hg. What changes of humoral
C. Cortisol regulation could have caused the rise of
D. Adrenocorticotropin arterial pressure in this case?
E. Natriuretic
130. A young man consulted a doctor
about disturbed urination. Examinati-
on of his external genitals revealed that
urethra is split on top and urine runs out
Krok 1 Medicine 2007 15

A. Activation of sympathoadrenal system A. Lisosomes


B. Activation of formation and excretion B. Mytochondrions
of aldosterone C. Granular endoplasmic reticulum
C. Activation of renin angiotensive system D. Golgi apparatus
D. Activation of kallikrein kinin system E. Ribosomes
E. Inhibition of sympathoadrenal system
140. A 55 y.o. woman consulted a doctor
135. A hypertensive glucose solution was about having continuous cyclic uterine
introduced to a patient. It will intensify hemorrhages for a year, weakness, dizzi-
water movement: ness. Examination revealed skin pallor.
Hemogram: Hb- 70 g/l, erythrocytes -
A. From the cells to the intercellular liquid 3, 2 · 1012 /l, color index - 0,6, leukocytes
B. From the intercellular liquid to the - 6, 0 · 109 /l, reticulocytes - 1%; erythrocyte
capillaries hypochromia. What anemia is it?
C. From the intercellular liquid to the cells
D. From the capillaries to the intercellular A. Chronic posthemorrhagic anemia
liquid B. Hemolytic anemia
E. There will be no changes of water C. Aplastic anemia
movement D. B12 -folate-deficiency anemia
E. Iron-deficiency anemia
136. A 36 y.o. man has a craniocerebral
trauma. Objectively: diminished breath 141. According to audiometry data a pati-
sounds, thready pulse, no reflexes. What ent has a disturbed perception of medium-
way of pyracetam introduction will be the frequency sounds. It might have been
most apropriate in this case? caused by a damage of:
A. Intravenous A. Middle part of helix
B. Rectal B. Cochlear nuclei
C. Subcutaneous C. Spiral ganglion
D. Peroral D. Quadritubercular structure
E. Inhalation E. Lateral geniculate bodies
137. A child complains of general 142. A patient diagnosed with carcinoid
weakness, loss of appetite, a troubled of bowels was admitted to the hospi-
sleep, itching in the perianal area. The tal. Analysis revealed high production of
provisional diagnosis is enterobiasis. serotonin. It is known that this substance
In order to specify this diagnosis it is is formed of tryptophane aminooacid.
necessary to perform: What biochemical mechanism underlies
this process?
A. Scraping from perianal folds
B. Roentgenoscopy A. Decarboxylation
C. Biopsy of muscle tissue B. Desamination
D. Immune diagnostics C. Microsomal oxydation
E. Duodenal contents analysis D. Transamination
E. Formation of paired compounds
138. A patient was ill with burn disease
that was complicated by DIC syndrome. 143. A 23 y.o. patient complains of
What stage of DIC syndrome can be weakness, temperature rise up to 38 −
suspected if it is known that the patient’s 400 C. Objectively: liver and spleen
blood coagulates in less than 3 minutes? are enlarged. Hemogram: Hb- 100 g/l,
A. Hypercoagulation erythrocytes - 2, 9 · 1012 /l, leukocytes -
B. Transition phase 4, 4 · 109 /l, thrombocytes - 48 · 109 /l,
C. Hypocoagulation segmentonuclear neutrophils - 17%,
D. Fibrinolysis lymphocytes - 15%, blast cells - 68%.
E. Terminal All cytochemical reactions are negative.
Make a hematological conclusion:
139. In course of practical training
students studied a stained blood smear A. Undifferentiated leukosis
of a mouse with bacteria phagocyted by B. Chronic myeloleukosis
leukocytes. What cell organella completes C. Acute myeloblastic leukosis
digestion of these bacteria? D. Acute lymphoblastic leukosis
E. Acute erythromyelosis
Krok 1 Medicine 2007 16

144. A patient ill with bronchial asthma di- A. Transamination and decarboxylation of
dn’t inform his doctor that he had attacks aminoacids
of stenocardia. Doctor administered him B. Oxidative decarboxylation of ketonic
a medication, which taking resulted in acids
less frequent attacks of bronchial asthma, C. Desamination of purine nucleotide
but stenocardia attacks became more D. Synthesis of purine and pyrimidine
frequent. What medication was admini- bases
stered? E. Protein synthesis
A. Isadrin 149. In course of an operation surgeon
B. Salbutamol removed a part of a lung that was venti-
C. Aminophylline lated by a tertiary bronchus accompani-
D. Cromolyn sodium ed by branches of pulmonary artery and
E. Phenotherol other vessels. What part of a lung was
removed?
145. A patient who has been sufferi-
ng from cardiac insufficiency for several A. Bronchopulmonary segment
months has been taking digoxin on B. Middle lobe
an outpatient basis. At a certain stage C. Inferior lobe
of treatment there appeared symptoms D. Superior lobe
of drug overdose. What phenomenon E. Pulmonary lobule
underlies the development of this compli-
cation? 150. A patient with clinical presentati-
ons of immunodeficiency went through
A. Material cumulation immunological examinations. They
B. Habituation revealed significant loss of cells that form
C. Sensibilization rosettes with erythrocytes of a ram. What
D. Functional cumulation conclusion can be made according to the
E. Tachyphylaxis analysis data?
146. A patient who has been treated A. Decrease of T-lymphocytes rate
with diazepam on account of neurosis B. Decrease of B-lymphocytes rate
complains of toothache. Doctor admi- C. Decrease of natural killer cell rate
nistered him an analgetic, but its dose D. Decrease of complement system rate
was lower than average therapeutic dose. E. Insufficiency of effector cells of humoral
What phenomenon did the doctor take immunity
into account while prescribing the patient
an underdose? 151. A woman with (B), Rh− blood
group born a child with (A) blood
A. Potentiation group. The child is diagnosed with
B. Summation hemolytic disease of newborn as a result
C. Cumulation of rhesus incompatibility. What blood
D. Drug dependence group is the child’s father likely to have?
E. Tolerance
A. (A), Rh+
147. A child’s blood presents high content B. I (0), Rh+
of galactose, glucose concentration is low. C. III (B), Rh+
There are such presentations as cataract, D. I (0), Rh−
mental deficiency, adipose degeneration E. II (A), Rh−
of liver. What disease is it?
152. A 45 y.o. woman suffers from
A. Galactosemia Cushing’s syndrome - steroid diabetes.
B. Diabetes mellitus Biochemical examination revealed:
C. Lactosemia hyperglycemia, hypochloremia. Which of
D. Steroid diabetes the under-mentioned processes is the first
E. Fructosemia to be activated?
148. According to clinical indications A. Gluconeogenesis
a patient was administered pyridoxal B. Glycogenolysis
phosphate. What processes is this medi- C. Glucose reabsorption
cation intended to correct? D. Glucose transport to the cell
E. Glycolysis
Krok 1 Medicine 2007 17

153. Histological specimen presents a A. Increase of ESR


vessel the wall of which consists of B. Decrease of ESR
endothelium, basal membrane and loose C. Increase of hematocrit
connective tissue. What type of vessel is D. Decrease of hematocrit
it? E. Hypercoagulation
A. Vein of non-muscular type 158. An experimental animal was fi-
B. Artery rst sensibilized whereupon an antigen
C. Vein of muscular type dose was introduced subcutaneously. This
D. Hemocapillary injection resulted in the development of
E. Lymphocapillary a fibrinous inflammation with alteration
of vessel walls, basal substance and fi-
154. Autopsy of a 48 y.o. man revealed a brous structures of connective tissue in
round formation 5 cm in diameter with form of mucoid and fibrinoid swelling and
clear-cut outlines in the region of the 1st necrosis. What immunological reaction
segment of his right lung. This formation took place?
was encircled with a thin layer of connecti-
ve tissue full of white brittle masses. Make A. Immediate hypersensitivity
a diagnosis of the secondary tuberculosis B. Delayed-type hypersensitivity
form: C. Reaction of transplantation immunity
D. Normergic reaction
A. Tuberculoma E. Granulomatosis
B. Caseous pneumonia
C. Acute cavernous tuberculosis 159. A patient suffers from vision impai-
D. Acute focal tuberculosis rment - hemeralopy (night blindness).
E. Fibrous cavernous tuberculosis What vitamin preparation should be
administered the patient in order to
155. A patient who suffers from restore his vision?
acute myocarditis has clinical signs
of cardiogenic shock. What of the A. Retinol acetate
under-mentioned pathogenetic mechani- B. Vicasol
sms plays the main part in shock C. Pyridoxine
development? D. Thiamine chloride
E. Tocopherol acetate
A. Disturbance of pumping ability of heart
B. Depositing of blood in organs 160. A 1 y.o. child with symptoms of
C. Reduction of diastolic flow to the heart muscle affection was admitted to the
D. Decrease of vascular tone hospital. Examination revealed carnitine
E. Increase of peripheral vascular resi- deficit in muscles. Biochemical base of this
stance pathology is disturbed process of:
156. On the 6th day of treatment a patient A. Transporting of fatty acids to mi-
with acute renal insufficiency developed tochondrions
polyuria. Diuresis intensification at the B. Regulation of Ca2+ rate in mitochondri-
beginning of polyuria stage of acute renal ons
insufficiency is caused by: C. Substrate phosphorylation
D. Lactic acid utilization
A. Renewal of filtration in nephrons E. Actin and myosin synthesis
B. Volume expansion of circulating blood
C. Growth of natriuretic factor 161. A patient suffers from severe
D. Reduction of aldosteron content in postoperative pseudomonadous infection.
plasma What of the following antibiotics should
E. Reduction of vasopressin content in be administered in this case?
plasma
A. Amicacin sulfate
157. Long-term starvation cure of a pati- B. Benzylpenicillin
ent resulted in diminished ratio of albumi- C. Cephazolin
nes and globulines in plasma. What of the D. Erythromycin
following will be result of these changes? E. Doxycycline
162. A 9 m.o. child has delayed dentition,
it is also out of order. Upper jaw confi-
guration is horizontal ("high"palate); mi-
Krok 1 Medicine 2007 18

croscopically - irregular mineralization of Blood analysis showed full-blown eosi-


tooth enamel, wrinkled enamel prisms, nophilia. What helminth could the patient
some of them are vacuolized. Predentin be infected with?
zone is extended; there are solitary denti-
cles. What disease is it? A. Trichina
B. Pinworm
A. Early rickets C. Ascarid
B. Late rickets D. Whipworm
C. Osteomalacia E. Hookworm
D. Gout
E. Hypervitaminosis D 167. After a tooth extraction a patient felt
persistent pain behind his breast bone.
163. Examination of a patient revealed an After sublingual intake of an antianginal
abscess of pterygopalatine fossa. Where drug the pain behind the breast bone di-
can the infection spread to unless the di- sappeared, but the patient complained of
sease is managed in time? headache and dizziness. What drug are
these properties typical for?
A. To the orbit
B. To the interpterygoid space A. Nitroglycerin
C. To the frontal sinus B. Propranolol
D. To the subgaleal temporal space C. Metoprolol
E. To the tympanic cavity D. Validol
E. Verapamil
164. Microscopical renal examination of
a 36 y.o. woman who died from renal 168. A patient with fracture of his lower
insufficiency revealed in the glomerules jaw was admitted to the maxillofaci-
proliferation of capsule nephrothelium al department. It was decided to fix
as well as of podocytes and phagocytes his bones surgically under anaesthetic.
accompanied by formation of "crescents", After intravenous introduction of muscle
capillary loop necrosis, fibrinous thrombs relaxant there arose short fibrillar
in their lumens; sclerosis and hyalinosis contractions of the patient’s facial
of glomerules, atrophy of tubules and fi- muscles. What muscle relaxant was appli-
brosis of renal stroma. What is the most ed?
probable diagnosis?
A. Dithylinum
A. Subacute glomerulonephritis B. Tubocurarin chloride
B. Acute glomerulonephritis C. Pipecuronium bromide
C. Chronic glomerulonephritis D. Diazepam
D. Focal segmentary sclerosis E. Melictine
E. Membranous nephropathy
169. Autopsy of a 56 y.o. man revealed in
165. A forensic medical expert exami- the right temporal part of brain a big focus
nes the body of a 58 y.o. man who of softened grey matter that was semi-
had been consuming large amounts of liquid and light grey. Arteries of cerebral
alcochol for a long time and died at home. tela contain multiple whitish-yellow thi-
Microscopicaly: the right lung is dense ckenings of intima that abruptly narrow
and enlarged, its incision revealed that the lumen. What is your diagnosis?
the tissue is greyish and homogenous,
pleura is covered with greyish layers. Mi- A. Ischemic stroke
croscopically - alveolar cavities contain B. Brain abscess
fibrin, hemolyzed erythrocytes. Make a C. Hemorrhage
diagnosis: D. Hemorrhagic infarction
E. Brain edema
A. Croupous pneumonia
B. Focal pneumonia 170. Vitamin A deficit results in the
C. Interstitial pneumonia impairment of twilight vision. Name the
D. Primary pulmonary tuberculosis cells that have the above-mentioned
E. Caseous pneumonia photoreceptor function:
166. Two days after consumption of
smoked pork a patient got face and eye-
lid edemata, gastrointestinal disturbances,
abrupt temperature rise, muscle pain.
Krok 1 Medicine 2007 19

A. Rod receptor cell cells of male sexual glands produce this


B. Horizontal neurocytes hormone?
C. Cone receptor cells
D. Bipolar neurons A. Leidig cells
E. Ganglion neurocytes B. Sustentocytes
C. Sertoli’s cells
171. Neurological examination of a 65 y.o. D. Supporting cells
patient revealed a haemorrhage within E. Spermatozoa
the superior temporal gyrus. In the blood
supply area of which artery is it? 176. An isolated cell of human heart
automatically generates excitation
A. Middle cerebral artery impulses with frequency 60 times pro mi-
B. Anterior cerebral artery nute. What heart structure was this cell
C. Posterior cerebral artery obtained from?
D. Anterior communicating artery
E. Basilar artery A. Sinoatrial node
B. Atrium
172. A 70 y.o. man has cut an abscess C. Ventricle
off in the area of mammiform process D. Atrioventricular node
during shaving. Two days later he was E. His’ bundle
admitted to the hospital with inflammati-
on of arachnoid membranes. How did 177. Examination of a patient revealed
the infection penetrate into the cavity of a strong, balanced, inert type of higher
skull? nervous activity according to Pavlov. What
temperament type does the patient have
A. V.emissaria mastoidea (according to Hippocrates classification)?
B. V.v.labyrinthi
C. V.v.tympanicae A. Phlegmatic
D. V.f acialis B. Sanguine
E. V.v.auriculares C. Choleric
D. Melancholic
173. A 22 y.o. woman has enlarged lymph E. -
nodes. Histologically: a lymph node
contains lymphocytes, histiocytes, reti- 178. A patient has a haemorrhage into
cular cells, small and big Hodgkin’s cells, the posterior central gyrus. What type of
multinucleated Sternberg cells, isolated sensitivity on the opposite side will be di-
foci of caseous necrosis. What disease are sturbed?
these changes typical for?
A. Skin and proprioceptive
A. Lymphogranulomatosis B. Visual
B. Lymphosarcoma C. Auditory
C. Chronic leukosis D. Olfactory
D. Acute leukosis E. Auditory and visual
E. Lung cancer metastasis
179. Analysis of a punction biopsy materi-
174. A lymph node punctate of a pati- al of liver revealed hepatocyte dystrophy
ent with suspected protozoal disease was with necroses as well as sclerosis with di-
examined. Examination of the stained sorder of beam and lobulous structure,
specimen (Romanovsky’s stain) revealed with formation of pseudolobules and
some crescent bodies with pointed end, regenerative nodes. What is the most
blue cytoplasm and red nucleus. What probable diagnosis:
protozoan were revealed in the smears?
A. Liver cirrhosis
A. Toxoplasms B. Chronic hepatosis
B. Malarial plasmodiums C. Chronic hepatitis
C. Dermotropic leishmania D. Progressive massive liver necrosis
D. Viscerotropic leishmania E. Acute hepatitis
E. Trypanosomes
180. Examination of a 60 y.o. patient
175. During pubescence the cells of revealed hyperglycemia and glucosuria.
male sexual glands begin to produce A doctor administered him a medication
male sex hormon testosterone that calls for internal use. What medication is it?
forth secondary sexual characters. What
Krok 1 Medicine 2007 20

A. Glibenclamid of basophilic cytoplasma that resemble


B. Furosemide of cells of basal epidermal layer. What
C. Oxytocin tumour is it?
D. Pancreatine
E. Corglycon A. Basalioma
B. Epidermal cancer
181. A patient who had been working hard C. Hydradenoma
under conditions of elevated temperature D. Trichoepithelioma
of the environment, has now a changed E. Syringoadenoma
quantity of blood plasma proteins. What
penomenon is the case? 186. A patient has pain, edema and
reddening of his skin in the anterosuperi-
A. Relative hyperproteinemia or area of his thigh and his foot’s thumb.
B. Absolute hyperproteinemia What lymph nodes of his lower extremity
C. Absolute hypoproteinemia responded to the inflammatory process?
D. Disproteinemia
E. Paraproteinemia A. Superficial inguinal
B. Deep inguinal
182. An experimental rat with extremity C. Internal longitudinal
paralysis has no tendon and cutaneous D. Superficial longitudinal
reflexes, muscle tone is decreased, but E. General longitudinal
muscles of the affected extremity mai-
ntain their ability to react with excitation 187. A laboratory received a material from
to the direct action of continious current. a patient’s wound. Ppreliminary diagnosis
What type of paralysis is it? is gaseous gangrene. What microbiologi-
cal method should be applied to determi-
A. Flaccid peripheral ne species of causative agent?
B. Flaccid central
C. Spastic peripheral A. Bacteriological
D. Spastic central B. Allergic
E. Extrapyramidal C. Bacterioscopic
D. Serological
183. A liquidator of a breakdown at a E. RIA
nuclear power plant who was irradiated
complained about vomiting that occurs all 188. Examination of cell culture got
of a sudden. What medication should be from a patient with lysosomal pathology
prescribed? revealed accumulation of great quantity
of lipids in the lysosomes. What of the
A. Metoclopramide following diseases is this disturbance typi-
B. Reserpine cal for?
C. Atropine
D. Aeron A. Tay-Sachs disease
E. De-Nol B. Gout
C. Phenylketonuria
184. Urine examination of a patient with D. Wilson disease
acute cystitis revealed leukocytes and a E. Galactosemia
lot of gram-negative bacilli. Inoculation
resulted in growth of colonies of mucous 189. A patient got a craniocerebral
nature that formed green soluble pigment. trauma that resulted in right-side
What microorganism is the most probable convergent strabismus. Damage of whi-
cause of the disease? ch craniocerebral nerve caused such
consequences?
A. P seudomonas aeruginosa
B. Escherihia coli A. n.abducens
C. Klebsiella pneumoniae B. n.f acialis
D. P roteus mirabilis C. n.trigeminus
E. Salmonella enteritidis D. n.trochlearis
E. n.aculomotorius
185. A 45 y.o. patient consulted a doctor
about plaque-shaped formation on his 190. In case of a penetrating wound of
neck. Histological examination of biopsy the anterior abdominal wall the wound
skin material revealed tumourous cells tract went above the lesser curvature of
of round and oval form with thin ring stomach. What peritoneum formation is
Krok 1 Medicine 2007 21

most likely to be injured? A. Serous diffuse


B. Interstitial proliferative
A. Ligamentum hepatogastricum C. Serous focal
B. Ligamentum gastrocolicum D. Purulent
C. Ligamentum hepatoduoduodenale E. Granulomatous
D. Ligamentum hepatorenale
E. Ligamentum triangulare sinistrum 194. A patient with chronic cardiac insuffi-
ciency has been treated with cardiotonic
191. A rabbit’s nerve that innervates the drugs and a thiazide diuretic, but in spi-
right ear was cut and its right superior te of it there are still edemata and risk
cervical ganglion was removed. Immedi- of ascites. What medication should be
ately after operation the temperature of prescribed to amplify diuretic effect of the
ear skin was measured. It was revealed applied drugs?
that the temperature of the rabbit’s ear
skin on the side of denervation was A. Spironolactone
by 1, 50 C higher than on the opposite B. Furosemide
intact side. What of the following is the C. Amyloride
most probable explanation of the above- D. Clopamide
mentioned effects? E. Manitole
A. Arterial neuroparalytic hyperemia 195. A patient ill with collagenesis has
B. Arterial neurotopical hyperemia been taking prednisolone for a long ti-
C. Atrerial hyperemia induced by me. Hypokaliemia development caused
metabolic factors spastic pain of skeletal muscles. What
D. Reactive arterial hyperemia medication should be used in order to
E. Physiological arterial hyperemia correct potassium exchange?
192. A 63 y.o. man fell ill with acute A. Panangin
tracheitis and bronchitis accompanied by B. Dithylinum
bronchial pneumonia. On the 10th day C. Diazepam
the patient died from cardiopulmonary D. Noshpa
insufficiency. Autopsy revealed fibrinous E. Thyrocalcitonin
hemorrhagic laryngotracheobronchi-
tis; lungs were enlarged, their incision 196. A boy is 7 y.o. Objectively: against
revealed the "coal-miner’s"effect caused the background of hyperemic skin there is
by interlacing of sections of bronchi- knobby bright-pink rash on his forehead,
al pneumonia, hemorrhages into the neck, at the bottom of abdomen, in
pulmonary parenchyma, acute abscesses the popliteal spaces; nasolabial triangle
and atelectases. Internal organs have di- is pale. Examination of oropharyngeal
scirculatory and dystrophic changes. What surface revealed localized bright-red
is the most probable diagnosis? hyperemia; tonsils are swollen, soft,
lacunas contain pus, tongue is crimson.
A. Influenza, severe form Cervical lymph nodes are enlarged, dense
B. Moderately severe influenza and painful. What is the most probable di-
C. Parainfluenza agnosis?
D. Respiratory syncytial infection
E. Adenoviral infection A. Scarlet fever
B. Rubella
193. Autopsy of a man who died from C. Whooping cough
influenza revealed that his heart was sli- D. Diphtheria
ghtly enlarged, pastous, myocardium was E. Infectious mononucleosis
dull and had specks. Microscopical exami-
nation of myocardium revealed signs of 197. A patient complains of dryness of
parenchymatous adipose and hydropic head skin, itching, fragility and loss of hair.
dystrophy; stroma was edematic with poor After examination he was diagnosed wi-
macrophagal and lymphocytic infiltrati- th seborrhea. Disturbed activity of which
on, vessels were plethoric; perivascular cells caused this condition?
analysis revealed petechial hemorrhages.
What type of myocarditis was developed
in this case?
Krok 1 Medicine 2007 22

A. Cells of sebaceous glands in reflex amplification of heart rate and


B. Cells of sudoriferous glands raise of systemic arterial pressure. What
C. Epithelial cells receptors activation was the main cause of
D. Adipocytes pressor reflex realization?
E. Melanocytes
A. Proprioreceptors of active muscles
198. In the surgical department of a B. Vascular chemoreceptors
hospital there was an outbreak of hospi- C. Vascular volume receptors
tal infection that showed itself in often D. Vascular baroceptors
postoperative wound abscesses. Bacteri- E. Hypothalamus thermoreceptors
ological examination of pus revealed
aurococcus. What examination shall be 200. In course of an experiment a skeletal
conducted to find out the source of this muscle is being stimulated by a series of
causative agent among the department electric impulses. What type of muscle
personnel? contraction will arise, if every subsequent
impulse comes in the period of relaxation
A. Phagotyping of single muscle contraction?
B. Microscopical examination
C. Serological identification A. Partial tetanus
D. Estimation of antibiotic susceptibility B. Holotetanus
E. Biochemical identification C. A series of single contractions
D. Muscle contructure
199. Short-term physical activity resulted E. Asynchronous tetanus
Krok 1 Medicine 2008 1

1. After consumption of rich food 6. A 37 year old patient suffering from


a patient has nausea and heartburn, obliterating vascular endarteritis of lower
steatorrhea. This condition might be limbs takes daily 60 microgram/kilogram
caused by: of phenylin. Because of presentations
of convulsive disorder (craniocerebral
A. Bile acid deficiency trauma in anamnesis) he was prescri-
B. Increased lipase secretion bed phenobarbital. Withholding this drug
C. Disturbed tripsin synthesis caused nasal hemorhage. What is this
D. Amylase deficiency complication connected with?
E. Disturbed phospholipase synthesis
A. Induction of enzymes of microsomal
2. An infant has apparent diarrhea resulti- oxidation in liver caused by phenobarbital
ng from improper feeding. One of the B. Aliphatic hydroxylation of phenobarbi-
main diarrhea effects is plentiful excreti- tal
on of sodium bicarbonate. What form of C. Conjugation of phenylin with glucuronic
acid-base balance disorder is the case? acid
A. Metabolic acidosis D. Oxidative deamination of phenylin
B. Metabolic alkalosis E. Inhibition of microsomal oxidation in
C. Respiratory acidosis liver caused by phenobarbital
D. Respiratory alkalosis 7. A 60 year old patient has impaired
E. No disorders of acid-base balance will perception of high-frequency sounds.
be observed These changes were caused by damage of
3. A 3 year old child with fever was the following auditory analyzer structures:
given aspirin. It resulted in intensifi- A. Main cochlea membrane near the oval
ed erythrocyte haemolysis. Hemolytic window
anemia might have been caused by B. Main cochlea membrane near the
congenital insufficiency of the following helicotrema
enzyme: C. Eustachian tube
A. Glucose 6-phosphate dehydrogenase D. Middle ear muscles
B. Glucose 6-phosphatase E. Tympanic membrane
C. Glycogen phosphorylase 8. Researchers isolated 5 isoenzymic
D. Glycerol phosphate dehydrogenase forms of lactate dehydrogenase from the
E. γ-glutamiltransferase human blood serum and studied their
4. A patient is ill with diabetes melli- properties. What property indicates that
tus accompanied by hyperglycemia on the isoenzymic forms were isolated from
the same enzyme?
an empty stomach (7,2 millimole/l). The
hyperglycemia rate can be retrospecti- A. Catalyzation of the same reaction
vely estimated (over the last 4-8 weeks B. The same molecular weight
before the examination) on the ground C. The same physicochemical properties
of the rate of the following blood plasma D. Tissue localization
protein: E. The same electrophoretic mobility
A. Glycated hemoglobin 9. It was proved that a molecule of
B. Albumin immature mRNA (precursor mRNA)
C. Fibrinogen contained more triplets than amino aci-
D. C-reactive protein ds found in the synthesized protein.
E. Ceruloplasmin The reason for that is that translation is
normally preceded by:
5. In course of laparotomy a surgeon
revealed gangrenous lesion of descendi- A. Processing
ng colon. It was caused by thrombosis of B. Initiation
the following artery: C. Reparation
D. Mutation
A. Sinister colic E. Replication
B. Median colic
C. Dexter colic 10. A patient has low rate of magnesium
D. Ileocolic ions that are necessary for affixion of ri-
E. Superior mesenteric artery bosomes to the endoplasmic reticulum.
It is known that it causes disturbance
Krok 1 Medicine 2008 2

of protein biosynthesis. At what stage is (purulent otitis media) was complicated


protein biosynthesis impaired? by inflammation of mammillary process
sockets. What wall of tympanic cavity
A. Translation did the pus penetrate into the sockets
B. Transcription through?
C. Replication
D. Amino acid activation A. Posterior
E. Termination B. Anterior
C. Medial
11. A student takes notes of a lecture. D. Lateral
Quality of his notes became significantly E. Superior
worse when his neighbours began talking.
What type of conditional reflex inhibition 16. In order to speed up healing of a
was the cause of it? wound of oral mucosa a patient was
prescribed a drug that is a thermostable
A. External protein occuring in tears, saliva, mother’s
B. Protective milk as well as in a new-laid hen’s egg. It
C. Extinctive is known that this protein is a factor of
D. Differentiated natural resistance of an organism. What is
E. Delayed it called?
12. A patient ill with neurodermatitis has A. Lysozyme
been taking prednisolone for a long time. B. Complement
Examination revealed high rate of sugar C. Interferon
in his blood. This complication is caused D. Interleukin
by the drug influence upon the following E. Imanine
link of carbohydrate metabolism:
17. An aged man had raise of arterial
A. Gluconeogenesis activation pressure under a stress. It was caused by
B. Glycogenogenesis activation activation of:
C. Intensification of glucose absorption in
the bowels A. Sympathoadrenal system
D. Inhibition of glycogen synthesis B. Parasympathetic nucleus of vagus
E. Activation of insulin decomposition C. Functions of thyroid gland
D. Functions of adrenal cortex
13. Labelled amino acids alanine and E. Hypophysis function
tryptophane were injected to a mouse
in order to study localization of protein 18. A 20 year old patient complains of
synthesis in its cells. The labelled ami- general weakness, dizziness, quick fati-
no acids will be accumulated near the gability. Blood analysis results: Hb- 80
following organellas: g/l. Microscopical examination results:
erythrocytes are of modified form. This
A. Ribosomes condition might be caused by:
B. Smooth endoplasmic reticulum
C. Cell centre A. Sickle-cell anemia
D. Lysosomes B. Hepatocellular jaundice
E. Golgi apparatus C. Acute intermittent porphyria
D. Obturative jaundice
14. A patient with frequent attacks of E. Addison’s disease
stenocardia was prescribed sustak-forte
to be taken one tablet twice a day. At first 19. Blood minute volume of a 30 year old
the effect was positive but on the second woman at rest is 5 l/m. What blood volume
day stenocardia attacks resumed. What is pumped through the pulmonary vessels
can explain inefficiency of the prescribed per minute?
drug?
A. 5 l
A. Tachyphylaxis B. 3,75 l
B. Cumulation C. 2,5 l
C. Sensibilization D. 2,0 l
D. Idiosyncrasy E. 1,5 l
E. Dependence
20. As a result of long-term starvation
15. Inflammation of the tympanic cavity the glomerular filtration of a man was
Krok 1 Medicine 2008 3

accelerated by 20%. The most probable indicated by depression of the following


cause of filtration changes under such ECG element:
conditions is:
A. R − R interval
A. Fall of oncotic pressure of blood plasma B. P − Q segment
B. Rise of systemic arterial pressure C. P − Q interval
C. Increased permeability of renal filter D. P − T interval
D. Growth of filtration coefficient E. QRS complex
E. Increase of renal plasma flow
26. A 62 year woman complai-
21. A patient has yellow skin colour, dark ns of frequent pain attacks in the
urine, dark-yellow feces. What substance area of her chest and backbone, rib
will have strengthened concentration in fractures. Her doctor suspected myeloma
the blood serum? (plasmocytoma). What of the following
laboratory characteristics will be of the
A. Unconjugated bilirubin greatest diagnostic importance?
B. Conjugated bilirubin
C. Mesobilirubin A. Paraproteinemia
D. Verdoglobin B. Hyperalbuminemia
E. Biliverdin C. Proteinuria
D. Hypoglobulinemia
22. A patient consulted a doctor about E. Hypoproteinemia
symmetric dermatitis of open skin areas.
It was found out that the patient lived 27. While playing volleyball a sportsman
mostly on cereals and ate too little meat, made a jump and landed on the outside
milk and eggs. What vitamin deficiency is edge of his foot. He felt acute pain in the
the most evident? talocrural joint, active movements are li-
mited, passive movements are unlimited
A. Nicotinamide but painful. A bit later there appeared a
B. Calciferol swelling in the area of external ankle, the
C. Folic acid skin became red and warm. What type of
D. Biotin peripheral circulation disturbance is the
E. Tocopherol case?
23. A 46 year old patient applied to a A. Arterial hyperemia
doctor complaining about joint pain that B. Stasis
becomes stronger the day before weather C. Embolism
changes. Blood examination revealed D. Venous hyperemia
strengthened concentration of uric acid. E. Thrombosis
The most probable cause of the disease
is the intensified disintegration of the 28. Bacterioscopic examination of a smear
following substance: from the pharynx of a diphtheria suspect
revealed bacilli with volutine granules.
A. Adenosine monophosphate What etiotropic drug should be chosen
B. Cytidine monophosphate in this case?
C. Uridine triphosphate
D. Uridine monophosphate A. Antidiphtheritic antitoxic serum
E. Thymidine monophosphate B. Bacteriophage
C. Diphtheritic anatoxin
24. A 38 year old patient suffers from D. Eubiotic
rheumatism in its active phase. What E. Interferon
laboratory characteristic of blood serum
is of diagnostic importance in case of this 29. A patient underwent an operation
pathology? on account of gall bladder excision that
resulted in obstruction of Ca absorption
A. C-reactive protein through the bowels wall. What vitamin
B. Uric acid wil stimulate this process?
C. Urea
D. Creatinine
E. Transferrin
25. ECG of a patient with hyperfunction
of thyroid gland showed heart hurry. It is
Krok 1 Medicine 2008 4

A. D3 34. A 65 year old man suffering from gout


B. P P complains of kidney pain. Ultrasound
C. C examination revealed renal calculi. The
D. B12 most probable cause of calculi formation
E. K is the strengthened concentration of the
following substance:
30. A woman underwent an operation on
account of extrauterine (tubal) pregnancy. A. Uric acid
In course of the operation the surgeon B. Cholesterol
should ligate the branches of the followi- C. Bilirubin
ng arteries: D. Urea
E. Cystine
A. Uterine and ovarian
B. Superior cystic and ovarian 35. Microscopic examination of a Gram-
C. Inferior cystic and ovarian stained scrape from patient’s tongue
D. Uterine and superior cystic revealed oval, round, elongated chains of
E. Uterine and inferior cystic dark-violet gemmating cells. What disease
can be caused by this causative agent?
31. A 6 month old baby ill with bronchitis
was taken for an X-ray of chest. Apart of A. Candidosis
changes associated with bronchi the X-ray B. Actinomycosis
film showed a shadow of thymus gland. C. Streptococcic infection
What might have caused such changes? D. Staphylococcic infection
E. Diphtheria
A. The above-mentioned condition is a
normal variant for this age 36. A 44 year old woman complains of
B. It’s the effect of bronchitis general weakness, heart pain, significant
C. It is caused by abnormal position increase of body weight. Objectively:
D. It is caused by thymus inflammation moon face, hirsutism, AP is 165/100 mm
E. It is caused by neoplastic process Hg, height - 164 cm, weight - 103 kg; the
fat is mostly accumulated on her neck,
32. A patient complains of frequent di- thoracic girdle, belly. What is the main
arrheas, especially after consumption of pathogenetic mechanism of obesity?
rich food, weight loss. Laboratory exami-
nation revealed steatorrhea; his feces A. Increased production of glucocorticoids
were hypocholic. What might have caused B. Reduced production of thyroid
such condition? hormones
C. Increased insulin production
A. Obturation of biliary tracts D. Reduced glucagon production
B. Inflammation of mucous membrane of E. Increased mineralocorticoid production
small intestine
C. Lack of pancreatic lipase 37. Blood of a patient with presumable
D. Lack of pancreatic phospholipase sepsis was inoculated into sugar broth.
E. Unbalanced diet There appeared bottom sediment.
Repeated inoculation into blood agar
33. Examination of a patient with pustular caused growth of small transparent round
skin lesions allowed to isolate a causati- colonies surrounded by hemolysis zone.
ve agent that forms in the blood agar Examination of a smear from the sedi-
roundish yellow middle-sized colonies ment revealed gram-positive cocci in form
surrounded by haemolysis zone. Smears of long chains. What microorganisms are
from the colonies contain irregular- present in blood of this patient?
shaped clusters of gram-positive cocci.
The culture is oxidase- and catalase- A. Streptococci
positive, ferments mannitol and synthesi- B. Micrococci
zes plasmocoagulase. What causative C. Staphylococci
agent was isolated? D. Tetracocci
E. Sarcina
A. Staphylococcus aureus
B. Streptococcus agalactiae 38. 2 years ago a patient underwent
C. Streptococcus pyogenes resection of pyloric part of stomach. He
D. Staphylococcus epidermidis complains of weakness, periodical dark
E. Staphylococcus saprophyticus shadows beneath his eyes, dyspnea. In
blood: Hb - 70 g/l, erythrocytes - 3, 0 ·
Krok 1 Medicine 2008 5

1012 /l, colour index - 0,7. What changes A. Acid resistance


of erythrocytes in blood smears are the B. Alkali resistance
most typical for this condition? C. Alcohol resistance
D. Capsule formation
A. Microcytes E. Sporification
B. Megalocytes
C. Schizocytes 43. Parents of a 10 year old boy consulted
D. Ovalocytes a doctor about extension of hair-covering,
E. Macrocytes growth of beard and moustache, low voi-
ce. Intensified secretion of which hormone
39. A patient is ill with hepatocirrhosis. must be assumed?
State of antitoxic liver function can be
characterized by examination of the A. Of testosterone
following substance exreted by urine: B. Of somatotropin
C. Of oestrogen
A. Hippuric acid D. Of progesterone
B. Ammonium salts E. Of cortisol
C. Creatinine
D. Uric acid 44. Examination of coronary arteries
E. Amino acids revealed atherosclerotic calcified plaques
closing vessel lumen by 1/3. The muscle
40. The territory of an old burial ground has multiple whitish layers of connective
for animal refuse that hasn’t been used for tissue. What process was revealed in the
over 50 years is meant for house building. myocardium?
But soil investigation showed the presense
of viable spores of a causative agent causi- A. Diffusive cardiosclerosis
ng a very dangerous disease. What mi- B. Tiger heart
croorganism might have been preserved C. Postinfarction cardiosclerosis
in soil for such a long period of time? D. Myocarditis
E. Myocardium infarction
A. Bacillus anthracis
B. Francisella tularensis 45. It was revealed that T-lymphocytes
C. Brucella abortus were affected by HIV. Virus enzyme -
D. Yersinia pestis reverse transcriptase (RNA-dependent
E. Mycobacterium bovis DNA polymerase) - catalyzes the
synthesis of:
41. Autopsy of a man who had been
working as a miner for many years and di- A. DNA on the matrix of virus mRNA
ed from cardiopulmonary decompensati- B. Virus informational RNA on the matrix
on revealed that his lungs were ai- of DNA
rless, sclerosed, their apexex had C. DNA on virus ribosomal RNA
emphysematous changes, the lung surface D. Viral DNA on DNA matrix
was greyish-black, the incised lung ti- E. mRNA on the matrix of virus protein
ssue was coal-black. What disease caused
death? 46. A 35 year old man with a trauma
of his left hand was admitted to the
A. Anthracosis traumatology department. Objectively:
B. Silicosis cut wound of palmar surface of left hand;
C. Talcosis middle phalanxes of II–V fingers don’t
D. Asbestosis bend. What muscles are damaged?
E. Aluminosis
A. Superficial finger flexor
42. Microscopy of stained (Ziehl-Neelsen B. Profound finger flexor
staining) smears taken from the sputum C. Lumbrical muscles
of a patient with chronic pulmonary di- D. Palmar interosseous muscles
sease revealed red bacilli. What property E. Dorsal interosseous muscles
of tuberculous bacillus was shown up?
47. Histological examination of a skin
tissue sampling revealed granulomas
consisting of macrophagal nodules with
lymphocytes and plasmatic cells. There
are also some big macrophages with fatty
vacuoles containing causative agents of
Krok 1 Medicine 2008 6

a disease packed up in form of spheres


(Virchow’s cells). Granulation tissue is A. To repeat the examination with serum
well vascularized. What disease is this taken 10 days later
granuloma typical for? B. To examine the same serum
C. To apply more sensitive reaction
A. Lepra D. To repeat examination with another
B. Tuberculosis diagnosticum
C. Syphilis E. To deny diagnosis of tick-borne
D. Rhinoscleroma encephalitis
E. Glanders
52. A 40 year old man noticed a reddeni-
48. In order to estimate toxigenity of di- ng and an edema of skin in the area
phtheria agents obtained from patients of his neck that later developed into a
the cultures were inoculated on Petri dish small abscess. The incised focus is dense,
with nutrient agar on either side of a filter yellowish-green. The pus contains whi-
paper strip that was put into the centre te granules. Histological examination
and moistened with antidiphtheric anti- revealed drusen of a fungus, plasmatic and
toxic serum. After incubation of inoculati- xanthome cells, macrophages. What type
ons in agar the strip-like areas of medium of mycosis is the most probable?
turbidity were found between separate
cultures and the strip of filter paper. What A. Actinomycosis
immunological reaction was conducted? B. Aspergillosis
C. Candidosis
A. Precipitation gel reaction D. Sporotrichosis
B. Coomb’s test E. Coccidioidomycosis
C. Agglutination reaction
D. Rings precipitation reaction 53. Continuous taking of some drugs
E. Opsonization reaction foregoing the pregnancy increase the ri-
sk of giving birth to a child with genetic
49. A patient taking clonidine for essenti- defects. What is this effect called?
al hypertension treatment was using
alcohol that caused intense inhibition of A. Mutagenic effect
central nervous system. What may it be B. Embryotoxic effect
connected with? C. Teratogenic effect
D. Fetotoxical effect
A. Effect potentiating E. Blastomogenic effect
B. Effect summation
C. Cumulation 54. A man with a wound of his limb that
D. Intoxication had been suppurating for a long time di-
E. Idiosyncrasy ed from intioxication. Autopsy revealed
extreme emaciation, dehydration, brown
50. Examination of an ovary specimen atrophy of liver, myocardium, spleen and
stained by hematoxylin-eosine revealed cross-striated muscles as well as renal
a follicle in which follicular epithelium amyloidosis. What diagnosis corresponds
consisted of 1-2 layers of cubic cells. There with the described picture?
was also a bright red membrane around
the ovocyte. What follicle is it? A. Chroniosepsis
B. Septicopyemia
A. Primary C. Septicemia
B. Primordial D. Chernogubov’s syndrome
C. Secondary E. Brucellosis
D. Mature
E. Atretic 55. 6 months after labour a woman
had uterine hemorrhage. Gynaecologi-
51. A patient with clinical signs of cal examination of uterine cavity revealed
encephalitis was delivered to the infecti- a dark-red tissue with multiple cavities
ous diseases hospital. Anamnesis registers resembling of a "sponge". Microscopic
a tick bite. Hemagglutination-inhibition examination of a tumour revealed in
reaction helped to reveal antibodies to the blood lacunas atypic light epithelial
causative agent of tick-borne encephalitis Langhans cells and giant cells of syncyti-
in the dilution 1:20 which is not diagnostic. otrophoblast. What tumour is it?
What actions should the doctor take after
he had got such result?
Krok 1 Medicine 2008 7

A. Chorioepithelioma 61. A patient suffering from chronic


B. Squamous cell nonkeratinous carci- cardiac insufficiency was recommended
noma to undergo a prophylactic course of
C. Adenocarcinoma treatment with a cardiological drug from
D. Fibromyoma the group of cardiac glycosides that is
E. Cystic mole to be taken enterally. What drug was
recommended?
56. An animal with aortic valve insuffi-
ciency got hypertrophy of its left A. Digoxin
heart ventricle. Some of its parts have B. Strophanthine
local contractures. What substance C. Corglycon
accumulated in the myocardiocytes D. Cordiamin
caused these contractures? E. Cordarone
A. Calcium 62. An ophthalmologist used a 1%
B. Potassium mesaton solution for the diagnostic
C. Lactic acid purpose (pupil dilation for eye-ground
D. Carbon dioxide examination). What is the cause of mydri-
E. Sodium asis induced by the drug?
57. A girl is diagnosed with adrenogeni- A. Activation of α1 adrenoreceptors
tal syndrome (pseudohermaphroditi- B. Activation of α2 adrenoreceptors
sm). This pathology was caused by C. Block of α1 adrenoreceptors
hypersecretion of the following adrenal D. Activation of β1 adrenoreceptors
hormone: E. Activation of M-cholinoreceptors
A. Androgen 63. Prophylactic medical examination of
B. Estrogen a 36 year old driver revealed that his
C. Aldosterone AP was 150/90 mm Hg. At the end of
D. Cortisol working day he usually hears ear noi-
E. Adrenalin se, feels slight indisposition that passes
after some rest. He was diagnosed with
58. In course of an experiment a big essential hypertension. What is the leadi-
number of stem cells of red bone marrow ng pathogenetic mechanism in this case?
was in some way destructed. Regenerati-
on of which cell populations in the loose A. Neurogenetic
connective tissue will be inhibited? B. Nephric
C. Humoral
A. Of macrophags D. Endocrinal
B. Of fibroblasts E. Reflexogenic
C. Of pigment cells
D. Of lipocytes 64. A cerebral trauma caused increased
E. Of pericytes ammonia generation. What amino acid
participates in the excretion of ammonia
59. Examination of a 70 year old pati- from the cerebral tissue?
ent rrevealed insulin-dependent diabetes.
What drug should be administered? A. Glutamic
B. Tyrosine
A. Glibenclamid C. Valine
B. Insulin D. Tryptophan
C. Mercazolilum E. Lysine
D. Parathyroidin
E. Cortisone 65. While studying maximally spiralized
chromosomes of human karyotype the
60. A human body cools in water much process of cell division was stopped in the
faster that in the air. What way of heat following phase:
emission in water is much more efficient?
A. Metaphase
A. Heat conduction B. Prophase
B. Convection C. Interphase
C. Heat radiation D. Anaphase
D. Sweat evaporation E. Telophase
E. -
Krok 1 Medicine 2008 8

66. 48 hours after tuberculine test were destroyed?


(Mantoux test) a child had a papule 10
mm in diameter on the spot of tuberculine A. Anterior tubercles of quadrigeminal
injection. What hypersensitivity mechani- plate
sm underlies these changes? B. Posterior tubercles of quadrigeminal
plate
A. Cellular cytotoxicity C. Red nuclei
B. Anaphylaxy D. Vestibular nuclei
C. Antibody-dependent cytotoxicity E. Black substance
D. Immunocomplex cytotoxicity
E. Granulomatosis 72. Myocyte cytoplasm contains a big
number of dissolved metabolites of
67. A patient was admitted to the surgical glucose oxidation. Name one of them that
department with suspected inflammation turns directly into a lactate:
of Meckel’s diverticulum. What part of
bowels should be examined in order to A. Pyruvate
discover the diverticulum in course of an B. Oxaloacetate
operation? C. Glycerophosphate
D. Glucose 6-phosphate
A. Ileum E. Fructose 6-phosphate
B. Duodenum
C. Jejunum 73. Mother of a 2 year old child
D. Caecum consulted a stomatologist. In the peri-
E. Colon ascendens od of pregnancy she was irregularly taki-
ng antibiotics for an infectious disease.
68. Power inputs of a man were measured. Examination of the child revealed inci-
In what state was this man if his power sor destruction, yellow enamel, brown rim
inputs were lower than basal metabolism? around the dental cervix. What drug has
apparent teratogenic effect?
A. Sleep
B. Relaxation A. Doxacycline
C. Simple work B. Furosemide
D. Nervous tension C. Ampiox
E. Rest D. Xantinol nicotinate
E. Octadine
69. Examination of a newborn boy’s geni-
talia revealed an urethral hiatus that 74. A 59 year old patient is a plant
opens on the undersite of his penis. What manager. After the tax inspection of his
malformation is it? plant he felt intense pain behind his
breastbone irradiating to his left arm.
A. Hypospadias 15 minutes later his condition came to
B. Hermaphroditism normal. Which of the possible mechani-
C. Epispadia sms of stenocardia development is the
D. Monorchism leading in this case?
E. Cryptorhidism
A. High catecholamine concentration in
70. A concentrated solution of sodium blood
chloride was intravenously injected to an B. Coronary atherosclerosis
animal. This caused decreased reabsorpti- C. Intravascular aggregation of blood
on of sodium ions in the renal tubules. It corpuscles
is the result of the following changes of D. Coronary thrombosis
hormonal secretion: E. Functional heart overload
A. Aldosterone reduction 75. A patient recovered from Sonne
B. Aldosterone increase dysentery and was once more infected wi-
C. Vasopressin reduction th the same causative agent. What is such
D. Vasopressin increase infection form called?
E. Reduction of atrial natriuretic factor
A. Reinfection
71. As a result of destruction of certain B. Recidivation
brainstem structures an animal has lost C. Superinfection
its orientative reflexes in response to D. Persisting infection
strong photic stimuli. What structures E. Chronic infection
Krok 1 Medicine 2008 9

76. A 56 year old patient came to a A. Thiamine pyrophosphate


hospital with complaints about general B. Nicotinamide adenine dinucleotide
weakness, tongue pain and burning, (NAD)
sensation of limb numbness. In the past C. Flavine adenine dinucleotide (FAD)
he underwent resection of forestomach. In D. Lipoic acid
blood: Hb- 80 g/l; erythrocytes - 2, 0·1012 /l; E. Coenzyme A
colour index - 1,2, leukocytes - 3, 5 · 109 /l.
What anemia type is it? 81. A chemical burn caused esophagus
stenosis. Difficulty of ingestion led to the
A. B12 -folate deficient abrupt loss of weight. In blood: 3, 0 · 1012/l,
B. Hemolytic Hb - 106 g/l, crude protein - 57 g/l. What
C. Posthemorrhagic type of starvation is it?
D. Aplastic
E. Iron-deficient A. Incomplete
B. Proteinic
77. A 35 year old patient applied to C. Complete
a doctor with complaints about havi- D. Water
ng intense rhinitis and loss of sense of E. Absolute
smell for a week. Objectively: nasal cavi-
ty contains a lot of mucus that covers 82. A 30 year old woman has face
mucous membrane and blocks olfactory edemata. Examination revealed protei-
receptors. In what part of nasal cavity are nuria (5,87 g/l), hypoproteinemia,
these receptors situated? dysproteinemia, hyperlipidemia. What
condition is the set of these symptoms
A. Superior nasal turbinate typical for?
B. Median nasal turbinate
C. Inferior nasal turbinate A. Nephrotic syndrome
D. Common nasal meatus B. Nephritic syndrome
E. Vestibule of nose C. Chronic pyelonephritis
D. Acute renal failure
78. A 17 year old boy fell seriously ill, E. Chronic renal failure
the body temperature rose up to 38, 5oC,
there appeared cough, rhinitis, lacrimati- 83. Skin of a man who died from cardi-
on, nasal discharges. What inflammation ac insufficiency has an eruption in form
is it? of spots and specks. There are also
bedsores in the area of sacrum and spi-
A. Catarrhal nous vertebral processes. Microscopical
B. Serous examination of CNS, skin, adrenal glands
C. Fibrinous revealed in the vessels of microcirculatory
D. Purulent bed and in small arteries destructive-
E. Hemorrhagic proliferative endothrombovasculitis with
Popov’s granulomas; interstitial myocardi-
79. A patient with disturbed cerebral ci- tis. What diagnosis corresponds with the
rculation has problems with deglutition. described picture?
What part of brain was damaged?
A. Spotted fever
A. Brainstem B. Q fever
B. Cervical part of spinal cord C. Enteric fever
C. Forebrain D. Nodular periarteritis
D. Interbrain E. HIV
E. Midbrain
84. Autopsy of a man who died from
80. Vitamin B1 deficiency results in di- the sepsis in his femoral bone revealed
sturbance of oxidative decarboxylation phlegmonous inflammation that affected
of α-ketoglutaric acid. This will disturb the marrow, haversian canals and peri-
synthesis of the following coenzyme: osteum. Under the periosteum there are
multiple abscesses, adjoining soft tissues
of thigh also have signs of phlegmonous
inflammation. What pathological process
was described?
Krok 1 Medicine 2008 10

A. Acute hematogenous osteomyelitis on revealed that the stone was in the


B. Osteoporosis common bile duct. What bile-excreting
C. Chronic hematogenous osteomielitis ducts make up the obturated duct?
D. Osteopetrosis
E. - A. Ductus hepaticus communis et ductus
cysticus
85. A patient has a decreased vasopressin B. Ductus hepaticus dexter et sinister
synthesis that causes polyuria and as a C. Ductus hepaticus dexter et ductus cysti-
result of it evident organism dehydratati- cus
on. What is the mechanism of polyuria D. Ductus hepaticus sinister et ductus
development? cysticus
E. Ductus hepaticus communis et ductus
A. Reduced tubular reabsorption of water choledochus
B. Reduced tubular reabsorption of Na
ions 90. A patient is 44 years old. Laboratory
C. Reduced tubular reabsorption of examination of his blood revealed that
protein content of proteins in plasma was 40 g/l.
D. Reduced glucose reabsorption What influence will be exerted on the
E. Acceleration of glomerular filtration transcapillary water exchange?
86. A patient with nephrotic syndrome A. Filtration will be increased, reabsorpti-
has massive edemata of his face and li- on - decreased
mbs. What is the leading pathogenetic B. Both filtration and reabsorption will be
mechanism of edemata development? increased
C. Both filtration and reabsorption will be
A. Drop of oncotic blood pressure decreased
B. Increase of vascular permeability D. Filtration will be decreased, reabsorpti-
C. Rise of hydrodynamic blood pressure on - increased
D. Lymphostasis E. Exchange will stay unchanged
E. Increase of lymph outflow
91. The cerebrospinal fluid is being exami-
87. Examination of a 55 year old woman ned for the purpose of diffrential meningi-
revealed under the skin of submandi- tis diagnostics. At what site is the lumbal
bular area a movable slowly growing pasty puncture safe?
formation with distinct borders 1,0x0,7 cm
large. Histological examination revealed A. L III-L IV
lipocytes that form segments of diffrent B. L II-L III
forms and sizes separated from each other C. L I-L II
by thin layers of connective tissue with D. Th XII-L I
vessels. What is the most probable di- E. L V-S I
agnosis?
92. An isolated cell of human heart
A. Lipoma automatically generates excitement
B. Fibroma impulses with frequency of 60 times per
C. Angioma minute. This cell was taken from the
D. Liposarcoma following heart structure:
E. Fibrosarcoma
A. Sinoatrial node
88. An unconscious young man with signs B. Atrium
of morphine poisoning entered admissi- C. Ventricle
on office. His respiration is shallow and D. Atrioventricular node
infrequent which is caused by inhibition E. His’ bundle
of respiratory centre. What type of respi-
ratory failure is it? 93. Examination of a 60 year old patient
revealed hyperglycemia and glucosuria. A
A. Ventilative dysregulatory doctor administered him a medication for
B. Ventilative obstructive internal use. What medication is it?
C. Ventilative restrictive
D. Perfusive A. Glibenclamid
E. Diffusive B. Furosemide
C. Oxytocin
89. A patient with cholelithiasis fell D. Pancreatine
ill with mechanic jaundice. Examinati- E. Corglycon
Krok 1 Medicine 2008 11

94. As a result of posttranslative modifi- crospcopic examination of scrapings from


cations some proteins taking part in blood the affected areas revealed living porrect
coagulation, particularly prothrombin, vermiform arthropoda 0,2-0,5 mm large
become capable of calcium binding. with four pairs of short extremities in the
The following vitamin takes part in this front part of their bodies. What is the
process: laboratory diagnosis?
A. K A. Demodicosis
B. C B. Scabies
C. A C. Myiasis
D. B1 D. Pediculosis
E. B2 E. Phthiriasis
95. Examination of a man who had been 100. A 28 year old man had a gunshot
working hard under higher temperature wound of shin that resulted in an
of the environment revealed abnormal ulcer from the side of the injury. What
quantity of blood plasma proteins. What is the main factor of neurodystrophy
phenomenon is the case? pathogenesis in this case?
A. Relative hyperproteinemia A. Traumatization of peripheral nerve
B. Absolute hyperproteinemia B. Psychical stress
C. Absolute hypoproteinemia C. Microcirculation disturbance
D. Dysproteinemia D. Infection
E. Paraproteinemia E. Tissue damage
96. A patient ill with thrombophlebitis of 101. A 45 year old man consulted a doctor
his lower limbs had chest pain, blood spi- about a plaque-like formation on his neck.
tting, progressing respiratory insufficiency Histological examination of a skin bi-
that led to his death. Autopsy diagnosed optate revealed clusters of round and
multiple lung infarctions. What is the most oval tumour cells with a narrow border of
probable cause of their development? basophilic cytoplasm resembling of cells
of basal epidermal layer. What tumour is
A. Thromboembolism of pulmonary artery it?
branches
B. Thrombosis of pulmonary artery A. Basal cell carcinoma
branches B. Epidermal cancer
C. Thrombosis of bronchial arteries C. Hydroadenoma
D. Thromboembolism of bronchial arteries D. Trichoepithelioma
E. Thrombosis of pulmonary veins E. Syringoadenoma
97. A woman suffering from osteochondrosis 102. A patient ill with essential
felt acute pain in her humeral articulation hypertension was recommended a drug
that became stronger when she abducted that prevents thrombosis. It is to be taken
her shoulder. These symptoms might be parenterally. What drug is it?
caused by damage of the following nerve:
A. Heparin
A. Axillary nerve B. Amben
B. Subscapular nerve C. Protamine sulfate
C. Dorsal scapular nerve D. Neodicumarin
D. Subclavicular nerve E. Syncumar
E. Throracodorsal nerve
103. Packed cell volume of a man was
98. A patient has a deep cut wound on the 40% before the trauma. What packed cell
posterior surface of his shoulder in its mi- volume will be observed 24 hours after
ddle third. What muscle might be injured? blood loss of 750 ml?
A. Triceps muscle of arm A. 30%
B. Biceps muscle of arm B. 40%
C. Anconeus muscle C. 55%
D. Brachial muscle D. 45%
E. Coracobrachial muscle E. 50%
99. A patient has acne on his face. Mi- 104. Two hours after an exam a student
Krok 1 Medicine 2008 12

had a blood count done and it was A. Kreatine


revealed that he had leukocytosis wi- B. Collagen
thout significant leukogram modifications. C. Porphyrin
What is the most probable mechanism of D. Myoglobin
leukocytosis development? E. Calmodulin
A. Redistribution of leukocytes in the 109. A patient being treated for viral
organism hepatitis type B got symptoms of hepatic
B. Leukopoiesis intensification insufficiency. What blood changes indi-
C. Deceleration of leukocyte lysis cative of protein metabolism disorder will
D. Deceleration of leukocyte migration to be observed in this case?
the tissues
E. Leukopoiesis intensification and A. Absolute hypoalbuminemia
deceleration of leukocyte lysis B. Absolute hyperalbuminemia
C. Absolute hyperfibrinogenemia
105. Golgi complex exports substances D. Proteinic blood composition is
from a cell due to the fusion of unchanged
the membrane saccule with the cell E. Absolute hyperglobulinemia
membrane. The saccule contents flows
out. What process is it? 110. A pregnant woman had her blood
group identified. Reaction of erythrocyte
A. Exocytosis agglutination with standard serums of 0αβ
B. Endocytosis (I), Bα (III) groups didn’t proceed wi-
C. Active transport th standard serum of Aβ (II) group. The
D. Facilitated diffusion blood group under examination is:
E. All answers are false
A. Аβ (II)
106. A boy found a spider with the followi- B. 0αβ (I)
ng morphological characteristics: it is 2 cm C. Вα (III)
long, has roundish black abdomen with D. АВ (IV)
two rows of red spots on its dorsal side; E. -
four pairs of jointed limbs are covered wi-
th small black hairs. What arthropod is it? 111. A sensitive neural ganglion consists
of roundish neurocytes with one extensi-
A. Karakurt spider on that divides into axon and dendrite at
B. Scorpion some distance from the perikaryon. What
C. Solpuga are these cells called?
D. Mite
E. Tarantula A. Pseudounipolar
B. Unipolar
107. A 3 year old child with symptoms C. Bipolar
of stomatitis, gingivitis and dermati- D. Multipolar
tis of open skin areas was delivered E. Apolar
to a hospital. Examination revealed
inherited disturbance of neutral amino 112. A doctor examined a child
acid transporting in the bowels. These and revealed symptoms of rachitis.
symptoms were caused by the deficiency Development of this desease was caused
of the following vitamin: by deficiency of the following compound:
A. Niacin A. 1,25 [ОН]-dichydroxycholecalciferol
B. Pantothenic acid B. Biotin
C. Vitamin A C. Tocopherol
D. Cobalamin D. Naphtaquinone
E. Biotin E. Retinol
108. A patient with suspected diagnosis 113. An embryo displays disturbed
"progressing muscular dystrophy"got his process of dorsal mesoderm segmentation
urine tested. What compound will confirm and somite formation. What part of skin
this diagnosis if found in urine? will have developmental abnormalities?
Krok 1 Medicine 2008 13

A. Derma are being stimulated in course of an


B. Hair experiment. As a result of it the exci-
C. Sebaceous glands tement conduction from atria to the
D. Epidermis ventricles was brought to a stop. It is
E. Sudoriferous glands caused by electrophysical changes in the
following structures:
114. It was found out that some
compounds, for instance fungi toxins and A. Atrioventricular node
some antibiotics can inhibit activity of B. His’ bundle
RNA-polymerase. What process will be C. Sinoatrial node
disturbed in a cell in case of inhibition of D. Ventricles
this enzyme? E. Atria
A. Transcription 119. If a man has an attack of bronchi-
B. Processing ospasm it is necessary to reduce the effect
C. Replication of vagus on smooth muscles of bronchi.
D. Translation What membrane cytoreceptors should be
E. Reparation blocked for this purpose?
115. A 7 year old child is ill with A. M-cholinoreceptors
bronchitis. It is necessary to administer B. N-cholinoreceptors
him an antibacterial drug. What drug C. α-adrenoreceptors
of fluoroquinolone group is CONTRA- D. β-adrenoreceptors
INDICATED at this age? E. α- and β-adrenoreceptors
A. Cyprofloxacin 120. A patient of surgical department
B. Ampicillin complains about pain in the small of her
C. Amoxicillin back and in the lower part of her belly;
D. Sulfadimethoxine painful and frequent urination. Bacteri-
E. Ampiox ological examination of urine revealed
gram-negative oxidase-positive rod-like
116. A culture of monkey cells (Vero) and bacteria forming greenish mucoid coloni-
a group of mouse sucklings were infected es with specific smell. What causative
with an inoculum taken from a child with agent can it be?
provisional diagnosis "enterovirus infecti-
on". There was no cytopathic effect on A. Pseudomonas aeruginosa
the cell culture but mouse sucklings died. B. Proteus mirabilis
What enteric viruses might have caused C. E.coli
disease of this child? D. Str.pyogenes
E. Mycoplasma pneumonie
A. Coxsackie A
B. Coxsackie B 121. There are several groups of
C. ECHO virus molecular mechanisms playing important
D. Polioviruses part in pathogenesis of insult to cells
E. Unclassified enteric viruses 68-71 which contributes to the pathology
development. What processes are sti-
117. A patient died from cardiopulmonary mulated by proteinic damage mechani-
decompensation. Histological examinati- sms?
on revealed diffused pulmonary lesi-
on together with interstitial edema, A. Enzyme inhibition
infiltration of tissue by limphocytes, B. Lipid peroxidation
macrophages, plasmocytes; pulmonary fi- C. Phospholipase activation
brosis, panacinar emphysema. What di- D. Osmotic membrane distension
sease corresponds with the described pi- E. Acidosis
cture?
122. A child was born with cleft palate.
A. Fibrosing alveolitis Examination revealed aorta defects and
B. Chronic bronchitis reduced number of T-lymphocytes in
C. Bronchopneumonia blood. What immunodeficient syndrome
D. Pulmonary atelectasis is it?
E. Bronchial asthma
118. Vagus branches that innervate heart
Krok 1 Medicine 2008 14

A. DiGeorge A. Inherited, dominant


B. Wiskott-Aldrich B. Inherited, recessive
C. Chediak-Higashi C. Inherited, sex-linked
D. Louis-Bar D. Congenital
E. Swiss-type E. Acquired
123. A 16 year old boy after an illness has 128. A patient was diagnosed with autoi-
diminished function of protein synthesis mmune hemolitic cytotoxic anemia. What
in liver as a result of vitamin K deficiency. substances are antigens in II type allergic
It will cause disturbance of: reactions?
A. Blood coagulation A. Modified receptors of cell membranes
B. Erythrocyte sedimentation rate B. Antibiotics
C. Anticoagulant generation C. Hormones
D. Erythropoietin secretion D. Serum proteins
E. Osmotic blood pressure E. Inflammation modulators
124. Life cycle of a cell includes the 129. While performing an operation in the
process of DNA autoreduplication. As a area of axillary crease a surgeon has to
result of it monochromatid chromosomes define an arterial vessel surrounded by
turn into bichromatid ones. What period fascicles of brachial plexus. What artery is
of cell cycle does this phenomenon fall it?
into?
A. A.axillaris
A. S B. A.vertebralis
B. Go C. A.transversa colli
C. G1 D. A.profunda brachii
D. G2 E. A.subscapularis
E. M
130. A patient takes digoxin for treatment
125. A driver who got a trauma in a road of cardiac insufficiency. What diuretic
accident and is shocked has reduction of may increase digoxin toxicity due to the
daily urinary output down to 300 ml. What intensified excretion of K + ions?
is the main pathogenetic factor of such di-
uresis change? A. Hydrochlorothiazide
B. Spironolactone
A. Drop of arterial pressure C. Panangine
B. Drop of oncotic blood pressure D. Siliborum
C. Increased vascular permeability E. Lisinopril
D. Decreased number of functioning
glomerules 131. During starvation muscle proteins
E. Secondary hyperaldosteronism break up into free amino acids. These
compounds will be the most probably
126. Systemic arterial pressure of an adult involved into the following process:
dropped from 120/70 to 90/50 mm Hg
that led to reflectory vasoconstriction. The A. Gluconeogenesis in liver
vasoconstriction will be maximal in the B. Gluconeogenesis in muscles
following organ: C. Synthesis of higher fatty acids
D. Glycogenolysis
A. Bowels E. Decarboxylation
B. Heart
C. Brain 132. A patient ill with amebiasis was
D. Kidneys prescribed a certain drug. The use of
E. Adrenals alcohol together with this drug is contra-
indicated because the drug inhibits
127. Examination of a 12 year old metabolism of ethyl alcohol. What drug
boy with developmental lag revealed is it?
achondroplasia: disproportional consti-
tution with evident shortening of upper A. Metronidazole
and lower limbs as a result of growth B. Reserpine
disorder of epiphyseal cartilages of long C. Clonidine
tubal bones. This disease is: D. Diazepam
E. Aminazine
Krok 1 Medicine 2008 15

133. Surgical removal of a part of stomach A. It will come to a standstill


resulted in disturbed absorption of vi- B. No changes will be observed
tamin B12 , it is excreted with feces. The C. Diaphragmal respiration will be mai-
patient was diagnosed with anemia. What ntained, thoracic respiration will disappear
factor is necessary for absorption of this D. Thoracic respiration will be maintained,
vitamin? diaphragmal respiration will disappear
E. It will become infrequent and deep
A. Gastromucoprotein
B. Gastrin 138. In course of an experiment a peri-
C. Hydrochloric acid pheral section of vagus of an expiremental
D. Pepsin animal is being stimulated. What changes
E. Folic acid will be observed?
134. A patient ill with chronic cardi- A. Heart rate fall
ac insufficiency was prescribed an B. Heart hurry
average therapeutic dose of digoxin. Two C. Pupil dilation
weeks after begin of its taking there D. Increase of respiration rate
appeared symptoms of drug intoxication E. Bronchi dilation
(bradycardia, extrasystole, nausea). Name
the phenomenon that caused accumulati- 139. 2 hours after a skeletal extension
on of the drug in the organism? was performed to a 27 year old patient
with multiple traumas (closed injury of
A. Material cumulation chest, closed fracture of right thigh) his
B. Functional cumulation condition abruptly became worse and the
C. Tolerance patient died from acute cardiopulmonary
D. Tachyphylaxis decompensation. Histological examinati-
E. Idiosyncrasy on of pulmonary and cerebral vessels stai-
ned with Sudan III revealed orange drops
135. In course of a preventive exami- occluding the vessel lumen. What compli-
nation of a miner a doctor revealed cation of polytrauma was developed?
changes of cardiovascular fitness which
was indicative of cardiac insufficiency at A. Fat embolism
the compensation stage. What is the main B. Gaseous embolism
proof of cardiac compensation? C. Microbal embolism
D. Thromboembolism
A. Myocardium hypertrophy E. Air embolism
B. Tachycardia
C. Rise of arterial pressure 140. In course of severe respiratory vi-
D. Dyspnea ral infection there appeared clinical si-
E. Cyanosis gns of progressing cardiac insufficiency
that caused death of a patient in the 2nd
136. Rest potential of a cell equals −80 week of disease. Autopsy revealed that
mV . At what stage of action potential did the heart was sluggish, with significant
the membrane potential equal +30 mV ? cavity dilatation. Histological examinati-
on of myocardium revealed plephora of
A. Reverse polarization microvessels and diffuse infiltration of
B. After hyperpolarization stroma by lymphocytes and histiocytes.
C. After depolarization What disease corresponds with the descri-
D. Depolarization bed picture?
E. -
A. Myocarditis
137. A 35 year old man got an injury B. Stenocardia
that caused complete disruption of spi- C. Acute coronary insufficiency
nal cord at the level of the first cervical D. Myocardium infarction
segment. What respiration changes will E. Cardiomyopathy
be observed?
141. A specimen stained by Ozheshko
method contains rod-like microorgani-
sms stained blue with round terminal
components stained red. What are these
components called?
Krok 1 Medicine 2008 16

A. Spores 146. A patient complains of dizziness and


B. Cilia hearing loss. What nerve is damaged?
C. Flagella
D. Capsules A. Vestibulocochlear
E. Mesosomas B. Trigeminus
C. Sublingual
142. A 38 year old patient with full-blown D. Vagus
jaundice, small cutaneous hemorrhages, E. Trochlear
general weakness and loss of appetite
underwent puncture biopsy of liver. Hi- 147. A 6 year old child was delivered to a
stological examination revealed dissemi- hospital. Examination revealed that the
nated dystrophy, hepatocyte necrosis, child couldn’t fix his eyes, didn’t keep
Councilman’s bodies. Lobule periphery his eyes on toys, eye ground had the
has signs of significant infiltration by cherry-red spot sign. Laboratory analyses
lymphocytes, there are also individual showed that brain, liver and spleen had hi-
multinuclear hepatocytes. What is the gh rate of ganglioside glycometide. What
most probable diagnosis? congenital disease is the child ill with?
A. Acute viral hepatitis A. Tay-Sachs disease
B. Acute alcoholic hepatitis B. Wilson’s syndrome
C. Miliary hepatic cirrhosis C. Turner’s syndrome
D. Toxic degeneration of liver D. Niemann-Pick disease
E. Chronic hepatitis E. MacArdle disease
143. A man with an injury of the dorsal 148. A patient has a cluster of matted
area of his neck was admitted to the together dense lymph nodes on his neck.
resuscitation department. What muscle Histological examination of a removed
occupies this area? lymph node revealed proliferation of reti-
cular cells, presense of Reed-Sternberg
A. M.trapezius cells. What disease is meant?
B. M.sternocleidomastoideus
C. M.latissimus dorsi A. Lymphogranulomatosis
D. M.rhomboideus minor B. Lymphoblastic leukosis
E. M.scalenus anterior C. Myeloblastic leukosis
D. Myelocytic leukosis
144. In course of an experiment a toad’s E. Lymphocytic leukosis
right labyrinth was destroyed. It will cause
amyotonia of the following muscles: 149. While palpating mammary gland of a
patient a doctor revealed an induration
A. Right extensors in form of a node in the inferior medial
B. Left flexors quadrant. Metastases may extend to the
C. Left extensors following lymph nodes:
D. Right flexors
E. Right and left extensors A. Parasternal
B. Posterior mediastinal
145. A 20 year old patient died from C. Profound lateral cervical
intoxication 8 days after artificial illegal D. Bronchopulmonary
abortion performed in her 14-15th E. Superior diaphragmal
week of pregnancy. Autopsy of the
corpse revealed yellowish colour of eye 150. A pregnant woman was registered
sclera and of skin, necrotic suppurati- in an antenatal clinic and underwent
ve endometritis, multiple pulmonary complex examination for a number of
abscesses, spleen hyperplasia with a big infections. Blood serum contained IgM
number of neutrophils in its sinuses. to the rubella virus. What is this result
What complication after abortion was indicative of?
developed?
A. Of primary infection
A. Septicopyemia B. Of a chronic process
B. Septicemia C. The woman is healthy
C. Hemorrhagic shock D. Of exacerbation of a chronic disease
D. Chroniosepsis E. Of recurring infection with rubella virus
E. Viral hepatitis type A
151. Parents of a 3 year old child have
Krok 1 Medicine 2008 17

been giving him antibiotics with purpose A. Urogenital trichomoniasis


of preventing enteric infections for a long B. Lambliasis
time. A month later the child’s conditi- C. Intestinal trichomoniasis
on changed for the worse. Blood exami- D. Toxoplasmosis
nation revealed apparent leukopenia E. Balantidiasis
and granulocytopenia. What is the most
probable mechanism of blood changes? 156. A boy has I (I 0 I 0 ) blood group and
his sister has IV (I A I B ) blood group. What
A. Myelotoxic blood groups do their parents have?
B. Autoimmune
C. Redistributive A. II (I I 0 ) and III (I I 0 )
D. Age-specific B. II (I I ) and III (I I 0 )
E. Hemolytic C. I (I 0 I 0 ) and IV (I A I B )
152. A pathological process in bronchi D. III (I I 0 ) and IV (I A I B )
resulted in epithelium desquamation. E. I (I 0 I 0 ) and III (I I 0 )
What cells will regenerate bronchial epi-
thelium? 157. A patient ill with enteritis accompani-
ed by massive diarrhea has low water rate
A. Basal in the extracellular space, high water rate
B. Intercalary inside the cells and low blood osmolari-
C. Ciliate ty. What is such disturbance of water-
D. Endocrinal electrolytic metabolism called?
E. Goblet
A. Hypo-osmolar hypohydration
153. A patient ill with tuberculosis di- B. Hyperosmolar hypohydration
ed from progressing cardiopulmonary C. Osmolar hypohydration
decompensation. Autopsy in the area of D. Hypo-osmolar hyperhydration
the right lung apex revealed a cavity 5 cm E. Hyperosmolar hyperhydration
in diameter communicating with lumen
of a segmental bronchus. On the insi- 158. For the purpose of myocardium
de cavity walls are covered with caseous infarction treatment a patient was injected
masses with epithelioid and Langhans with embryonal stem cells derived from
cells beneath them. What morphological this very patient by means of therapeutic
form of tuberculosis is it? cloning . What transplantation type is it?

A. Acute cavernous tuberculosis A. Autotransplantation


B. Tuberculoma B. Allotransplantation
C. Caseous pneumonia C. Xenotransplantation
D. Infiltrative tuberculosis D. Isotransplantation
E. Acute focal tuberculosis E. Heterotransplantation

154. Workers of a hothouse farm work 159. A histological specimen of spleen


under conditions of unfavourable mi- shows a vessel with a wall consisting of
croclimate: air temperature is +37oC, endothelium and subendothelial layer,
relative humidity is 90%, air speed is 0,2 median membrane is absent, exterior
m/s. The way of heat emission under these membrane inosculates with the layers of
conditions will be: spleen connective tissue. What vessel is it?

A. Evaporation A. Vein of non-muscular type


B. Heat conduction B. Vein of muscular type
C. Convection C. Artery of muscular type
D. Radiation D. Arteriole
E. All the ways E. Capillary

155. A gynaecologist was examining a 160. In course of a conditional experiment


patient and revealed symptoms of genital the development of mesenchyma cells
tract inflammation. A smear from vagi- was completely inhibited. Development
na contains pyriform protozoa with a spi- of the following muscular tissue will be di-
ne, flagella at their front; there is also an sturbed:
undulating membrane. What disease can
be suspected?
Krok 1 Medicine 2008 18

A. Smooth muscular tissue A. Sinoatrial node


B. Neural muscular tissue B. Atrioventricular node
C. Epidermal muscular tissue C. His’ bundle branches
D. Cardiac muscular tissue D. His’ bundle fibers
E. Skeletal muscular tissue E. Purkinje’s fibers
161. A patient ill with chronic gastri- 166. A stomatologist injected a patient wi-
tis went for endogastric pH-metry that th a certain drug in order to reduce sali-
allowed to reveal decreased acidity of vation during tooth filling. What drug is it?
gastric juice. It is indicative of diminished
function of the following cells: A. Atropine sulfate
B. Adrenaline hydrochloride
A. Parietal exocrinocytes C. Proserin
B. Chief exocrinocytes D. Pilocarpine hydrochloride
C. Endocrinocytes E. Mesaton
D. Cervical cells
E. Accessory cells 167. A 30 year old woman has applied
a lipstick with a fluorescent substance
162. Vagi of an experimental animal for a long time. Then she got a limited
were cut on both sides. What respirati- erythema and slight peeling on her lip
on changes will be observed? border, later there appeared transversal
striae and cracks. Special methods of mi-
A. It will become deep and infrequent croscopic examination of the affected area
B. It will become shallow and frequent helped to reveal sensibilized lymphocytes
C. It will become deep and frequent and macrophages in the connective tissue;
D. It will become shallow and infrequent cytolysis. What type of immunological
E. No changes will be observed hypersensitivity was developed?
163. A 70 year old man is ill with A. IV type (cellular cytotoxicity)
vascular atherosclerosis of lower extremi- B. I type (reaginic)
ties and coronary heart disease. Exami- C. II type (antibody cytotoxicity)
nation revealed disturbance of lipidic
blood composition. The main factor of D. III type (immune complex cytotoxicity)
atherosclerosis pathogenesis is the excess E. Granulomatosis
of the following lipoproteins: 168. A patient ill with essential arterial
A. Low-density lipoproteins hypertension had a hypertensic crisis that
B. Cholesterol resulted in an attack of cardiac asthma.
C. High-density lipoproteins What is the leading mechanism of cardiac
D. Intermediate density lipoproteins insufficiency in this case?
E. Chylomicrons A. Heart overload caused by high pressure
164. On the 2-3rd day after stomach B. Heart overload caused by increased
resection intestinal peristalsis wasn’t blood volume
restored. What is to be administered for C. Absolute coronary insufficiency
stimulation of gastrointestinal tract? D. Myocardium damage
E. Blood supply disturbance
A. Proserin
B. Prasosin 169. A man who has been taking a
C. Cyclodole drug for a long time cannot withhold it
D. Atropine sulfate because this causes impairment of psychic,
E. Acetylcholine somatic and vegetative functions. Name
the syndrome of different disturbances
165. A cardiac electric stimulator was caused by drug discontinuation:
implanted to a 75 year old man with
heart rate of 40 bpm. Thereafter the heart A. Abstinence
rate rose up to 70 bpm. The electric sti- B. Sensibilization
mulator has undertaken the function of C. Idiosyncrasy
the following heart part: D. Tachyphylaxis
E. Cumulation
170. A patient consulted a stomatologist
about purulent inflammation of his gums.
What drug will be the most effective if it
Krok 1 Medicine 2008 19

is suspected that a causative agent is an tumour contains cell bundles with oval
anaerobe? nuclei. Cellular fibrous bundles form
rhythmic structures made up by parallel
A. Metronidazole rows of regularly oriented cells arranged
B. Gentamicin in form of a palisade with cell-free
C. Oxacillin sodium homogenous zone (Verocay bodies)
D. Co-trimoxazole between them. What tumour is it?
E. Nitroxoline
A. Neurinoma
171. Rats being under stress have B. Malignant neurinoma
muscular hypertonia and high arterial C. Ganglioneurinoma
pressure, high glucose concentration in D. Neuroblastoma
blood and intensified secretion of corti- E. Ganglioneuroblastoma
cotropin and corticosteroids. In what
stress phase are these animals? 176. A 45 year old woman is ill with
breast cancer. Her left arm has symptoms
A. Antishock phase of lymphatic system insufficiency - limb
B. Exhaustion edema, lymph node enlargement. What
C. Shock phase form of lymphatic circulation insufficiency
D. Erectile is it?
E. Terminal
A. Mechanic insufficiency
172. Ultramicroscopical examination of B. Dynamic insufficiency
"dark"hepatocyte population in the cell C. Resorption insufficiency
cytoplasm detected a developed granular D. Combined insufficiency
endoplasmic reticulum. What function has E. -
this organella in these cells?
177. An endocrinal gland with
A. Synthesis of blood plasma proteins parenchyma consisting of epithelium
B. Carbohydrate synthesis and neural tissue is under morphologi-
C. Deintoxicative function cal examination. Epithelial trabecules
D. Bile production have two types of cells: chromophilic and
E. Calcium ion depositing chromophobic. Identify this organ:
173. A patient came to the hospital A. Hypophysis
complaining about quick fatigability and B. Adrenal glands
apparent muscle weakness. Examinati- C. Hypothalamus
on revealed an autoimmune disease that D. Thyroid gland
causes disorder of functional receptor E. Parathyroid gland
condition in neuromuscular synapses.
What transmitter will be blocked? 178. While performing an inguinal canal
operation on account of hernia a surgeon
A. Acetylcholine damaged the canal’s contents. What
B. Noradrenalin exactly was damaged?
C. Dopamine
D. Serotonin A. Funiculus spermaticus
E. Glycine B. Urarchus
C. Lig. teres uteri
174. Examination of a 35 year old patient D. Lig. inguinalе
revealed high acidity of gastric juice. What E. -
receptors should be blocked in order to
reduce it? 179. A 12 year old child has intolerance
to some foodstuffs. Their consumption
A. Histamine causes an allergic reaction in form of itchi-
B. α1 -adrenoreceptors ng skin eruptions. What antihistaminic
C. α2 -adrenoreceptors drug should be admistered so that the chi-
D. β1 -adrenoreceptors ld could attend school?
E. β2 -adrenoreceptors
175. Examination of a young woman
revealed a node-like, soft and elastic
homogenous tumour of pinkish-white
colour along the acoustic nerve. The
Krok 1 Medicine 2008 20

A. Loratadine in disturbance of tryptophane absorpti-


B. Dimedrol on in the bowels and its resorption in the
C. Diclofenac renal tubules. It is the reason for disorder
D. Aminophylline of both digestive and urination systems.
E. Ephedrine What genetic phenomenon is observed in
this case?
180. A patient that entered the admi-
ssion office had the following signs of A. Pleiotropy
acute cardiac insufficiency: paleness, B. Complementary interaction
acrocyanosis, frequent shallow respirati- C. Polymery
on. What drug is indicated in this case? D. Codominance
E. Semidominance
A. Corglycon
B. Digitoxin 185. A patient who has been treated in
C. Cordiamin a neural clinic and has been taking a
D. Nitroglycerine sedative for a long time got the followi-
E. Adrenaline hydrochloride ng complication: cough, rhinitis, epiphora.
What drug caused these disturbances?
181. A 26 year old man is in the torpid
shock phase as a result of a car accident. A. Sodium bromide
In blood: 3, 2 · 109 /l. What is the leading B. Diazepam
mechanism of leukopenia development? C. Valerian
D. Phenazepam
A. Redistribution of leukocytes in E. Reserpine
bloodstream
B. Leikopoiesis inhibition 186. In order to accelerate healing of a
C. Disturbed going out of mature radiation ulcer a vitamin drug was admi-
leukocytes from the marrow into the nistered. What drug is it?
blood
D. Lysis of leukocytes in the blood-forming A. Retinol acetate
organs B. Retabolil
E. Intensified elimination of leukocytes C. Prednisolone
from the organism D. Levamisole
E. Methyluracil
182. A patient had hemorrhagic stroke.
Blood examination revealed strengthened 187. Inoculum from pharynx of a patient
kinin concentration.The patient was ill with angina was inoculated into blood-
prescribed contrical. It was administered tellurite agar. It resulted in growth of
in order to inhibit the following protei- grey, radially striated (in form of rosettes)
nase: colonies 4-5 mm in diameter. Gram-
positive bacilli with clublike thickenings
A. Kallikrein on their ends placed in form of spread wi-
B. Pepsin de apart fingers are visible by microscope.
C. Trypsin What microorganisms are these?
D. Chemotrypsin
E. Collagenase A. Diphtheria corynebacteria
B. Botulism clostridia
183. A patient with bacterial pneumonia C. Diphtheroids
was prescribed benzylpenicillin. What is D. Streptococci
the mechanism of its antibacterial effect? E. Streptobacilli
A. Inhibition of synthesis of microorgani- 188. Plasmic factors of blood coagulation
sm wall are exposed to post-translational modifi-
B. Inhibition of intracellular protein cation with the participation of vitamin
synthesis K. It is necessary as a cofactor in the
C. Abnormal permeability of cytoplasmic enzyme system of γ-carboxylation of
membrane protein factors of blood coagulation due
D. Inhibition of SH-groups of microorgani- to the increased affinity of their molecules
sm enzymes with calcium ions. What amino acid is
E. Antagonism with p-amino-benzoic acid carboxylated in these proteins?
184. Hartnup disease is caused by point
mutation of only one gene which results
Krok 1 Medicine 2008 21

A. Glutamic hypertensive drug - salt of an alkaline-


B. Valine earth metal. What drug was injected?
C. Serine
D. Phenylalanine A. Magnesium sulfate
E. Arginine B. Potassium chloride
C. Sodium hydrocarbonate
189. Pharmacological effects of anti- D. Calcium lactate
depressants are connected with inhibition E. Benzohexamethonium
of an enzyme catalyzing biogenic amines
noradrenaline and serotonine in the mi- 194. In course of a small pelvis operation
tochondrions of cerebral neurons. What it became necessary to ligate an ovari-
enzyme participates in this process? an artery. What formation may be acci-
dentally ligated together with it?
A. Monoamine oxidase
B. Transaminase A. Ureter
C. Decarboxylase B. Uterine tube
D. Peptidase C. Round ligament of uterus
E. Lyase D. Internal iliac vein
E. Urethra
190. An oncological patient was prescri-
bed methotrexate. With the lapse of time 195. A patient with acute morphine poi-
target cells of the tumour lost susceptibi- soning was delivered to a hospital. What
lity to this drug. There is change of gene specific narcotic antagonist should be
expression of the folowing enzyme: chosen in this case?
A. Dehydrofolate reductase A. Naloxone
B. Thiaminase B. Paracetamol
C. Deaminase C. Methacin
D. Folate oxidase D. Digoxin
E. Folate decarboxylase E. Unithiol
191. As a result of prophylactic medi- 196. The upper lobe of the right lung
cal examination a 7 year old boy was is enlarged, grey and airless, the inscisi-
diagnosed with Lesch-Nyhan syndrome on surface is dripping with turbid liquid,
(only boys fall ill with it). The boy’s the pleura has many fibrinogenous films;
parents are healthy but his grandfather microscopical examination of alveoles
by his mother’s side suffers from the same revealed exudate containing neutrophi-
disease. What type of disease inheritance ls, desquamated alveolocytes and fibrin
is it? fibers. The bronchus wall is intact. What is
the most probable diagnosis?
A. Recessive, sex-linked
B. Dominant, sex-linked A. Croupous pneumonia
C. Autosomal recessive B. Interstitial pneumonia
D. Autosomal dominant C. Pulmonary abscess
E. Semidominance D. Focal pneumonia
E. Influenzal pneumonia
192. Planned mass vaccination of all
newborn 5-7 day old children against 197. A 28 year old patient had high
tuberulosis plays an important role in arterial pressure, hematuria and faci-
tuberculosis prevention. In this case the al edemata. In spite of treatment renal
following vaccine is applied: insufficiency was progressing. 6 months
later the patient died from uremia. Mi-
A. BCG croscopic examination of his kidneys and
B. Diphteria and tetanus toxoids and their glomerules revealed proliferation of
pertussis vaccine capsule nephrothelium and of podocytes
C. Diphtheria and tetanus anatoxin vacci- with "demilune"formation, sclerosis and
ne hyalinosis of glomerules. What disease
D. Adsorbed diphtheria vaccine corresponds with the described picture?
E. -
193. A patient with hypertensic crisis was
admitted to the cardiological department,
he was injected intravenously with an anti-
Krok 1 Medicine 2008 22

A. Subacute glomerulonephritis de from his body. What nerve is most


B. Acute pyelonephritis probably damaged?
C. Nephrotic syndrome
D. Chronic glomerulonephritis A. N.axillaris
E. Acute glomerulonephritis B. N.medianus
C. N.radialis
198. A patient with a knife wound in D. N.ulnaris
the left lumbal part was delivered to the E. N.subclavius
emergency hospital. In course of operati-
on a surgeon found that internal organs 200. Autopsy of a man ill with severe
were not damaged but the knife injured hypothyroidism revealed that connecti-
one of muscles of renal pelvis. What ve tissue, organ stroma, adipose and
muscle is it? cartilaginous tissues were swollen, semi-
transparent, mucus-like. Microscopic
A. Greater psoas muscle examination of tissues revealed stellate
B. Iliac muscle cells having processes with mucus
C. Erector muscle of spine between them. What type of dystrophy
D. Abdominal internal oblique muscle is it?
E. Abdominal external oblique muscle
A. Stromal-vascular carbohydrate
199. A man with a stab wound in the B. Stromal-vascular adipose
area of quadrilateral foramen applied to C. Stromal-vascular proteinaceous
a doctor. Examination revealed that the D. Parenchymatous proteinaceous
patient was unable to draw his arm asi- E. Parenchymatous adipose
Krok 1 Medicine 2009 1

1. A 30-year-old patient complains A. Fibroma


about having abdominal pain and di- B. Myoma
arrhea for five days; body temperature C. Histiocytoma
rise up to 37, 5o C along with chills. The D. Dermatofibroma
day before a patient had been in a E. Desmoid
forest and drunk from an open water
reservoir. Laboratory analyses enabled 6. A patient underwent a surgery for exci-
to make the following diagnosis: amebic sion of a cyst on pancreas. After this he
dysentery. What is the drug of choice for developed haemorrhagic syndrome with
its treatment? apparent disorder of blood coagulation.
Development of this complication can be
A. Metronidazole explained by:
B. Furazolidonum
C. Levomycetin A. Activation of fibrinolytic system
D. Phthalazol B. Insufficient fibrin production
E. Emetine hydrochloride C. Reduced number of thrombocytes
D. Activation of anticoagulation system
2. A man suffering from a hereditary di- E. Activation of Christmas factor
sease married a healthy woman. They got
5 children, three girls and two boys. All 7. A mother consulted a doctor about her
the girls inherited their father’s disease. 5-year-old child who develops erythemas,
What is the type of the disease inheri- vesicular rash and skin itch under the
tance? influence of sun. Laboratory studies
revealed decreased iron concentration in
A. Dominant, X-linked the blood serum, increased uroporphyri-
B. Autosomal recessive nogen I excretion with the urine. What is
C. Asutosomal dominant the most likely inherited pathology in this
D. Y -linked child?
E. Recessive, X-linked
A. Erythropoietic porphyria
3. An electronic microphotograph shows B. Methemoglobinemia
a macrophagic cell with erythrocytes at C. Hepatic porphyria
different stages of differentiation located D. Coproporphyria
along its processes. This is the cell of the E. Intermittent porphyria
following organ:
8. A baby refuses the breast, he is anxious,
A. Red bone marrow presents with arrhythmic respiration. The
B. Thymus urine smells of "brewer’s yeast"or "maple
C. Spleen syrup". This pathology was caused by the
D. Tonsil inherited defect of the following enzyme:
E. Lymph node
A. Dehydrogenase of branched-chain
4. Quite often the cause of secondary alpha-keto acids
immunodeficiency is an infection B. Glucose 6-phosphate dehydrogenase
involvement, when the causative agents C. Glycerol kinase
propagate directly in the cells of immune D. Aspartate aminotransferase
system and destroy it. The following di- E. UDP-glucuronil transferase
seases are characterized by:
9. While studying a microslide obtained
A. Infectious mononucleosis, AIDS from the punctuate of a regional lymph
B. Tuberculosis, mycobacteriosis node and stained by Romanovsky-Giemsa
C. Poliomyelitis, type A hepatitis method a physician revealed some light-
D. Dysentery, cholera pink thin microorganisms with 12-14
E. Q-febris, epidemic typhus regular spiral coils and pointed ends, up
to 10-13 micrometer long. This might be
5. Examination of a patient revealed a the causative agent of the following di-
dense, movable skin tumour that is standi- sease:
ng out distinctly from the surrounding ti-
ssues. Its section is found to be white and
composed of fibrous tissue. Microscopic
examination revealed interlacing collagen
fibers and few cells. What tumour is it?
Krok 1 Medicine 2009 2

A. Syphilis to the granular endoplasmatic reticulum.


B. Trypanosomiasis It is known that this causes protein bi-
C. Leptospirosis osynthesis disturbance. What stage of
D. Relapsing fever protein biosynthesis will be disturbed?
E. Leishmaniasis
A. Translation
10. Sanitary bacteriological research on B. Transcription
water by the membrane filter method C. Replication
revealed two red colonies on a membrane D. Amino acid activation
filter (Endo agar) through which 500 ml E. Termination
of analyzed water were passed. Calculate
the coli index and coli titer of the analyzed 15. A patient suffering from stomach ulcer
water: has been treated with an antacid drug
almagel. For acute bronchitis treatment he
A. 4 and 250 was prescribed the antibiotic methacycli-
B. 2 and 500 ne. However within next 5 days the
C. 250 and 4 fever didn’t fall, cough and sputum
D. 500 and 2 nature remained unchanged. A physici-
E. 250 and 2 an came to the conclusion that the drugs
were incompatible. What type of drug
11. While examining a patient an incompatibility is the case?
otolaryngologist noticed hyperaemia and
significantly edematous tonsils with a A. Pharmacokinetic, absorption stage
grayish film upon them. Microscopical B. Pharmacokinetic, biotransformation
examination of this film revealed some stage
gram-positive bacilli placed at an angle C. Pharmaceutic
with each other. What disease might be D. Pharmacodynamic
suspected? E. Direct antagonism
A. Diphtheria 16. A 70-year-old patient suffers from
B. Angina atherosclerosis complicated by the lower
C. Scarlet fever limb thrombosis that has caused gangrene
D. Meningococcal nasopharyngitis on his left toes. What is the most likely
E. Epidemic parotitis cause of the thrombosis origin?
12. A 38-year-old patient with an uterine A. Thrombocyte adhesion
haemorrhage lasting for 2 days was deli- B. Prothrombinase activation
vered to the admission ward. Which of the C. Transformation of prothrombin into
following will be revealed in the patient’s thrombin
blood? D. Transformation of fibrinogen into fibrin
E. Impaired heparin synthesis
A. Decrease in the haematocrite index
B. Eosinophilia 17. ECG of a 44-year-old patient shows
C. Deceleration in ESR signs of hypertrophy of both ventricles
D. Leukocytosis and the right atrium. The patient was di-
E. Increase in the colour index agnosed with the tricuspid valve insuffici-
ency. What pathogenetic variant of cardiac
13. After a hypertonic crisis a pati- dysfunction is usually observed in case of
ent presents with lacking spontaneous such insufficiency?
movements in his right arm and leg,
muscle tone of these extremities is A. Heart overload by volume
increased. What type of motor dysfuncti- B. Heart overload by resistance
on has developed in this case? C. Primary myocardial insufficiency
D. Coronary insufficiency
A. Central paralysis E. Cardiac tamponade
B. Peripheral paralysis
C. Peripheral paresis 18. Bacteriological examination of
D. Reflectrory paresis purulent discharges from the urethra
E. Central paresis revealed gram-negative bacteria looking
like coffee beans. They were localized
14. A patient’s organism has decreased in the leukocytes and could decompose
concentration of magnesium ions that are glucose and maltose to acid. These are the
necessary for attachment of ribosomes causative agents of the following disease:
Krok 1 Medicine 2009 3

family hemophilia may be passed to:


A. Gonorrhoea
B. Syphilis A. Half of sons
C. Veneral lymphogranulomatosis B. Both sons and daughters
D. Soft chancre C. Daughters only
E. Melioidosis D. Half of daughters
E. All the children
19. Proserin increases skeletal muscle tone
when given systematically. Halothane 24. The minute blood volume in a patient
induces relaxation of skeletal muscles with transplanted heart has increased as a
and reduces proserin effects. What is result of physical activity. What regulati-
the nature of proserin and halothane ve mechanism is responsible for these
interaction? changes?
A. Indirect functional antagonism A. Catecholamines
B. Direct functional antagonism B. Sympathetic unconditioned reflexes
C. Competitive antagonism C. Parasympathetic unconditioned reflexes
D. Independent antagonism D. Sympathetic conditioned reflexes
E. Noncompetitive antagonism E. Parasympathetic conditioned reflexes
20. A 62-year-old female patient has 25. A female patient consulted a
developed a cataract (lenticular opacity) physician about digestive disorder,
secondary to the diabetes mellitus. What extended abdominal pain. Examination
type of protein modification is observed revealed drastic decrease in hemoglobin
in case of diabetic cataract? concentration. It is known from the
anamnesis that while living in the Far East
A. Glycosylation the patient used to eat freshly-salted cavi-
B. Phosphorylation ar. Some relatives living with her had the
C. ADP-ribosylation similar condition. What is the most likely
D. Methylation diagnosis?
E. Limited proteolysis
A. Diphyllobothriasis
21. Examination of a 2-year-old child B. Echinococcosis
revealed physical developmental lag, the C. Teniasis
child often has pneumonias. The child D. Trichiniasis
was diagnosed with nonclosure of ductus E. Ascaridiasis
arteriosus. Haemodynamics disorder was
caused by the intercommunication of the 26. A man has normal sensitivity of his fi-
following vessels: nger skin, however he doesn’t sense his
wedding ring around the finger. What
A. Aorta and pulmonary trunk process induced by wearing of the ring
B. Pulmonary trunk and pulmonary veins has caused this phenomenon?
C. Superior cava and aorta
D. Superior cava and pulmonary trunk A. Receptor adaptation
E. Aorta and pulmonary veins B. Development of the fibrous tissue
C. Abnormality of the epidermis structure
22. The temperature of the ambient envi- D. Impaired circulation
ronment is 38o C and relative air humidity E. Abnormality of the receptor structure
is 50%. What ways of heat emission provi-
de maintaining a constant temperature of 27. It is necessary to take the cerebrospi-
the human body? nal fluid from a patient with suspected
inflammation of brain tunics. Diagnostic
A. Evaporation puncture was performed between the
B. Radiation arches of the lumbar vertebras. During
C. Heat conduction the puncture the needle went through the
D. Convection following ligament:
E. Convection and conduction
A. Yellow (flaval)
23. A married couple consulted a speci- B. Iliolumbar
alist at the genetic consultation about C. Anterior longitudinal
probability of having children with D. Posterior longitudinal
hemophilia. Both spouses are healthy, but E. Intertransverse
the wife’s father has hemophilia. In this
Krok 1 Medicine 2009 4

28. ECG study showed that the T -waves A. Transport of FFA (free fatty acids) from
were positive in the standard extremity cytosol to the mitochondria
leads, their amplitude and duration were B. Transport of FFA from fat depots to the
normal. The right conclusion would be tissues
that the following process runs normally C. It takes part in one of reactions of FFA
in the heart ventricles: beta-oxidation
D. FFA activation
A. Repolarization E. Activation of intracellular lipolysis
B. Depolarization
C. Excitement 33. In a 2-year-old child with catarrhal
D. Contraction presentations and skin rash a pediatri-
E. Relaxation cian suspected scarlet fever. The child
was given intracutaneously a small dose
29. A patient has an increased pyruvate of serum antibody to the streptococcal
concentration in blood. A large amount erythrogenic toxin; on the site of injection
of it is excreted with the urine. What vi- the rash disappeared. What do the reacti-
tamin is lacking in this patient? on results mean?
A. B1 A. The clinical diagnosis was confirmed
B. E B. The child has hypersensitivity to the
C. B3 erythrogenic toxin
D. B6 C. The disease wasn’t caused by haemolytic
E. B2 streptococcus
D. The whole serum dose may be injected
30. A patient has pellagra. Interrogation intravenously
revealed that he had lived mostly on mai- E. The child has very weak immune system
ze for a long time and eaten little meat.
This disease had been caused by the defi- 34. A patient has a transversal lacerati-
cit of the following substance in the maize: on in the spinal cord. What respiratory
changes will result from this?
A. Tryptophan
B. Tyrosine A. Respiration will present no significant
C. Proline changes
D. Alanine B. Respiration will stop
E. Histidine C. Respiration will become less frequent
D. Respiration will become deeper
31. Study of bacteriological sputum speci- E. Respiration will become more frequent
mens stained by the Ziel-Neelsen method
revealed some bright-red acid-resistant 35. A female patient suffering from
bacilli that were found in groups or si- bronchial asthma had got a viral infecti-
ngularly. When inoculated onto the nutri- on that provoked status asthmaticus wi-
ent media, the signs of their growth show th fatal outcome. Histological examinati-
up on the 10-15 day. These bacteria relate on of lungs revealed spasm and edema of
to the following family: bronchioles, apparent infiltration of their
walls with lymphocytes, eosinophils and
A. Micobacterium tuberculosis other leukocytes; labrocyte degranulati-
B. Yersinia pseudotuberculosis on. What mechanism of hypersensitivity
C. Histoplasma dubrosii underlies the described alterations?
D. Klebsiella rhinoscleromatis
E. Coxiella burnettii A. Reagin reaction
B. Inflammatory
32. A patient with high rate of obesity was C. Autoimmune
advised to use carnitine as a food additive D. Immune complex
in order to enhance "fat burning". What E. Immune cytolysis
is the role of carnitine in the process of fat
oxidation? 36. In the perianal folds of a 5-year-old
girl her mother has found some white
"worms"that caused itch and anxiety in
the child. The "worms"were sent to the
laboratory. During examination the physi-
cian saw white filiform helminths 0,5-1 cm
long, with pointed ends, some helminths
had twisted ends. What is the most likely
Krok 1 Medicine 2009 5

diagnosis? A. Paraproteinemia
B. Hyperalbuminemia
A. Enterobiasis C. Proteinuria
B. Diphyllobothriasis D. Hypoglobulinemia
C. Teniasis E. Hypoproteinemia
D. Ascaridiasis
E. Opisthorchiasis 41. An 8-year-old child was admitted
to the infectious department with fever
37. Examination of a 27-year-old patient (up to 38o C) and punctuate bright-red
revealed pathological changes in liver and skin rash. The child was diagnosed as
brain. Blood plasma analysis revealed an having scarlet fever. Objectively: mucous
abrupt decrease in the copper concentrati- membrane of pharynx is apparently
on, urine analysis revealed an increased hyperaemic and edematic, the tonsils are
copper concentration. The patient was di- enlarged and have dull yellowish-grey foci
agnosed with Wilson’s degeneration. To with some black areas. What inflammati-
confirm the diagnosis it is necessary to on is the reason for the pharynx alterati-
study the activity of the following enzyme ons?
in blood serum:
A. Purulent necrotic
A. Ceruloplasmin B. Fibrinous
B. Carbonic anhydrase C. Haemorrhagic
C. Xanthine oxidase D. Serous
D. Leucine aminopeptidase E. Catarrhal
E. Alcohol dehydrogenase
42. Nappies of a newborn have dark spots
38. A patient complains about dyspnea being the evidence of homogentisic acid
provoked by the physical activity. Clini- formation. This is caused by the metabolic
cal examination revealed anaemia and disorder of the following substance:
presence of the paraprotein in the zone
of gamma-globulins. To confirm the A. Tyrosine
myeloma diagnosis it is necessary to B. Galactose
determine the following index in the pati- C. Methionine
ent’s urine: D. Cholesterol
E. Tryptophan
A. Bence Jones protein
B. Bilirubin 43. A histological specimen presents a
C. Haemoglobin receptor zone of a sensoepithelial sense
D. Ceruloplasmin organ. Cells of this zone are placed upon
E. Antitrypsin the basal membrane and include the
following types: external and internal
39. After a serious psycho-emotional receptor cells, external and internal
stress a 45-year-old patient suddenly felt phalangeal cell, stem cells, external limiti-
constricting heart pain irradiating to the ng cells and external supporting cell. The
left arm, neck and left scapula. His face described receptor zone belongs to the
turned pale, the cold sweat stood out on following sense organ:
it. The pain attack was stopped with ni-
troglycerine. What process has developed A. Acoustic organ
in this patient? B. Visual organ
C. Gustatory organ
A. Stenocardia D. Equilibrium organ
B. Myocardial infarction E. Olfactory organ
C. Stroke
D. Psychogenic shock 44. A patient was admitted to the hospi-
E. Stomach ulcer perforation tal with an asphyxia attack provoked
by a spasm of smooth muscles of the
40. A 62-year-old female patient complai- respiratory tracts. This attack was mai-
ns about frequent pains in the region of nly caused by alterations in the following
thorax and vertebral column, rib fractures. parts of the airways:
A physician suspected myelomatosis
(plasmocytoma). Which of the following
laboratory indices will be of the greatest
diagnostic importance?
Krok 1 Medicine 2009 6

A. Small bronchi A. Atropine sulfate


B. Median bronchi B. Physostigmine
C. Large bronchi C. Pyridostigmine bromide
D. Terminal bronchioles D. Isadrine
E. Respiratory part E. Mesatonum
45. While eating a child choked on 49. A man died from an acute infecti-
food and aspirated it. The child has ous disease accompanied by fever, jaundi-
severe cough, cyanotic skin and mucous ce, haemorrhagic rash on the skin
membranes, rapid pulse, infrequent respi- and mucous membranes as well as by
ration, prolonged expiration. The child acute renal insufficiency. Histological
has developed the following disorder of examination of renal tissue (stained by
the external respiration: Romanovsky-Giemsa method) revealed
some convoluted bacteria looking like
A. Expiratory dyspnea under asphyxia C und S letters. What bacteria were
B. Inspiratory dyspnea under asphyxia revealed?
C. Stenotic respiration
D. Alternating respiration A. Leptospira
E. Biot’s respiration B. Treponema
C. Spirilla
46. A 50-year-old patient complains D. Borrelia
about general weakness, appetite loss E. Campilobacteria
and cardiac arrhythmia. The patient
presents with muscle hypotonia, flaccid 50. A 12-year-old adolescent suffering
paralyses, weakened peristaltic activity from bronchial asthma has a severe attack
of the bowels. Such condition might be of asthma: he presents with marked expi-
caused by: ratory dyspnea, skin pallor. What type of
alveolar ventilation disorder is observed?
A. Hypokaliemia
B. Hypoproteinemia A. Obstructive
C. Hyperkaliemia B. Restrictive
D. Hypophosphatemia C. Thoracodiaphragmatic
E. Hyponatremia D. Central
E. Neuromuscular
47. While playing volleyball a sportsman
jumped and then landed across the 51. A 46-year-old patient suffering from
external edge of his foot. This caused the diffuse toxic goiter underwent resecti-
acute pain in the talocrural articulation, on of the thyroid gland. After the
active movements became limited, passive surgery the patient presents with appetite
movements remained unlimited but pai- loss, dyspepsia, increased neuromuscular
nful. In the region of the external ankle excitement. The body weight remained
a swelling appeared, the skin turned red unchanged. Body temperature is normal.
and became warmer to the touch. What Which of the following has caused such a
type of peripheral circulation disorder has condition in this patient?
developed in this case?
A. Reduced production of parathormone
A. Arterial hyperaemia B. Increased production of thyroxin
B. Stasis C. Increased production of calcitonin
C. Embolism D. Increased production of thyroliberin
D. Venous hyperaemia E. Reduced production of thyroxin
E. Thrombosis
52. A patient who had myocardial infarcti-
48. A patient suffering from myasthenia on was administered 75 mg of acetylsalici-
has been administered proserin. After its nic acid a day. What is the purpose of this
administration the patient has got nausea, administration?
diarrhea, twitch of tongue and skeletal
muscles. What drug would help to elimi- A. Reduction of thrombocyte aggregation
nate the intoxication? B. Inflammation reduction
C. Pain relief
D. Temperature reduction
E. Coronary vessel dilatation
53. Medical examination at the mili-
Krok 1 Medicine 2009 7

tary registration and enlistment office A. Phenylketonuria


revealed that a 15-year-old boy was hi- B. Alkaptonuria
gh, with eunuchoid body proportions, C. Tyrosinosis
gynecomastia, female pattern of pubic D. Albinism
hair distribution. The boy had also fat E. Xanthinuria
deposits on the thighs, no facial hair, hi-
gh voice, subnormal intelligence quotient. 58. Autopsy of a 73-year-old man who had
Which karyotype corresponds with this di- been suffering from the coronary heart di-
sease? sease along with cardiac insufficiency for
a long time revealed: nutmeg liver, brown
A. 47, XXY induration of lungs, cyanotic induration
B. 45, XO of kidneys and spleen. What kind of ci-
C. 46, XX rculation disorder was the cause of such
D. 46, XY effects?
E. 47, XXX
A. General chronic venous congestion
54. To prevent the transplant rejection B. Arterial hyperaemia
after organ transplantation it is requi- C. General acute venous congestion
red to administer hormonotherapy for D. Acute anaemia
the purpose of immunosuppression. What E. Chronic anaemia
hormones are used for this purpose?
59. A patient suffering from chronic
A. Glucocorticoids hyperacidic gastritis takes an antacid
B. Mineralocorticoids drug for heartburn elimination. After its
C. Sexual hormones ingestion the patient feels better but at the
D. Catecholamines same time he has a sensation of stomach
E. Thyroid swelling. Which of the following drugs mi-
ght be the cause of such side effect?
55. A 53-year-old female patient was di-
agnosed with liver rupture resulting from A. Sodium hydrocarbonate
a blunt abdominal injury. The escaped B. Magnesium oxide
blood will be assembled in the followi- C. Magnesium trisilicate
ng anatomic formation: D. Aluminium hydrooxide
E. Pepsin
A. Rectouterine pouch
B. Vesicouterine pouch 60. Continuous use of a certain drug may
C. Right mesenteric sinus cause osteoporosis, erosions of stomach
D. Omental bursa mucosa, hypokaliemia, retention of sodi-
E. Left mesenteric sinus um and water in the organism, decreased
concentration of corticotropin in blood.
56. A patient complains about edemata What drug is it?
of legs, skin cyanosis, small ulcers on
one side of the lateral condyle. Exami- A. Prednisolone
nation revealed a swelling, enlarged vei- B. Hypothiazide
ns, formation of nodes. The pathological C. Digoxin
process has started in the following vein: D. Indometacin
E. Reserpine
A. V. saphena parva
B. V. saphena magna 61. A patient has been given high doses
C. V. femoralis of hydrocortisone for a long time. This
D. V. profunda femoris caused atrophy of one of the adrenal
E. V. iliaca externa cortex zones. Which zone is it?
57. A 1,5-year-old child presents with both A. Fascial
mental and physical lag, decolorizing of B. Glomerular
skin and hair, decrease in catecholamine C. Reticular
concentration in blood. When a few drops D. Glomerular and reticular
of 5% solution of trichloroacetic iron had E. -
been added to the child’s urine it turned
olive green. Such alteration are typical for 62. A 63-year-old patient with collapse
the following pathology of the amino acid presentations was delivered to the
metabolism: emergency hospital. A physician has
chosen noradrenalin against hypotension.
Krok 1 Medicine 2009 8

What is its mechanism of action? and mucous membranes revealed proline


and lysine within the collagen fibers. Di-
A. Activation of α1 -adrenoreceptors sorder of their hydroxylation is caused by
B. Activation of serotonin receptors lack of the following vitamin:
C. Activation of β-adrenoreceptors
D. Activation of dopamine receptors A. Vitamin C
E. Block of M-cholinoreceptors B. Vitamin K
C. Vitamin A
63. The greater amount of nitrogen is D. Vitamin B1
excreted from the organism in form of E. Vitamin E
urea. Inhibition of urea synthesis and
accumulation of ammonia in blood and 68. During a prophylactic medical exami-
tissues are induced by the decreased acti- nation a 7-year-old boy was diagnosed
vity of the following liver enzyme: with daltonism. His parents are healthy
and have normal colour vision, but his
A. Carbamoyl phosphate synthetase grandfather on his mother’s side has the
B. Aspartate aminotransferase same abnormality. What is the type of the
C. Urease abnormality inheritance?
D. Amylase
E. Pepsin A. Recessive, sex-linked
B. Dominant, sex-linked
64. A patient suffering from syphilis has C. Semidominance
been treated with bismuth preparations. D. Autosomal recessive
As a result of it some grey spots turned E. Autosomal dominant
up on the mucous membrane of the oral
cavity; nephropathy symptoms were also 69. In a healthy adult speed of the
present. What drug should be used for excitement conduction through the atri-
treatment of bismuth intoxication? oventricular node is 0,02-0,05 m/sec. Atri-
oventricular delay enables:
A. Unithiol
B. Methylene blue A. Sequence of atrial and ventricular
C. Naloxone contractions
D. Bemegride B. Simultaneity of both atria contractions
E. Nalorphine C. Simultaneity of both ventricles
contractions
65. ECG of a patient shows such alterati- D. Sufficient force of atrial contractions
ons: P -wave is normal, P − Q-interval is E. Sufficient force of ventricular contracti-
short, ventricular QRST complex is wi- ons
de, R-wave is double-peak or two-phase.
What form of arrhythmia is it? 70. A histological specimen of a kidney
shows a part of the distal tubule going
A. WPW syndrome (Wolff-Parkinson- between the afferent and efferent arteri-
White) ole. The cells building the tubule wall have
B. Frederick’s syndrome (atrial flutter) dense nuclei; basal membrane is absent.
C. Atrioventricular block Such structural formation is called:
D. Ventricular fibrillation
E. Ciliary arrhythmia A. Macula densa
B. Juxtaglomerular cells
66. In a histological specimen parenchyma C. Mesangial cells
of an organ is represented by lymphoid ti- D. Juxtavascular cells
ssue that forms lymph nodes; the latter are E. -
arranged in a diffuse manner and enclose
a central artery. What anatomic formation 71. Stimulation of an excitable cell by the
has such morphological structure? electric current has led to the depolarizati-
on of its membrane. The depolarization
A. Spleen has been caused mainly by the following
B. Tonsil ions penetrating into the cell through its
C. Lymph node membrane:
D. Thymus
E. Red bone marrow
67. Examination of a patient with frequent
haemorrhages from the internal organs
Krok 1 Medicine 2009 9

A. Na+ blurring of its structure, absence of


B. HCO 3− lymphoid follicles; all the microscopic fi-
C. Ca2+ elds showed cells with roundish nuclei and
D. Cl− thin limbus of basophil cytoplasm. It is
E. K + known from the clinical data that other
groups of lymph nodes are also enlarged
72. Products of some proteins hydrolysis as well as spleen and liver. What disease
and modification are the biologically might be suspected?
active substances called hormones. Li-
potropin, corticotropin, melanotropin A. Lymphoid leukosis
and endorphins are synthesized in the B. Lymphogranulomatosis
hypophysis of the following protein: C. Lymphosarcoma
D. Myeloid leukosis
A. Proopiomelanocortin (POMC) E. Multiple myeloma
B. Neuroalbumin
C. Neurostromin 77. Two weeks after lacunar tonsillitis a 20-
D. Neuroglobulin year-old man started complaining about
E. Thyreoglobulin general weakness, lower eyelid edemata.
After examination the patient was di-
73. In patients with the biliary tract agnosed with acute glomerulonephritis.
obstruction the blood coagulation is What are the most likely pathological
inhibited; the patients have frequent changes in the urine formula?
haemorrhages caused by the subnormal
assimilation of the following vitamin: A. Proteinuria
B. Cylindruria
A. K C. Presence of fresh erythrocytes
B. A D. Pyuria
C. D E. Natriuria
D. E
E. C 78. A 38-year-old patient came to a
traumatology centre and complained
74. A 48-year-old patient was admitted about an injury of his right hand. Objecti-
to the hospital with complaints about vely: the patient has a cut wound in the
weakness, irritability, sleep disturbance. region of the thenar eminence on the right
Objectively: skin and scleras are of yellow hand; distal phalanx of the I finger cannot
colour. In blood: increased concentration be flexed. What muscle was injured?
of total bilirubin with prevailing direct bi-
lirubin. The feces are acholic. The urine is A. Long flexor muscle of thumb
dark (contains bile pigments). What type B. Short flexor muscle of thumb
of jaundice is it? C. Short abductor muscle of thumb
D. Opposer muscle of thumb
A. Mechanic E. Abductor muscle of thumb
B. Haemolytic
C. Parenchymatous 79. Lung ventilation in a person is
D. Gilbert’s syndrome increased as a result of physical activi-
E. Crigler-Najjar syndrome ty. Which of the following indices of the
external respiration is much higher than
75. A bacteriological laboratory has recei- in a state of rest?
ved smears from the sputum of a pati-
ent with a chronic pulmonary disease. A. Respiratory volume
Microscopical examination of the smears B. Vital capacity of lungs
stained by the Ziehl-Neelsen technique C. Inspiratory reserve volume
revealed red bacilli. What property of the D. Expiratory reserve volume
tuberculosis bacillus has shown itself? E. Total lung capacity
A. Acid resistance 80. As a result of continuous starvati-
B. Alkali resistance on the glomerular filtration rate has
C. Alcohol resistance increased by 20%. The most probable
D. Capsule formation cause of the glomerular filtration alterati-
E. Spore formation on under the mentioned conditions is:
76. Microscopical examination of an
enlarged cervical lymph node revealed
Krok 1 Medicine 2009 10

A. Decrease in the oncotic pressure of A. Atrioventricular block


blood plasma B. Extrasystole
B. Increase in the systemic arterial pressure C. Sinus bradycardia
C. Increase in the permeability of the renal D. Ciliary arrhythmia
filter E. Sinoatrial block
D. Increase of the filtartion quotient
E. Increase of the renal blood flow 85. A patient suffering from coronary
artery disease had taken a certain
81. After transfusion of 200 ml of blood a drug many times a day in order to
patient presented with body temperature arrest stenocardia attacks. Overdose of
rise up to 37, 9o C. Which of the followi- this drug finally caused intoxication.
ng substances is the most likely cause of Objectively: cyanotic skin and mucous
temperature rise? membranes, dramatic fall in the arterial
pressure, tachycardia, respiration inhibiti-
A. Interleukin-1 on. Blood has increased concentration of
B. Interleukin-2 methemoglobin. The drug the patient had
C. Tumour necrosis factor taken relates to the following group:
D. Interleukin-3
E. Interleukin-4 A. Organic nitrates
B. α-adrenoceptor blockers
82. A man who is riding the carousel C. Calcium channel blockers
presents with increased heart rate, sweati- D. Adenosine drugs
ng, nausea. This condition is caused pri- E. Myotropic spamolytics
marily by the stimulation of the following
receptors: 86. A patient with android-type obesi-
ty had been suffering from arterial
A. Vestibular ampullar hypertension, hyperglycemia, glycosuria
B. Proprioceptors for a long time and died from the cerebral
C. Vestibular otolithic haemorrhage. Pathologic examination
D. Auditory revealed pituitary basophil adenoma,
E. Visual adrenal cortex hyperplasia. What is the
most likely diagnosis?
83. A worker of a cattle farm fell acutely
ill and then died from the progressing A. Itsenko-Cushing’s syndrome
intoxication. Autopsy revealed enlarged, B. Diabetes mellitus
hyposthenic spleen of dark-cherry colour C. Acromegalia
when dissected; excessive pulp scraping. D. Pituitary nanism
At the base and fornix of brain pia maters E. Adiposogenital dystrophy
are edematous, soaked with blood, dark-
red ("scarlet hat"). Microscopic exami- 87. A middle-aged man went to a foreign
nation revealed serous haemorrhagic country because he had been offered a job
inflammation of brain tissues and tuni- there. However he had been unemployed
cs along with destruction of small vessel for quite a long time. What endocrine
walls. What is the most likely diagnosis? glands were exhausted most of all in this
man?
A. Anthrax
B. Tularemia A. Adrenal glands
C. Brucellosis B. Parathyroid glands
D. Plaque C. Seminal glands
E. Cholera D. Substernal gland
E. Thyroid gland
84. A 49-year-old patient consulted a
doctor about increased fatigability and 88. A girl has been diagnosed with
dyspnea provoked by physical activity. adrenogenital syndrome (pseudohermaphrodi-
ECG results: heart rate - 50/min, P Q- tism). This pathology is caused by
interval is prolonged, QRS- complex hypersecretion of the following adrenal
is unchanged, the number of P -waves hormone:
exceeds the number of QRS-complexes.
What type of arrhythmia is it?
Krok 1 Medicine 2009 11

A. Androgens typical for the following vessel:


B. Estrogens
C. Mineralocorticoids A. Muscular-type artery
D. Glucocorticoids B. Elastic-type artery
E. Catecholamines C. Capillary
D. Non-muscular vein
89. A woman delivered a dead child wi- E. Muscular-type vein
th multiple developmental defects. What
protozoan disease might have caused the 94. Cooling of the human body in water is
intrauterine death? much more faster than in the air. What
way of heat emission in water is much
A. Toxoplasmosis more effective?
B. Leishmaniasis
C. Malaria A. Heat conduction
D. Amebiasis B. Convection
E. Lambliasis C. Heat radiation
D. Sweat evaporation
90. A patient has a massive haemorrhage E. -
caused by damage of the dorsal li-
ngual artery by cancer of tongue back. 95. An 18-year-old man was delivered to
What vessel should be ligated for the the hospital after a road accident. Exami-
haemorrhage arrest? nation at the traumatological department
revealed multiple injuries of soft tissues of
A. Lingual artery face in the region of the medial eye angle.
B. Dorsal lingual artery The injuries caused massive haemorrhage.
C. Deep artery of tongue What arterial anastomosis might have
D. Facial artery been damaged in this region?
E. Ascending pharyngeal artery
A. a. carotis externa et a. carotis interna
91. A 12-year-old teenager has signifi- B. a. carotis externa et a. subclavia
cantly put off weight within 3 months; C. a. carotis interna et a. subclavia
glucose concentration rose up to 50 mi- D. a. subclavia et a. ophthalmica
llimoleł. He fell into a coma. What is the E. a. carotis interna et a. ophthalmica
main mechanism of its development?
96. After a surgery a 36-year-old woman
A. Hyperosmolar was given an intravenous injection of
B. Hypoglycemic concentrated albumin solution. This has
C. Ketonemic induced intensified water movement in
D. Lactacidemic the following direction:
E. Hypoxic
A. From the intercellular fluid to the
92. As a result of increased permeabi- capillaries
lity of the erythrocyte membrane B. From the intercellular fluid to the cells
in a patient with microspherocytic C. From the cells to the intercellular fluid
anaemia (Minkowsky-Shauffard disease) D. From the capillaries to the intercellular
cells receive sodium ions and water. fluid
Erythrocytes take form of spherocytes E. No changes of water movement will be
and can be easily broken down. What observed
is the leading mechanism of erythrocyte
damage in this case? 97. Autopsy of a man with a mali-
gnant stomach tumour who had di-
A. Electrolytic osmotic ed from cancer intoxication revealed
B. Calcium in the posteroinferior lung fields some
C. Acidotic dense, grayish-red irregular foci protrudi-
D. Protein ng above the section surface. Microscopic
E. Nucleic examination revealed exudate containi-
ng a large amount of neutrophils in the
93. A histological specimen shows a lumen and walls of small bronchi and
blood vessel. Its inner coat is composed alveoles. Such pulmonary alterations indi-
by endothelium, subendothelium and cate the following disease:
internal elastic membrane. The middle
coat is enriched with smooth myocytes.
Such morphological characteristics are
Krok 1 Medicine 2009 12

A. Acute purulent bronchopneumonia has been released from the repressor


B. Acute bronchitis gene. Immediately after this the followi-
C. Croupous pneumonia ng process will start in the cell:
D. Intermittent pneumonia
E. Acute serous bronchopneumonia A. Transcription
B. Translation
98. Autopsy of a 1,5-year-old chi- C. Replication
ld revealed haemorrhagic skin rash, D. Processing
moderate hyperaemia and edema of E. Repression
nasopharyngeal mucous membrane, small
haemorrhages in the mucous membranes 103. While determining power inputs of a
and internal organs; dramatic dystrophic patient’s organism it was established that
alterations in liver and myocardium; acute the respiratory coefficient equaled 1,0.
necrotic nephrosis; massive haemorrhages This means that in the cells of the pati-
in the adrenal glands. What disease are ent the following substances are mainly
these alterations the most typical for? oxidized:
A. Meningococcal infection A. Carbohydrates
B. Scarlet fever B. Proteins
C. Diphtheria C. Fats
D. Measles D. Proteins and carbohydrates
E. Epidemic typhus E. Carbohydrates and fats
99. During examination of an 11- 104. During an experiment the dorsal
month-old infant a pediatrician revealed roots of the spinal cord of an animal have
osteoectasia of the lower extremities and been cut. What changes will be observed
delayed mineralization of cranial bones. in the innervation zone?
Such pathology is usually provoked by the
deficit of the following vitamin: A. Sensitivity loss
B. Loss of motor functions
A. Cholecalciferol C. Decrease in muscle tone
B. Thiamin D. Increase in muscle tone
C. Pantothenic acid E. Sensitivity loss and loss of motor functi-
D. Bioflavonoids ons
E. Riboflavin
105. An experimental animal has lost ori-
100. Examination of a patient suffering entative reflexes as a result of destructi-
from chronic hepatitis revealed a signifi- on of certain brainstem structures. What
cant decrease in the synthesis and secreti- structures had been destroyed?
on of bile acids. What process will be mai-
nly disturbed in the patient’s bowels? A. Quadrigeminal plate
B. Medial nuclei of the reticular formation
A. Fat emulsification C. Red nuclei
B. Protein digestion D. Vestibular nuclei
C. Carbohydrate digestion E. Black substance
D. Glycerin absorption
E. Amino acid absorption 106. A patient has osmotic pressure of
blood plasma at the rate of 350 mOsmol/l
101. Retrospective diagnostics of bacterial (norm is 300 mOsmol/l). This will cause
dysentery involved serological analysis of hypersecretion of the following hormone:
blood serum intended for determination
of Shigella antibody titre. Which of the A. Vasopressin
following reactions should be applied for B. Aldosterone
this purpose? C. Cortisol
D. Adrenocorticotropin
A. Passive haemagglutination E. Natriuretic
B. Complement binding
C. Precipitation 107. After a sprint an untrained person
D. Haemolysis develops muscle hypoxia. This leads to the
E. Bacteriolysis accumulation of the following metabolite
in muscles:
102. You are studying functioning of
a bacteria operon. The operator gene
Krok 1 Medicine 2009 13

A. Lactate
B. Ketone bodies A. Intensification of leukopoiesis
C. Acetyl CoA B. Redistribution of the leukocytes in the
D. Glucose 6-phosphate organism
E. Oxaloacetate C. Deceleration of leukocyte breakdown
D. Deceleration of leukocyte migration to
108. Cytoplasm of the myocytes contains a the tissues
lot of dissolved metabolites resulting from E. Intensification of leukopoiesis and
glucose oxidation. Name the metabolite deceleration of leukocyte migration to the
that turns directly into lactate: tissues
A. Pyruvate 113. A 35-year-old patient complains
B. Oxaloacetate about having severe rhinitis and loss of
C. Glycerophosphate sense of smell for a week. Objectively: the
D. Glucose-6-phosphate nasal cavity contains a lot of mucus coveri-
E. Fructose-6-phosphate ng the mucous membrane and blocking
olfactory receptors. In what region of the
109. A young man complains about nasal cavity are these receptors located?
urination disorder. Examination of the
external genitals revealed that the urethra A. Superior nasal concha
was split and urine could flow out of this B. Median nasal concha
orifice. What anomaly of the external C. Inferior nasal concha
genitals development is it? D. Common nasal meatus
E. Vestibule of nose
A. Epispadia
B. Phimosis 114. A 10-year-old child had the mantoux
C. Hermaphroditism tuberculin test administered. 48 hours
D. Paraphimosis later a papule up to 8 mm in diameter
E. Hypospadia appeared on the site of the injection.
What type of hypersensitivity reaction
110. A patient consulted an urologist developed after the tuberculin injection?
about pain during urination. Analysis of
his urine taken in the daytime revealed A. Type IV hypersensitivity reaction
eggs with a characteristic sharp point. It is B. Arthus phenomenon
known from the anamnesis that the pati- C. Seroreaction
ent has recently returned from Australia. D. Atopic reaction
What is the most likely diagnosis? E. Type II hypersensitivity reaction
A. Urogenital schistosomiasis 115. Vitamin B1 deficiency causes di-
B. Intestinal schistosomiasis sturbance of oxidative decarboxylati-
C. Japanese schistosomiasis on of α-ketoglutaric acid. This leads to
D. Opisthorchiasis the impaired synthesis of the following
E. Dicroceliasis coenzyme:
111. Colonoscopy of a patient with A. Thiamine pyrophosphate
dysentery revealed that the mucous B. Nicotinamide adenine dinucleotide
membrane of the large intestine was C. Flavine adenine dinucleotide
hyperemic, edematic, and its surface was D. Lipoic acid
covered with grey-and-green layerings. E. Coenzyme A
What morphological form of dysenteric
colitis is it? 116. On the ground of clinical presentati-
ons a patient was prescribed pyri-
A. Fibrinous doxal phosphate. This medication is
B. Catarrhal recommended for correction of the
C. Ulcerous following processes:
D. Purulent
E. Necrotic A. Transamination and decarboxylation of
amino acids
112. Examination of a patient 24 hours B. Oxidative decarboxylation of ketonic
after appendectomy revealed neutrophi- acids
lic leukocytosis with a regenerative shi- C. Desamination of purine nucleotides
ft. What is the most likely mechanism of D. Synthesis of purine and pyrimidin bases
leukocytosis development? E. Protein synthesis
Krok 1 Medicine 2009 14

117. A patient with clinical presentati- A. Immediate hypersensitivity


ons of immunodeficiency has undergone B. Delayed-type hypersensitivity
immunological tests. They revealed signi- C. Reaction of transplantation immunity
ficant decrease in number of cells that D. Normergic reaction
form rosettes with sheep erythrocytes. E. Granulomatosis
What conclusion can be drown on the
ground of the analysis data? 121. While examining the oral cavity a
stomatologist revealed inflammation of
A. Decrease in T -lymphocyte level papillae on the border of the median
B. Decrease in B-lymphocyte level and posterior third of the back of tongue.
C. Decrease in natural killer level (NK- What papillae are inflamed?
cells)
A. Papillae vallatae
D. Decrease in complement system level B. Papillae fungiformes
E. Lack of effector cells of the humoral C. Papillae foliatae
immunity
D. Papillae filiformes
118. A man had worked in a coal mi- E. Papillae conicae
ne for over 20 years. After his death 122. Autopsy of a 50-year-old man
autopsy revealed that his lungs were
revealed the following changes: his right
dense, grayish-black and had large areas lung was moderately compact in all parts,
of neogenic connective tissue containing the dissected tissue was found to be ai-
a lot of microphages with black pigment
rless, fine-grained, dryish. Visceral pleura
in the cytoplasm. What is the most likely had greyish-brown layers of fibrin. What
diagnosis?
is the most likely diagnosis?
A. Anthracosis
A. Croupous pneumonia
B. Anthracosilicosis
B. Tuberculosis
C. Silicoanthracosis C. Bronchopneumonia
D. Talcosis
D. Interstitial pneumonia
E. Siderosis E. Pneumofibrosis
119. Autopsy of a man who died 123. In the pubertal period cells of
from sepsis revealed a phlegmonous
the male sexual glands start producing
inflammation in the femoral bone of lower
the male sexual hormone testosterone
extremity. The inflammation was seen that is responsible for formation of the
in the bone marrow, haversian canals
secondary sexual characters. What cells
and periosteum. There were also multi- of the male sexual glands produce this
ple abscesses underneath the periosteum; hormone?
the surrounding soft tissues of the thigh
were also affected by the phlegmonous A. Leidig cells
inflammation. What pathological process B. Sustenocytes
is it? C. Sertoli’s cells
D. Sustentacular cells
A. Acute haematogenous osteomyelitis
E. Spermatozoa
B. Osteoporosis
C. Chronic haematogenous osteomyelitis 124. Examination of a patient revealed
D. Osteopetrosis overgrowth of facial bones and soft
E. - tissues, tongue enlargement, wide
interdental spaces in the enlarged dental
120. After the prior sensibilization
arch. What changes of the hormonal
an experimental animal was given a
secretion are the most likely?
subcutaneous injection of an antigen.
The place of injection exhibited a fi- A. Hypersecretion of the somatotropic
brinous inflammation with alteration of hormone
the vessel walls, basal substance and fi- B. Hyposecretion of the somatotropic
brous structures of the connective tissue hormone
in form of mucoid and fibrinoid swelling C. Hypersecretion of insulin
and necrosis. What immunological reacti- D. Hyposecretion of thyroxin
on is it? E. Hyposecretion of insulin
125. Autopsy of a man, who had been
suffering from the multiple bronchiectasis
Krok 1 Medicine 2009 15

for 5 years and died from chronic renal A. Glibenclamid


insufficiency, revealed that kidneys were B. Insulin
dense and enlarged, with thickened corti- C. Retabolil
cal layer of white colour with greasy lustre. D. Lovastatin
What renal disease might be suspected? E. Amlodipine
A. Secondary amyloidosis 130. A 32-year-old patient consulted a
B. Glomerulonephritis doctor about the absence of lactation
C. Chronic pyelonephritis after parturition. Such disorder might be
D. Necrotic nephrosis explained by the deficit of the following
E. - hormone:
126. Autopsy of a 49-year-old woman A. Prolactin
who died from chronic renal insufficiency, B. Somatotropin
revealed: kidneys were dense, reduced, C. Vasopressin
multicoloured, with haemorrhagic areas. D. Thyrocalcitonin
Microscopic examination revealed some E. Glucagon
hematoxylin bodies in the nuclei of
the renal tubule epithelium; "wire- 131. Preventive examination of a pati-
loop"thickening of the glomerular capi- ent revealed an enlarged lymph node of
llary basement membrane; here and there metastatic origin on the medial wall of the
in the capillaries some hyaline thrombi left axillary crease. Specify the most likely
and foci of fibrinoid necrosis were present. localization of the primary tumour:
What is the most likely diagnosis? A. Mammary gland
A. Systemic lupus erythematosus B. Submandibular salivary gland
B. Rheumatism C. Lung
C. Arteriosclerotic pneumosclerosis D. Stomach
D. Amyloidosis E. Thyroid gland
E. Atherosclerotic nephrosclerosis 132. A man suffering from osteochondrosis
127. As a result of durative antibiotic got acute pain in the abdominal muscles
therapy a 37-year old patient developed (lateral and anterior). During objective
intestinal dysbacteriosis. What type of examination a physician diagnosticated
drugs should be used in order to normali- increased pain sensitivity of skin in the
ze intestinal microflora? hypogastric region. This pain might be
caused by affection of the following nerve:
A. Eubiotics
B. Sulfanilamides A. Iliohypogastric
C. Bacteriophages B. Sciatic
D. Autovaccines C. Obturator
E. Vitamins D. Femoral
E. Genitofemoral
128. A married couple came to the genetic
counseling. The husband suffers from the 133. Pyeloureterography X-ray photo
insulin-dependant diabetes, the wife is showed a renal pelvis with minor calyces
healthy. What is the probability that this only (major calyces were absent). What
couple will have an insulin-dependant chi- form of urinary tracts of a kidney was
ld? revealed?
A. Higher than throughout the population A. Embryonal
B. The same as throughout the population B. Fetal
C. Lower than throughout the population C. Mature
D. 100% D. Ampullar
E. 50% E. -
129. An elderly female patient suffers 134. Autopsy of a man who died
from the type 2 diabetes mellitus from influenza revealed that the heart
accompanied by obesity, atherosclerosis, was slightly enlarged and pastose. The
coronary artery disease. Basal hyperi- surface of the incision of myocardium
nsulinemia is also present. What appeared to be pale, with specks. Mi-
treatment would be the most appropriate? croscopic examination revealed signs of
parenchymatous adipose and hydropic
Krok 1 Medicine 2009 16

degeneration, edematic stroma with scant A. Retinol acetate


lymphocytic and macrophage infiltration; B. Cyanocobalamin
plethoric vessels; perivascular petechial C. Pyridoxine hydrochloride
haemorrhages. What type of myocarditis D. Ascorbic acid
is it? E. Nicotinic acid
A. Serous diffuse 139. A female patient consulted a doctor
B. Interstitial proliferative about pain and limited movements in
C. Serous focal the knee joints. Which of the followi-
D. Purulent ng nonsteroid anti-inflammatory drugs
E. Granulomatous should be administered taking into consi-
deration that the patient has a history of
135. In spite of treatment with cardiotoni- chronic gastroduodenitis?
cs and thiazide diuretic a patient suffering
from chronic cardiac failure still presents A. Celecoxib
with edemata and faces a risk of ascites. B. Diclofenac sodium
What medication should be administered C. Promedol
in order to increase the diuretic effect of D. Acetylsalicilic acid
the above mentioned drugs? E. Butadiounum
A. Spironolactone 140. Depressions and emotional insaniti-
B. Furosemide es result from the deficit of noradrenalin,
C. Amiloride serotonin and other biogenic amines in
D. Clopamide the brain. Their concentration in the
E. Manithol synapses can be increased by means of the
antidepressants that inhibit the following
136. Gynecological examination of the enzyme:
uterine cervix in a 30-year-old woman
revealed some bright-red lustrous spots A. Monoamine oxidase
that easily bleed when touched. Biopsy B. Diamine oxidase
showed that a part of the uterine cervix C. L-amino-acid oxidase
was covered with cylindrical epithelium D. D-amino-acid oxidase
with papillary outgrowths; in the depth of E. Phenylalanine-4-monooxygenase
tissue the growth of glands was present.
What pathology of the uterine cervix was 141. A patient presents with icteriti-
revealed? ousness of skin, scleras and mucous
membranes. Blood plasma the total bili-
A. Pseudoerosion rubin is increased, stercobilin is increased
B. True erosion in feces, urobilin is increased in urine.
C. Endocervicitis What type of jaundice is it?
D. Glandular hyperplasia
E. Leukoplakia A. Haemolytic
B. Gilbert’s disease
137. A stillborn child was found to have C. Parenchymatous
thickened skin resembling of the tortoi- D. Obturational
se shell, underdeveloped auricles. Hi- E. Cholestatic
stological examination of skin revealed
hyperkeratosis, atrophy of the granular 142. A pathology-histology laboratory
epidermis layer; inflammatory changes received a vermiform appendix up to 2,0
were not present. What is the most likely cm thick. Its serous membrane was pale,
diagnosis? thick and covered with yellowish-green fi-
lms. The wall was flaccid, of grayish-red
A. Ichthyosis colour. The appendix lumen was dilated
B. Leukoplakia and filled with yellowish-green substance.
C. Xerodermia Histological examination revealed that
D. Erythroplakia the appendix wall was infiltrated with
E. Dermatomyositis neutrophils. Specify the appendix disease:
138. A patient presents with twilight visi-
on impairment. Which of the following
vitamins should be administered?
Krok 1 Medicine 2009 17

A. Acute phlegmonous appendicitis 147. A student came to see a doctor


B. Acute gangrenous appendicitis and asked to administer him a drug for
C. Acute superficial appendicitis treatment of allergic rhinitis that occurs in
D. Acute simple appendicitis the period of linden flowering. What drug
E. Chronic appendicitis may be used?
143. While on holiday in the countryside A. Loratadine
a boy found a spider with the followi- B. Noradrenaline hydrotartrate
ng morphological peculiarities: body C. Propanolol
length of 2 cm, round black abdomen D. Ambroxol
with two rows of red dots on its dorsal E. Losartan
surface, four pairs of segmented extremi-
ties covered with tiny black hairs. Identify 148. In an embryo the process of dorsal
this arthropod: mesoderm segmentation and somite
formation is disturbed. What part of
A. Karakurt spider skin will probably have developmental
B. Scorpion abnormalities?
C. Solifugae
D. Mite A. Dermis
E. Tarantula B. Hair
C. Sebaceous glands
144. As a result of a trauma a patient has D. Epidermis
developed traumatic shock that led to the E. Perspiratory glands
following disorders: AP is 140/90 mm Hg,
Ps is 120 bpm. The patient is fussy, talkati- 149. A 9-month-old infant is fed with arti-
ve, pale. Such state relates to the following ficial formulas with unbalanced vitamin
shock phase: B6 concentration. The infant presents
with pellagral dermatitis, convulsions,
A. Erectile anaemia. Convulsion development might
B. Latent period be caused by the disturbed formation of:
C. Terminal
D. Torpid A. GABA
E. - B. Histamine
C. Serotonin
145. Examination of a patient admitted to D. DOPA
the surgical department with symptoms of E. Dopamine
acute appendicitis revealed the following
changes in the white blood cells: the total 150. During an experiment the myotatic
count of leukocytes is 16 · 109 /l. Leukocyte reflex has been studied in frogs. After
formula: basophils - 0, eosinophils - 2%, extension in a skeletal muscle its
juvenile forms - 2%, stabnuclear - 8%, reflectory contraction was absent. The
segmentonuclear - 59%, lymphocytes reason for it might be a dysfunction of
- 25%, monocytes- 4%. The described the following receptors:
changes can be classified as:
A. Muscle spindles
A. Neutrophilia with regenerative left shift B. Nociceptors
B. Neutrophilia with right shift C. Articular
C. Neutrophilia with degenerative left shift D. Golgi tendon organs
D. Neutrophilic leukemoid reaction E. Tactile
E. Neutrophilia with hyperregenerative
left shift 151. During an experiment vagus
branches that innervate heart are being
146. After a trauma a patient lost ability stimulated. This has stopped conduction
of elbow extension. This might have been of excitement from the atria to the ventri-
caused by dysfunction of the following cles. The reason for it are electrophysical
main muscle: changes in the following structures:
A. m. triceps brachii A. Atrioventricular node
B. m. subscapularis B. His’ bundle
C. m. teres major C. Sinoatrial node
D. m. infraspinatus D. Ventricles
E. m. levator scapulae E. Atria
Krok 1 Medicine 2009 18

152. A man has worked in an Afri- on in the diuresis?


can country for 3 years. A month after
his return to Ukraine he consulted A. Arterial pressure drop
an ophthalmologist and complained B. Oncotic blood pressure drop
about eye ache, eyelid edema, lacrimati- C. Increase in vascular permeability
on and temporary visual impairment. D. Decrease in number of the functioning
Underneath the eye conjunctiva the glomerules
doctor revealed helminths 30-50 mm long E. Secondary hyperaldosteronism
with elongated filiform body. What di-
agnosis might be suspected? 157. Examination of a patient revealed
autoimmune haemolytic anaemia
A. Filariasis (cytotoxic type). What substances act as
B. Diphyllobothriasis antigens in the II-type allergic reactions?
C. Ascaridiasis
D. Enterobiasis A. Modified receptors of cell membranes
E. Trichocephaliasis B. Antibiotics
C. Hormones
153. After a disease a 16-year-old boy D. Serum proteins
is presenting with decreased function of E. Inflammation modulators
protein synthesis in the liver as a result of
vitamin K deficiency. This may cause di- 158. A patient suffers from the
sorder of: haemorrhagic syndrome that shows itself
in frequent nasal bleedings, posttraumatic
A. Blood coagulation and spontaneous intracutaneous and
B. Erythrocyte sedimentation rate intra-articular haemorrhages. After a
C. Anticoagulant production laboratory study a patient was diagnosed
D. Erythropoietin production with the type B haemophilia. This disease
E. Osmotic blood pressure is provoked by the deficit of the following
factor of blood coagulation:
154. In response to a change in body posi-
tion from horizontal to vertical blood A. IX
circulation system develops reflectory B. VIII
pressor reaction. Which of the following C. XI
is its compulsory component? D. V
E. VII
A. Systemic constriction of the venous
vessels 159. After a craniocerebral trauma a pati-
B. Systemic dilatation of the arterial resi- ent lost the ability to execute learned
stive vessels purposeful movements (apraxia). The
C. Decrease in the circulating blood injury is most likely localized in the
volume following region of the cerebral cortex:
D. Increase in the heart rate
E. Weakening of the pumbing ability of A. Gyrus supramarginalis
heart B. Gyrus angularis
C. Gyrus paracentralis
155. Life cycle of a cell includes a D. Gyrus lingualis
process of DNA autoreduplication. As E. Gyrus parahippocampalis
a result of this process monochromatid
chromosomes become bichromatid. This 160. A 58-year-old patient suffers from
phenomenon is observed within the the cerebral atherosclerosis. Examinati-
following period of the cell cycle: on revealed hyperlipoidemia. What class
of lipoproteins will most probably show
A. S increase in concentration in this patient’s
B. G0 blood serum?
C. G1
D. G2 A. Low-density lipoproteins
E. M B. High-density lipoproteins
C. Fatty acid complexes with albumins
156. As a result of a road accident a driver D. Chylomicrons
has gotten a trauma. Now he is in shock E. Cholesterol
condition and presents with a decrease in
daily diuresis down to 300 ml. What is the 161. A patient consulted a physician about
main pathogenetic factor of such alterati- muscle rigidity, constrained movements,
Krok 1 Medicine 2009 19

permanent arm tremor. The patient was 166. A man presents with increased heart
diagnosed with Parkinson’s disease. What rate, mydriatic pupils, dry mouth. This
preparation should be administered? condition results from the activation of
the following system of function regulati-
A. Levodopa on:
B. Phenytoin
C. Phenobarbital A. Sympathetic
D. Diazepam B. Parasympathetic
E. Ethosuximide C. Metasympathetic
D. Vago-insular
162. A patient with coronary artery di- E. Hypothalamo-pituitary-adrenal
sease was admitted to the cardiological
department. For stenocardia prevention 167. A patient with a limb fracture must
a drug from the group of β-adrenoceptor be administered a depolarizing drug from
blockers was administered. What drug is the myorelaxant group for the purpose of
it? a short-time surgery. What drug is it?
A. Metoprolol A. Dithylinum
B. Atropine sulfate B. Tubocurarine chloride
C. Morphine hydrochloride C. Cytitonum
D. Oxytocin D. Atropine sulfate
E. Furosemide E. Pentaminum
163. A 28-year-old female patient 168. A patient suffering from chronic
consulted a gynecologist about sterility. bronchitis takes a synthetic mucolytic
Examination revealed underdeveloped drug that facilitates the sputum thinning.
ovaries and uterus, irregular menstrual What drug is it?
cycle. Analysis of the sex chromatin
revealed 2 Barr’s bodies in most somatic A. Acetylcysteine
cells. What chromosome disease is most B. Diazepam
likely? C. Heparin
D. Furosemide
A. Triple X syndrome E. Enalapril
B. Edwards’ syndrome
C. Patau’s syndrome 169. A patient with massive burns
D. Klinefelter’s syndrome developed acute renal insufficiency
E. Turner’s syndrome characterized by a significant and
rapid deceleration of glomerular fi-
164. A patient with drug intoxication ltration. What is the mechanism of its
presented with the dryness of oral mucous development?
membrane and mydriatic pupils. Such
action of this drug is associated with the A. Reduction of renal blood flow
following effect: B. Damage of glomerular filter
C. Reduction of functioning nephron
A. Muscarinic cholinoreceptor block number
B. Muscarinic cholinoreceptor stumulation D. Rise of pressure of tubular fluid
C. Nicotinic cholinoreceptor stumulation E. Renal artery embolism
D. Adrenoreceptor stimulation
E. Adrenoreceptor block 170. A patient with severe course of respi-
ratory viral infection presented with clini-
165. A newborn develops dyspepsia cal signs of progressing heart failure that
after the milk feeding. When the mi- led to his death in the 2nd week of disease.
lk is substituted by the glucose solution Autopsy revealed that the heart caviti-
the dyspepsia symptoms disappear. The es were significantly dilated, the heart
newborn has the subnormal activity of the was flabby. Histological examination of
following enzyme: the myocardium revealed microvascular
plethora and diffuse stroma infiltration
A. Lactase with lymphocytes and histiocytes. What
B. Invertase is the most likely diagnosis?
C. Maltase
D. Amylase
E. Isomaltase
Krok 1 Medicine 2009 20

A. Myocarditis cells of the immune system:


B. Stenocardia
C. Acute coronary insufficiency A. T -lymphocytes-killers
D. Myocardial infarction B. Plasmoblasts
E. Cardiomyopathy C. T -lymphocytes-supressors
D. B-lymphocyte
171. During the regular sanitary- E. Stem cells
epidemiological inspection of a pharmacy,
the bacteriological analysis of air was 176. A 65-year-old man has purulent
performed. The air was found to have abscess on his neck. Analyses revealed
bacilli, yeast fungi, hemolytic streptococci, a culture of gram-positive cocci with
micrococci. Which of the detected mi- plasmocoagulase activity. This culture
croorganisms indicate the direct epidemic relates most likely to:
danger?
A. Staphylococcus aureus
A. Haemolytic streptococci B. Streptococcus pyogenes
B. Micrococci C. Staphylococcus epidermidis
C. Bacilli D. Staphylococcus saprophyticus
D. Yeast fungi E. -
E. -
177. In clinical practice tuberculosis is
172. A man with an injury in the nuchal treated with izoniazid preparation - that
region (regio nuchae) was admitted to the is an antivitamin able to penetrate into
resuscitation department. What muscle the tuberculosis bacillus. Tuberculostatic
occupies this region? effect is induced by the interference wi-
th replication processes and oxidation-
A. m. trapezius reduction reactions due to the buildup of
B. m. sternocleidomastoideus pseudo-coenzyme:
C. m. latissimus dorsi
D. m. rhomboideus minor A. NAD
E. m. scalenus anterior B. FAD
C. FMN
173. A section of the left lung was found D. TDP
to have an area of dense red tissue. The E. CoQ
area was cone-shaped, stood out distinctly
from the healthy tissue, with its base di- 178. A newborn child was found to have
rected to the pleura. The dissected tissue reduced intensity of sucking, frequent
was granular, dark-red. What is the most vomiting, hypotonia. Urine and blood
likely diagnosis? exhibit increased concentration of citrulli-
ne. What metabolic process is disturbed?
A. Haemorrhagic infarction
B. Lung abscess A. Ornithinic cycle
C. Lung gangrene B. Tricarboxylic acid cycle
D. Primary tuberculous affection C. Glycolysis
E. Croupous pneumonia D. Glyconeogenesis
E. Cori cycle
174. An animal has an increased tonus
of extensor muscles. This the result 179. Vagus nerves of an experimental ani-
of intensified information transmissi- mal have been cut on the both sides. What
on to the motoneurons of the spinal respiratory changes will result from this?
cord through the following descending
pathways: A. Respiration will become deep and
infrequent
A. Vestibulospinal B. Respiration will become shallow and
B. Medial corticospinal frequent
C. Reticulospinal C. Respiration will become deep and
D. Rubrospinal frequent
E. Lateral corticospinal D. Respiration will become shallow and
infrequent
175. Following exposure to radiation a E. There will be no respiratory changes
lot of mutant cells appeared in a pati-
ent. Some time later most of them were 180. Before tooth extraction a patient
detected and destroyed by the following was advised to take a certain drug for
Krok 1 Medicine 2009 21

haemorrhage prevention. What drug was A. Medial brachial cutaneous nerve


advised? B. Medial antebrachial cutaneous nerve
C. Radial nerve
A. Vicasolum D. Ulnar nerve
B. Heparin E. Axillary nerve
C. Asparcam
D. Magnesium sulfate 186. Examination of a young woman
E. Dimedrol revealed a tumour up to 3 cm in di-
ameter in form of a knot localized
181. A patient who has been taking a drug along the acoustic nerve. The tumour is
for a long time cannot abruptly stop its homogenous, soft and elastic, of pink-
use, because this may lead to psychic and and-white colour. Microscopically the
somatic dysfunctions. Name the syndrome tumour contains clusters of cells with oval
of different disorders caused by a drug wi- nuclei. Fibrous cell clusters form regular
thdrawal: structures made up by parallel rows of
regularly oriented cells arranged in form
A. Abstinence of a palisade. Zones between the rows
B. Sensibilization of cells are acellular and homogenous
C. Idiosyncrasy (Verocai bodies). What tumour is it?
D. Tachyphylaxis
E. Cumulation A. Neurinoma
B. Malignant neurinoma
182. A 45-year-old patient suffers from C. Ganglioneuroma
neurosis characterized by irritability, D. Neuroblastoma
sleeplessness, motiveless anxiety. What E. Ganglioneuroblastoma
drug would eliminate all the symptoms?
187. A patient with myocardial infarcti-
A. Diazepam on was admitted to the cardiological
B. Valerian extract department. For pain relief it was deci-
C. Pyracetam ded to potentiate fentanyl action wi-
D. Caffeine sodium benzoate th a neuroleptic. Which of the followi-
E. Levodopa ng neuroleptics is the most suitable for
183. After taking poor-quality food a pati- neuroleptanalgesia?
ent developed repeated episodes of di- A. Droperidol
arrhea. On the next day he presented with B. Aminazine
decreased arterial pressure, tachycardia, C. Triftazine
extrasystole. Blood pH is 7,18. These D. Haloperidol
abnormalities were caused by the E. Sulpiride
development of:
188. Treatment course of bacterial
A. Nongaseous acidosis pneumonia included benzylpenicillin
B. Gaseous acidosis sodium salt. What is the mechanism of
C. Nongaseous alkalosis its antimicrobial action?
D. Gaseous alkalosis
E. Metabolic alkalosis A. Inhibition of cell wall synthesis of the
microorganism
184. Which muscle contraction will be B. Inhibition of the intracellular protein
observed in the upper extremity during synthesis
holding (not moving) a load in a certain C. Inhibition of the cholinesterase activity
position? D. Inhibition of the SH enzyme groups of
the microorganisms
A. Isometric E. Antagonism with the paraaminobenzoic
B. Isotonic acid
C. Auxotonic
D. Concentric 189. A 49-year-old driver complains about
E. Excentric unbearable constricting pain behind the
breastbone irradiating to the neck. The
185. A patient has lost skin sensitivity in pain arose 2 hours ago. Objectively: the
the region of the medial surface of his patient’s condition is grave, he is pale,
shoulder. This is the result of dysfuncti- heart tones are decreased. Laboratory
on of the following nerve: studies revealed high activity of creatine
kinase and LDH1 . What disease are these
Krok 1 Medicine 2009 22

symptoms typical for? A. B, α (III) Rh−


B. A, β (II) Rh−
A. Acute myocardial infarction C. 0, α, β, (I) Rh+
B. Acute pancreatitis D. AB (IV ), Rh+
C. Stenocardia
D. Cholelithiasis E. AB (IV ), Rh−
E. Diabetes mellitus 194. Pharmacological effects of anti-
190. A 60-year-old patient presents wi- depressants are based upon blocking
th weakened peristaltic activity of the (inhibiting) the enzyme that acts as a
bowels. Which of the following foodstuffs catalyst for the breakdown of biogenic
would stimulate peristalsis most of all? amines noradrenalin and serotonin in the
mitochondria of cephalic neurons. What
A. Brown bread enzyme takes part in this process?
B. White bread
C. Meat A. Monoamine oxidase
D. Lard B. Transaminase
E. Tea C. Decarboxylase
D. Peptidase
191. After inoculation of the material E. Lyase
obtained from the pharynx of an angina
patient onto the blood-tellurite agar, grey 195. An oncological patient was admini-
colonies could be observed. They were stered methotrexate. With the lapse of
4-5 mm in diameter, radially striated (in time the target cells of the tumour lost
sensitivity to this preparation. We can
form of rosettes). Microscopical exami- observe changes in the gene expression
nation revealed gram-positive bacilli wi- of the following enzyme:
th clavate swollen ends arranged in form
of wide-spread fingers. Identify these mi- A. Dihydrofolate reductase
croorganisms: B. Thiminase
C. Desaminase
A. Diphtheria corynebacteria D. Folate oxidase
B. Clostridium botulinum E. Folate decarboxylase
C. Diphtheroids
D. Streptococci 196. A patient with a hypertensive cri-
E. Streptobacilli sis was admitted to the cardiological
department. He was given an intravenous
192. A microspecimen of the submandi- injection of an antihypertensive drug -
bular salivary gland shows some basket- alkali-earth metal salt. What drug was
shaped cells concentrated around the injected?
acines and excretory ducts. These cells
surround bases of the serous cells and A. Magnesium sulfate
are called myoepitheliocytes. These cells B. Potassium chloride
relate to the following tissue: C. Sodium hydrocarbonate
D. Calcium lactate
A. Muscular tissue E. Benzohexonium
B. Epithelial tissue
C. Neural tissue 197. An injured person was delivered to
D. Special connective tissue the hospital with a penetrating wound in
E. Loose fibrous connective tissue the left lateral region of abdomen. What
part of the large intestine is most likely
193. It was established that agglutinati- damaged?
on of the recipient’s blood erythrocytes
had been caused by the standard sera A. Colon descendens
from the I and II groups. Serum from B. Colon ascendens
the III group as well as anti-Rh serum C. Colon transverses
hadn‘t provoke any agglutination. Which D. Caecum
blood group and rhesus is allowed to be E. Rectum
transfused this recipient?
198. After a road accident a driver was
delivered to the hospital with an injury of
the medial epicondyle of humerus. What
nerve might be damaged in this case?
Krok 1 Medicine 2009 23

A. n. ulnaris 200. Examination of the anterior abdomi-


B. n. radialis nal wall of a pregnant woman revealed
C. n. axillaris a tumour-like formation that arose on
D. n. muscolocutaneus the spot of a tumour that was removed
E. n. medianus two years ago. The neoplasm was well-
defined, dense, 2х1 cm large. Histological
199. A man with a stab wound in the regi- examination revealed that the tumour was
on of the quadrilateral foramen consulted composed of differentiated connective ti-
a doctor about it. Examination revealed ssue with prevailing collagen fibres. What
that the injured couldn’t abduct his arm tumour might be suspected?
from the body. What nerve is most likely
damaged? A. Desmoid
B. Lipoma
A. n. axillaris C. Fibrosarcoma
B. n. medianus D. Hibernoma
C. n. radialis E. Leiomyoma
D. n. ulnaris
E. n. subclavius
Krok 1 Medicine 2010 1

1. An unconscious patient was deli- A. Tryptophan


vered to the admission ward. Objecti- B. Alanine
vely: the patient’s skin is cold, pupils are C. Histidine
myotic, he has laboured Cheyne-Stokes D. Methionine
respiration, arterial pressure is low, uri- E. Tyrosine
nary bladder is full. What substance has
caused intoxication? 6. Blood of a 12 year old boy presents low
concentration of uric acid and accumulati-
A. Narcotic analgetics on of xanthine and hypoxanthine. This
B. Tranquilizers child has genetic defect of the following
C. Non-narcotic analgetics enzyme:
D. Muscarinic receptor blockers
E. - A. Xanthine oxidase
B. Arginase
2. Characteristic sign of glycogenosis C. Urease
is muscle pain during physical work. D. Ornithine carbamoyltransferase
Blood examination reveals usually E. Glycerylkinase
hypoglycemia. This pathology is caused
by congenital deficiency of the following 7. A patient who suffers from insomnia
enzyme: caused by emotional disorder was prescri-
bed a hypnotic drug with tranquillizing
A. Glycogen phosphorylase effect. What hypnotic was prescribed?
B. Glucose 6-phosphate dehydrogenase
C. Alpha amylase A. Nitrazepam
D. Gamma amylase B. Phenobarbital
E. Lysosomal glycosidase C. Chloral hydrate
D. Sodium ethaminal
3. A woman works as railway traffic E. Bromisoval
controller. She suffers from seasonal
vasomotor rhinitis and gets treatment in 8. A patient is ill with diabetes melli-
the outpatient setting. She was prescribed tus accompanied by hyperglycemia on
an antihistamine that has no effect upon an empty stomach (7,2 millimole/l). The
central nervous system. What drug is it? hyperglycemia rate can be retrospecti-
vely estimated (over the last 4-8 weeks
A. Loratadine before the examination) on the ground
B. Dimedrol of the rate of the following blood plasma
C. Promethazine protein:
D. Suprastin
E. Tavegil A. Glycated hemoglobin
B. Albumin
4. During an operation a patient got C. Fibrinogen
injection of muscle relaxant dithylinum. D. C-reactive protein
Relaxation of skeletal muscles and inhibi- E. Ceruloplasmin
tion of respiration lasted two hours. This
condition was caused by absence of the 9. In course of laparotomy a surgeon
following enzyme in blood serum: revealed gangrenous lesion of descendi-
ng colon. It was caused by thrombosis of
A. Butyrylcholin esterase the following artery:
B. Catalase
C. Acetylcholinesterase A. Sinister colic
D. Glucose 6-phosphatase B. Median colic
E. Glutathione peroxidase C. Dexter colic
D. Ileocolic
5. Examination of a patient suffering from E. Superior mesenteric artery
cancer of urinary bladder revealed high
rate of serotonin and hydroxyanthranilic 10. A 60 year old patient has impai-
acid. It is caused by excess of the followi- red perception of high-frequency sounds.
ng amino acid in the organism: These changes were caused by damage of
the following auditory analyzer structures:
Krok 1 Medicine 2010 2

A. Main cochlea membrane near the oval


window A. Ribosomes
B. Main cochlea membrane near the B. Smooth endoplasmic reticulum
helicotrema C. Cell centre
C. Eustachian tube D. Lysosomes
D. Middle ear muscles E. Golgi apparatus
E. Tympanic membrane
16. Students who are taking examinations
11. A patient has low rate of magnesium often have dry mouth. The mechanism
ions that are necessary for affixion of ri- that causes this state is the realization of
bosomes to the endoplasmic reticulum. the following reflexes:
It is known that it causes disturbance
of protein biosynthesis. At what stage is A. Conditioned sympathetic
protein biosynthesis impaired? B. Unconditioned parasympathetic
C. Conditioned parasympathetic
A. Translation D. Unconditioned sympathetic
B. Transcription E. Unconditioned peripheral
C. Replication
D. Amino acid activation 17. Inflammation of the tympanic cavity
E. Termination (purulent otitis media) was complicated
by inflammation of mammillary process
12. A healthy man is in a region with hi- sockets. What wall of tympanic cavity
gh risk of catching malaria. What drug did the pus penetrate into the sockets
should be administered for individual through?
chemoprophylaxis of malaria?
A. Posterior
A. Chingamin B. Anterior
B. Sulfalen C. Medial
C. Tetracycline D. Lateral
D. Metronidazole E. Superior
E. Biseptol
18. Examination of a patient revealed
13. A full-term newborn child has yellowi- typical presentations of collagenosis. This
sh skin and mucous membranes. This mi- pathology is characterized by increase of
ght be probably caused by temporary defi- the following urine index:
ciency of the following enzyme:
A. Hydroxyproline
A. UDPglucoronyltransferase B. Arginine
B. Uridine transferase C. Glucose
C. Heme synthetase D. Mineral salts
D. Heme oxygenase E. Ammonium salts
E. Biliverdin reductase
19. Untrained people often have muscle
14. A patient ill with neurodermatitis has pain after sprints as a result of lactate
been taking prednisolone for a long time. accumulation. This might be caused by
Examination revealed high rate of sugar intensification of the following biochemi-
in his blood. This complication is caused cal process:
by the drug influence upon the following
link of carbohydrate metabolism: A. Glycolysis
B. Gluconeogenesis
A. Gluconeogenesis activation C. Pentose phosphate pathway
B. Glycogenogenesis activation D. Lipogenesis
C. Intensification of glucose absorption in E. Glycogenesis
the bowels
D. Inhibition of glycogen synthesis 20. Examination of a patient sufferi-
E. Activation of insulin decomposition ng from frequent haemorrhages in the
inner organs and mucous membranes
15. Labelled amino acids alanine and revealed proline and lysine being included
tryptophane were injected to a mouse in collagen fibers. Impairment of their
in order to study localization of protein hydroxylation is caused by lack of the
synthesis in its cells. The labelled ami- following vitamin:
no acids will be accumulated near the
following organellas:
Krok 1 Medicine 2010 3

A. C resulted in remission but the patient got


B. E exacerbation of chronic tonsillitis. This
C. K complication is induced by the following
D. A effect of glucocorticosteroids:
E. D
A. Immunosuppressive
21. During an experiment a skeletal B. Anti-inflammatory
muscle is stimulated by a series of electric C. Anti-shock
impulses. What type of muscle contracti- D. Antiallergenic
on will be observed provided that each E. Antitoxic
subsequent impulse comes in the peri-
od of shortening of the previous single 26. A 62 year woman complai-
muscle contraction? ns of frequent pain attacks in the
area of her chest and backbone, rib
A. Holotetanus fractures. Her doctor suspected myeloma
B. Partial tetanus (plasmocytoma). What of the following
C. Asynchronous tetanus laboratory characteristics will be of the
D. A series of single contractures greatest diagnostic importance?
E. Muscle contracture
A. Paraproteinemia
22. A 48 year old patient complai- B. Hyperalbuminemia
ned about intense pain, slight swelli- C. Proteinuria
ng and reddening of skin over the joi- D. Hypoglobulinemia
nts, temperature rise up to 38o C. Blood E. Hypoproteinemia
analysis revealed high concentration of
urates. This condition might be caused by 27. A newborn child has convulsions that
disturbed metabolism of: have been observed after prescription of
vitamin B6 . This most probable cause of
A. Purines this effect is that vitamin B6 is a componet
B. Collagen of the following enzyme:
C. Cholesterol
D. Pyrimidines A. Glutamate decarboxylase
E. Carbohydrates B. Pyruvate dehydrostase
C. Netoglubarate dehydromine
23. An experimantal animal that was kept D. Aminolevulinate synthase
on protein-free diet developed fatty li- E. Glycogen phosphorylase
ver infiltration, in particular as a result
of deficiency of methylating agents. This 28. A patient who has been abusing
is caused by disturbed generation of the tobacco smoking for a long time has got
following metabolite: cough accompanied by excretion of vi-
scous mucus; weakness after minor physi-
A. Choline cal stress, pale skin. The patient has also
B. DOPA lost 12,0 kg of body weight. Endoscopic
C. Cholesterol examination of biosy material his illness
D. Acetoacetate was diagnosed as squamous cell carci-
E. Linoleic acid noma. Name a pathological process that
preceded formation of the tumour:
24. A 46 year old woman suffering from
chololithiasis developed jaundice. Her A. Metaplasia
urine became dark-yellow and feces B. Hypoplasia
became colourless. Blood serum will have C. Hyperplasia
the highest concentration of the following D. Necrosis
substance: E. Sclerosis
A. Conjugated bilirubin 29. Bacterioscopic examination of a smear
B. Unconjugated bilirubin from the pharynx of a diphtheria suspect
C. Biliverdin revealed bacilli with volutine granules.
D. Mesobilirubin What etiotropic drug should be chosen
E. Urobilinogen in this case?
25. A patient suffering from infecti-
ous mononucleosis has been taking
glucocorticosteroids for two weeks. This
Krok 1 Medicine 2010 4

A. Antidiphtheritic antitoxic serum A. Obturation of biliary tracts


B. Bacteriophage B. Inflammation of mucous membrane of
C. Diphtheritic anatoxin small intestine
D. Eubiotic C. Lack of pancreatic lipase
E. Interferon D. Lack of pancreatic phospholipase
E. Unbalanced diet
30. A patient caught a cold after whi-
ch there appeared facial expression di- 35. Examination of a patient with pustular
sorder. He cannot close his eyes, raise his skin lesions allowed to isolate a causati-
eyebrows, bare his teeth. What nerve is ve agent that forms in the blood agar
damaged? roundish yellow middle-sized colonies
surrounded by haemolysis zone. Smears
A. Facial from the colonies contain irregular-
B. Vagus shaped clusters of gram-positive cocci.
C. Trigeminus The culture is oxidase- and catalase-
D. Glossopharyngeal positive, ferments mannitol and synthesi-
E. Infraorbital zes plasmocoagulase. What causative
agent was isolated?
31. A patient underwent an operation
on account of gall bladder excision that A. Staphylococcus aureus
resulted in obstruction of Ca absorption B. Streptococcus agalactiae
through the bowels wall. What vitamin C. Streptococcus pyogenes
wil stimulate this process? D. Staphylococcus epidermidis
E. Staphylococcus saprophyticus
A. D3
B. P P 36. A patient complained about being
C. C unable to adduct and abduct fingers in
D. B12 the metacarpophalangeal articulations
E. K towards and away from the 3rd finger.
Which muscles’ function is impaired?
32. A 70 year old female patient was di-
agnosed with fracture of left femoral neck A. Interosseous muscles
accompanied by disruption of ligament of B. Lumbrical muscles
head of femur. The branch of the followi- C. Breviflexors of fingers
ng artery is damaged: D. Long flexors of fingers
E. Extensors
A. Obturator
B. Femoral 37. A 44 year old woman complains of
C. External iliac general weakness, heart pain, significant
D. Inferior gluteal increase of body weight. Objectively:
E. Internal pudendal moon face, hirsutism, AP is 165/100 mm
Hg, height - 164 cm, weight - 103 kg; the
33. ECG of a patient shows prolongation fat is mostly accumulated on her neck,
of T-wave. This is caused by deceleration thoracic girdle, belly. What is the main
in ventricles of: pathogenetic mechanism of obesity?
A. Repolarization A. Increased production of glucocorticoids
B. Depolarization and repolarization B. Reduced production of thyroid
C. Depolarization hormones
D. Contraction C. Increased insulin production
E. Relaxation D. Reduced glucagon production
34. A patient complains of frequent di- E. Increased mineralocorticoid production
arrheas, especially after consumption of 38. A 34 year old woman was di-
rich food, weight loss. Laboratory exami- agnosed with hereditary microspherocytic
nation revealed steatorrhea; his feces
hemolytic anemia (Minkowsky-Shauffard
were hypocholic. What might have caused
such condition? disease). What mechanism caused
haemolysis of erythrocytes?
Krok 1 Medicine 2010 5

A. Membranopathy function is realized by the following form


B. Enzymopathy of vitamin A:
C. Hemoglobinopathy
D. Autoimmune disorder A. Trans-retinoic acid
E. Bone marrow hypoploasia B. Trans-retinal
C. Cis-retinal
39. According to the model of double D. Retinol
DNA helix that was suggested by Watson E. Carotin
and Creek, it was established that one of
chains would not be lost during replication 44. During preparation of a patient to a
and the second chain would be synthesi- heart surgery it was necessary to measure
zed complementary to the first one. What pressure in heart chambers. In one of
way of replication is it? them pressure varied from 0 mm Hg up
to 120 mm Hg within one cardiac cycle.
A. Semiconservative What heart chamber is it?
B. Analogous
C. Identical A. Left ventricle
D. Dispersed B. Right ventricle
E. Conservative C. Right atrium
D. Left atrium
40. A 4 year old child was admitted to E. -
the orthopaedic department with shin
fracture together with displacement. Bone 45. Examination of coronary arteries
fragments reposition requires preliminary revealed atherosclerotic calcified plaques
analgesia. What preparation should be closing vessel lumen by 1/3. The muscle
chosen? has multiple whitish layers of connective
tissue. What process was revealed in the
A. Promedol myocardium?
B. Analgin
C. Morphine hydrochloride A. Diffusive cardiosclerosis
D. Panadol B. Tiger heart
E. - C. Postinfarction cardiosclerosis
D. Myocarditis
41. A patient suffers from hepatoci- E. Myocardium infarction
rrhosis. State of antitoxic liver function
can be characterized by examination of 46. In order to determine toxigenicity of
the following substance exreted by urine: diphtheria bacilli a strip of filter paper
impregnated with antitoxic diphtherial
A. Hippuric acid serum was put on the dense nutrient medi-
B. Ammonium salts um. There were also inoculated a mi-
C. Creatinine crobal culture under examination and a
D. Uric acid strain that is known to be toxigenic. If
E. Amino acids the microbal culture under examinati-
on produces exotoxin, this wil result in
42. A 22 year old patient from the formation of:
West Ukraine complains of laboured
nasal breathing. Morphological exami- A. Precipitin lines
nation of biopsy material of nasal mucous B. Haemolysis zones
membrane revealed lymphoid, epitheli- C. Zones of diffuse opacification
oid, plasma cells as well as Mikulicz’s cells. D. Zones of lecithovitellinous activity
What is the most probable diagnosis? E. Precipitin ring
A. Rhinoscleroma 47. A 63 year old male patient who
B. Glanders had been suffering from chronic di-
C. Tuberculosis ffuse obstructive disease, pulmonary
D. Leprosy emphysema, for 15 years died from cardi-
E. Syphilis ac insufficiency. Autopsy revealed nutmeg
liver cirrhosis, cyanotic induration of ki-
43. Vitamin A together with specific dneys and spleen, ascites, edemata of
cytoreceptors penetrates through the lower limbs. These changes of internal
nuclear membranes, induces transcripti- organs are typical for the following di-
on processes that stimulate growth and sease:
differentiation of cells. This biological
Krok 1 Medicine 2010 6

A. Chronic right-ventricular insufficiency method can be applied for confirmation


B. Acute right-ventricular insufficiency of this diagnosis?
C. Chronic left-ventricular insufficiency
D. Acute left-ventricular insufficiency A. Microscopy of pathological material
E. General cardiac insufficiency B. Infection of laboratory animals
C. Test with bacteriophage
48. Examination of an ovary specimen D. Hemagglutination reaction
stained by hematoxylin-eosine revealed E. Immobilization reaction
a follicle in which follicular epithelium
consisted of 1-2 layers of cubic cells. There 53. Autopsy of a man who died from
was also a bright red membrane around burn disease revealed brain edema, li-
the ovocyte. What follicle is it? ver enlargement as well as enlargement
of kidneys with wide light-grey cortical
A. Primary layer and plethoric medullary area. Mi-
B. Primordial croscopic examination revealed necrosis
C. Secondary of tubules of main segments along with
D. Mature destruction of basal membranes, intersti-
E. Atretic cium edema with leukocytic infiltrati-
on and haemorrhages. What is the most
49. A newborn child suffers from mi- probable postmortem diagnosis?
lk curdling in stomach, this means that
soluble milk proteins (caseins) transform A. Necrotic nephrosis
to insoluble proteins (paracaseins) by B. Tubulointerstitial nephritis
means of calcium ions and a certain C. Pyelonephritis
enzyme. What enzyme takes part in this D. Gouty kidney
process? E. Myeloma kidney
A. Renin 54. A 30 year old man had been suffering
B. Pepsin from acute respiratory disease and died
C. Gastrin from cardiopulmonary decompensation.
D. Secretin Autopsy revealed fibrinous-haemorrhagic
E. Lipase inflammation in the mucous membrane
of larynx and trachea, destructive
50. A patient suffers from diabetes meli- panbronchitis, enlarged lungs that look
tus. After the regular insulin injecti- black due to the multiple abcesses,
on his condition grew worse: there haemorrhages, necrosis. What is the most
appeared anxiety, cold sweat, tremor probable postmortem diagnosis?
of limbs, general weakness, dizziness.
What preparation can eliminate these A. Influenza
symptoms? B. Parainfluenza
C. Respiratory syncytial infection
A. Adrenaline hydrochloride D. Measles
B. Butamide E. Adenoviral infection
C. Caffeine
D. Noradrenaline 55. A 19 year old woman suffers from
E. Glibutide primary syphilis. Doctor administered
her complex therapy that includes
51. A patient has difficulties with benzylpenicillin sodium salt. What is the
hand movement. Examination revealed mechanism of action of this drug?
inflammation of common synovial sheath
of flexor muscles. It is known from the A. It blocks synthesis of peptidoglycan of
patient’s anamnesis that he got a stab microbal membrane
wound of finger a week ago. Which finger B. It blocks synthesis of cytoplasm proteins
was most probably damaged? C. It blocks thiol enzymes
D. It blocks RNA synthesis
A. Digitus minimus E. It blocks DNA synthesis
B. Pollex
C. Digitus medius 56. A 65 year old female patient
D. Index suffers from chronic renal insufficiency
E. Digitus anularis accompanied by evident edemata caused
by chronic glomerulonephritis. What di-
52. Clinical diagnosis of a female pati- uretic should be administered for forced
ent was gonorrhoea. What examination diuresis?
Krok 1 Medicine 2010 7

old man’s thymus revealed reduced share


A. Furosemide of parenchymatous elements, increased
B. Hydrochlorothiazide share of adipose and loose connective ti-
C. Chlorthalidone ssue, its enrichment with thymus bodies.
D. Cyclometazide The organ’s mass was unchanged. What is
E. Acetazolamide this phenomenon called?
57. 6 months after labour a woman A. Age involution
had uterine hemorrhage. Gynaecologi- B. Accidental involution
cal examination of uterine cavity revealed C. Hypotrophy
a dark-red tissue with multiple cavities D. Dystrophy
resembling of a "sponge". Microscopic E. Atrophy
examination of a tumour revealed in
blood lacunas atypic light epithelial 62. A 56 year old patient suffering from
Langhans cells and giant cells of syncyti- cardiac insufficiency has edema of feet
otrophoblast. What tumour is it? and shins, edematous skin is pale and cold.
What is the leding mechanism of edema
A. Chorioepithelioma pathogenesis?
B. Squamous cell nonkeratinous carci-
noma A. Rise of hydrostatic pressure in venules
C. Adenocarcinoma B. Drop of oncotic pessure in capillaries
D. Fibromyoma C. Increase of capillary permeability
E. Cystic mole D. Disorder of lymph outflow
E. Positive water balance
58. Atria of an experimental animal were
superdistended by blood that resulted in 63. Researchers of a bacteriological
decreased reabsorption of Na+ and water laboratory examine tinned meat for
in renal tubules. This can be explained by botulinic toxin. For this purpose a group
the influence of the following factor upon of mice was injected with an extract of the
kidneys: material under examination and antitoxic
antibotulinic serum of A, B, E types. A
A. Natriuretic hormone control group of mice was injected with
B. Aldosterone the same extract but without antibotuli-
C. Renin nic serum. What serological reaction was
D. Angiotensin applied?
E. Vasopressin
A. Neutralization
59. A patient suffering from periodical B. Precipitation
attacks caused by inhalation of different C. Complement binding
flavoring substances was diagnosed with D. Opsonocytophagic
atopic bronchial asthma. IgE level was E. Double immune diffusion
increased. This is typical for the followi-
ng type of reactions: 64. A clinic observes a 49 year old
patient with significant prolongation
A. Anaphylactic reactions of coagulation time, gastrointestinal
B. Cytotoxic reactions haemorrhages, subcutaneous hematomas.
C. Immunocomplex reactions These symptoms might be explained by
D. delayed-type hypersensitivity the deficiency of the following vitamin:
E. Autoimmune reactions
A. K
60. During regular examination of B. B1
schoolchildren it was revealed that a 10 C. B6
year old girl had asymmetric oval eggs wi- D. H
th a larva in the scrape from her perianal E. E
folds. What diagnosis should be made?
65. A patient suffering from chronic
A. Enterobiasis cardiac insufficiency was recommended
B. Ascariasis to undergo a prophylactic course of
C. Amebiasis treatment with a cardiological drug from
D. Trichocephalosis the group of cardiac glycosides that is
E. Ankylostomiasis to be taken enterally. What drug was
recommended?
61. Histological examination of a 40 year
Krok 1 Medicine 2010 8

A. Digoxin
B. Strophanthine A. Hypospadias
C. Corglycon B. Hermaphroditism
D. Cordiamin C. Epispadia
E. Cordarone D. Monorchism
E. Cryptorhidism
66. A cerebral trauma caused increased
ammonia generation. What amino acid 71. A patient complains about impaired
participates in the excretion of ammonia evacuatory function of stomach (long-
from the cerebral tissue? term retention of food in stomach).
Examination revealed a tumour of initi-
A. Glutamic al part of duodenum. Specify localization
B. Tyrosine of the tumour:
C. Valine
D. Tryptophan A. Pars superior
E. Lysine B. Pars inferior
C. Pars descendens
67. During the repeated Widal’s aggluti- D. Pars ascendens
nation test it was noticed that the ratio E. Flexura duodeni inferior
of antibody titers and O-antigens S.typhi
in the patient’s serum had increased from 72. A concentrated solution of sodium
1:100 to 1:400. How would you interpret chloride was intravenously injected to an
these results? animal. This caused decreased reabsorpti-
on of sodium ions in the renal tubules. It
A. The patient has typhoid fever is the result of the following changes of
B. The patient is an acute carrier of typhoid hormonal secretion:
microbes
C. The patient is a chronic carrier of A. Aldosterone reduction
typhoid microbs B. Aldosterone increase
D. The patient previously had typhoid C. Vasopressin reduction
fever D. Vasopressin increase
E. The patient was previously vaccinated E. Reduction of atrial natriuretic factor
against typhoid fever
73. People adapted to high external
68. It is known that the gene responsible temperatures have such pecularity:
for development of blood groups accordi- profuse sweating isn’t accompanied by
ng to AB0 system has three allele vari- loss of large volumes of sodium chloride.
ants. If a man has IV blood group, it can This is caused by the effect of the followi-
be explained by the following variability ng hormone upon the perspiratory glands:
form:
A. Aldosterone
A. Combinative B. Vasopressin
B. Mutational C. Cortisol
C. Phenotypic D. Tgyroxin
D. Genocopy E. Natriuretic
E. Phenocopy
74. Emotional stress causes activation of
69. 48 hours after tuberculine test hormon-sensitive triglyceride lipase in
(Mantoux test) a child had a papule 10 the adipocytes. What secondary mediator
mm in diameter on the spot of tuberculine takes part in this process?
injection. What hypersensitivity mechani-
sm underlies these changes? A. Cyclic adenosine monophosphate
B. Cyclic guanosine monophosphate
A. Cellular cytotoxicity C. Adenosine monophosphate
B. Anaphylaxy D. Diacylglycerol
C. Antibody-dependent cytotoxicity E. Ions of 2+
D. Immunocomplex cytotoxicity
E. Granulomatosis 75. A patient has been syffering from
diarrhea for 5 day. On the fith day
70. Examination of a newborn boy’s geni- colonoscopy revealed that membrane
talia revealed an urethral hiatus that of rectum was inflamed, there were
opens on the undersite of his penis. What greyish-green films closely adhering to
malformation is it? the subjacent tissue. What is the most
Krok 1 Medicine 2010 9

probable diagnosis? 80. A 42 year old woman with neuralgia


of trifacial nerve complains about peri-
A. Dysentery odical reddening of the right part of her
B. Typhoid fever face and neck, sense of warmth gush,
C. Nonspecific ulcerous colitis increased skin sensitivity. These effects
D. Salmonellosis can be explained by the following type
E. Crohn’s disease of arterial hyperemia:
76. A 17 year old boy fell seriously ill, A. Neurotonic
the body temperature rose up to 38, 5oC, B. Neuroparalytic
there appeared cough, rhinitis, lacrimati- C. Metabolic
on, nasal discharges. What inflammation D. Functional
is it? E. Reactive
A. Catarrhal 81. A 30 year old woman has face
B. Serous edemata. Examination revealed protei-
C. Fibrinous nuria (5,87 g/l), hypoproteinemia,
D. Purulent dysproteinemia, hyperlipidemia. What
E. Hemorrhagic condition is the set of these symptoms
typical for?
77. A 25 year old Palestinian woman
complains of weakness, dizziness, A. Nephrotic syndrome
dyspnea. In anamnesis: periodically B. Nephritic syndrome
exacerbating anemia. In blood: Hb - 60 C. Chronic pyelonephritis
g/l, erythrocytes - 2, 5 · 1012 /l, reticulocytes D. Acute renal failure
- 35o /oo , anisocytosis and poikilocytosis E. Chronic renal failure
of erythrocytes, a lot of target cells and
polychromatophils. What type of anemia 82. Autopsy of a 17 year old girl who
is it? died from pulmonary failure revealed
a small area of caseous necrosis in the
A. Thalassemia inferior lobe of the right lung, and
B. Sickle-cell anemia occurences of caseous necrosis in the
C. Minkowsky-Shauffard disease bronchopulmonary, bronchial and bi-
D. Addison-Biermer disease furcational lymph nodes. What is the most
E. Glucose 6-phosphate dehydrogenase- probable postmortem diagnosis?
deficient anemia
A. Primary tuberculosis
78. A patient suffering from initial B. Hematogenous progression of primary
hypertension has been taking an anti- tuberculosis
hypertensive preparation for a long ti- C. Hematogenous tuberculosis with
me. Suddenly he stopped taking this predominant lung affection
preparation. After this his condition D. Tuberculoma
grew worse, this led to development of E. Caseous pneumonia under secondary
hypertensive crisis. This by-effect can be tuberculosis
classified as:
83. A patient has a decreased vasopressin
A. Abstinence syndrome synthesis that causes polyuria and as a
B. Cumulation result of it evident organism dehydratati-
C. Tolerance on. What is the mechanism of polyuria
D. Sensibilization development?
E. Dependence
A. Reduced tubular reabsorption of water
79. Inflammatory processes cause B. Reduced tubular reabsorption of Na
synthesis of protein of acute phase in ions
an organism. What substances stumulate C. Reduced tubular reabsorption of
their synthesis? protein
D. Reduced glucose reabsorption
A. Interleukin-1 E. Acceleration of glomerular filtration
B. Immunoglobulins
C. Interferons 84. A 35 year old man consulted a denti-
D. Biogenic amins st about reduced density of dental ti-
E. Angiotensin ssue, high fragility of teeth during eati-
ng solid food. This patient suffers the
Krok 1 Medicine 2010 10

most probably from the deficiency of the A. Ductus hepaticus communis et ductus
following mineral element: cysticus
B. Ductus hepaticus dexter et sinister
A. Calcium C. Ductus hepaticus dexter et ductus
B. Potassium cysticus
C. Sodium D. Ductus hepaticus sinister et ductus
D. Magnesium cysticus
E. Iron E. Ductus hepaticus communis et ductus
choledochus
85. A patient staying in the pulmonologi-
cal department was diagnosed with 89. A patient is 44 years old. Laboratory
pulmonary emphysema accompanied by examination of his blood revealed that
reduced elasticity of pulmonary tissue. content of proteins in plasma was 40 g/l.
What type of respiration is observed? What influence will be exerted on the
transcapillary water exchange?
A. Expiratory dyspnea
B. Inspiratory dyspnea A. Filtration will be increased, reabsorpti-
C. Superficial respiration on - decreased
D. Infrequent respiration B. Both filtration and reabsorption will be
E. Periodic respiration increased
C. Both filtration and reabsorption will be
86. A 4 year old child complained of decreased
pain during deglutition, indisposition. D. Filtration will be decreased, reabsorpti-
Objectively: palatine arches and tonsils on - increased
are moderately edematic and hyperemic, E. Exchange will stay unchanged
there are greyish-white films up to 1 mm
thick closely adhering to the subjacent ti- 90. After destruction of CNS structures
ssues. What pathological process are these an animal lost orientative reflexes. What
changes typical for? structure was destroyed?
A. Inflammation A. Quadrigeminal plate
B. Dystrophy B. Red nucleus
C. Necrosis C. Lateral vestibular nuclei
D. Metaplasia D. Black substance
E. Organization E. Medial reticular nuclei
87. An unconscious young man with signs 91. An isolated cell of human heart
of morphine poisoning entered admissi- automatically generates excitement
on office. His respiration is shallow and impulses with frequency of 60 times per
infrequent which is caused by inhibition minute. This cell was taken from the
of respiratory centre. What type of respi- following heart structure:
ratory failure is it?
A. Sinoatrial node
A. Ventilative dysregulatory B. Atrium
B. Ventilative obstructive C. Ventricle
C. Ventilative restrictive D. Atrioventricular node
D. Perfusive E. His’ bundle
E. Diffusive
92. A 62 year old patient who previ-
88. A patient with cholelithiasis fell ously worked as stoker was admitted to
ill with mechanic jaundice. Examinati- a hospital with complaints about general
on revealed that the stone was in the weakness, abrupt weight loss, hoarse voi-
common bile duct. What bile-excreting ce, dyspnea, dry cough. Laryngoscopy
ducts make up the obturated duct? revealed a tumour in the pharynx that
invaded vocal cords and epiglottis. What
is the most probable cause of tumour
development?
Krok 1 Medicine 2010 11

A. Polycyclic aromatic carbohydrates


B. Nitrosamines A. Proserin
C. Aromatic amines and amides B. Dichlothiazide
D. Retroviruses C. Reserpine
E. Ionizing radiation D. Mannitol
E. Propanolol
93. Examination of a man who had been
working hard under higher temperature 98. A patient has acne on his face. Mi-
of the environment revealed abnormal crospcopic examination of scrapings from
quantity of blood plasma proteins. What the affected areas revealed living porrect
phenomenon is the case? vermiform arthropoda 0,2-0,5 mm large
with four pairs of short extremities in the
A. Relative hyperproteinemia front part of their bodies. What is the
B. Absolute hyperproteinemia laboratory diagnosis?
C. Absolute hypoproteinemia
D. Dysproteinemia A. Demodicosis
E. Paraproteinemia B. Scabies
C. Myiasis
94. A patient ill with thrombophlebitis of D. Pediculosis
his lower limbs had chest pain, blood spi- E. Phthiriasis
tting, progressing respiratory insufficiency
that led to his death. Autopsy diagnosed 99. Among junior children of an
multiple lung infarctions. What is the most orphanage an outbreak of intestinal
probable cause of their development? infection with signs of colienteritis was
registered. In order to identify isolated
A. Thromboembolism of pulmonary artery causative agent it is necessary to:
branches
B. Thrombosis of pulmonary artery A. Study antigenic properties of the
branches causative agent
C. Thrombosis of bronchial arteries B. To determine sensitivity to antibiotics
D. Thromboembolism of bronchial arteries C. To study sensitivity to bacteriophages
E. Thrombosis of pulmonary veins D. To study biochemical properties of the
causative agent
95. During influenza epidemic 40% of E. To study virulence of the causative agent
pupils who didn’t go in for sports were
affected by the disease, and among the 100. A 45 year old man consulted a doctor
pupils who regularly did physical exercises about a plaque-like formation on his neck.
this index was only 20%. What adaptati- Histological examination of a skin bi-
ve mechanisms determined such a low si- optate revealed clusters of round and
ckness rate of pupils participating in the oval tumour cells with a narrow border of
sports? basophilic cytoplasm resembling of cells
of basal epidermal layer. What tumour is
A. Cross adaptation it?
B. Specific adaptation
C. Physiological adaptation A. Basal cell carcinoma
D. Biochemical adaptation B. Epidermal cancer
E. Genetic adaptation C. Hydroadenoma
D. Trichoepithelioma
96. A patient suffers from stenocardia E. Syringoadenoma
and takes isosorbide mononitrate. He was
prescribed a complementary drug with di- 101. A 45 year old patient was admitted to
saggregating effect. What drug is it? the cardiological department. ECG data:
negative P wave overlaps QRS complex,
A. Acetylsalicinic acid diastolic interval is prolonged after
B. Nitroglycerine extrasystole. What type of extrasystole is
C. Propranolol it?
D. Nifedipine
E. Validol A. Atrioventricular
B. Sinus
97. A patient in postoperative period was C. Atrial
prescribed an anticholinesterase drug for D. Ventricular
stimulation of intestinal peristalsis and E. Bundle-branch
tonus of urinary bladder. What drug is it?
Krok 1 Medicine 2010 12

102. A virological laboratory obtained 106. Hepatitis has led to the development
pathological material (mucous discharges of hepatic failure. Mechanism of edemata
from nasal meatuses) taken from a pati- formation is activated by the impairment
ent with provisional diagnosis "influenza". of the following liver function:
What quick test will allow to reveal speci-
fic viral antigen in the material under A. Protein-synthetic
examination? B. Barrier
C. Chologenetic
A. Direct and indirect D. Antitoxic
immunofluorescence test E. Glycogen-synthetic
B. Direct and indirect fluorescence
immunoassay 107. A patient working at a pig farm
C. Hemagglutination inhibition assay complains about paroxysmal abdomi-
D. Radioimmunoassay nal pain, liquid feces with admixtures of
E. - mucus and blood, headache, weakness,
fever. Examination of large intestine
103. A family of students who came revealed ulcers from 1 mm up to several
from Africa got a child with anemia si- cm large, feces contained oval unicellular
gns. The child died soon. Examinati- organisms with cilia. What disease should
on revealed that the child’s erythrocytes be suspected?
have abnormal semilunar shape. Specify
genotypes of the child’s parents: A. Balantidiasis
B. Amebiasis
A. Аа х Аа C. Toxoplasmosis
B. Аа х аа D. Lambliasis
C. АА х АА E. Trichomoniasis
D. аа х аа
E. Аа х АА 108. A patient staggers and walks
astraddle. He has hypomyotonia of arm
104. Two hours after an exam a student and leg muscles, staccato speech. In what
had a blood count done and it was brain section is this affection localized?
revealed that he had leukocytosis wi-
thout significant leukogram modifications. A. Cerebellum
What is the most probable mechanism of B. Putamen
leukocytosis development? C. Caudate nucleus
D. Motor cortex
A. Redistribution of leukocytes in the E. Red nucleus
organism
B. Leukopoiesis intensification 109. During hypersensitivity test a patient
C. Deceleration of leukocyte lysis got subcutaneous injection of an antigen
D. Deceleration of leukocyte migration to which caused reddening of skin, edema,
the tissues pain as a result of histamine action. This
E. Leukopoiesis intensification and biogenic amine is generated as a result of
deceleration of leukocyte lysis transformation of the following histidine
amino acid:
105. A 66 year old female patient
got intravenous injection of magnesi- A. Decarboxylation
um sulfate solution for the purpose of B. Methylation
elimination of hypertensive crisis. But C. Phosphorylation
arterial pressure didn’t go down and D. Isomerization
after repeated introduction of the same E. Deaminization
preparation there appered sluggishness, 110. Normal, actively dividing cells of
slow response, inhibition of consciousness human red bone marrow are analyzed.
and respiration. What preparation is What number of cells’ chromosomes is
antagonist of magnesium sulfate and can typical for G1 period?
eliminate symptoms of its overdose?
A. 46
A. Calcium chloride B. 48
B. Potassium chloride C. 47
C. Sodium chloride D. 45
D. Activated carbon E. 23
E. Potassium permanganate
Krok 1 Medicine 2010 13

111. Blood group of a 30 year old man was 116. A sportsman needs to improve his
specified before an operation. His blood sporting results. He was recommended to
is Rh-positive. Reaction of erythrocyte take a preparation that contains carnitine.
agglutination was absent with standard What process is activated the most by this
sera of 0αβ (I), Аβ (II), Вα (III) groups. compound?
The blood under examination is of the
following group: A. Fatty acids transporting
B. Amino acids transporting
A. 0αβ (I) C. Calcium ions transporting
B. Аβ (II) D. Glucose transporting
C. Вα (III) E. Vitamin K transporting
D. АВ (IV) 117. Laboratory examination of a child
E. - revealed increased concentration of leuci-
112. A 64 year old woman has impairment ne, valine, isoleucine and their ketoderi-
of twilight vision (hemeralopy). What vi- vatives in blood and urine. Urine smelt of
maple syrup. This disease is characterized
tamin should be recommended in the first
by the deficit of the following enzyme:
place?
A. Vitamin A A. Dehydrogenase of branched amino
B. Vitamin B2 acids
B. Aminotransferase
C. Vitamin E
D. Vitamin C C. Glucose-6-phosphatase
D. Phosphofructokinase
E. Vitamin B6
E. Phosphofructomutase
113. A patient underwent appendectomy.
In the postoperative period he has been 118. A 7 year old child often suffers from
taking an antibiotic. The patient complai- streprococcic angina. Doctor suspected
development of rheumatism and admini-
ns about hearing impairment and vesti-
bular disorders. What group of antibiotics stered serological examination. The provi-
sional diagnosis will be most probably
has such by-effects?
confirmed by presence of antibodies to
A. Aminoglycosides the following streptococcic antigen:
B. Penicillins
C. Tetracyclines A. O-streptolysin
B. C-carbohydrate
D. Macrolides
E. Cephalosporins C. M-protein
D. Erythrogenic toxin
114. A patient was stung by a bee. Exami- E. Capsular polysaccharide
nation revealed that his left hand was hot,
119. A 46 year old patient who had been
pink, edematic, there was a big red blister
on the site of sting. What is the leading suffering from tuberculosis for 6 years di-
ed from massive pulmonary haemorrhage.
mechanism of edema development?
Autopsy revealed different-sixed foci of
A. Increased vessel permeability sclerosis and caseous necrosis in lungs, in
B. Reduced vessel filling the upper part of the right lung there was
C. Injury of vessels caused by the sting a cavity 5 cm in diameter with dense grey
D. Drop of oncotic pressure in tissue walls, the cavity contained liquid blood
E. Drop of osmotic pressure in tissue and blood clots. What type of tuberculosis
is it?
115. A patient complained about di-
zziness, memory impairment, periodical A. Fibrocavernous
convulsions. It was revealed that these B. Acute cavernous
changes were caused by a product of C. Infiltrative
decarboxylation of glutamic acid. Name D. Fibrous focal
this product: E. Acute focal

A. GABA 120. A patient suffering from pheochromocytoma


B. Pyridoxal phosphate complains of thirst, dry mouth,
C. TDP hunger. Blood test for sugar revealed
D. ATP hyperglycemia. What type of hyperglycemia
E. THFA is it?
Krok 1 Medicine 2010 14

A. Adrenal 125. A man weighs 80 kg, after long physi-


B. Hypercorticoid cal activity his circulating blood volume is
C. Alimentary reduced down to 5,4 l, hematocrit makes
D. Somatotropic up 50%, whole blood protein is 80 g/l.
E. Hypoinsulinemic These blood characteristics are determi-
ned first of all by:
121. A patient has been suffering from
elevated temperature and attacks of typi- A. Water loss with sweat
cal cough for 10 days. Doctor admini- B. Increased number of erythrocytes
stered inoculation of mucus from the pati- C. Increased protein concentration in
ent’s nasopharynx on the agar. What mi- plasm
croorganism is presumed? D. Increased circulating blood volume
E. Increased diuresis
A. Pertussis bacillus
B. Pfeiffer’s bacillus 126. Examination of a pregnant woman
C. Listeria revealed twice as much concentration of
D. Klebsiella fibrinogen in blood plasm. What ESR can
E. Staphylococcus this woman have?
122. A patient suffering from stenocardia A. 40-50 mm/h
was taking nitroglycerine which caused B. 10-15 mm/h
restoration of blood supply of myocardi- C. 2-12 mm/h
um and relieved pain in the cardiac area. D. 5-10 mm/h
What intracellular mechanism provides E. 0-5 mm/h
restoration of energy supply of insulted
cells? 127. Examination of a child who
frequently suffers from infectious diseases
A. Intensification of ATP resynthesis revealed that IgG concentration in blood
B. Reduction of ATP resynthesis serum was 10 times less than normal, IgA
C. Increased permeability of membranes and IgM concentration was also signifi-
D. Intensification of oxygen transporting cantly reduced. Analysis showed also lack
into the cell of B-lymphocytes and plasmocytes. What
E. Intensification of RNA generation disease are these symptoms typical for?
123. A 50 year old patient has been taki- A. Bruton’s disease
ng treatment thrice for the last 6 months B. Swiss-type agammaglobulinemia
because of fractures caused by domestic C. Dysimmunoglobulinemia
accidents. Microscopical examination of D. Louis-Bar syndrome
bony tissue revealed foci of lacunar E. Di George syndrome
resolution, giant-cell granulomas in the
tumour-like formations, cysts. Bony tissue 128. Introduction of a big dose of hi-
was substituted by fibrous connective ti- stamine to an experimental animal caused
ssue. Examination revealed also adenoma abrupt drop of arterial pressure as a result
of parathyroid gland and hypercalcemia. of:
What is the most probable diagnosis?
A. Dilatation of resistance vessels
A. Parathyroid osteodystrophy B. Constriction of resistance vessels
B. Myelomatosis C. Increase of heart rate
C. Osteomyelitis D. Decrease of heart rate
D. Osteopetrosis E. Decrease of heart rate and force
E. Paget’s disease
129. Examination of a 42 year old patient
124. A couple had a child with Down’s di- revealed a tumour of adenohypophysis.
sease. Mother is 42 years old. This disease Objectively: the patient’s weight is 117
is most probably caused by the following kg, he has moon-like hyperemic face, red-
impairment of prenatal development: blue striae of skin distension on his belly.
Osteoporosis and muscle dystrophy are
A. Gametopathy present. AP is 210/140 mm Hg. What is
B. Blastopathy the most probable diagnosis?
C. Embryopathy
D. Non-specific fetopathy
E. Specific fetopathy
Krok 1 Medicine 2010 15

A. Cushing’s disease
B. Cushing’s syndrome A. Levodopa
C. Conn’s disease B. Diphenylhydantoin
D. Diabetes mellitus C. Phenobarbital
E. Essential hypertension D. Diazepam
E. Ethosuximide
130. Vagus branches that innervate heart
are being stimulated during an experi- 135. Examination of a patient with
ment. This caused reduction of heart rate impaired blood coagulation revealed
due to the intensification of the followi- thrombosis of a branch of inferior
ng process (through the cell membrane of mesenteric artery. What bowel segment
cardiac pacemaker): is damaged?
A. Potassium ion yield A. Colon sigmoideum
B. Potassium ion entry B. Ileum
C. Calcium ion entry C. Caecum
D. Calcium ion yield D. Colon transversum
E. Calcium and potassium ion yield E. Colon ascendens
131. A patient got an injury of spinal 136. When blood circulation in the
marrow in a road accident that caused damaged tissue is restored, then lactate
loss of tactile sensation, posture sense, vi- accumulation comes to a stop and
bration sense. What conduction tracts are glucose consumption decelerates. These
damaged? metabolic changes are caused by activati-
on of the following process:
A. Fascicle of Goll and cuneate fascicle
B. Anterior spinocerebellar tract A. Aerobic glycolysis
C. Rubrospinal tract B. Anaerobic glycolysis
D. Reticulospinal tract C. Lipolysis
E. Tectospinal tract D. Gluconeogenesis
E. Glycogen biosynthesis
132. A scheme presents an exocrinous
gland that has unbranched excretory duct 137. A woman was delivered to a hospi-
with a terminal part in form of a saccule tal for trachea intubation. What of the
openining into the duct. How is this gland following drugs should be applied in this
called according to the morphological case?
classification of exocrinous glands?
A. Dithylinum
A. Simple unbranched alveolar B. Nitroglycerine
B. Compound branched alveolar C. Metronidazole
C. Simple branched tubular D. Atropine sulfate
D. Compound unbranched alveolar E. Gentamycin sulfate
E. Compound unbranched alveolar tubular
138. A patient suffers from pulmonary
133. Examination of a patient revealed tuberculosis. During treatment neuritis of
hypertrophy and inflammation of visual nerve arose. What drug has caused
lymphoid tissue, edema of mucous this by-effect?
membrane between palatine arches (acute
A. Isoniazid
tonsillitis). What tonsil is normally si- B. Ethambutol
tuated in this area? C. Kanamycin
A. Tonsilla palatina D. Rifampicin
B. Tonsilla pharyngealis E. Streptomycin
C. Tonsilla tubaria 139. A patient suffers from chronic left-
D. Tonsilla lingualis ventricular insufficiency. What medication
E. - should be administered?
134. A patient complained about muscle A. Digoxin
rigidity, constrained motions, constant B. Bemegride
tremor of arms. On the grounds of exami- C. Etimizole
nation his disease was diagnosed as Parki- D. Vinpocetine
nson’s disease. What drug should be admi- E. Pyracetam
nistered?
Krok 1 Medicine 2010 16

140. Roentgenological examination of 145. A patient with skin mycosis has di-
skull base bones revealed enlargement of sorder of cellular immunity. The most
sellar cavity, thinning of anterior clinoid typical characteristic of it is reduction of
processes, destruction of different parts, the following index:
destruction of different parts of sella turci-
ca. Such bone destruction might be caused A. T-lymphocytes
by a tumour of the following wndocrinous B. Immunoglobulin G
gland: C. Immunoglobulin E
D. B-lymphocytes
A. Hypophysis E. Plasmocytes
B. Epiphysis
C. Thymus gland 146. A 50 year old patient underwent
D. Adrenal glands resection of tumour of large intestine
E. Thyroid gland wall. Microscopically it presents itself as
fascicles of divergent collagen fibers of
141. Patients who suffer from severe di- different thickness and form and some
abetes and don’t receive insulin have monomorphous fusiform cells that are
metabolic acidosis. This is caused by irregularly distributed among the fibers.
increased concentration of the following Cellular atypia is not evident. What
metabolites: tumour is it?
A. Ketone bodies A. Hard fibroma
B. Fatty acids B. Fibromyoma
C. Unsaturated fatty acids C. Soft fibroma
D. Triacylglycerols D. Desmoma
E. Cholesterol E. Fibrosarcoma
142. A 4 year old child with hereditary 147. A cell at the stage of mitosis anaphase
renal lesion has signs of rickets, vitamin D was stimulated by colchicine that inhibi-
concentration in blood is normal. What ts chromosome separation to the poles.
is the most probable cause of rickets What type of mutation will be caused?
development?
A. Polyploidy
A. Impaired synthesis of calcitriol B. Inversion
B. Increased excretion of calcium C. Deletion
C. Hyperfunction of parathyroid glands D. Duplication
D. Hypofunction of parathyroid glands E. Translocation
E. Lack of calcium in food
148. Autopsy of a 5 year old child revealed
143. A man was intoxicated with in the area of vermis of cerebellum a
mushrooms. They contain muscarine that soft greyish-pink node 2 cm in diameter
stimulates muscarinic cholinoreceptors. with areas of haemorrhage. Histologi-
What symptoms signalize intoxication wi- cally this tumour consisted of atypical
th inedible mushrooms? monomorphous small roundish cells wi-
th big polymorphous nuclei. What tumour
A. Myotic pupils is it?
B. Mydriatic pupils
C. Bronchi dilatation A. Medulloblastoma
D. Increased heart rate B. Meningioma
E. Rise of arterial pressure C. Glioblastoma
D. Astrocytoma
144. A 47 year old man with myocardi- E. Oligodendroglioma
um infarction was admitted to the cardi-
ological department. What changes of 149. Voluntary breath-holding caused
cellular composition of peripheral blood increase of respiration depth and
are induced by necrotic changes in the frequency. The main factor stimulating
myocardium? these changes of external respiration is:
A. Neutrophilic leukocytosis
B. Monocytosis
C. Eosinophilic leukocytosis
D. Thrombocytopenia
E. Lymphopenia
Krok 1 Medicine 2010 17

A. Increased tension of CO2 in blood A. Clostridia


B. Increased tension of O2 in blood B. Streptococci
C. Decreased tension of O2 in blood C. Spirochaete
D. Decreased tension of CO2 in blood D. Actinomycete
E. Decreased concentration of H + in blood E. Diplococci
150. A patient was admitted to the surgi- 155. A bacteriological laboratory received
cal department with inguinal hernia. Duri- sputum sample of a patient suffering from
ng the operation the surgeon performs tuberculosis. Bacterioscopic examination
plastic surgery on posterior wall of ingui- of smears and detection of tuberculosis
nal canal. What structure forms this wall? bacillus can be realized by one of enri-
chment methods that involves processing
A. Transverse fascia of sputum only with solution of caustic
B. Aponeurosis of abdominal external soda. What is this method called?
oblique muscle
C. Inguinal ligament A. Homogenization
D. Loose inferior edge of transverse B. Inactivation
abdominal muscle C. Flotation
E. Peritoneum D. Filtration
E. Neutralization
151. A patient has delayed conduction
of excitement through the atrioventri- 156. Electronic microphotography of
cular node. What changes of ECG will pulmonary alveole’s wall presents a big
be observed? cell. Its cytoplasm has a lot of mi-
tochondria, developed Golgi apparatus,
A. Prolongation of P − Q interval osmiophil lamellated corpuscles. What is
B. Prolongation of Q − S interval the main function of this cell?
C. Negative T wave
D. S − T -segment displacement A. It produces surfactant
E. Prolongation of Q − T interval B. It is a component of blood-air barrier
C. It warms the air
152. Surface with an intact toad on it was D. It purifies the air
inclined to the right. Tone of extensor E. It absorbs microorganisms
muscles became reflectory higher due to
the activation of the following receptors: 157. A patient in a transplantation centre
underwent heart transplantation. The
A. Vestibuloreceptors of utricle and organ was taken from a donor who di-
saccule ed in a road accident. Foreign heart can
B. Vestibuloreceptors of semicircular ducts be rejected as a result of development
C. Mechanoreceptors of foot skin of transplantation immunity. It is usually
D. Photoreceptors of retina prevented by means of:
E. Proprioreceptors
A. Immunosuppressors
153. A female patient underwent liver B. Chemotherapy
transplantation. 1,5 month after it her C. Ultrasound
condition became worse because of reacti- D. Enzymes
on of transplant rejection. What factor of E. X-ray therapy
immune system plays the leading part in
this reaction? 158. A child is pale, pastose, muscular ti-
ssue is bad developed, lymph nodes are
A. T-killers enlarged. He often suffers from angi-
B. Interleukin-1 na and pharyngitis, blood has signs of
C. Natural killers lymphocytosis. The child is also predi-
D. B-lymphocytes sposed to autoallergic diseases. What type
E. T-helpers of diathesis can be presumed in this case?
154. Microscopical examination of a mi- A. Lymphohypoplastic
crobal culture revealed fusiform spore- B. Exudative
forming microorganisms that get violet- C. Gouty
blue Gram’s stain. What microorganisms D. Asthenic
were revealed? E. Hemorrhagic
159. A patient had been suffering from
Krok 1 Medicine 2010 18

profuse diarrhea and vomiting for 2 days. activity of head muscles. In what parts
He died from acute dehydration. Autopsy of cerebral cortex is the respective centre
revealed that the intestinal wall was normally localized?
edematic and hyperemic, with multiple
haemorrhages in the mucous membrane. A. Inferior part of precentral gyrus
Intestine lumen contains whitish fluid B. Superior part of precentral gyrus
resembling of rice water. What disease C. Supramarginal gyrus
caused death? D. Superior parietal lobule
E. Angular gyrus
A. Cholera
B. Dysentery 164. Material taken from a patient wi-
C. Salmonellosis th provisional diagnosis "influenza"was
D. Typhoid fever referred to a laboratory. For virological
E. Enterocolitis examination the hemadsorption reaction
was applied. This reaction can be applied
160. Examination of a 66 year old for detection of the following viruses:
patient revealed a lytic tumour in the
locus of pathological rib fracture. Hi- A. Viruses containing hemagglutinins
stologically this tumour consists of atypi- B. All the simple viruses
cal plasmoblasts. Further examinati- C. All the complex viruses
on revealed osteoporosis in the bones D. DNA-genomic viruses
of vertebral column and pelvis. These E. Any viruses
changes are typical for:
165. Lungs of a preterm infant have areas
A. Myelomatosis of atelectasis (pulmonary collapse). The
B. Tuberculous osteomyelitis main cause is:
C. Ewing’s osteosarcoma
D. Neuroblastoma A. Surfactant deficiency
E. Metastatic lung cancer B. Increased viscous resistance
C. Underdeveloped inspiration muscles
161. A patient died from acute cardiac D. Diminished force of surface tension of
insufficiency, among clinical presentations lungs
there was gastrointestinal haemorrhage. E. Surfactant excess
Examination of mucous membrane of
sromach revealed some defects reachi- 166. An alcoholic woman has born a girl
ng myenteron; their edges and bottom with mental and physical developmental
were mostly even and loose, some of lag. Doctors diagnosed the girl with fetal
them contained dark-red blood. What alcohol syndrome. What effect is the cause
pathological process was revealed? of the girl’s state?

A. Acute ulcers A. Teratogenic


B. Chronic ulcers B. Mutagenic
C. Erosions C. Malignization
D. Thrombosis D. Carcinogenic
E. Inflammation E. Mechanic

162. A 33 year old man died from uraemia. 167. A viral infection has damaged cells
Autopsy revealed enlarged kidneys wei- that form walls of bile capillaries. This sti-
ghing 500,0 each and consisting of multi- mulated conditions for inflow of bile into
ple cavities 0,5-2 cm in diameter. The cavi- the blood of sinusoidal capillaries. What
ties were full of light-yellow transparent cells are damaged?
liquid. Renal pelvis and ureters had no A. Hepatocytes
pecularities. What renal disease caused B. Kupffer’s cells
uraemia? C. Ito cells
A. Bilateral polycystic renal disease D. Pit-cells
B. Chronic pyelonephritis E. Endotheliocytes
C. Renal tumour 168. Dietary intake of a 30 year old nursi-
D. Renal tuberculosis ng woman contains 1000 mg of calcium,
E. Rapidly progressing glomerulonephritis 1300 mg of phosphorus and 20 mg of iron
163. A patient got a trauma that caused per day. It is necessary to change content
dysfunction of motor centres regulating of these mineral substances in the followi-
Krok 1 Medicine 2010 19

ng way: These symptoms result from:


A. To increase phosphorus content A. Material cumulation
B. To increase calcium content B. Tachyphylaxis
C. To reduce fluorine content C. Idiosyncrasy
D. To increase iron content D. Antagonism
E. To reduce iron content E. Sensibilization
169. A patient with obliterating 174. A patient was diagnosed wi-
atherosclerosis underwent sympathectomy th paralysis of facial and masticatory
of femoral artery in the region of femoral muscles. The haematoma is inside the
trigone. What type of arterial hyperemia genu of internal capsule. What conduction
was induced by the operation? tract is damaged?
A. Neuroparalytic A. Tr. cortico-nuclearis
B. Reactive B. Tr. cortico-spinalis
C. Metabolic C. Tr. cortico-thalamicus
D. Neurotonic D. Tr. cortico-fronto-pontinus
E. Functional E. Tr. cortico-temporo-parieto-occipito-
pontinus
170. A 15 year old girl has pale skin, glossi-
tis, gingivitis. Blood count: erythrocytes - 175. A 5 year old child is ill with measles.
3, 3 cot 1012 /l, hemoglobin - 70 g/l, colour Blood analysis revealed increase of total
index - 0,5. Examination of blood smear number of leukocytes up to 13 · 109 /l.
revealed hypochromia, microcytosis, poi- Leukogram: basophils - 0, eosinophils
kilocytosis. What type of anemia is it? - 1, myelocytes - 0, juvenile neutrophi-
ls - 0, band neutrophils - 2, segmented
A. Iron-deficient neutrophils - 41, lymphocytes - 28,
B. B12 -folic acid-deficient monocytes - 28. Name this phenomenon:
C. Sickle-cell
D. Hemolytic A. Monocytosis
E. Thalassemia B. Agranulocytosis
C. Lymphocytosis
171. Burned skin surface was treated wi- D. Eosinopenia
th a certain preparation. Its antiseptic E. Neutropenia
properties are provided by atomic oxygen
that is formed in presence of organic 176. A patient was admitted to the infecti-
substances. What preparation was appli- ous department. His symptoms: dry skin,
ed? decreased skin turgor, rice-water stool.
The patient was diagnosed with cholera.
A. Potassium permanganate What disorder of water-electrolytic
B. Furacillin balance is most often observed in this di-
C. Chlorhexidine bigluconate sease?
D. Alcoholic iodine solution
E. Sodium hydrocarbonate A. Isoosmotic hypohydration
B. Hyperosmotic hyperhydration
172. Cardinal symptoms of primary C. Hypoosmotic hypohydration
hyperparathyroidism are osteoporosis and D. Hyperosmotic hypohydration
renal lesion along with development of E. Hypoosmotic hyperhydration
urolithiasis. What substance makes up the
basis of these calculi in this disease? 177. A foreign body (a button) closed
space of the right superior lobar bronchus.
A. Calcium phosphate What segments of the right lung won’t be
B. Uric acid supplied with air?
C. Cystine
D. Bilirubin A. Apical, posterior, anterior
E. Cholesterol B. Superior and inferior lingular
C. Apical and posterior basal
173. A patient with chronic cardiac insuffi- D. Apical and median basal
ciency has been taking foxglove (Digi- E. Medial and lateral
talis) preparations for a long time. Due
to the violation of intake schedule the 178. A patient was diagnosed with
woman got symptoms of intoxication. bartholinitis (inflammation of greater
Krok 1 Medicine 2010 20

vulvovaginal glands). In which organ of tis, cellular infiltration composed of


urogenital system are these glands locali- lymphocytes, epithelioid cells (mainly
zed? plasmocytes). What is the most probable
diagnosis?
A. Large lips of pudendum
B. Small lips of pudendum A. Syphilis
C. Clitoris B. Tuberculosis
D. Vagina C. Scleroma
E. Uterus D. Sarcoma
E. Leprosy
179. A histological specimen presents an
artery. One of the membranes of its wall 184. A young woman who entered a
has flat cells lying on the basal membrane. production department where it strongly
What type of cells is it? smelt of paints and varnishes had a
bronchospasm. This reflex was caused by
A. Endothelium irritation of the following receptors:
B. Mesothelium
C. Smooth myocytes A. Irritant
D. Fibroblasts B. Juxtaglomerular
E. Macrophages C. Pleura receptors
D. Central chemoreceptors
180. Study of conversion of a food colouri- E. Peripheral chemoreceptors
ng agent revealed that neutralization of
this xenobiotic takes place only in one 185. A 50 year old man who was referred
phase - microsomal oxydation. Name a to the hospital for treatment of cervical
component of this phase: lymphadenitis underwent test for induvi-
dual sensitivity to penicillin. 30 seconds
A. Cytochrome Р-450 after he went hot all over, AP dropped
B. Cytochrome B down to 0 mm Hg that led to cardiac
C. Cytochrome C arrest. Resuscitation was unsuccessful.
D. Cytochrome A Autopsy results: acute venous plethora
E. Cytochrome oxidase of internal organs; histological examinati-
181. Ultrasonic examination of a patient on of skin (from the site of injection)
revealed aneurism in the area of aortic revealed degranulation of mast cells (ti-
arch that caused alteration of vocal functi- ssue basophils). Degranulation was also
on of larynx. What nerve was constricted? revealed in myocardium and lungs. What
type of hypersensitivity reaction is it?
A. Recurrent laryngeal
B. Diaphragmatic A. Anaphylactic
C. Superior laryngeal B. Delayed-type hypersensitivity
D. Mandibular C. Complement-mediated cytotoxic
E. Sublingual D. Immunocomplex-mediated
E. -
182. A 25 year old man has spent a long
time in the sun under high air humidity. 186. One of sections of central nervous
As a result of it his body temperature rose system has layerwise arrangement of
up to 39oC. What pathological process is neurocytes. Among them there are cells
it? of the following forms: stellate, fusiform,
horizontal, pyramidal. What section of
A. Hyperthermia central nervous system is this structure
B. Infectious fever typical for?
C. Hypothermia
D. Noninfectious fever A. Cortex of cerebrum
E. Burn disease B. Spinal cord
C. Cerebellum
183. A 23 year old man has perforation of D. Medulla oblongata
hard palate. In the area of this perforati- E. Hypothalamus
on there was a compact well-defined
formation. Microscopic examination of 187. Study of fingerprints (dactylography)
the resected formation revealed a large is used by criminalists for personal identi-
focus of caseous necrosis surrounded fication as well as for diagnostics of
by granulation tissue with endovasculi- genetic abnormalities, particularly Dawn’s
Krok 1 Medicine 2010 21

disease. What layer of skin determines structure localized in this space is


individuality of fingerprints? dangerous to be damaged?
A. Dermopapillary A. Jugular venous arch
B. Horny B. External jugular vein
C. Reticular C. Subclavicular vein
D. Clear (stratum lucidum epidermidis) D. Inferior thyroid arthery
E. Basal E. Superior thyroid arthery
188. A patient has herpetic rash. What 193. A doctor revealed tissues injury on
medication should be administered? patient’s scalp with localized suppurati-
ons and diagnosed his disease as myiasis.
A. Acyclovir This infestation is caused by larvae of the
B. Gentamycin following insect:
C. Clotrimazole
D. Benzylpenicillin sodium salt A. Wohlfahrt fly
E. Biseptol B. Kissing bug
C. Stable fly (Stomoxys calcitrans)
189. Cytogenetic examination of a patient D. Malarial mosquito
with reproductive dysfunction revealed E. Mosquito
normal karyotype 46 ХY in some cells, but
most cells have karyotype of Klinefelter’s 194. A patient with hip fracture was
syndrome - 47 ХХY. Such cell heterogeni- prescribed a narcotic analgetic. Its
ty is called: anesthetic action is determined by
interaction with the following receptors:
A. Mosaicism
B. Inversion A. Opiate receptors
C. Transposition B. Adrenoreceptors
D. Duplication C. Cholinoreceptors
E. Monomorphism D. Benzodiazepine receptors
E. GABA-ergic receptors
190. An isolated muscle fiber is under
examination. It was established that the 195. A patient consumed a lot of reach in
threshold of stimulation force became si- proteins food that caused increase of rate
gnificantly lower. What is the cause of this of proteolytic enzymes of pancreatic juice.
phenomenon? It is also accompanied by increase of rate
of the following enzyme:
A. Activation of sodium channels of
membrane A. Tripsin
B. Activation of potassium channels of B. Pepsin
membrane C. Enterokinase
C. Inactivation of sodium channels of D. Gastricsin
membrane E. Renin
D. Inactivation of potassium channels of
membrane 196. In course of an experiment
E. Block of energy production in the cell thalamocortical tracts of an animal were
cut. What type of sensory perception
191. During examination of a pati- remained intact?
ent a dentist revealed a lot of "white
spots zones of enamel demineralization. A. Olfactory
What microorganisms take part in the B. Auditory
development of this process? C. Exteroreceptive
D. Visual
A. Streptococcus mutans E. Nociceptive
B. Streptococcus salivarius
C. Streptococcus pyogenes 197. A 2 year old child had acute
D. Veilonella parvula respiratory viral infection and died
E. Staphylococcus epidermidis from cardiopulmonary decompensati-
on. Autopsy revealed that his right lung
192. Surgical approach to the thyroid was hyperemic; in the 2nd, 6th and 10th
gland from the transverse (collar) segments and on the incision there were
approach involves opening of interaponeurotic airless yellowish foci of irregular form,
suprasternal space. What anatomic from several mm up to 1 cm large. Mi-
Krok 1 Medicine 2010 22

croscopical examination revealed exudate 199. An infectious disease caused


consisting mainly of neutrophils in the contractive activity of muscles that
given areas of pulmonary tissue in the contract and dilate eye pupil (paralytic
alveoles, bronchioles and bronchial tubes. state). What functional eye system was
What is the most probable diagnosis? damaged?
A. Focal pneumonia A. Accomodative
B. Interstitial pneumonia B. Dioptric
C. Croupous pneumonia C. Ancillary
D. Acute bronchitis D. Photosensory
E. Pulmonary abscess E. Lacrimal apparatus
198. A 25 year old patient was examined 200. Brain tomography revealed a tumour
by a medical board. Examination revealed in the region of red nucleus. What part of
pathology of chest. Transverse dimensi- brain isdamaged?
ons were to small and the sternum was
strongly protruding. What chest type is it? A. Midbrain
B. Medulla oblongata
A. Keeled chest C. Cerebellum
B. Funnel chest D. Interbrain
C. Flat chest E. Pons cerebelli
D. Cylindrical chest
E. Barrel chest
Krok 1 Medicine 2011 1

1. A 30-year-old patient complains A. Creatine phosphokinase


about having abdominal pain and di- B. Lactate dehydrogenase
arrhea for five days; body temperature C. Pyruvate dehydrogenase
rise up to 37, 5o C along with chills. The D. Glutamate dehydrogenase
day before a patient had been in a E. Adenylate cyclase
forest and drunk from an open water
reservoir. Laboratory analyses enabled 6. Extensive thromboembolic infarcti-
to make the following diagnosis: amebic on of the left cerebral hemispheres,
dysentery. What is the drug of choice for large septic spleen, immunocomplex
its treatment? glomerulonephritis, ulcers on the edges of
the aortic valves, covered with polypous
A. Metronidazole thrombus with colonies of staphylococcus
B. Furazolidonum were revealed on autopsy of the young
C. Levomycetin man who died in coma. What disease
D. Phthalazol caused cerebral thromboemboly?
E. Emetine hydrochloride
A. Septic bacterial endocarditis
2. Quite often the cause of secondary B. Septicemia
immunodeficiency is an infection C. Acute rheumatic valvulitis
involvement, when the causative agents D. Septicopyemia
propagate directly in the cells of immune E. Rheumatic thromboendocarditis
system and destroy it. The following di-
seases are characterized by: 7. A 10-year-old girl has a history of
repeated acute respiratory viral infection.
A. Infectious mononucleosis, AIDS After recovering she presents with multi-
B. Tuberculosis, mycobacteriosis ple petechial hemorrhages on the sites of
C. Poliomyelitis, type A hepatitis friction from clothing rubbing the skin.
D. Dysentery, cholera What kind of hypovitaminosis has this gi-
E. Q-febris, epidemic typhus rl?
3. Heart rate of a 30-year-old man under A. C
emotional stress reached 112 bpm. The B. B6
reason for the heart rate increase is C. B1
the altered condition of the following D. A
conducting system of heart: E. B2
A. Sinoatrial node 8. Autopsy of a patient who suffered
B. Purkinje’s fibers from croupous pneumonia and died from
C. His’ bundle branches pneumococcal sepsis revealed 900 ml of
D. Atrioventricular node turbid greenish-yellow liquid in the ri-
E. His’ bundle ght pleural cavity. Pleural leaves are dull,
plephoric. Name the clinicopathological
4. Sanitary bacteriological research on form of inflammation in the pleural cavity:
water by the membrane filter method
revealed two red colonies on a membrane A. Empyema
filter (Endo agar) through which 500 ml B. Fibrinous inflammation
of analyzed water were passed. Calculate C. Phlegmon
the coli index and coli titer of the analyzed D. Chronic abscess
water: E. Acute abscess
A. 4 and 250 9. Researchers isolated 5 isoenzymic
B. 2 and 500 forms of lactate dehydrogenase from the
C. 250 and 4 human blood serum and studied their
D. 500 and 2 properties. What property indicates that
E. 250 and 2 the isoenzymic forms were isolated from
the same enzyme?
5. A 46-year-old female patient has
a continuous history of progressive A. Catalyzation of the same reaction
muscular (Duchenne’s) dystrophy. Whi- B. The same molecular weight
ch blood enzyme changes will be of di- C. The same physicochemical properties
agnostic value in this case? D. Tissue localization
E. The same electrophoretic mobility
Krok 1 Medicine 2011 2

10. A patient suffering from stomach ulcer A. Sympathoadrenal system


has been treated with an antacid drug B. Parasympathetic nucleus of vagus
almagel. For acute bronchitis treatment he C. Functions of thyroid gland
was prescribed the antibiotic methacycli- D. Functions of adrenal cortex
ne. However within next 5 days the E. Hypophysis function
fever didn’t fall, cough and sputum
nature remained unchanged. A physici- 15. Autopsy of a man who died from
an came to the conclusion that the drugs chronic cardiovascular collapse revealed
were incompatible. What type of drug "tiger heart". Sidewards of endocardium
incompatibility is the case? a yellowish-white banding can be seen;
myocardium is dull, dark-yellow. What
A. Pharmacokinetic, absorption stage process caused this pathology?
B. Pharmacokinetic, biotransformation
stage A. Fatty parenchymatous degeneration
C. Pharmaceutic B. Carbohydrate degeneration
D. Pharmacodynamic C. Hyaline degeneration
E. Direct antagonism D. Fatty vascular-stromal degeneration
E. Amyloidosis
11. ECG of a 44-year-old patient shows
signs of hypertrophy of both ventricles 16. A 55-year-old male patient was hospi-
and the right atrium. The patient was di- talised to a surgical clinic for suspected
agnosed with the tricuspid valve insuffici- septicemia. What material should be
ency. What pathogenetic variant of cardiac taken for analysis?
dysfunction is usually observed in case of A. Blood, sugar broth
such insufficiency? B. Liquor, serum agar
A. Heart overload by volume C. Urine, beef-extract broth
B. Heart overload by resistance D. Pus, yolk saline agar
C. Primary myocardial insufficiency E. Lymph node punctate, cysteine agar
D. Coronary insufficiency 17. A patient has pellagra. Interrogation
E. Cardiac tamponade revealed that he had lived mostly on mai-
12. Proserin increases skeletal muscle tone ze for a long time and eaten little meat.
when given systematically. Halothane This disease had been caused by the defi-
induces relaxation of skeletal muscles cit of the following substance in the maize:
and reduces proserin effects. What is A. Tryptophan
the nature of proserin and halothane B. Tyrosine
interaction? C. Proline
A. Indirect functional antagonism D. Alanine
B. Direct functional antagonism E. Histidine
C. Competitive antagonism 18. During examination of a 6-year-
D. Independent antagonism old child a doctor revealed greyi-
E. Noncompetitive antagonism sh films on the pharyngeal tonsi-
13. The minute blood volume in a patient ls. Their removal provoked moderate
with transplanted heart has increased as a haemorrhage. Bacterioscopy revealed
result of physical activity. What regulati- gram-positive clublike bacteria. What
ve mechanism is responsible for these symptoms will develop in this child within
changes? the next few days if no specific treatment
is provided?
A. Catecholamines
B. Sympathetic unconditioned reflexes A. Toxic lesions of myocard, liver and
C. Parasympathetic unconditioned reflexes kidney
D. Sympathetic conditioned reflexes B. Pulmonary edema
E. Parasympathetic conditioned reflexes C. Strong paroxysmal cough
D. Papulous skinrash
14. An aged man had raise of arterial E. Intermittent fever
pressure under a stress. It was caused by
activation of: 19. A 46-year-old patient consulted a
doctor complaining about joint pain that
becomes stronger the day before the
weather changes. Blood examination
Krok 1 Medicine 2011 3

revealed an increased concentration of A. Vanishing of elastic fibers


uric acid. The most probable cause of the B. Vanishing of collagen fibers
disease is the intensified disintegration of C. Muscle layer atrophy
the following substance: D. Intima changes by shagreen leather
type
A. Adenosine monophosphate E. Vascularization
B. Cytidine monophosphate
C. Uridine triphosphate 24. A patient suffering from myasthenia
D. Uridine monophosphate has been administered proserin. After its
E. Thymidine monophosphate administration the patient has got nausea,
diarrhea, twitch of tongue and skeletal
20. 12 hours after an accute attack of muscles. What drug would help to elimi-
retrosternal pain a patient presented a nate the intoxication?
jump of aspartate aminotransferase activi-
ty in blood serum. What pathology is this A. Atropine sulfate
deviation typical for? B. Physostigmine
C. Pyridostigmine bromide
A. Myocardium infarction D. Isadrine
B. Viral hepatitis E. Mesatonum
C. Collagenosis
D. Diabetes mellitus 25. A man died from an acute infecti-
E. Diabetes insipidus ous disease accompanied by fever, jaundi-
ce, haemorrhagic rash on the skin
21. After a trauma of the upper third and mucous membranes as well as by
of the anterior forearm surface a patient acute renal insufficiency. Histological
presents with difficult pronation, weakeni- examination of renal tissue (stained by
ng of palmar flexor muscles and altered Romanovsky-Giemsa method) revealed
skin sensitivity of 1-3 fingers. Which nerve some convoluted bacteria looking like
is damaged? C und S letters. What bacteria were
revealed?
A. n. medianus
B. n. musculocutaneus A. Leptospira
C. n. ulnaris B. Treponema
D. n. cutaneus antebrachii medialis C. Spirilla
E. n. radialis D. Borrelia
E. Campilobacteria
22. A 2-year-old child with mental and
physical retardation has been delivered 26. A 46-year-old patient suffering from
to a hospital. He presents with frequent the diffuse toxic goiter underwent resecti-
vomiting after having meals. There is on of the thyroid gland. After the
phenylpyruvic acid in urine. Which surgery the patient presents with appetite
metabolism abnormality is the reason for loss, dyspepsia, increased neuromuscular
this pathology? excitement. The body weight remained
unchanged. Body temperature is normal.
A. Amino-acid metabolism Which of the following has caused such a
B. Lipidic metabolism condition in this patient?
C. Carbohydrate metabolism
D. Water-salt metabolism A. Reduced production of parathormone
E. Phosphoric calcium metabolism B. Increased production of thyroxin
C. Increased production of calcitonin
23. A 38-year-old man died in the attempt D. Increased production of thyroliberin
of lifting weight. He had collaptoid state. E. Reduced production of thyroxin
Autopsy revealed an extensive aneurism
rupture of thoracic aorta. He suffered 27. Medical examination at the mili-
from visceral syphilis during his lifeti- tary registration and enlistment office
me. What pathological process caused revealed that a 15-year-old boy was hi-
weakness of aortic wall, its dilatation and gh, with eunuchoid body proportions,
rupture? gynecomastia, female pattern of pubic
hair distribution. The boy had also fat
deposits on the thighs, no facial hair, hi-
gh voice, subnormal intelligence quotient.
Which karyotype corresponds with this di-
sease?
Krok 1 Medicine 2011 4

drug for heartburn elimination. After its


A. 47, XXY ingestion the patient feels better but at the
B. 45, XO same time he has a sensation of stomach
C. 46, XX swelling. Which of the following drugs mi-
D. 46, XY ght be the cause of such side effect?
E. 47, XXX
A. Sodium hydrocarbonate
28. A doctor recommends a patient wi- B. Magnesium oxide
th duodenal ulcer to drink cabbage and C. Magnesium trisilicate
potato juice after the therapy course. D. Aluminium hydrooxide
Which substances contained in these E. Pepsin
vegetables help to heal and prevent the
ulcers? 33. A 30-year-old male patient with acute
pancreatitis has been found to have a
A. Vitamin U disorder of cavitary protein digestion.
B. Pantothenic acid The reason for such condition can be the
C. Vitamin C hyposynthesis and hyposecretion of the
D. Vitamin B1 following enzyme:
E. Vitamin K
A. Tripsin
29. Urine analysis of a 12-year-old boy B. Pepsin
reveals high concentration of all aliphatic C. Lipase
amino acids with the highest excretion D. Dipeptidase
of cystine and cysteine. US of kidneys E. Amylase
revealed kidney concrements. What is the
most likely pathology? 34. A patient has been given high doses
of hydrocortisone for a long time. This
A. Cystinuria caused atrophy of one of the adrenal
B. Alkaptonuria cortex zones. Which zone is it?
C. Cystitis
D. Phenylketonuria A. Fascial
E. Hartnup disease B. Glomerular
C. Reticular
30. A 5-year-old child has been diagnosed D. Glomerular and reticular
with acute right distal pneumonia. E. -
Sputum inoculation revealed that the
causative agent is resistant to penicillin, 35. A victim of an accident has bleeding
but it is senstive to macrolides. What drug from the soft tissues anteriad the mandi-
should be prescribed? bular angle. Which vessel should be li-
gated for the bleeding arrest?
A. Azithromycin
B. Tetracycline A. A.facialis
C. Gentamycin B. A.carotis interna
D. Streptomycin C. A.temporalis superficialis
E. Ampicillin D. A.alveolaris inferior
E. A.lingvalis
31. Autopsy of a 73-year-old man who had
been suffering from the coronary heart di- 36. Microscopic examination of a gram-
sease along with cardiac insufficiency for stained scrape from a patient’s tongue
a long time revealed: nutmeg liver, brown revealed oval, round, elongated chains of
induration of lungs, cyanotic induration dark-violet gemmating cells. What disease
of kidneys and spleen. What kind of ci- can be caused by this causative agent?
rculation disorder was the cause of such
effects? A. Candidiasis
B. Actinomycosis
A. General chronic venous congestion C. Streptococcic infection
B. Arterial hyperaemia D. Staphylococcic infection
C. General acute venous congestion E. Diphtheria
D. Acute anaemia
E. Chronic anaemia 37. The greater amount of nitrogen is
excreted from the organism in form of
32. A patient suffering from chronic urea. Inhibition of urea synthesis and
hyperacidic gastritis takes an antacid accumulation of ammonia in blood and
Krok 1 Medicine 2011 5

tissues are induced by the decreased acti- A. Left ventricle


vity of the following liver enzyme: B. Right ventricle
C. Right atrium
A. Carbamoyl phosphate synthetase D. Left atrium
B. Aspartate aminotransferase E. -
C. Urease
D. Amylase 43. A patient with essential hypertensi-
E. Pepsin on has a high rate of blood renin. Whi-
ch of antihypertensive drugs should be
38. A 36-year-old female patient has a hi- preferred?
story of collagen disease. Urine analysis is
likely to reveal an increased concentrati- A. Lisinopril
on of the following metabolite: B. Propranolol
C. Prazosinum
A. Oxyproline D. Nifedipine
B. Indican E. Dichlothiazide
C. Creatinine
D. Urea 44. In order to determine toxigenicity of
E. Urobilinogen diphtheria bacilli a strip of filter paper
impregnated with antitoxic diphtherial
39. A coprological survey revealed light- serum was put on the dense nutrient medi-
colored feces containing drops of neutral um. There were also inoculated a mi-
fat. The most likely reason for this condi- crobal culture under examination and a
tion is the disorder of: strain that is known to be toxigenic. If
the microbal culture under examinati-
A. Bile inflow into the bowel on produces exotoxin, this wil result in
B. Gastric juice acidity formation of:
C. Pancreatic juice secretion
D. Intestinal juice secretion A. Precipitin lines
E. Intestinal absorption B. Haemolysis zones
C. Zones of diffuse opacification
40. The secretion of whixh hypophysi- D. Zones of lecithovitellinous activity
al hormones will be inhibited after taki- E. Precipitin ring
ng the oral contraceptives containing sex
hormones? 45. It was revealed that T-lymphocytes
were affected by HIV. Virus enzyme -
A. Gonadotropic hormone reverse transcriptase (RNA-dependent
B. Vasopressin DNA-polymerase) - catalyzes the
C. Thyrotrophic hormone synthesis of:
D. Somatotropic hormone
E. Ocytocin A. DNA on the matrix of virus mRNA
B. Virus informational RNA on the matrix
41. A histological specimen of a kidney of DNA
shows a part of the distal tubule going C. DNA on virus ribosomal RNA
between the afferent and efferent arteri- D. Viral DNA on DNA matrix
ole. The cells building the tubule wall have E. mRNA on the matrix of virus protein
dense nuclei; basal membrane is absent.
Such structural formation is called: 46. Microscopical examination of an
enlarged cervical lymph node revealed
A. Macula densa blurring of its structure, absence of
B. Juxtaglomerular cells lymphoid follicles; all the microscopic fi-
C. Mesangial cells elds showed cells with roundish nuclei and
D. Juxtavascular cells thin limbus of basophil cytoplasm. It is
E. - known from the clinical data that other
42. During preparation of a patient to a groups of lymph nodes are also enlarged
heart surgery it was necessary to measure as well as spleen and liver. What disease
pressure in heart chambers. In one of might be suspected?
them pressure varied from 0 mm Hg up
to 120 mm Hg within one cardiac cycle.
What heart chamber is it?
Krok 1 Medicine 2011 6

A. Lymphoid leukosis A. Dobutamine


B. Lymphogranulomatosis B. Caffeine sodium benzoate
C. Lymphosarcoma C. Cordiamin
D. Myeloid leukosis D. Aethimizolum
E. Multiple myeloma E. Bemegride
47. A 38-year-old patient came to a 52. After transfusion of 200 ml of blood a
traumatology centre and complained patient presented with body temperature
about an injury of his right hand. Objecti- rise up to 37, 9oC. Which of the followi-
vely: the patient has a cut wound in the ng substances is the most likely cause of
region of the thenar eminence on the right temperature rise?
hand; distal phalanx of the I finger cannot
be flexed. What muscle was injured? A. Interleukin-1
B. Interleukin-2
A. Long flexor muscle of thumb C. Tumour necrosis factor
B. Short flexor muscle of thumb D. Interleukin-3
C. Short abductor muscle of thumb E. Interleukin-4
D. Opposer muscle of thumb
E. Abductor muscle of thumb 53. A man who is riding the carousel
presents with increased heart rate, sweati-
48. Lung ventilation in a person is ng, nausea. This condition is caused pri-
increased as a result of physical activi- marily by the stimulation of the following
ty. Which of the following indices of the receptors:
external respiration is much higher than
in a state of rest? A. Vestibular ampullar
B. Proprioceptors
A. Respiratory volume C. Vestibular otolithic
B. Vital capacity of lungs D. Auditory
C. Inspiratory reserve volume E. Visual
D. Expiratory reserve volume
E. Total lung capacity 54. A worker of a cattle farm fell acutely
ill and then died from the progressing
49. As a result of activation of the ion intoxication. Autopsy revealed enlarged,
channels of the external membrane the hyposthenic spleen of dark-cherry colour
rest potential of an excitable cell has when dissected; excessive pulp scraping.
greatly increased. What channels were At the base and fornix of brain pia maters
activated? are edematous, soaked with blood, dark-
red ("scarlet hat"). Microscopic exami-
A. Potassium channels nation revealed serous haemorrhagic
B. Sodium channels inflammation of brain tissues and tuni-
C. Fast calcium channels cs along with destruction of small vessel
D. Slow calcium channels walls. What is the most likely diagnosis?
E. Sodium and calcium channels
A. Anthrax
50. As a result of continuous starvati- B. Tularemia
on the glomerular filtration rate has C. Brucellosis
increased by 20%. The most probable D. Plaque
cause of the glomerular filtration alterati- E. Cholera
on under the mentioned conditions is:
55. Which of the listed diuretic agents WI-
A. Decrease in the oncotic pressure of LL NOT have diuretic effect on a patient
blood plasma with Addison’s disease?
B. Increase in the systemic arterial pressure
C. Increase in the permeability of the renal A. Spironolactone
filter B. Furosemide
D. Increase of the filtartion quotient C. Hydrochlorothiazide
E. Increase of the renal blood flow D. Triamterene
E. Ethacrynic acid
51. A patient with cardiogenic shock,
hypotension, asphyxia and edemata was 56. A man with a wound of his limb that
given an injection of non-glycosidic cardi- had been suppurating for a long time di-
otonic. What drug was injected? ed from intoxication. Autopsy revealed
extreme emaciation, dehydration, brown
Krok 1 Medicine 2011 7

atrophy of liver, myocardium, spleen and alterations are indicative of the following
cross-striated muscles as well as renal disease:
amyloidosis. What diagnosis corresponds
with the described presentations? A. Silicosis
B. Acute pneumonia
A. Chroniosepsis C. Multiple bronchiectasis
B. Septicopyemia D. Chronic bronchitis
C. Septicemia E. Bronchial asthma
D. Chernogubov’s syndrome
E. Brucellosis 62. A 12-year-old teenager has signifi-
cantly put off weight within 3 months;
57. This drug has a destructive effect on glucose concentration rose up to 50 mi-
erythrocytic forms of malarial plasmodia llimole/l. He fell into a coma. What is the
and dysenteric amoebae. It is used for main mechanism of its development?
treatment and prevention of such diseases
as malaria, amebiasis and interstitial di- A. Hyperosmolar
sease. What drug is it? B. Hypoglycemic
C. Ketonemic
A. Chingamin D. Lactacidemic
B. Emetine hydrochloride E. Hypoxic
C. Tetracycline
D. Erythromycin 63. Which way of heat emission by the
E. Quinine bodies of greenhouse workers is the most
effective at the temperature of 36o C
58. A patient has an increased pyruvate degrees and relative humidity of 70%?
concentration in blood, most of it is
excreted with the urine. What kind of avi- A. Liquid evaporation
taminosis has this patient? B. Thermal conduction
C. Heat radiation
A. B1 D. Convection
B. E E. -
C. B3
D. B6 64. Examination of a 70-year-old pati-
E. B2 ent revealed insulin-dependent diabetes.
What drug should be administered?
59. A child suffers from drug idiosyncrasy.
What is the cause of such reaction? A. Glibenclamid
B. Insulin
A. Hereditary enzymopathy C. Mercazolilum
B. Exhaustion of substrate interacting with D. Parathyroidin
pharmaceutical substance E. Cortisone
C. Accumulation of pharmaceutical
substance 65. A disaster fighter at a nuclear power
D. Inhibition of microsomal liver enzymes plant developed hemorrhagic syndrome
E. Associated disease of target organ on the background of acute radiation di-
sease. What is the most important factor
60. One of the parents is suspected of of syndrome pathogenesis?
having phenylketonuria recessive gene.
What is the risk of giving birth to a chi- A. Thrombocytopenia
ld with inborn phenylketonuria? B. Vascular wall damage
C. Increased activity of fibrinolysis factors
A. 0% D. Increased activity of anticoagulative
B. 25% system factors
C. 50% E. Decreased activity of coagulative factors
D. 75%
E. 100% 66. Jaundice treatment involves admi-
nistration of barbiturates inducing the
61. Pulmonary examination of a pati- synthesis of UDP-glucuronyl transferase.
ent who has worked as a stone grinder A medicinal effect is caused by the
for 9 years revealed small dense roundi- production of:
sh nodules consisting of connective ti-
ssue. The nodules were found to have
peripheral macrophages. Such pulmonary
Krok 1 Medicine 2011 8

A. Direct reacting (conjugated) bilirubin


B. Indirect reacting (unconjugated) bili- A. Acute purulent bronchopneumonia
rubin B. Acute bronchitis
C. Biliverdin C. Croupous pneumonia
D. Protoporphyrin D. Intermittent pneumonia
E. Heme E. Acute serous bronchopneumonia

67. A histological specimen shows a 71. A surgeon has to find the common
blood vessel. Its inner coat is composed hepatic duct during the operative
by endothelium, subendothelium and intervention on account of concrements in
internal elastic membrane. The middle the gall ducts. The common hepatic duct
coat is enriched with smooth myocytes. is located between the leaves of:
Such morphological characteristics are
typical for the following vessel: A. Hepatoduodenal ligament
B. Hepatogastric ligament
A. Muscular-type artery C. Hepatorenal ligament
B. Elastic-type artery D. Round ligament of liver
C. Capillary E. Venous ligament
D. Non-muscular vein
E. Muscular-type vein 72. Autopsy of a 1,5-year-old chi-
ld revealed haemorrhagic skin rash,
68. An 18-year-old man was delivered to moderate hyperaemia and edema of
the hospital after a road accident. Exami- nasopharyngeal mucous membrane, small
nation at the traumatological department haemorrhages in the mucous membranes
revealed multiple injuries of soft tissues of and internal organs; dramatic dystrophic
face in the region of the medial eye angle. alterations in liver and myocardium; acute
The injuries caused massive haemorrhage. necrotic nephrosis; massive haemorrhages
What arterial anastomosis might have in the adrenal glands. What disease are
been damaged in this region? these alterations the most typical for?

A. a. carotis externa et a. carotis interna A. Meningococcal infection


B. a. carotis externa et a. subclavia B. Scarlet fever
C. a. carotis interna et a. subclavia C. Diphtheria
D. a. subclavia et a. ophthalmica D. Measles
E. a. carotis interna et a. ophthalmica E. Epidemic typhus

69. Autopsy of a 75-year-old man with a 73. A 75-year-old-female patient wi-


long history of atherosclerosis revealed a th complaints of visual impairment has
grey irregular-shaped focus of loose consi- been delivered to the ophthalmologic
stency in the right parietotemporal region department. Objective examination
of brain. What is the most likely cause of revealed a brain tumor in area of the left
this process? optic tract. The patient has a visual field
defect in the following area:
A. Thrombosis of the right medial cerebral
artery A. Left half of both eyes retina
B. Thrombosis of the right anterior B. Right half of both eyes retina
cerebral artery C. Left and right halves of the left eye
C. Thrombosis of the right posterior retina
cerebral artery D. Left and right halves of the right eye
D. Thrombosis of basilar artery retina
E. Thrombosis of tomentum cerebri E. Left and right halves of both eyes retina

70. Autopsy of a man with a mali- 74. A patient with coronary disease
gnant stomach tumour who had di- has been diagnosed with myocardial
ed from cancer intoxication revealed hypertrophy, tachycardia and a decrease
in the posteroinferior lung fields some in minute blood volume. What is the
dense, grayish-red irregular foci protrudi- leading mechanism of cardiac hystiocyte
ng above the section surface. Microscopic damage in this case?
examination revealed exudate containi-
ng a large amount of neutrophils in the
lumen and walls of small bronchi and
alveoles. Such pulmonary alterations indi-
cate the following disease:
Krok 1 Medicine 2011 9

A. Damage of specific membrane pumps A. Nongaseous alkalosis


B. Increase in α and β adrenoreceptors B. Gaseous alkalosis
quantity C. Gaseous acidosis
C. Mg 2+ loss by cardiac hystiocytes D. Metabolic acidosis
D. Ca2+ loss by cardiac hystiocytes E. Excretory acidosis
E. Cardiac hystiocyte dehydration
80. A concentrated solution of sodium
75. Blood analysis of a patient wi- chloride was intravenously injected to an
th jaundice reveals conjugated bilirubi- animal. This caused decreased reabsorpti-
nemia, increased concentration of bile aci- on of sodium ions in the renal tubules. It
ds. There is no stercobilinogen in urine. is the result of the following changes of
What type of jaundice is it? hormonal secretion:

A. Obstructive jaundice A. Aldosterone reduction


B. Hepatocellular jaundice B. Aldosterone increase
C. Parenchymatous jaundice C. Vasopressin reduction
D. Hemolytic jaundice D. Vasopressin increase
E. Cythemolytic jaundice E. Reduction of atrial natriuretic factor

76. While studying maximally spiralized 81. A patient has been diagnosed with
chromosomes of human karyotype the alkaptonuria. Choose an enzyme whose
process of cell division was stopped in the deficiency can be the reason for this
following phase: pathology:

A. Metaphase A. Homogentisic acid oxidase


B. Prophase B. Phenylalanine hydroxylase
C. Interphase C. Glutamate dehydrogenase
D. Anaphase D. Pyruvate dehydrogenase
E. Telophase E. Dioxyphenylalanine decarboxylase

77. A man is being measured power inputs 82. A patient consulted an urologist about
on an empty stomach, in the lying posi- pain during urination. Analysis of his uri-
tion, under conditions of physical and ne taken in the daytime revealed eggs wi-
psychic rest at a comfortable temperature. th a characteristic sharp point. It is known
Power inputs will reach the maximum at: from the anamnesis that the patient has
recently returned from Australia. What is
A. 5-6 p.m. the most likely diagnosis?
B. 7-8 a.m.
C. 10-12 a.m. A. Urogenital schistosomiasis
D. 2-3 p.m. B. Intestinal schistosomiasis
E. 3-4 a.m. C. Japanese schistosomiasis
D. Opisthorchiasis
78. When measuring power inputs of a E. Dicroceliasis
man by the method of indirect calori-
metry the following results were obtained: 83. A 49-year old female patient has limi-
1000 ml oxygen consumption and 800 ml tation of left limbs arbitrary movements.
carbon dioxide liberation per minute. The Muscular tonus of left hand and leg is
man under examination has the following overstrained and spasmodic, local tendon
respiratory coefficient: reflexes are strong, pathological reflexes
are presented. What is the most likely
A. 0,8 development mechanism of hypertensi-
B. 1,25 on and hyperreflexia?
C. 0,9
D. 0,84 A. Reduction of descending inhibitory
E. 1,0 influence
B. Motoneuron activation induced by
79. A newborn child with pylorostenosis stroke
has often repeating vomiting accompani- C. Activation of excitatory influence from
ed by apathy, weakness, hypertonicity, the focus of stroke
sometimes convulsions. What disorder D. Activation of synaptic transmission
form of acid-base balance is it? E. Ihibition of cerebral cortex
motoneurons
Krok 1 Medicine 2011 10

84. A 35-year-old patient complains about um hydrocarbonate injected for?


having severe rhinitis and loss of sense
of smell for a week. Objectively: the nasal A. For increasing renal excretion of
cavity contains a lot of mucus covering the phenobarbital
mucous membrane and blocking olfactory B. For breathing stimulation
receptors. In what region of the nasal cavi- C. For arterial pressure normalization
ty are these receptors located? D. For phenobarbital inactivation
E. For bringing the patient to consci-
A. Superior nasal concha ousness
B. Median nasal concha
C. Inferior nasal concha 89. While examining the oral cavity a
D. Common nasal meatus stomatologist revealed inflammation of
E. Vestibule of nose papillae on the border of the median
and posterior third of the back of tongue.
85. A 10-year-old child had the mantoux What papillae are inflamed?
tuberculin test administered. 48 hours
later a papule up to 8 mm in diameter A. Papillae vallatae
appeared on the site of the injection. B. Papillae fungiformes
What type of hypersensitivity reaction C. Papillae foliatae
developed after the tuberculin injection? D. Papillae filiformes
E. Papillae conicae
A. Type IV hypersensitivity reaction
B. Arthus phenomenon 90. A patient is 44 years old. Laboratory
C. Seroreaction examination of his blood revealed that
D. Atopic reaction content of proteins in plasma was 40 g/l.
E. Type II hypersensitivity reaction What influence will be exerted on the
transcapillary water metabolism?
86. Vitamin B1 deficiency causes di-
sturbance of oxidative decarboxylati- A. Filtration will be increased, reabsorpti-
on of α-ketoglutaric acid. This leads to on - decreased
the impaired synthesis of the following B. Both filtration and reabsorption will be
coenzyme: increased
C. Both filtration and reabsorption will be
A. Thiamine pyrophosphate decreased
B. Nicotinamide adenine dinucleotide D. Filtration will be decreased, reabsorpti-
C. Flavine adenine dinucleotide on - increased
D. Lipoic acid E. Metabolism will stay unchanged
E. Coenzyme A
91. A patient has corestenoma. What is
87. Examination of a pregnant woman the reason of such condition?
having Rh-negative blood revealed hi-
gh level of antierythrocytic antibodies. A. Increased tonus of parasympathetic
For its reduction she was implanted wi- centres
th her husband’s Rh-positive skin graft. B. Increased tonus of sympathetic centres
The graft was rejected in two weeks. C. Increased activity of sympathoadrenal
Its microscopic examination revealed ci- system
rculatory disturbance, edema and cellular D. Adrenaline action
infiltration with lymphocytes, neutrophils E. Noradrenaline action
and macrophages predominance. What is
the most likely pathology? 92. A 46-year-old man had a bulging
dark macula on skin that caused no di-
A. Graft immunity scomfort. With time it began to increase
B. Immediate hypersensitivity in size and became painful. It turned dark
C. Delayed-type hypersensitivity brown and there was a nodule on palpati-
D. Granulomatous inflammation on. Histological examination of tissues
E. Interstitial inflammation revealed spindle and polymorphous cells
with multiple mitoses. Their cytoplasm
88. A 17-year-old girl took a high dose contained brown pigment. What tumour
of phenobarbital to commit a suici- is it?
de. An ambulance doctor cleansed her
stomach and gave her an intravenous
injection of bemegride and sodium
hydrocarbonate solution. What was sodi-
Krok 1 Medicine 2011 11

A. Melanoma A. Axillary nerve


B. Basalioma B. Subscapular nerve
C. Hemangioma C. Dorsal scapular nerve
D. Nevus D. Subclavicular nerve
E. - E. Thoracodorsal nerve
93. The patient with complaints of 98. Pyeloureterography X-ray photo
permanent thirst applied to the doctor. showed a renal pelvis with minor calyces
Hyperglycemia, polyuria and increased only (major calyces were absent). What
concentration of 17-ketosteroids in the form of urinary tracts of a kidney was
urine were revealed. What disease is the revealed?
most likely?
A. Embryonal
A. Steroid diabetes B. Fetal
B. Insulin-dependent diabetes mellitus C. Mature
C. Myxoedema D. Ampullar
D. Type I glycogenosis E. -
E. Addison’s disease
99. A female patient presents with
94. A 32-year-old patient consulted a endocrine dysfunction of follicular cells
doctor about the absence of lactation of the ovarian follicles resulting from
after parturition. Such disorder might be an inflammation. The synthesis of the
explained by the deficit of the following following hormone will be inhibited:
hormone:
A. Estrogen
A. Prolactin B. Progesterone
B. Somatotropin C. Lutropin
C. Vasopressin D. Follicle stimulating hormone
D. Thyrocalcitonin E. Follistatine
E. Glucagon
100. A 19-year-old female patient has
95. An infant has pylorospasm, weakness, had low haemoglobin rate of 90-95 g/l si-
hypodynamia, convulsions as a result of nce childhood. Blood count results obtai-
frequent vomiting. What kind of acid-base ned after hospitalisation are as follows:
disbalance is it? erythrocytes - 3, 2 · 1012 /l, Hb- 85 g/l,
A. Excretory alkalosis colour index - 0,78; leukocytes - 5, 6 · 109 /l,
B. Excretory acidosis platelets - 210 · 109 /l. Smear examinati-
C. Metabolic acidosis on revealed anisocytosis, poikilocytosis
D. Exogenous nongaseous acidosis and target cells. Reticulocyte rate is 6%.
E. Gaseous alkalosis Iron therapy was ineffective. What blood
pathology corresponds with the described
96. A weightlifter has a disruption of clinical presentations?
thoracic lymphatic duct as a result of li-
fting a weight. Choose the most likely site A. Thalassemia
of injury: B. Enzymopathy
C. Membranopathy
A. In the region of aortic hiatus D. Sickle-cell anemia
B. In the region of lumbosacral plexus E. Favism
C. In the posterior mediastinum
D. In the region of venous angle 101. A liquidator of a breakdown at a
E. In the region of neck nuclear power plant who was irradiated
complained about vomiting that occurs all
97. A woman suffering from osteochondrosis of a sudden. What medication should be
has acute pain in her humeral articulation prescribed?
that gets worse when she tries to abduct
her shoulder. These symptoms might be A. Metoclopramide
caused by damage of the following nerve: B. Reserpine
C. Atropine
D. Aeron
E. De-Nol
102. A patient with acute myocardial
infarction has been administered heparin
Krok 1 Medicine 2011 12

as a part of complex therapy. Some A. Fibroadenoma


time after heparin injection the pati- B. Fibroma
ent developed hematuria. What heparin C. Metastatic cancer
antagonist should be injected in order to D. Adenoma
manage the complication? E. Fibrocarcinoma
A. Protamine sulfate 107. A 38-year-old male patient has been
B. Vicasol ill with systemic lupus erythematosus for
C. Aminocaproic acid three years. He was diagnosed with diffuse
D. Neodicumarin renal affection accompanied by massi-
E. Fibrinogen ve edemata and expressive proteinuria.
What is the most likely cause of protei-
103. A 45-year-old patient was admitted to nuria development?
the cardiological department. ECG data:
negative P wave overlaps QRS complex, A. Autoimmune renal affection
diastolic interval is prolonged after B. Aseptic renal affection
extrasystole. What type of extrasystole is C. Ischemic renal affection
it? D. Urinary bladder inflammation
E. Urinary tracts inflammation
A. Atrioventricular
B. Sinus 108. A student failed to answer all the
C. Atrial questions of examination paper correctly.
D. Ventricular As a result he blushed, felt hot and
E. Bundle-branch lost confidence. What type of arterial
hyperemia has developed in this case?
104. A patient complains of hydruria (7
liters per day) and polydipsia. Examinati- A. Neurotonic hyperemia
on reveals no disorders of carbohydrate B. Neuroparalytic hyperemia
metabolism. These abnormalities might C. Metabolic hyperemia
be caused by the dysfunction of the D. Pathologic hyperemia
following endocrine gland: E. Postishemic hyperemia

A. Neurohypophysis 109. Acute renal impairment caused


B. Adenohypophysis death of a bleeding patient. Autopsy
C. Islets of Langerhans (pancreatic islets) revealed enlarged kidneys with a broad
D. Adrenal cortex pale pink cortical layer expressively
E. Adrenal medulla demarcated from dark red renal pyrami-
ds. Macroscopic examination revealed
105. Before a surgery a blood sample of a lack of epithelial nuclei of convoluted
30-year-old man has been typed. Blood tubules, tubulorrhexis, phlebostasis. The
is Rh-positive. Standard serums of such cell nuclei of choroid glomus and straight
groups as 0 αβ (I), Аβ (II), Вα (III) didn’t tubules were present. What pathology is
activate erythrocyte agglutination reacti- it?
on. The group of the analyzed blood is:
A. Necronephrosis
A. 0 αβ (I) B. Infarction
B. Аβ (II) C. Glomerulonephritis
C. Вα (III) D. Pyelonephritis
E. Nephrosis
D. АВ (IV)
E. - 110. A female patient consulted a doctor
about pain and limited movements in
106. Medical examination of a 20-year- the knee joints. Which of the followi-
old woman revealed a dense incapsulated ng nonsteroid anti-inflammatory drugs
node 1 cm in diameter that was palpated should be administered taking into consi-
in the mammary gland. The postoperati- deration that the patient has a history of
ve biopsy revealed connective tissue chronic gastroduodenitis?
overgrowth around the mammary ducts
and glandular components of different di-
ameter that didn’t make lobules and bore
no signs of cellular abnormality. What di-
agnosis will be made?
Krok 1 Medicine 2011 13

A. Celecoxib upper region of anterior abdominal wall.


B. Diclofenac sodium Which of the described mechanisms might
C. Promedol have provoked the cardiac arrest?
D. Acetylsalicilic acid
E. Butadiounum A. Parasympathetic unconditioned reflexes
B. Sympathetic unconditioned reflexes
111. A patient presents with icteriti- C. Parasympathetic conditioned reflexes
ousness of skin, scleras and mucous D. Sympathetic conditioned reflexes
membranes. Blood plasma the total bili- E. Peripheric reflexes
rubin is increased, stercobilin is increased
in feces, urobilin is increased in urine. 116. There is a severe time restriction for
What type of jaundice is it? people’s staying at a height of over 800
m above the sea level without oxygen
A. Haemolytic bombs. What is the life limiting factor in
B. Gilbert’s disease this case?
C. Parenchymatous
D. Obturational A. Partial oxygen pressure
E. Cholestatic B. Ultraviolet intensity
C. Moisture level
112. Blood plasma of a healthy man D. Temperature
contains several dozens of proteins. Duri- E. Earth gravity
ng an illness new proteins can originate,
namely the protein of "acute phase". 117. A patient with marked pneumofi-
Select such protein from the listed below: brosis that developed after infiltrati-
ng pulmonary tuberculosis has been di-
A. C-reactive protein agnosed with respiratory failure. What is
B. Prothrombin its pathogenetic type?
C. Fibrinogen
D. G immunoglobulin A. Restrictive
E. A immunoglobulin B. Obstructive
C. Dysregulatory
113. A patient working at a pig farm D. Reflex
complains about paroxysmal abdomi- E. Apneistic
nal pain, liquid feces with admixtures of
mucus and blood, headache, weakness, 118. During surgical manipulations a pati-
fever. Examination of large intestine ent has been given novocaine injection for
revealed ulcers from 1 mm up to several anesthesia. 10 minutes later the patient
cm large, feces contained oval unicellular developed paleness, dyspnea, hypotensi-
organisms with cilia. What disease should on. What type of allergic reaction is it?
be suspected?
A. Anaphylactic immune reaction
A. Balantidiasis B. Cellulotoxic immune reaction
B. Amebiasis C. Aggregate immune reaction
C. Toxoplasmosis D. Stimulating immune reaction
D. Lambliasis E. Cell-mediated immune reaction
E. Trichomoniasis
119. A sensitive neural ganglion consists
114. During hypersensitivity test a patient of roundish neurocytes with one extensi-
got subcutaneous injection of an antigen on that divides into axon and dendrite at a
which caused reddening of skin, edema, some distance from the perikaryon. What
pain as a result of histamine action. This are these cells called?
biogenic amine is generated as a result of
transformation of the following histidine A. Pseudounipolar
amino acid: B. Unipolar
C. Bipolar
A. Decarboxylation D. Multipolar
B. Methylation E. Apolar
C. Phosphorylation
D. Isomerization 120. A patient complained about di-
E. Deaminization zziness, memory impairment, periodical
convulsions. It was revealed that these
115. During fighting a man had a cardiac changes were caused by a product of
arrest as a result of a hard blow to the decarboxylation of glutamic acid. Name
Krok 1 Medicine 2011 14

this product: a big irregular round-shaped cell. There


are a lot of intracellular tubules and mi-
A. GABA tochondria in the cytoplasm. Specify this
B. Pyridoxal phosphate cell:
C. TDP
D. ATP A. Parietal cell
E. THFA B. Principal cell
C. Undifferentiated cell
121. It was found out that some D. Mucous cell
compounds, for instance fungi toxins and E. Endocrine cell
some antibiotics can inhibit activity of
RNA-polymerase. What process will be 126. Blood count of an athlete is as
disturbed in a cell in case of inhibition of follows: erythrocytes - 5, 5 · 1012 /l, Hb-
this enzyme? 180 g/l, leukocytes - 7 · 109 /l, neutrophils -
64%, basophils - 0,5%, eosinophils - 0,5%,
A. Transcription monocytes - 8%, lymphocytes - 27%. First
B. Processing of all, such results indicate the stimulation
C. Replication of:
D. Translation
E. Reparation A. Erythropoiesis
B. Leukopoiesis
122. During an experiment the myotatic C. Lymphopoiesis
reflex has been studied in frogs. After D. Granulocytopoiesis
extension in a skeletal muscle its E. Immunogenesis
reflectory contraction was absent. The
reason for it might be a dysfunction of 127. A patient died from cardiopulmonary
the following receptors: decompensation. Histological examinati-
on revealed diffused pulmonary affection
A. Muscle spindles along with interstitial edema, infiltration
B. Nociceptors of tissue by limphocytes, macrophages,
C. Articular plasmocytes; pulmonary fibrosis, panaci-
D. Golgi tendon organs nar emphysema. What is the most likely
E. Tactile diagnosis?
123. An adult man presents with systemic A. Fibrosing alveolitis
arterial pressure drop from 120/70 to B. Chronic bronchitis
90/50 mm Hg. This resulted in reflex C. Bronchopneumonia
vasoconstriction. Vasoconstriction will be D. Pulmonary atelectasis
minimal in the following organ: E. Bronchial asthma
A. Heart 128. If a man has an attack of bronchi-
B. Skin ospasm it is necessary to reduce the effect
C. Bowels of vagus on smooth muscles of bronchi.
D. Skeletal muscles What membrane cytoreceptors should be
E. Liver blocked for this purpose?
124. A culture of monkey cells (Vero) and A. M-cholinoreceptors
a group of mouse sucklings were infected B. N-cholinoreceptors
with an inoculum taken from a child with C. α-adrenoreceptors
provisional diagnosis "enterovirus infecti- D. β-adrenoreceptors
on". There was no cytopathic effect on E. α- and β-adrenoreceptors
the cell culture but mouse sucklings died.
What enteric viruses might have caused 129. A male patient has been di-
disease of this child? agnosed with gastric ulcer. Bacteriologi-
cal examination of biopsy material from
A. Coxsackie A the affected part of stomach revealed
B. Coxsackie B small colonies of gram-negative, oxide
C. ECHO virus reductase-positive flexibacteria that grew
D. Polioviruses on the chocolate agar on the fifth day.
E. Unclassified enteric viruses 68-71 Which of the following microorganisms
is the most likely causative agent?
125. An electron microphotography of a
fragment of proper gastric gland shows
Krok 1 Medicine 2011 15

A. Helicobacter pylori A. Accumulation of Na+ and Ca2+ ions in


B. Campilobacter jejuni the myocardium cells
C. Campilobacter fetus B. Accumulation of K + and Mg 2+ ions in
D. Mycoplasma hominis the myocardium cells
E. Chlamydia trachomatis C. Reduction of Na+ and Ca2+ ions in the
130. There are several groups of myocardium cells
molecular mechanisms playing important D. Reduction of K + and Mg 2+ ions in the
part in pathogenesis of insult to cells extracellular space
which contributes to the pathology E. Reduction of Na+ and Ca2+ ions in the
development. What processes are sti- extracellular space
mulated by proteinic damage mechani-
sms? 134. A child with a history of
frequent angine and pharyngitis has
A. Enzyme inhibition been diagnosed with lymphadenopathy
B. Lipid peroxidation and splenomegaly. His appearance is
C. Phospholipase activation characterised by pastosity and paleness,
D. Osmotic membrane distension muscular tissue is poorly developed.
E. Acidosis Lymphocytosis is present. What kind of
diathesis is it?
131. A patient has been diagnosed wi-
th a compression fracture of a lumbar A. Lymphohypoplastic diathesis
vertebra. As a result he has a consi- B. Exudative diathesis
derable increase in curvature of the C. Gouty diathesis
lumbar lordosis. Which ligament damage D. Asthenic diathesis
can induce such changes in the spine E. Hemorrhagic diathesis
curvature?
135. In response to a change in body posi-
A. Anterior longitudinal ligament tion from horizontal to vertical blood
B. Posterior longitudinal ligament circulation system develops reflectory
C. Yellow ligament pressor reaction. Which of the following
D. Iliolumbar ligament is its compulsory component?
E. Interspinous ligament
A. Systemic constriction of the venous
132. A 49-year-old patient with croupous vessels
pneumonia died from pneumococcal B. Systemic dilatation of the arterial resi-
septicemia. Autopsy revealed up to 700 stive vessels
ml of turbid greenish-yellow foul-smelling C. Decrease in the circulating blood
liquid in the left pleural cavity. The pleural volume
leaflets were dull and plethoric. What D. Increase in the heart rate
form of pleural inflammation is it? E. Weakening of the pumbing ability of
heart
A. Empyema
B. Chronic abscess 136. After an immunoassay a chi-
C. Acute abscess ld was diagnosed with immunodefici-
D. Phlegmon ency of humoral immunity. What is the
E. Fibrinous inflammation reason for the primary immunodeficiency
development in the child?
133. A 35-year-old man developed acute
heart failure while running for a long time. A. Hereditary abnormality of immune
What changes in ionic composition can be system
observed in the cardiac muscle? B. Embryonal development abnormalities
C. Pathometabolism in mother’s organism
D. Immune responsiveness and resistance
disorders
E. Toxic damage of B-lymphocytes
137. Life cycle of a cell includes the
process of DNA autoreduplication. As a
result of it monochromatid chromosomes
turn into bichromatid ones. What period
of cell cycle does this phenomenon fall
into?
Krok 1 Medicine 2011 16

142. A patient with coronary disease


A. S and arrhythmia has been administered a
B. Go drug that blocks potassium channels and
C. G1 prolongs the action potential. What drug
D. G2 is it?
E. M
A. Amiodarone
138. Examination of a 12-year-old B. Corglyconum
boy with developmental lag revealed C. Nitroglycerin
achondroplasia: disproportional consti- D. Dobutamine
tution with evident shortening of upper E. Lisinopril
and lower limbs as a result of growth
disorder of epiphyseal cartilages of long 143. A patient takes digoxin for cardiac
tubal bones. This disease is: insufficiency. What diuretic may increase
digoxin toxicity due to the intensified
A. Inherited, dominant excretion of K + ions?
B. Inherited, recessive
C. Inherited, sex-linked A. Hydrochlorothiazide
D. Congenital B. Spironolactone
E. Acquired C. Panangine
D. Siliborum
139. 2 days after labour a woman E. Lisinopril
developed shock along with DIC
syndrome that caused her death. Autopsy 144. During starvation muscle proteins
revealed purulent endomyometritis, regi- break up into free amino acids. These
onal purulent lymphangitis, lymphadeni- compounds will be the most probably
tis and purulent thrombophlebitis. There involved into the following process:
were also dystrophic alterations and
interstitial inflammation of parenchymal A. Gluconeogenesis in liver
organs. What is the most likely diagnosis? B. Gluconeogenesis in muscles
C. Synthesis of higher fatty acids
A. Septicemia D. Glycogenolysis
B. Syphilis E. Decarboxylation
C. Tuberculosis of genital organs
D. Chorioadenoma destruens 145. A female patient consulted a doctor
E. Hydatid mole about a sense of epigastric discomfort,
nausea and anorexia. A duodenal content
140. After a craniocerebral trauma a pati- analysis revealed lamblia. What drug
ent lost the ability to execute learned should be prescribed?
purposeful movements (apraxia). The
injury is most likely localized in the A. Metronidazole
following region of the cerebral cortex: B. Chingamin
C. Rifampicin
A. Gyrus supramarginalis D. Isoniazid
B. Gyrus angularis E. Acyclovir
C. Gyrus paracentralis
D. Gyrus lingualis 146. A 28-year-old female patient
E. Gyrus parahippocampalis consulted a gynecologist about sterility.
Examination revealed underdeveloped
141. Tissue inosytol triphosphates are ovaries and uterus, irregular menstrual
generated as a result of the phosphati- cycle. Analysis of the sex chromatin
dylinositol diphosphate hydrolysis and revealed 2 Barr’s bodies in most somatic
act as secondary agents (mediators) in cells. What chromosome disease is most
the mechanism of hormone action. Their likely?
effect in cells is directed at:
A. Triple X syndrome
A. Calcium ion liberation from cellular B. Edwards’ syndrome
depot C. Patau’s syndrome
B. Adenylate cyclase activation D. Klinefelter’s syndrome
C. Protein kinase A activation E. Turner’s syndrome
D. Phosphodiesterase inhibition
E. Protein kinase A inhibition 147. A 1-year-old baby has been hospitali-
sed for body and limbs lesions. Exami-
Krok 1 Medicine 2011 17

nation revealed carnitine deficiency in the by eight histone proteins and a part of
child’s muscles. A biochemical reason for DNA molecule which makes about 1,75
this pathology is the disorder of: revolutions around the molecules. Which
structure has been identified?
A. Transport of fatty acids to mitochondria
B. Regulation of Ca2+ rate in mitochondria A. Nucleosoma
C. Substrate-linked phosphorylation B. Elemetary fibril
D. Utilization of lactic acid C. Half-chromatid
E. Oxidative phosphorylation D. Chromatid
E. Chromosome
148. During anesthetization a pati-
ent presented with symptoms of tonus 153. A patient with massive burns
increase of parasympathetic nervous developed acute renal insufficiency
system such as hypersalivation and characterized by a significant and
laryngospasm. What drug could have rapid deceleration of glomerular fi-
prevented these undesirable effects? ltration. What is the mechanism of its
development?
A. Atropine sulphate
B. Adrenaline hydrochloride A. Reduction of renal blood flow
C. Neostigmine B. Damage of glomerular filter
D. Analgin C. Reduction of functioning nephron
E. Pyracetam number
D. Rise of pressure of tubular fluid
149. Some students developed myodynia E. Renal artery embolism
after continuous physical activity during
physical education. The reason for such 154. A three-year-old child has had
condition was accumulation of lactic acid marked diarrhea for three days. Immune
in the skeletal muscles. It was generated electron microscopy of his excrements
in the students’ bodies after activation of revealed bilayer pseudocovered capsid vi-
the following process: ruses that looked like small spoke wheels.
What viruses have been revealed?
A. Glycolysis
B. Gluconeogenesis A. Rotaviruses
C. Pentose-phosphate cycle B. Coxsackie viruses
D. Lipolysis C. ECHO viruses
E. Glycogeny D. Coronaviruses
E. Reoviruses
150. A newborn develops dyspepsia
after the milk feeding. When the mi- 155. A male patient has fever and
lk is substituted by the glucose solution enanthesis. As a result of the exami-
the dyspepsia symptoms disappear. The nation involving serological tests he has
newborn has the subnormal activity of the been diagnosed with fasciola hepatica. It
following enzyme: was found out that the patient had been
infected through raw river water. Which
A. Lactase stage of fasciola life cycle is invasive for
B. Invertase humans?
C. Maltase
D. Amylase A. Adolescaria
E. Isomaltase B. Metacercaria
C. Ovum
151. A doctor asked a patient to breath D. Miracidium
out fully after taking a normal breath. E. Cysticercus
What muscles contract during such
exhalation? 156. A patient with high-titer antinuclear
antibodies died from progressing renal
A. Abdominal muscles impairment. Autopsy revealed mesangi-
B. External intercostal muscles oproliferative glomerulonephritis and
C. Diaphragm abacterial polypous endocarditis. There
D. Trapezius muscles was periarterial bulbar sclerosis in spleen
E. Pectoral muscles and productive proliferative vasculitis in
skin. What is the most likely diagnosis?
152. On an electron micrograph a sci-
entist has identified a structure formed
Krok 1 Medicine 2011 18

A. Systemic lupus erythematosus A. Atrophic arthritis


B. Nephrotic syndrome B. Periarteritis nodosa
C. Rheumatism C. Ankylosing spondylitis
D. Dermatomyositis D. Tuberculous arthritis
E. Periarteritis nodosa E. Deforming arthrosis
157. Electronic microphotography of 161. A man having a hearing loss after
pulmonary alveole’s wall presents a big a head trauma was delivered to the
cell. Its cytoplasm has a lot of mi- neurosurgery department. The cause of
tochondria, developed Golgi apparatus, the hearing loss might be the damage of
osmiophil lamellated corpuscles. What is the following lobe of cerebral cortex:
the main function of this cell?
A. Temporal
A. It produces surfactant B. Postcentral gyrus
B. It is a component of blood-air barrier C. Parietal
C. It warms the air D. Occipital
D. It purifies the air E. Frontal
E. It absorbs microorganisms
162. A pregnant woman was registered
158. A patient has food poisoning. in an antenatal clinic and underwent
Laboratory analysis revealed a culture complex examination for a number of
of anaerobic gram-positive spore-forming infections. Blood serum contained IgM
bacteria. What is the most likely kind of to the rubella virus. What is this result
the isolated causative agent? indicative of?
A. C. perfringens A. Of primary infection
B. Proteus vulgaris B. Of a chronic process
C. P. mirabilis C. The woman is healthy
D. Vibrio parahemolyticus D. Of exacerbation of a chronic disease
E. Esherichia coli E. Of recurring infection with rubella virus
159. An animal has an increased tonus 163. A patient with tuberculosis di-
of extensor muscles. This the result ed from progressing cardiopulmonary
of intensified information transmissi- decompensation. Autopsy in the region of
on to the motoneurons of the spinal the right lung apex revealed a cavity 5 cm
cord through the following descending in diameter communicating with lumen
pathways: of a segmental bronchus. On the insi-
de cavity walls are covered with caseous
A. Vestibulospinal masses with epithelioid and Langhans
B. Medial corticospinal cells beneath them. What morphological
C. Reticulospinal form of tuberculosis is it?
D. Rubrospinal
E. Lateral corticospinal A. Acute cavernous tuberculosis
B. Tuberculoma
160. A 70-year-old male patient died from C. Caseous pneumonia
acute coronary insufficiency. He had knee D. Infiltrative tuberculosis
joint swelling, gonycampsis and gonalgia E. Acute focal tuberculosis
during his lifetime. Pathomorphologic
examination of the deformed joi- 164. Vomiting matters of a patient
nts and synovial membranes revealed suspected of having cholera were deli-
membrane hyperaemia with multiple vered to the bacteriological laboratory.
perivascular inflammatory infltrations The material was used for preparing a
made by lymphocytes, plasmocytes "hanging drop"specimen. What type of
and macrophagocytes. There was an microscopy will be applied for identificati-
accumulation of organized fibrin covering on of the causative agent by its mobility?
some areas of synovium membrane and
looking like rice grains in the articular li- A. Phase-contrast microscopy
quid. What is the most likely diagnosis? B. Electron microscopy
C. Immune and electron microscopy
D. Fluorescence microscopy
E. Immersion microscopy
165. A patient with enteritis accompanied
Krok 1 Medicine 2011 19

by massive diarrhea has low water rate by lymphoid tissue making some lymph
in the extracellular space, high water rate nodes. The nodes are located diffusi-
inside the cells and low blood osmolari- vely and contain a central artery. What
ty. What is such disturbance of water- anatomic formation might have such
electrolytic metabolism called? morphological structure?
A. Hypo-osmolar hypohydration A. Spleen
B. Hyperosmolar hypohydration B. Red bone marrow
C. Osmolar hypohydration C. Thymus
D. Hypo-osmolar hyperhydration D. Tonsil
E. Hyperosmolar hyperhydration E. Lymph node
166. Examination of a 6-month-old child 171. A victim with a head trauma in the
revealed a delay in closure of the occipi- temporal region has been diagnosed with
tal fontanelle. When should it normally epidural hematoma. What artery is most
close? likely to be damaged?
A. Until 3 months A. Medial membranous artery
B. Before the child is born B. Medial cerebral artery
C. Until 6 months C. Superficial temporal artery
D. Until the end of the first year of life D. Anterior membranous artery
E. Until the end of the second year of life E. Posterior auricular artery
167. A newborn child was found to have 172. A child has mental and physical
reduced intensity of sucking, frequent retardation, grave damage of internal
vomiting, hypotonia. Urine and blood connective tissue. Urine analysis reveals
exhibit increased concentration of citrulli- keratan sulfates. What metabolic process
ne. What metabolic process is disturbed? is disturbed?
A. Ornithinic cycle A. Glycosaminoglycans
B. Tricarboxylic acid cycle B. Collagen
C. Glycolysis C. Elastin
D. Glyconeogenesis D. Fibronectin
E. Cori cycle E. Hyaluronic acid
168. A male patient has been diagnosed 173. A tooth extraction in a patient
with acute radiation disease. Laboratory with chronic persistent hepatitis was
examination revealed a considerable complicated with prolonged hemorrhage.
reduction of platelet serotonin level. The What is the reason for the haemorrhagic
likely cause of platelet serotonin reducti- syndrome?
on is the disturbed metabolism of the
following substance: A. Decrease in thrombin production
B. Increase in thromboplastin production
A. 5-oxytryptofane C. Decrease in fibrin production
B. Tyrosine D. Increase in fibrinogen synthesis
C. Histidine E. Fibrinolysis intensification
D. Phenylalanine
E. Serine 174. Sex chromosomes of a woman didn’t
separate and move to the opposite poles
169. In course of a conditional experiment of a cell during gametogenesis (meiosis).
the development of mesenchyma cells The ovum was impregnated with a normal
was completely inhibited. Development spermatozoon. Which chromosomal di-
of the following muscular tissue will be di- sease can be found in her child?
sturbed:
A. Turner’s syndrome
A. Smooth muscular tissue B. Down’s syndrome
B. Neural muscular tissue C. Patau’s syndrome
C. Epidermal muscular tissue D. Edwards’ syndrome
D. Cardiac muscular tissue E. Cat cry syndrome
E. Skeletal muscular tissue
175. A female patient was administered
170. A histologic specimen shows an loratadine for allergic cheilitis treatment.
organ’s parenchyma which is presented What is the mechanism of the drug’s acti-
Krok 1 Medicine 2011 20

on? 180. A patient consulted a doctor about


loss of taste sensitivity on the tongue root.
A. It blocks the activity of H1 histamine The doctor revealed that it is caused by
receptors nerve affection. Which nerve is it?
B. It blocks the adrenergic receptors
C. It stimulates the activity of monoamine A. Glossopharyngeal
oxidase B. Vagus nerve
D. It inhibits the activity of Na,K-ATP C. Facial nerve
E. It inhibits the activity of choline esterase D. Superlaryngeal nerve
E. Trigeminal nerve
176. Ultramicroscopical examination of
"dark"hepatocyte population in the cell 181. Which muscle contraction will be
cytoplasm detected a developed granular observed in the upper extremity duri-
endoplasmic reticulum. What function has ng holding (but not moving) a load in a
this organella in these cells? certain position?
A. Synthesis of blood plasma proteins A. Isometric
B. Carbohydrate synthesis B. Isotonic
C. Deintoxicative function C. Auxotonic
D. Bile production D. Concentric
E. Calcium ion depositing E. Excentric
177. A doctor prescribed a cephalosporin 182. An older woman has been hospitali-
antibiotic to the patient after appendectomy sed for acute pain and edema of the right
for infection prevention. Antimicrobial hip joint that appered after a fall. Objecti-
activity of this group of antibiotics is based vely: the hip is adduced inwards, hip joi-
upon the disturbance of the following nt movements are impaired. The patient
process: is most likely to have a fracture of the
following bone or bone part:
A. Microbial wall formation
B. Nucleic acid synthesis A. Femoral neck
C. Ribosome protein synthesis B. Shaft of femur
D. Energy metabolism C. Condyle of femur
E. Choline esterase block D. Pubic bone
E. Ischial bone
178. A patient underwent an extraction of
a part of a CNS structures by medical indi- 183. A 45-year-old woman has breast
cations. As a result of the extraction the cancer. Her left arm has symptoms of
patient developed atony, astasia, intenti- lymphatic system insufficiency - limb
on tremor, ataxy and adiadochokinesis. edema, lymph node enlargement. What
Which part of CNS structure had been form of lymphatic circulation insufficiency
extracted? is it?
A. Cerebellum A. Mechanic insufficiency
B. Amygdaloid corpus B. Dynamic insufficiency
C. Hippocamp C. Resorption insufficiency
D. Basal ganglions D. Combined insufficiency
E. Limbic system E. -
179. A patient has been diagnosed with 184. Study of conversion of a food colouri-
influenza. His condition became drasti- ng agent revealed that neutralization of
cally worse after taking antipyretic drugs. this xenobiotic takes place only in one
His consciousness is confused, AP is 80/50 phase - microsomal oxydation. Name a
mm Hg, Ps is 140/m, body temperature component of this phase:
droped down to 35, 8o C. What complicati-
on developed in this patient? A. Cytochrome Р-450
B. Cytochrome B
A. Collapse C. Cytochrome C
B. Hyperthermia D. Cytochrome A
C. Hypovolemia E. Cytochrome oxidase
D. Acidosis
E. Alkalosis 185. While performing an inguinal canal
operation on account of hernia a surgeon
Krok 1 Medicine 2011 21

damaged the canal’s contents. What for emergency care?


exactly was damaged?
A. Aminazine
A. Funiculus spermaticus B. Diazepam
B. Urarchus C. Sodium bromide
C. Lig. teres uteri D. Reserpine
D. Lig. inguinalе E. Halothane
E. -
191. A patient who has been treated in
186. A patient with myocardial infarcti- a neural clinic and has been taking a
on was admitted to the cardiological sedative for a long time got the followi-
department. For pain relief it was deci- ng complication: cough, rhinitis, epiphora.
ded to potentiate fentanyl action wi- What drug caused these disturbances?
th a neuroleptic. Which of the followi-
ng neuroleptics is the most suitable for A. Sodium bromide
neuroleptanalgesia? B. Diazepam
C. Valerian
A. Droperidol D. Phenazepam
B. Aminazine E. Reserpine
C. Triftazine
D. Haloperidol 192. The liver puncture biopsy of a patient
E. Sulpiride with hepatocellular insufficiency revealed
hydropic and ballooning degenerationof
187. A 25-year-old man has spent a long ti- hepatocytes, necrosis of certain cells,
me in the sun under high air humidity. As presence of Kaunsilmen’s bodies. Portal
a result of it his body temperature rose up and lobular stroma were infiltrated mostly
to 39oC. What pathological process is it? with lymphocytes and macrophages
as well as with a small number of
A. Hyperthermia polymorphonuclear lymphocytes. What is
B. Infectious fever the most likely diagnosis?
C. Hypothermia
D. Noninfectious fever A. Acute viral hepatitis
E. Burn disease B. Chronic persistent hepatitis
C. Chronic aggressive hepatitis
188. A 26-year-old man is in the torpid D. Autoimmune hepatitis
shock phase as a result of a car accident. E. Alcoholic hepatitis
In blood: 3, 2 · 109 /l. What is the leading
mechanism of leukopenia development? 193. In order to accelerate healing of a
radiation ulcer a vitamin drug was admi-
A. Redistribution of leukocytes in nistered. What drug is it?
bloodstream
B. Leikopoiesis inhibition A. Retinol acetate
C. Disturbed going out of mature B. Retabolil
leukocytes from the marrow into the C. Prednisolone
blood D. Levamisole
D. Lysis of leukocytes in the blood-forming E. Methyluracil
organs
E. Intensified elimination of leukocytes 194. After inoculation of the material
from the organism obtained from the pharynx of an angina
patient onto the blood-tellurite agar, grey
189. A patient consulted a doctor about a colonies could be observed. They were
sensation of imbalance which appeared 4-5 mm in diameter, radially striated (in
after a trauma. Which nerve is damaged? form of rosettes). Microscopical exami-
nation revealed gram-positive bacilli wi-
A. Vestibulocochlear nerve th clavate swollen ends arranged in form
B. Trigeminal nerve of wide-spread fingers. Identify these mi-
C. Facial nerve croorganisms:
D. Intermediate nerve
E. Vagus nerve
190. An alcoholic has alcoholic psychosis
with evident psychomotor agitation. What
neuroleptic drug should be administered
Krok 1 Medicine 2011 22

A. Diphtheria corynebacteria 198. An injured person was delivered to


B. Clostridium botulinum the hospital with a penetrating wound in
C. Diphtheroids the left lateral region of abdomen. What
D. Streptococci part of the large intestine is most likely
E. Streptobacilli damaged?
195. Cytogenetic examination of a patient A. Colon descendens
with reproductive dysfunction revealed B. Colon ascendens
normal karyotype 46 ХY in some cells, but C. Colon transverses
most cells have karyotype of Klinefelter’s D. Caecum
syndrome - 47 ХХY. Such cell heterogeni- E. Rectum
ty is called:
199. A 4-year-old child presents with
A. Mosaicism general weakness, sore throat and degluti-
B. Inversion tive problem. After his examination a
C. Transposition doctor suspected diphtheria and sent the
D. Duplication material to the bacteriological laboratory.
E. Monomorphism In order to determine the diphtheria
causative agent the material should be
196. A patient presents with acne and inoculated into the following differenti-
inflammatory alterations of facial skin. al diagnostic medium:
Microscopial investigation of lesion foci
has revealed live arthropods sized 0,2- A. Blood tellurite agar
0,5 mm. They have prolate vermiform B. Endo’s agar
form and four pairs of thin short limbs C. Ploskyrev’s agar
located in the middle part of the body. D. Sabouraud’s agar
The revealed arthropods cause: E. Levenshtein-Yessen agar
A. Demodicosis 200. Examination of the anterior abdomi-
B. Scabies nal wall of a pregnant woman revealed
C. Pediculosis a tumour-like formation that arose on
D. Phthiriasis the spot of a tumour that was removed
E. Dermamyiasis two years ago. The neoplasm was well-
defined, dense, 2х1 cm large. Histological
197. Surgical approach to the thyroid examination revealed that the tumour was
gland from the transverse (collar) composed of differentiated connective ti-
approach involves opening of interaponeurotic ssue with prevailing collagen fibres. What
suprasternal space. What anatomic tumour might be suspected?
structure localized in this space is
dangerous to be damaged? A. Desmoid
B. Lipoma
A. Jugular venous arch C. Fibrosarcoma
B. External jugular vein D. Hibernoma
C. Subclavicular vein E. Leiomyoma
D. Inferior thyroid arthery
E. Superior thyroid arthery
Krok 1 Medicine 2012 1

1. A man suffering from a hereditary 6. A patient has a right-sided fracture in


disease married a healthy woman. They the region of the frontal third of mandible
got 5 children, three girls and two boys. accompanied by a haematoma in the regi-
All the girls inherited their father’s di- on of chin. It is caused by the injury of the
sease. What is the type of the disease following artery:
inheritance?
A. Mental
A. Dominant, X-linked B. Inferior labial
B. Autosomal recessive C. Lingual
C. Asutosomal dominant D. Facial
D. Y -linked E. Palatine
E. Recessive, X-linked
7. A 46 year old female patient has a conti-
2. Characteristic sign of glycogenosis nuous history of progressive muscular
is muscle pain during physical work. (Duchenne’s) dystrophy. Which blood
Blood examination reveals usually enzyme changes will be of diagnostic
hypoglycemia. This pathology is caused value in this case?
by congenital deficiency of the following
enzyme: A. Creatine phosphokinase
B. Lactate dehydrogenase
A. Glycogen phosphorylase C. Pyruvate dehydrogenase
B. Glucose 6-phosphate dehydrogenase D. Glutamate dehydrogenase
C. Alpha amylase E. Adenylate cyclase
D. Gamma amylase
E. Lysosomal glycosidase 8. As a result of a cold a patient has the
abnormal pain and temperature sensitivi-
3. During an operation a patient got ty of the frontal 2/3 of his tongue. Which
injection of muscle relaxant dithylinum. nerve must have been damaged?
Relaxation of skeletal muscles and inhibi-
tion of respiration lasted two hours. This A. Trigeminus
condition was caused by absence of the B. Sublingual
following enzyme in blood serum: C. Accessory
D. Vagus
A. Butyrylcholin esterase E. Glossopharyngeal
B. Catalase
C. Acetylcholinesterase 9. A 60 year old patient has impaired
D. Glucose 6-phosphatase perception of high-frequency sounds.
E. Glutathione peroxidase These changes were caused by damage of
the following auditory analyzer structures:
4. A couple has a son with haemophilia.
The parents are healthy but the maternal A. Main cochlea membrane near the oval
grandfather also has haemophilia. Specify window
the type of inheritance: B. Main cochlea membrane near the
helicotrema
A. Recessive sex-linked C. Eustachian tube
B. Recessive autosomal D. Middle ear muscles
C. Dominant sex-linked E. Tympanic membrane
D. Semidominance
E. Autosomal dominant 10. After a trauma of soft tissues in the
region of the posterior sorface of medi-
5. Examination of newborns in one of al condyle of humerus a patient has got
the Ukrainian cities revealed a baby wi- a skin prickle of medial forearm surface.
th phenylketonuria. The baby’s parents Which of the listed nerves is located in the
don’t suffer from this disease and have affected region?
two other healthy children. Specify
the most likely parents’ genotype with A. N.ulnaris
phenylketonuria gene: B. N.musculocutaneu
C. N.dorsalis scapularis
A. Аа х Аа D. N.subscapularis
B. АА х аа E. N.radialis
C. аа х аа
D. Аа х аа 11. A boy has fallen down from a tree.
E. Аа х АА Now he finds it difficult to abduct his arm
Krok 1 Medicine 2012 2

into horizontal position. Which muscle is membrane of the bronchus bioptate of


most likely to be injured? the patient with smoke abuse. Which of
the processes is the most likely?
A. M.deltoideus
B. M.triceps brachii A. Metaplasia
C. M.anconeus B. Hyperplasia of the epithelium
D. M.coracobrachialis C. Squamous cancer
E. M.supinator D. Leucoplacia
E. Epithelium hypertrophy
12. A female patient with a tumour of
pancreas has developed mechanic jaundi- 17. During postembryonal haemopoiesis
ce resulting from compression of a bile- in the red bone marrow the cells of one
excreting duct. Which duct is compressed? of the cellular differons demonstrate a
gradual decrease in cytoplasmic basophi-
A. Ductus choledochus lia as well as an increase in oxyphilia,
B. Ductus cysticus the nucleus is being forced out. Such
C. Ductus hepaticus communis morphological changes are typical for the
D. Ductus hepaticus dexter following haemopoiesis type:
E. Ductus hepaticus sinister
A. Erythropoiesis
13. A 28 year old woman has been di- B. Lymphopoiesis
agnosed with extrauterine pregnancy C. Neutrophil cytopoiesis
complicated by tha fallopian tube rupture. D. Eosinophil cytopoiesis
The blood is most likely to penetrate the E. Basophil cytopoiesis
following peritoneal space:
18. Students who are taking examinations
A. Rectouterine often have dry mouth. The mechanism
B. Vesicouterine that causes this state is the realization of
C. Right mesenteric sinus the following reflexes:
D. Left mesenteric sinus
E. Intersigmoid sinus A. Conditioned sympathetic
B. Unconditioned parasympathetic
14. Patients with similar complaints appli- C. Conditioned parasympathetic
ed to the doctor: weakness, pain in the D. Unconditioned sympathetic
intestines, disorder of GIT. Examination E. Unconditioned peripheral
of the faeces revealed that one patient wi-
th four nucleus cysts should be hospitali- 19. The temperature of the ambient envi-
zed immidiately. For what protozoa are ronment is 38o C and relative air humidity
such cysts typical? is 50%. What ways of heat emission provi-
de maintaining a constant temperature of
A. Dysenteric amoeba the human body?
B. Intestinal amoeba
C. Balantidium A. Evaporation
D. Trichomonas B. Radiation
E. Lamblia C. Heat conduction
D. Convection
15. Autopsy of a 58 year old man revealed E. Convection and conduction
that bicuspid valve was deformed, thi-
ckened and unclosed. Microscopically: 20. A microspecimen of heart shows
foci of collagen fibrilla are eosinophi- rectangular cells from 50 to 120 mcm
lic, react positively to fibrin. The most large with central position of nucleus,
probably it is: developed myofibrils. The cells are
connected by intercalated discs. These
A. Fibrinoid swelling cells are responsible for the following
B. Fibrinous inflammation function:
C. Mucoid swelling
D. Hyalinosis A. Function of heart contractions
E. Amyloidosis B. Function of impulse conduction
C. Endocrine
16. Chronic inflammation and transformati- D. Protective
on of the one-layer ciliated epitheli- E. Regeneratory
um into multiple-layers flat epithelium
was revealed in the thickened mucous 21. A 30 year old woman has subnormal
Krok 1 Medicine 2012 3

concentration of enzymes in the


pancreatic juice. This might be caused by A. Lymphogranulomatosis
the hyposecretion of the following gastroi- B. Acute lympholeucosis
ntestinal hormone: C. Chronic lympholeucosis
D. Berkitt’s lymphoma
A. Cholecystokinin-pancreozymin E. Fungoid mycosis
B. Somatostatin
C. Secretin 26. A child has abnormal formation of
D. Gastro-inhibiting peptide tooth enamel and dentin as a result of low
E. Vaso-intestinal peptide concentration of calcium ions in blood.
Such abnormalities might be caused by
22. A female patient consulted a deficiency of the following hormone:
physician about digestive disorder,
extended abdominal pain. Examination A. Parathormone
revealed drastic decrease in hemoglobin B. Thyrocalcitonin
concentration. It is known from the C. Thyroxin
anamnesis that while living in the Far East D. Somatotropic hormone
the patient used to eat freshly-salted cavi- E. Triiodothyronine
ar. Some relatives living with her had the
similar condition. What is the most likely 27. A sportsman was examined after an
diagnosis? intensive physical activity. The examinati-
on revealed disorder of movement coordi-
A. Diphyllobothriasis nation but the force of muscle contracti-
B. Echinococcosis ons remained the same. It can be explai-
C. Teniasis ned by retarded speed of excitement
D. Trichiniasis conduction through:
E. Ascaridiasis
A. Central synapses
23. A patient has a trauma of sternoclei- B. Neuromuscular synapses
domastoid muscle. This caused a decrease C. Efferent nerves
in value of the following indicator of D. Afferent nerves
external respiration: E. Conduction tracts
A. Inspiratory reserve volume 28. After a long training session
B. Expiratory reserve volume a sportsman has developed fatigue
C. Respiratory capacity accompanied by abrupt performance
D. Residual volume decrement. What link of the reflex arch
E. Functional residual lung capacity was the fatigue initiated in?
24. A month after surgical constriction A. Nerve centres
of rabbit’s renal artery the considerable B. Afferent conductor
increase of systematic arterial pressure C. Receptors
was observed. What of the following D. Efferent conductor
regulation mechanisms caused the ani- E. Muscles
mal’s pressure change?
29. Blood minute volume of a 30 year old
A. Angiotensin-II woman at rest is 5 l/m. What blood volume
B. Vasopressin is pumped through the pulmonary vessels
C. Adrenaline per minute?
D. Noradrenaline
E. Serotonin A. 5 l
B. 3,75 l
25. Microscopic examination of the C. 2,5 l
enlarged neck gland of a 14 year old D. 2,0 l
girl revealed destruction of the ti- E. 1,5 l
ssue structure of the node, absence of
the lymph follicles, sclerotic areas and 30. After a tourniquet application a
necrosis foci, cell constitution of the node patient was found to have petechial
is polymorphous, lymphocites, eosinophi- haemorrhages. The reason for it is the
les, big atypical cells with multilobular dysfunction of the following cells:
nuclei (Beresovsky-Sternberg cells) and
mononuclear cells of the large size are
present. What is the most likely diagnosis?
Krok 1 Medicine 2012 4

A. Platelets A. Choline
B. Eosinophils B. DOPA
C. Monocytes C. Cholesterol
D. Lymphocytes D. Acetoacetate
E. Neutrophils E. Linoleic acid
31. A patient under test was subjected to 36. Vegetative abnormalities in the sleep,
a moderate physical stress. His minute heat regulation, all kinds of metabolism,
blood volume amounted 10 l/min. What diabetes insipidus are developing in the
blood volume was pumped through his patient due to grouth of the tumour in
lung vessels every minute? the III ventricle of brain. Irritation of the
nucleus of what part of the brain can cause
A. 10 l/min this symptoms?
B. 5 l/min
C. 4 l/min A. Hypothalamus
D. 6 l/min B. Cerebral peduncles (cruces cerebri)
E. 7 l/min C. Mesencephalic tegmentum
D. Pons cerebelli
32. A patient presents with the following E. Medulla
motor activity disturbances: tremor, ataxia
and asynergia movements, dysarthria. The 37. A patient complains about dyspnea
disturbances are most likely to be locali- provoked by the physical activity. Clini-
zed in: cal examination revealed anaemia and
presence of the paraprotein in the zone
A. Cerebellum of gamma-globulins. To confirm the
B. Basal ganglions myeloma diagnosis it is necessary to
C. Limbic system determine the following index in the pati-
D. Brainstem ent’s urine:
E. Medulla oblongata
A. Bence Jones protein
33. A man has a considerable decrease B. Bilirubin
in diuresis as a result of 1,5 l blood loss. C. Haemoglobin
The primary cause of such diuresis di- D. Ceruloplasmin
sorder is the hypersecretion of the followi- E. Antitrypsin
ng hormone:
38. Examination of a child who hasn’t
A. Vasopressin got fresh fruit and vegetables during wi-
B. Corticotropin nter revealed numerous subcutaneous
C. Natriuretic hemorrhages, gingivitis, carious cavities in
D. Cortisol teeth. What vitamin combination should
E. Parathormone be prescribed in this case?
34. The patient with thymoma (thymus A. Ascorbic acid and rutin
gland tumour) has cyanosis, extention of B. Thiamine and pyridoxine
subcutaneous venous net and edema of C. Folic acid and cobalamin
the soft tissues of face, neck, upper part D. Riboflavin and nicotinamide
of the trunk and upper extremities. What E. Calciferol and ascorbic acid
venous trunk is pressed with tumour?
39. A 2 year old child with mental
A. Superior vena cava and physical retardation has been deli-
B. External jugular vein vered to a hospital. He presents with
C. Clavicular vein frequent vomiting after having meals.
D. Internal jugular vein There is phenylpyruvic acid in urine. Whi-
E. Frontal jugular vein ch metabolism abnormality is the reason
for this pathology?
35. An experimantal animal that was kept
on protein-free diet developed fatty li- A. Amino-acid metabolism
ver infiltration, in particular as a result B. Lipidic metabolism
of deficiency of methylating agents. This C. Carbohydrate metabolism
is caused by disturbed generation of the D. Water-salt metabolism
following metabolite: E. Phosphoric calcium metabolism
40. 46 chromosomes were revealed on
Krok 1 Medicine 2012 5

karyotype examination of the 5 year old hemorrhage. What substance content has
girl. One of the 15th pair of chromosomes decreased in blood?
is longer than usual due to connected
chromosome from the 21 pair. What type A. Prothrombin
of mutation does this girl have? B. Haptoglobin
C. Heparin
A. Translocation D. Creatin
B. Deletion E. Ceruloplasmin
C. Inversion
D. Insufficiency 46. After severe viral hepatitis a 4 year
E. Duplication old boy presents with vommiting, occasi-
onal loss of consciousness, convulsions.
41. A 62 year old woman complai- Blood test revealed hyperammoniemia.
ns of frequent pain attacks in the Such condition is caused by a disorder of
area of her chest and backbone, rib the following biochemical hepatic process:
fractures. Her doctor suspected myeloma
(plasmocytoma). What of the following A. Disorder of ammonia neutralization
laboratory characteristics will be of the B. Disorder of biogenic amines neutrali-
greatest diagnostic importance? zation
C. Protein synthesis inhibition
A. Paraproteinemia D. Activation of amino acid decarboxylati-
B. Hyperalbuminemia on
C. Proteinuria E. Inhibition of transamination enzymes
D. Hypoglobulinemia
E. Hypoproteinemia 47. Having helped to eliminate
consequences of a failure at a nuclear
42. Before the cells can utilize the glucoze, power plant, a worker got an irradiati-
it is first transported from the extracellular on doze of 500 roentgen. He complai-
space through the plasmatic membrane ns of headache, nausea, dizziness. What
inside theml. This process is stimulated by changes in leukocytes quantity can be
the following hormone: expected 10 hours after irradiation?
A. Insulin A. Neutrophilic leukocytosis
B. Glucagon B. Lymphocytosis
C. Thyroxin C. Leukopenia
D. Aldosterone D. Agranulocytosis
E. Adrenalin E. Leukemia
43. Parodontitis is treated with calcium 48. Bacterioscopic examination of a smear
preparations and a hormone that sti- from the pharynx of a diphtheria suspect
mulates tooth mineralization and inhibits revealed bacilli with volutine granules.
tissue resorption. What hormone is it? What etiotropic drug should be chosen
in this case?
A. Calcitonin
B. Parathormone A. Antidiphtheritic antitoxic serum
C. Adrenalin B. Bacteriophage
D. Aldosterone C. Diphtheritic anatoxin
E. Thyroxine D. Eubiotic
E. Interferon
44. A child has an acute renal failure.
What biochemical factor found in saliva 49. A patient with a history of
can confirm this diagnosis? chronic glomerulonephritis presents
with azotemia, oliguria, hypo- and
A. Increase in urea concentration isosthenuria, proteinuria. What is the
B. Increase in glucose concentration leading factor in the pathogenesis of these
C. Decrease in glucose concentration symptoms development under chronic
D. Increase in concentration of higher fatty renal failure?
acids
E. Decrease in nucleic acid concentration
45. After implantation of a cardiac valve a
young man constantly takes indirect anti-
coagulants. His state was complicated by
Krok 1 Medicine 2012 6

A. Mass decrease of active nephrons A. Acute left-ventricular failure


B. Intensification of glomerular filtration B. Chronic right-ventricular failure
C. Tubular hyposecretion C. Chronic left-ventricular failure
D. Disturbed permeability of glomerular D. Collapse
membranes E. Cardiac tamponade
E. Intensification of sodium reabsorption
55. A 5 year old child has been diagnosed
50. After an attack of bronchial asthma a with acute right distal pneumonia.
patient had his peripheral blood tested. Sputum inoculation revealed that the
What changes can be expected? causative agent is resistant to penicillin,
but it is senstive to macrolides. What drug
A. Eosinophilia should be prescribed?
B. Leukopenia
C. Lymphocytosis A. Azithromycin
D. Thrombocytopenia B. Tetracycline
E. Erythrocytosis C. Gentamycin
D. Streptomycin
51. A 70 year old female patient was di- E. Ampicillin
agnosed with fracture of left femoral neck
accompanied by disruption of ligament of 56. A patient suffering from chronic
head of femur. The branch of the followi- myeloleukemia has got the followi-
ng artery is damaged: ng symptoms of anemia: decreased
number of erythrocytes and low
A. Obturator haemoglobin concentration, oxyphilic
B. Femoral and polychromatophilic normocytes, mi-
C. External iliac crocytes. What is the leading pathogenetic
D. Inferior gluteal mechanism of anemia development?
E. Internal pudendal
A. Substitution of haemoblast
52. ECG of a patient shows prolongation B. Intravascular hemolysis of erythrocytes
of T-wave. This is caused by deceleration C. Deficiency of vitamin B12
in ventricles of: D. Reduced synthesis of erythropoietin
E. Chronic haemorrhage
A. Repolarization
B. Depolarization and repolarization 57. A patient with obliterating endarteri-
C. Depolarization tis underwent ganglionic sympathectomy.
D. Contraction What type of arterial hyperaemia should
E. Relaxation have developed as a result of the surgery?
53. After the traumatic tooth extraction A. Neuroparalytic
a patient is complaining of acute, dull, B. Neurotonic
poorly-localized pain in gingiva, body C. Metabolic
temperature rise up to 37, 5oC. The pati- D. Functional
ent has been diagnosed with alveolitis. E. Reactive
Specify the kind of pain in this patient:
58. Autopsy of a woman with cerebral
A. Protopathic atherosclerosis revealed in the left
B. Epicritic cerebral hemisphere a certain focus that
C. Visceral is presented by flabby, anhistic, greyish
D. Heterotopic and yellowish tissue with indistinct edges.
E. Phantom What pathological process is the case?
54. A 50 year old patient suffers from A. Ischemic stroke
essential hypertension. After a physical B. Multifocal tumor growth with cystic
stress he experienced muscle weakness, degeneration
breathlessness, cyanosis of lips, skin and C. Multiple foci of fresh and old cerebral
face. Respiration was accompanied by hemorrhage
distinctly heard bubbling rales. What D. Focal encephalitis
mechanism underlies the development of E. Senile encephalopathy
this syndrome?
59. A patient suffering from chronic
hyperacidic gastritis takes an antacid
drug for heartburn elimination. After its
Krok 1 Medicine 2012 7

ingestion the patient feels better but at the A. Necrotic nephrosis


same time he has a sensation of stomach B. Acute pyelonephritis
swelling. Which of the following drugs mi- C. Acute glomerulonephritis
ght be the cause of such side effect? D. Acute tubular-interstitial nephritis
E. Lipoid nephrosis
A. Sodium hydrocarbonate
B. Magnesium oxide 64. A 44 year old woman complains of
C. Magnesium trisilicate general weakness, heart pain, significant
D. Aluminium hydrooxide increase of body weight. Objectively:
E. Pepsin moon face, hirsutism, AP is 165/100 mm
Hg, height - 164 cm, weight - 103 kg; the
60. A 63 year old patient with collapse fat is mostly accumulated on her neck,
presentations was delivered to the thoracic girdle, belly. What is the main
emergency hospital. A physician has pathogenetic mechanism of obesity?
chosen noradrenalin against hypotension.
What is its mechanism of action? A. Increased production of glucocorticoids
B. Reduced production of thyroid
A. Activation of α1 -adrenoreceptors hormones
B. Activation of serotonin receptors C. Increased insulin production
C. Activation of β-adrenoreceptors D. Reduced glucagon production
D. Activation of dopamine receptors E. Increased mineralocorticoid production
E. Block of M-cholinoreceptors
65. A man with a long-term history of
61. Autopsy of a man who had bronchial asthma died from asphyxia.
tuberculosis revealed a 3x2 cm large cavi- Histological examination of his lungs
ty in the superior lobe of the right lung. revealed that the lumens of bronchioles
The cavity was interconnected with a and minor bronchi contained a lot of
bronchus, its wall was dense and consi- mucus with some eosinophils. There was
sted of three layers: the internal layer also sclerosis of interalveolar septa, di-
was pyogenic, the middle layer was made latation of alveole lumens. What mechani-
by tuberculous granulation tissue and the sm accounts for the development of
external one was made by connective ti- hypersensitivity reaction?
ssue. What is the most likely diagnosis?
A. Reagine reaction
A. Fibrous cavernous tuberculosis B. Cytotoxic reaction
B. Fibrous focal tuberculosis C. Immune complex reaction
C. Tuberculoma D. Lymphocyte-mediated cytolysis
D. Acute focal tuberculosis E. Granulomatosis
E. Acute cavernous tuberculosis
66. A 36 year old female patient has a hi-
62. A patient suffering from syphilis has story of collagen disease. Urine analysis is
been treated with bismuth preparations. likely to reveal an increased concentrati-
As a result of it some grey spots turned on of the following metabolite:
up on the mucous membrane of the oral
cavity; nephropathy symptoms were also A. Oxyproline
present. What drug should be used for B. Indican
treatment of bismuth intoxication? C. Creatinine
D. Urea
A. Unithiol E. Urobilinogen
B. Methylene blue
C. Naloxone 67. Bacterioscopy of nasopharyngeal
D. Bemegride mucus taken from a 2,5 year old chi-
E. Nalorphine ld with nasopharyngitis revealed gram-
positive diplococci looking like coffee
63. Autopsy of a man who died from grains. What organs of the child are most
ethylene glycol poisoning revealed that likely to be affected if these microorgani-
his kidneys are a little bit enlarged, sms penetrate the blood?
edematic; their capsule can be easily
removed. Cortical substance is broad and
light-grey. Medullary substance is dark-
red. What pathology had this man?
Krok 1 Medicine 2012 8

A. Brain tunics A. CD 4
B. Cardiac valves B. CD 3
C. Renal glomeruli C. CD 8
D. Urogenital tracts D. CD 19
E. Lymph nodes E. CD 28
68. Histologic analysis of uterus mucous 73. A 65 year old patient with chronic
membrane revealed twisting glands, heart failure has been taking digitoxin in
serrated and spinned, they were extended self-administered dosages for a long ti-
by stroma growth with proliferation of its me. She was admitted to the hospital for
cells. Formulate a diagnosis: general health aggravation, arrhythmia,
nausea, reduced diuresis, insomnia. What
A. Glandular hyperplasia of endometrium is the primary action to be taken?
B. Acute endometritis
C. Leiomyoma A. To withhold digitoxin
D. Cystic mole B. To reduce digitoxin dosage
E. Placental polyp C. To administer strophanthine
intravenously
69. Analysis of sputum taken from a pati- D. To administer digoxin
ent with suspected pneumonia revealed E. To give an intravenous injection of
rather elongated gram-positive diplococci calcium gluconate solution
with somewhat pointed opposite ends.
What microorganisms were revealed in 74. A 20 year old patient complains of
the sputum? morbid thirst and huperdiuresis (up to 10
l daily). Glucose concentration in blood is
A. Streptococcus pneumoniae normal but it is absent in urine. The pati-
B. Staphylococcus aureus ent has been diagnosed with diabetes insi-
C. Klebsiella pneumoniae pidus. What hormonal drug is the most
D. Neisseria meningitidis appropriate for management of this di-
E. Neisseria gonorrhoeae sorder?
70. Serological diagnostics of infectious A. Vasopressin
diseases is based upon specific interacti- B. Cortisol
on with antigenes. Specify the serologi- C. Thyroxin
cal reaction that underlies adhesion of D. Oxytocin
microorganisms when they are affected E. Insulin
by specific antibodies in presence of an
electrolyte: 75. A patient has myocardial infarcti-
on with thrombosis of the left coronary
A. Agglutination reaction artery. What pharmocological preparation
B. Precipitation reaction group should be used to reestablish blood
C. Complement-binding reaction flow?
D. Hemadsorption reaction
E. Neutralization reaction A. Fibrinolysis activators
B. Narcotic analgesics
71. The immunoblot detected gp120 C. β-adrenergic blockers
protein in the blood serum. This protein is D. Angiotensin-converting enzyme inhibi-
typical for the following disease: tors
A. HIV-infection E. Glucocorticoids
B. Virus B hepatitis 76. A nurse accidentally injected a nearly
C. Tuberculosis double dose of insulin to a patient with di-
D. Syphilis abetes mellitus. The patient lapsed into a
E. Poliomyelitis hypoglycemic coma. What drug should be
72. HIV has gp41 and gp120 on its injected in order to help him out of coma?
surface interacts with target cells of A. Glucose
an organism. Which of the following B. Lidase
human lymphocyte antigens is gp120 C. Insulin
complementary bound with? D. Somatotropin
E. Noradrenaline
77. A patient has a slowly healing fracture.
Krok 1 Medicine 2012 9

What medicine can be used to accelerate A. Paschen bodies


formation of connective tissue matrix? B. Babes-Negri bodies
C. Guarnieri bodies
A. Methyluracil D. Babes-Ernst bodies
B. Prednisolone E. Syncytium
C. Cyclophosphan
D. Methotrexate 82. Vitamin A together with specific
E. Cyclosporine cytoreceptors penetrates through the
nuclear membranes, induces transcripti-
78. From the group of children who were on processes that stimulate growth and
eating sweet sappy watermelon two kids differentiation of cells. This biological
developed the signs of poisoning: rapid function is realized by the following form
weakness, dizziness, headache, vomiting, of vitamin A:
edema, tachycardia, cyanosis of mouth,
ears, tips of the fingers cyanosis. High A. Trans-retinoic acid
concentration of nitrates was detected. B. Trans-retinal
What is the leading mechanism of the C. Cis-retinal
pathogenesis of the poisoning in the two D. Retinol
children? E. Carotin
A. Insufficiency of met-Hb-reductase 83. In order to determine toxigenicity of
B. Insufficiency of superoxiddismutase diphtheria bacilli a strip of filter paper
C. Block cytochrome oxidase impregnated with antitoxic diphtherial
D. Insufficiency glutathione pyroxidase serum was put on the dense nutrient medi-
E. Insufficiency of catalase um. There were also inoculated a mi-
crobal culture under examination and a
79. As a result of a trauma a patient has strain that is known to be toxigenic. If
damaged anterior roots of spinal cord. the microbal culture under examinati-
What structures have been affected? on produces exotoxin, this wil result in
formation of:
A. Axons of motoneurons and axons of
neurons of lateral horns A. Precipitin lines
B. Central processes of sensitive neurons B. Haemolysis zones
of spinal ganglions C. Zones of diffuse opacification
C. Peripheral processes of sensitive spinal D. Zones of lecithovitellinous activity
ganglions E. Precipitin ring
D. Axons of neurons of lateral horns
E. Dendrites of neurons of spinal ganglions 84. A 63 year old male patient who
had been suffering from chronic di-
80. During a prophylactic medical exami- ffuse obstructive disease, pulmonary
nation a 7-year-old boy was diagnosed emphysema, for 15 years died from cardi-
with daltonism. His parents are healthy ac insufficiency. Autopsy revealed nutmeg
and have normal colour vision, but his liver cirrhosis, cyanotic induration of ki-
grandfather on his mother’s side has the dneys and spleen, ascites, edemata of
same abnormality. What is the type of the lower limbs. These changes of internal
abnormality inheritance? organs are typical for the following di-
sease:
A. Recessive, sex-linked
B. Dominant, sex-linked A. Chronic right-ventricular insufficiency
C. Semidominance B. Acute right-ventricular insufficiency
D. Autosomal recessive C. Chronic left-ventricular insufficiency
E. Autosomal dominant D. Acute left-ventricular insufficiency
E. General cardiac insufficiency
81. The contents of vesicles that appeared
on the mucous membrane of a patient 85. A man who is riding the carousel
with variola was sent to a virological presents with increased heart rate, sweati-
laboratory. Which of the listed changes ng, nausea. This condition is caused pri-
were revealed during the smear mi- marily by the stimulation of the following
croscopy? receptors:
Krok 1 Medicine 2012 10

A. Vestibular ampullar
B. Proprioceptors A. Natriuretic hormone
C. Vestibular otolithic B. Aldosterone
D. Auditory C. Renin
E. Visual D. Angiotensin
E. Vasopressin
86. A patient underwent esophagogastroduodenoscopy.
Analysis of the biopsy material enabled 91. After a serious psychoemotional stress
doctors to diagnose him with heli- a 48 year old patient suddenly developed
cobacteriosis. What property of the acute heart ache irradiating to the left
bacteria found in this patient had to be arm. Nitroglycerine relieved pain after 10
obligatory taken into account during their minutes. What is the leading pathogenetic
cultivation? mechanism of this process development?
A. Microaerophilic ability A. Spasm of coronary arteries
B. Presence of urease B. Dilatation of peripheral vessels
C. Colonisation of gastral cells C. Obstruction of coronary vessels
D. Absence of spores and capsules D. Compression of coronary vessels
E. Presence of six polar flagella E. Increase in myocardial oxygen
consumption
87. A 38 year old man died all of a sudden.
Autopsy revealed myocardial infarction 92. A 42 year old patient complains
in the posterior wall of the left ventri- of pain in the epigastral area, vomiti-
cle. What are the most likely alterations ng; vomit masses have the colour of
in myocardiocyte structure that can be "coffee-grounds", the patient has also
revealed microscopically in the infarcti- melena. Anamnesis records gastric ulcer.
on focus? Blood formula: erythrocytes - 2, 8 · 1012 /l,
leukocytes - 8 · 109 /l, Hb- 90 g/l. What
A. Karyolysis complication is it?
B. Adipose degeneration
C. Carbohydrate degeneration A. Haemorrhage
D. Calcification B. Penetration
E. Protein degeneration C. Perforation
D. Canceration
88. During cytoscopy mucous membrane E. Pyloric stenosis
of urinary bladder normally makes folds
except for a single triangular area with 93. A 56 year old patient suffering from
smooth mucosa. This triangle is located in cardiac insufficiency has edema of feet
the following part of urinary bladder: and shins, edematous skin is pale and cold.
What is the leding mechanism of edema
A. Bladder floor pathogenesis?
B. Bladder cervix
C. Bladder apex A. Rise of hydrostatic pressure in venules
D. Bladder body B. Drop of oncotic pessure in capillaries
E. Bladder isthmus C. Increase of capillary permeability
D. Disorder of lymph outflow
89. A child suffers from drug idiosyncrasy. E. Positive water balance
What is the cause of such reaction?
94. A disaster fighter at a nuclear power
A. Hereditary enzymopathy plant developed hemorrhagic syndrome
B. Exhaustion of substrate interacting with on the background of acute radiation di-
pharmaceutical substance sease. What is the most important factor
C. Accumulation of pharmaceutical of syndrome pathogenesis?
substance
D. Inhibition of microsomal liver enzymes A. Thrombocytopenia
E. Associated disease of target organ B. Vascular wall damage
C. Increased activity of fibrinolysis factors
90. Atria of an experimental animal were D. Increased activity of anticoagulative
superdistended by blood that resulted in system factors
decreased reabsorption of Na+ and water E. Decreased activity of coagulative factors
in renal tubules. This can be explained by
the influence of the following factor upon 95. To prevent postoperative bleeding a 6-
kidneys: year-old child was administered vicasol
Krok 1 Medicine 2012 11

that is a synthetic analogue of vitamin K. 100. Power inputs of a man were


Name post-translational changes of blood measured. In what state was this man if
coagulation factors that will be activated his power inputs were lower than basal
by vicasol: metabolism?
A. Carboxylation of glutamin acid A. Sleep
B. Phosphorylation of serine radicals B. Relaxation
C. Partial proteolysis C. Simple work
D. Polymerization D. Nervous tension
E. Glycosylation E. Rest
96. While under barbituric anaesthesia a 101. A man is being measured power
65 year old male patient developed respi- inputs on an empty stomach, in the lying
ratory inhibition. Anesthesiologist made position, under conditions of physical and
him a 10 ml intravenous injection of 0,5% psychic rest at a comfortable temperature.
bemegride solution. The patient’s condi- Power inputs will reach the maximum at:
tion got better, the pulmonary ventilati-
on volume increased. What phenomenon A. 5-6 p.m.
underlies the interaction of these medi- B. 7-8 a.m.
vcations? C. 10-12 a.m.
D. 2-3 p.m.
A. Direct antagonism E. 3-4 a.m.
B. Indirect antagonism
C. Unilateral antagonism 102. When measuring power inputs of
D. Direct synergism a man by the method of indirect calori-
E. Indirect synergism metry the following results were obtained:
1000 ml oxygen consumption and 800 ml
97. A surgeon has to find the common carbon dioxide liberation per minute. The
hepatic duct during the operative man under examination has the following
intervention on account of concrements in respiratory coefficient:
the gall ducts. The common hepatic duct
is located between the leaves of: A. 0,8
B. 1,25
A. Hepatoduodenal ligament C. 0,9
B. Hepatogastric ligament D. 0,84
C. Hepatorenal ligament E. 1,0
D. Round ligament of liver
E. Venous ligament 103. A patient complains about impaired
evacuatory function of stomach (long-
98. Blood analysis of a patient wi- term retention of food in stomach).
th jaundice reveals conjugated bilirubi- Examination revealed a tumour of initi-
nemia, increased concentration of bile aci- al part of duodenum. Specify localization
ds. There is no stercobilinogen in urine. of the tumour:
What type of jaundice is it?
A. Pars superior
A. Obstructive jaundice B. Pars inferior
B. Hepatocellular jaundice C. Pars descendens
C. Parenchymatous jaundice D. Pars ascendens
D. Hemolytic jaundice E. Flexura duodeni inferior
E. Cythemolytic jaundice
104. As a result of damage to certain
99. It is known that the gene responsible structures of brainstem an animal lost ori-
for development of blood groups accordi- entation reflexes. What structures were
ng to AB0 system has three allele vari- damaged?
ants. If a man has IV blood group, it can
be explained by the following variability A. Quadritubercular bodies
form: B. Medial nuclei of reticular formation
C. Red nuclei
A. Combinative D. Vestibular nuclei
B. Mutational E. Black substance
C. Phenotypic
D. Genocopy 105. Emotional stress causes activation
E. Phenocopy of hormon-sensitive triglyceride lipase in
Krok 1 Medicine 2012 12

the adipocytes. What secondary mediator toxoplasmosis. Which drug should be used
takes part in this process? for toxoplasmosis treatment?
A. Cyclic adenosine monophosphate A. Co-trimoxazole
B. Cyclic guanosine monophosphate B. Itraconazole
C. Adenosine monophosphate C. Mebendazole
D. Diacylglycerol D. Azidothimidine
E. Ions of 2+ E. Acyclovir
106. A patient has been diagnosed with 111. Autopsy of a 17 year old girl who
alkaptonuria. Choose an enzyme whose died from pulmonary failure revealed
deficiency can be the reason for this a small area of caseous necrosis in the
pathology: inferior lobe of the right lung, and
occurences of caseous necrosis in the
A. Homogentisic acid oxidase bronchopulmonary, bronchial and bi-
B. Phenylalanine hydroxylase furcational lymph nodes. What is the most
C. Glutamate dehydrogenase probable postmortem diagnosis?
D. Pyruvate dehydrogenase
E. Dioxyphenylalanine decarboxylase A. Primary tuberculosis
B. Hematogenous progression of primary
107. A 10 year old child had the mantoux tuberculosis
tuberculin test administered. 48 hours C. Hematogenous tuberculosis with
later a papule up to 8 mm in diameter predominant lung affection
appeared on the site of the injection. D. Tuberculoma
What type of hypersensitivity reaction E. Caseous pneumonia under secondary
developed after the tuberculin injection? tuberculosis
A. Type IV hypersensitivity reaction 112. After 4 months of treatment for
B. Arthus phenomenon tuberculosis the patient began complaini-
C. Seroreaction ng of toes and fingers numbness, sensati-
D. Atopic reaction on of creeps. He was diagnosed with
E. Type II hypersensitivity reaction polyneuritis. What antituberculous drug
might have caused these complications?
108. The patient with acute miocardi-
al infarction was given intravenously A. Isoniazid
different solutions during 8 hours B. Rifampicin
with medical dropper 1500 ml and C. Ciprofloxacin
oxygen intranasally. He died because D. Sodium salt of benzylpenicillin
of pulmonary edema. What caused the E. Alcohol iodine solution
pulmonary edema?
113. Examination of a 55 year old woman
A. Volume overload of the left ventricular revealed under the skin of submandi-
B. Decreased oncotic pressure due to bular area a movable slowly growing pasty
hemodilution formation with distinct borders 1,0x0,7 cm
C. Allergic reaction large. Histological examination revealed
D. Neurogenic reaction lipocytes that form segments of diffrent
E. Inhalation of the oxygen forms and sizes separated from each other
by thin layers of connective tissue with
109. In course of an operation surgeon vessels. What is the most probable di-
removed a part of a lung that was venti- agnosis?
lated by a tertiary bronchus accompani-
ed by branches of pulmonary artery and A. Lipoma
other vessels. What part of a lung was B. Fibroma
removed? C. Angioma
D. Liposarcoma
A. Bronchopulmonary segment E. Fibrosarcoma
B. Middle lobe
C. Inferior lobe 114. According to the phenotypic di-
D. Superior lobe agnosis a female patient has been provi-
E. Pulmonary lobule sionally diagnosed with X-chromosome
polysomia. This diagnosis can be confi-
110. After the second abortion a 23 year rmed by a cytogenetic method. What
old woman has been diagnosed with karyotype will allow to confirm the di-
Krok 1 Medicine 2012 13

agnosis?
A. Sodium valproate
A. 47(ХХХ) B. Ethosuxemide
B. 48(XXXY) C. Amitriptyline
C. 48(XXYY) D. Phenytoin
D. 47(XXY) E. Phenobarbital
E. 46(XX) 120. A patient diagnosed with morphini-
115. A patient has been diagnosed with sm has been admitted to the narcological
acute glomerulonephritis that developed department. A doctor noted a decrease
after he had had streptococcal infecti- in pharmacological activity of morphi-
on. It is most likely that the affection of ne. Repetitive use of a drug may result
basal glomerular membrane is caused by in tolerance to its effect, and this
an allergic reaction of the following type: phenomenon is called:

A. Immune complex A. Addiction


B. Anaphylactic B. Cumulation
C. Cytotoxic C. Tachyphylaxis
D. Delayed D. Antagonism
E. Stimulating E. Allergy

116. An elderly patient has chronic 121. As a result of durative antibiotic


constipations induced by large intestine therapy a 37 year old patient developed
hypotonia. What drug should be admini- intestinal dysbacteriosis. What type of
stered? drugs should be used in order to normali-
ze intestinal microflora?
A. Bisacodyl
B. Sodium sulphate A. Eubiotics
C. Castor oil B. Sulfanilamides
D. Atropine C. Bacteriophages
E. Novocaine amide D. Autovaccines
E. Vitamins
117. Examination of patients with peri-
odontitis revealed the interdependence 122. A man got poisoned with
between the rate of affection of peri- mushrooms. They contain muscarine that
odontal tissues and the amount of stimulates muscarinic cholinoreceptors.
lysozymes in saliva and gingival liquid. What symptom is typical for poisoning
These results can be obtained during with inedible mushrooms?
studying the following protection system A. Miosis
of an organism: B. Mydriasis
A. Non-specific resistance C. Bronchi dilation
B. Humoral immunity D. Heart rate rise
C. Cellular immunity E. Arterial pressure rise
D. Autoresponsiveness 123. A patient complains of skin itch,
E. Tolerance especially between fingers, in the inguinal
118. An isolated cell of human heart creases, on the lower abdomen. Exami-
automatically generates excitement nation of these regions revealed there
impulses with frequency of 60 times per some small vesicles. Laboratory diagnosti-
minute. This cell was taken from the cs allowed to establish that this condition
following heart structure: had been caused by a representative of
Arthropoda. Specify the disease caused
A. Sinoatrial node by this arthropod:
B. Atrium
C. Ventricle A. Scabies
D. Atrioventricular node B. Demodicosis
E. His’ bundle C. Myiasis
D. Pediculosis
119. A patient with epilepsy and depressi- E. Dermatotropic leishmaniasis
ve reaction has been administered a drug
that reduced epilepsy manifestations and 124. An infant has pylorospasm,
improved the patient’s psychic condition. weakness, hypodynamia, convulsions as
Krok 1 Medicine 2012 14

a result of frequent vomiting. What kind 129. A patient suffers from stenocardia
of acid-base disbalance is it? and takes isosorbide mononitrate. He was
prescribed a complementary drug with di-
A. Excretory alkalosis saggregating effect. What drug is it?
B. Excretory acidosis
C. Metabolic acidosis A. Acetylsalicinic acid
D. Exogenous nongaseous acidosis B. Nitroglycerine
E. Gaseous alkalosis C. Propranolol
D. Nifedipine
125. During influenza epidemic 40% of E. Validol
pupils who didn’t go in for sports were
affected by the disease, and among the 130. A 49 year old woman spent a lot of
pupils who regularly did physical exercises time standing. As a result of it she got leg
this index was only 20%. What adaptati- edema. What is the most likely cause of
ve mechanisms determined such a low si- the edema?
ckness rate of pupils participating in the
sports? A. Increase in hydrostatic pressure of
blood in veins
A. Cross adaptation B. Decrease in hydrostatic pressure of
B. Specific adaptation blood in veins
C. Physiological adaptation C. Decrease in hydrostatic pressure of
D. Biochemical adaptation blood in arteries
E. Genetic adaptation D. Increase in oncotic pressure of blood
plasma
126. A 39 year old man who had been E. Increase in systemic arterial pressure
operated for the stomach ulcer died 7 days
after the surgery. Autopsy revealed that 131. A patient presented to a hospital wi-
peritoneal leaves were dull, plephoric, th complaints about quick fatigability and
covered with massive yellow-greenish fi- significant muscle weakness. Examinati-
lms, the peritoneal cavity contained for on revealed an autoimmune disease that
about 300 ml of thick yellow-greenish causes functional disorder of receptors
liquid. What pathologic process was in the neuromuscular synapses. This wi-
revealed in the peritoneal cavity? ll result in the disturbed activity of the
following mediator:
A. Fibrinous suppurative peritonitis
B. Serous peritonitis A. Acetylcholine
C. Fibrinous serous peritonitis B. Noradrenaline
D. Peritoneal commissures C. Dopamine
E. Fibrinous haemorrhagic peritonitis D. Serotonin
E. Glycine
127. Preventive examination of a pati-
ent revealed an enlarged lymph node of 132. Blood test of a patient suffering
metastatic origin on the medial wall of the from atrophic gastritis gave the followi-
left axillary crease. Specify the most likely ng results: RBCs - 2, 0 · 1012 /l, Hb- 87 g/l,
localization of the primary tumour: colour index - 1,3, WBCs - 4, 0 · 109 /l,
A. Mammary gland thrombocytes - 180 · 109 /l. Anaemia mi-
B. Submandibular salivary gland gh have been caused by the following
C. Lung substance deficiency:
D. Stomach A. Vitamin B12
E. Thyroid gland B. Vitamin A
128. A 60 year old patient was found C. Vitamin K
to have a dysfunction of main digesti- D. Iron
ve enzyme of saliva. This causes the di- E. Zinc
sturbance of primary hydrolysis of: 133. A 45 year old man consulted a doctor
A. Carbohydrates about a plaque-like formation on his neck.
B. Fats Histological examination of a skin bi-
C. Proteins optate revealed clusters of round and
D. Cellulose oval tumour cells with a narrow border of
E. Lactose basophilic cytoplasm resembling of cells
of basal epidermal layer. What tumour is
it?
Krok 1 Medicine 2012 15

infiltration and growth of granulation ti-


A. Basal cell carcinoma ssue bulging into the bronchial lumen in
B. Epidermal cancer form of a polyp. What is the most likely
C. Hydroadenoma diagnosis?
D. Trichoepithelioma
E. Syringoadenoma A. Chronic bronchitis
B. Lobar pneumonia
134. A 71 year old man had been presenti- C. Acute bronchitis
ng with diarrhea for 10 days. The feces D. Bronchopneumonia
had admixtures of blood and mucus. He E. Interstitial pneumonia
was delivered to a hospital in grave condi-
tion and died 2 days later. Autopsy of the 138. Acute renal impairment caused
body revealed the following: diphtheri- death of a bleeding patient. Autopsy
tic colitis with multiple irregularly-shaped revealed enlarged kidneys with a broad
ulcers of different depth in both sigmoid pale pink cortical layer expressively
colon and rectus. Bacteriological analysis demarcated from dark red renal pyrami-
revealed Shigella. What was the main di- ds. Macroscopic examination revealed
sease? lack of epithelial nuclei of convoluted
tubules, tubulorrhexis, phlebostasis. The
A. Dysentery cell nuclei of choroid glomus and straight
B. Typhoid fever tubules were present. What pathology is
C. Salmonellosis it?
D. Nonspecific ulcerous colitis
E. Yersiniosis A. Necronephrosis
B. Infarction
135. In spite of treatment with cardiotoni- C. Glomerulonephritis
cs and thiazide diuretic a patient suffering D. Pyelonephritis
from chronic cardiac failure still presents E. Nephrosis
with edemata and faces a risk of ascites.
What medication should be administered 139. Osteolaterism is charcterized by a
in order to increase the diuretic effect of decrease in collagen strength caused by
the above mentioned drugs? much less intensive formation of cross-
links in collagen fibrils. This phenomenon
A. Spironolactone is caused by the low activity of the followi-
B. Furosemide ng enzyme:
C. Amiloride
D. Clopamide A. Lysyl oxidase
E. Manithol B. Monoamino-oxidase
C. Prolyl hydroxylase
136. Autopsy of a 75 year old patient D. Lysyl hydroxylase
who had been suffering from dissemi- E. Collagenase
nated atherosclerosis and died under
chronic cardiac failure revealed constricti- 140. A histological specimen of ki-
on and deformation of coronary arteries, dney shows a structure consisting of a
tuberous intima whose section appeared glomerulus of fenestrated capillaries and
to be white and petrosal. Specify the stage a bilayer epithelial capsule. Specify this
of atherosclerosis morphogenesis: structure:
A. Atherocalcinosis A. Renal corpuscle
B. Lipoidosis B. Proximal tubule
C. Liposclerosis C. Distal tubule
D. Bilipid D. Henle’s loop
E. Atheromatosis E. Receiving tube
137. Examination of a bronchial tissue 141. A 66 year old female patient
sample revealed atrophy of mucous got intravenous injection of magnesi-
membrane, cystic degeneration of glands, um sulfate solution for the purpose of
focal metaplastic changes of lining pri- elimination of hypertensive crisis. But
smatic epithelial cells into multilayer arterial pressure didn’t go down and
squamous cells; increase in goblet cell after repeated introduction of the same
number; in some parts of bronchial wall preparation there appered sluggishness,
and especially in the mucous membrane slow response, inhibition of consciousness
there was marked cellular inflammatory and respiration. What preparation is
Krok 1 Medicine 2012 16

antagonist of magnesium sulfate and can A. Mosaicism


eliminate symptoms of its overdose? B. Inversion
C. Transposition
A. Calcium chloride D. Duplication
B. Potassium chloride E. Heterogeneity
C. Sodium chloride
D. Activated carbon 146. An oncological patient had been
E. Potassium permanganate administered methotrexate. With time
target cells of the tumour lost sensitivity to
142. As a result of a road accident a 26- this drug. At the same time the change in
year-old man is in the torpid phase of gene expression of the following enzyme
shock. Blood count: leukocytes - 3, 2 · is observed:
109 /l. What is the leading mechanism of
leukopenia development? A. Dehydropholate reductase
B. Thiaminase
A. Leukocyte redistribution in the C. Deaminase
bloodstream D. Pholate oxidase
B. Leukopoiesis inhibition E. Pholate decarboxylase
C. Faulty release of mature leukocytes
from the bone marrow into the blood 147. A 64 year old woman has impairment
D. Leukocyte destruction in the hematopi- of twilight vision (hemeralopy). What vi-
etic organs tamin should be recommended in the first
E. Increased excretion of the leukocytes place?
from the organism
A. A
143. A 5-month-old boy was hospitali- B. B2
zed for tonic convulsions. He has a life- C. E
time history of this disease. Examination D. C
revealed coarse hair, thinned and fragile E. B6
nails, pale and dry skin. In blood: calci-
um - 1,5 millimole/l, phosphor - 1,9 milli- 148. A patient underwent appendectomy.
mole/l. These changes are associated with: In the postoperative period he has been
taking an antibiotic. The patient complai-
A. Hypoparathyroidism ns about hearing impairment and vesti-
B. Hyperparathyroidism bular disorders. What group of antibiotics
C. Hyperaldosteronism has such by-effects?
D. Hypoaldosteronism
E. Hypothyroidism A. Aminoglycosides
B. Penicillins
144. As a result of a trauma a patient has C. Tetracyclines
developed traumatic shock that led to the D. Macrolides
following disorders: AP is 140/90 mm Hg, E. Cephalosporins
Ps is 120 bpm. The patient is fussy, talkati-
ve, pale. Such state relates to the following 149. While playing a child got a punch
shock phase: in the presternum region. As a result
of this trauma an organ located behind
A. Erectile the presternum was damaged. Name this
B. Latent period organ:
C. Terminal
D. Torpid A. Thymus
E. - B. Thyroid gland
C. Heart
145. Cytogenetic examination of a pati- D. Pericardium
ent with dysfunction of the reproductive E. Larynx
system revealed normal karyotype 46,ХУ
in some cells, but most cells have Kli- 150. In an embryo the process of dorsal
nefelter’s syndrome karyotype - 47,ХХУ. mesoderm segmentation and somite
Such phenomenon of cell inhomogeneity formation is disturbed. What part of
is called: skin will probably have developmental
abnormalities?
Krok 1 Medicine 2012 17

A. Dermis A. Parathyroid osteodystrophy


B. Hair B. Myelomatosis
C. Sebaceous glands C. Osteomyelitis
D. Epidermis D. Osteopetrosis
E. Perspiratory glands E. Paget’s disease
151. A patient consulted a dentist about 155. A patient has been diagnosed wi-
itching and burning in the oral cavity; high th a compression fracture of a lumbar
temperature. The patient was diagnosed vertebra. As a result he has a consi-
with trichomonal gingivostomatitis. What derable increase in curvature of the
drug should be chosen for his treatment? lumbar lordosis. Which ligament damage
can induce such changes in the spine
A. Metronidazole curvature?
B. Ampicillin
C. Doxycycline hydrochloride A. Anterior longitudinal ligament
D. Gentamicin sulfate B. Posterior longitudinal ligament
E. Nystatin C. Yellow ligament
D. Iliolumbar ligament
152. An electron microphotography of a E. Interspinous ligament
fragment of proper gastric gland shows
a big irregular round-shaped cell. There 156. When water affects mucous
are a lot of intracellular tubules and mi- membrane of the inferior nasal meatuses,
tochondria in the cytoplasm. Specify this this causes "diver reflex"that provokes:
cell:
A. Reflectory apnea
A. Parietal cell B. Reflectrory dyspnea
B. Principal cell C. Reflectory hyperpnea
C. Undifferentiated cell D. Cough
D. Mucous cell E. Bronchospasm
E. Endocrine cell
157. Life cycle of a cell includes a
153. A patient suffering from stenocardia process of DNA autoreduplication. As
was taking nitroglycerine which caused a result of this process monochromatid
restoration of blood supply of myocardi- chromosomes become bichromatid. This
um and relieved pain in the cardiac area. phenomenon is observed within the
What intracellular mechanism provides following period of the cell cycle:
restoration of energy supply of insulted
cells? A. S
B. G0
A. Intensification of ATP resynthesis C. G1
B. Reduction of ATP resynthesis D. G2
C. Increased permeability of membranes E. M
D. Intensification of oxygen transporting
into the cell 158. Examination of a child who
E. Intensification of RNA generation frequently suffers from infectious diseases
revealed that IgG concentration in blood
154. A 50 year old patient has been taki- serum was 10 times less than normal, IgA
ng treatment thrice for the last 6 months and IgM concentration was also signifi-
because of fractures caused by domestic cantly reduced. Analysis showed also lack
accidents. Microscopical examination of of B-lymphocytes and plasmocytes. What
bony tissue revealed foci of lacunar disease are these symptoms typical for?
resolution, giant-cell granulomas in the
tumour-like formations, cysts. Bony tissue A. Bruton’s disease
was substituted by fibrous connective ti- B. Swiss-type agammaglobulinemia
ssue. Examination revealed also adenoma C. Dysimmunoglobulinemia
of parathyroid gland and hypercalcemia. D. Louis-Bar syndrome
What is the most probable diagnosis? E. Di George syndrome
159. Examination of a 42 year old patient
revealed a tumour of adenohypophysis.
Objectively: the patient’s weight is 117
kg, he has moon-like hyperemic face, red-
blue striae of skin distension on his belly.
Krok 1 Medicine 2012 18

Osteoporosis and muscle dystrophy are consumes a lot of sweets and rich food,
present. AP is 210/140 mm Hg. What is has sedentary way of life. That’s why
the most probable diagnosis? anabolic metabolism has the priority in
his organism. Which of the following
A. Cushing’s disease pathways is amphibolic?
B. Cushing’s syndrome
C. Conn’s disease A. Cycle of tricarboxylic acids
D. Diabetes mellitus B. Glyconeogenesis
E. Essential hypertension C. Lipolysis
D. Glycolysis
160. Microelectrode technique allowed E. Fatty acids oxidation
to register a potential following
"all-or-none"law and being able of 165. A patient who abuses smoking has
undecremental spreading. Specify this chronic bronchitis. Biopsy of his primary
potential: bronchus revealed multilayer pavement
epithelium. What pathological process
A. Action potential was revealed in the bronchus?
B. Excitatory postsynaptic potential
C. Rest potential A. Metaplasia
D. Inhibitory postsynaptic potential B. Physiological regeneration
E. Receptor potential C. Reparative regeneration
D. Hyperplasia
161. A patient has been diagnosed with E. Dysplasia
transmural myocardial infarction. What
drug should be given in order to prevent 166. A patient suffers from pulmonary
cardiogenic shock? tuberculosis. During treatment neuritis of
visual nerve arose. What drug has caused
A. Promedol this by-effect?
B. Reserpin
C. Octadine A. Isoniazid
D. Phentolamine B. Ethambutol
E. Analgin C. Kanamycin
D. Rifampicin
162. 2 days after labour a woman E. Streptomycin
developed shock along with DIC
syndrome that caused her death. Autopsy 167. A 67 year old patient complains of
revealed purulent endomyometritis, regi- periodic heart ache, dyspnea during light
onal purulent lymphangitis, lymphadeni- physical activities. ECG reveals extraordi-
tis and purulent thrombophlebitis. There nary contractions of heart ventricles. Such
were also dystrophic alterations and arrhythmia is called:
interstitial inflammation of parenchymal
organs. What is the most likely diagnosis? A. Extrasystole
B. Bradycardia
A. Septicemia C. Tachycardia
B. Syphilis D. Flutter
C. Tuberculosis of genital organs E. Fibrillation
D. Chorioadenoma destruens
E. Hydatid mole 168. A patient with ischemic heart disease
has been administered an anti-anginal
163. A patient got an injury of spinal drug that reduces the myocardial oxygen
marrow in a road accident that caused consumption and improves blood supply
loss of tactile sensation, posture sense, vi- of myocardium. What drug is it?
bration sense. What conduction tracts are
damaged? A. Nitroglycerine
B. Validol
A. Fascicle of Goll and cuneate fascicle C. Propranolol
B. Anterior spinocerebellar tract D. Promedol
C. Rubrospinal tract E. Retabolil
D. Reticulospinal tract
E. Tectospinal tract 169. A doctor asked a patient to breath
out fully after taking a normal breath.
164. Examination of a patient revealed What muscles contract during such
II grade obesity. It is known that he exhalation?
Krok 1 Medicine 2012 19

A. Gamma-amino butyric acid


A. Abdominal muscles B. Serotonin
B. External intercostal muscles C. Dopamine
C. Diaphragm D. Acetylcholine
D. Trapezius muscles E. Histamine
E. Pectoral muscles
175. A 50 year old patient underwent
170. A 4 year old child with hereditary resection of tumour of large intestine
renal lesion has signs of rickets, vitamin D wall. Microscopically it presents itself as
concentration in blood is normal. What fascicles of divergent collagen fibers of
is the most probable cause of rickets different thickness and form and some
development? monomorphous fusiform cells that are
irregularly distributed among the fibers.
A. Impaired synthesis of calcitriol Cellular atypia is not evident. What
B. Increased excretion of calcium tumour is it?
C. Hyperfunction of parathyroid glands
D. Hypofunction of parathyroid glands A. Hard fibroma
E. Lack of calcium in food B. Fibromyoma
C. Soft fibroma
171. A man was intoxicated with D. Desmoma
mushrooms. They contain muscarine that E. Fibrosarcoma
stimulates muscarinic cholinoreceptors.
What symptoms signalize intoxication wi- 176. Toxic affection of liver results in
th inedible mushrooms? dysfunction of protein synthesis. It is
usually accompanied by the following ki-
A. Myotic pupils nd of dysproteinemia:
B. Mydriatic pupils
C. Bronchi dilatation A. Absolute hypoproteinemia
D. Increased heart rate B. Relative hypoproteinemia
E. Rise of arterial pressure C. Absolute hyperproteinemia
D. Relative hyperproteinemia
172. There was a record of some anthrax E. Paraproteinemia
cases among animals in a countryside.
The spread of disease can be prevented 177. A cell at the stage of mitosis anaphase
by means of immunization. What kind of was stimulated by colchicine that inhibi-
vaccine should be used? ts chromosome separation to the poles.
What type of mutation will be caused?
A. STI live vaccine
B. BCG vaccine A. Polyploidy
C. Salk vaccine B. Inversion
D. Sabin’s vaccine C. Deletion
E. Diphteria and tetanus toxoids and D. Duplication
pertussis vaccine E. Translocation
173. An infectious disease unit admitted a 178. Examination of duodenal contents
patient with signs of jaundice caused by revealed some pyriform protozoa with
hepatitis virus. Select an indicator that is twin nuclei and four pairs of flagella.
specific only for parenchymatous jaundi- There were two supporting filaments
ce: between the nuclei and a suctorial disc
on the ventral side. What representative
A. Increase in ALT and AST rate of protozoa was revealed in this patient?
B. Hyperbilirubinemia
C. Bilirubinuria A. Lamblia
D. Cholaemia B. Toxoplasma
E. Urobilinuria C. Leishmania
D. Intestinal trichomonad
174. A patient presents with dysfuncti- E. Trypanosome
on of cerebral cortex accompanied by
epileptic seizures. He has been admini- 179. In order to prevent massive
stered a biogenic amine synthetized from haemorrhage in the region of oral cavi-
glutamate and responsible for central ty floor it is required to ligate an artery
inhibition. What substance is it? which is located within Pirogov’s triangle.
What artery is it?
Krok 1 Medicine 2012 20

A. Homogenization
A. Lingual artery B. Inactivation
B. Superior thyroid artery C. Flotation
C. Facial artery D. Filtration
D. Ascending pharyngeal artery E. Neutralization
E. Maxillary artery
184. Electronic microphotography of
180. A patient with high-titer antinuclear pulmonary alveole’s wall presents a big
antibodies died from progressing renal cell. Its cytoplasm has a lot of mi-
impairment. Autopsy revealed mesangi- tochondria, developed Golgi apparatus,
oproliferative glomerulonephritis and osmiophil lamellated corpuscles. What is
abacterial polypous endocarditis. There the main function of this cell?
was periarterial bulbar sclerosis in spleen
and productive proliferative vasculitis in A. It produces surfactant
skin. What is the most likely diagnosis? B. It is a component of blood-air barrier
C. It warms the air
A. Systemic lupus erythematosus D. It purifies the air
B. Nephrotic syndrome E. It absorbs microorganisms
C. Rheumatism
D. Dermatomyositis 185. A patient with diabetes mellitus
E. Periarteritis nodosa complicated by angiopathy has been
recommended a drug which is a sulphonyl
181. A shepherd who has tended urease derivate of the second generati-
sheep together with dogs consulted a on. It improves microcirculation and is
doctor about pain in his right subcostal known for its relatively good tolerance.
area, nausea, vomiting. Roentgenoscopy What drug is it?
revealed a tumour-like formation. What
kind of helminthiasis might be suspected? A. Glibenclamide
B. Glibutidum
A. Echinococcosis C. Insulin
B. Ascaridiasis D. Acarbose
C. Enterobiasis E. Adrenalin
D. Taeniarhynchosis
E. Taeniasis 186. A child is pale, pastose, muscular ti-
ssue is bad developed, lymph nodes are
182. For relief of hypertensive crisis a enlarged. He often suffers from angi-
doctor administered a patient a drug na and pharyngitis, blood has signs of
that apart from antihypertensive effect lymphocytosis. The child is also predi-
has also sedative, spasmolytic and anti- sposed to autoallergic diseases. What type
convulsive effect. The drug was taken of diathesis can be presumed in this case?
parenterally. When it is taken enterally
it acts as a laxative and cholagogue. What A. Lymphohypoplastic
drug was administered? B. Exudative
C. Gouty
A. Magnesium sulfate D. Asthenic
B. Dibasolum E. Hemorrhagic
C. Reserpine
D. No-spa 187. A 26 year old pregnant woman is
E. Apressin under treatment at an in-patient hospi-
tal. After a continuous attack of vomiting
183. A bacteriological laboratory received she was found to have reduced volume of
sputum sample of a patient suffering from circulating blood. What kind of change in
tuberculosis. Bacterioscopic examination general blood volume is the case?
of smears and detection of tuberculosis
bacillus can be realized by one of enri- A. Polycythemic hypovolemia
chment methods that involves processing B. Simple hypovolemia
of sputum only with solution of caustic C. Oligocythemic hypovolemia
soda. What is this method called? D. Polycythemic hypervolemia
E. Oligocythemic hypervolemia
188. Vomiting matters of a patient
suspected of having cholera were deli-
vered to the bacteriological laboratory.
Krok 1 Medicine 2012 21

The material was used for preparing a 193. Sex chromosomes of a woman didn’t
"hanging drop"specimen. What type of separate and move to the opposite poles
microscopy will be applied for identificati- of a cell during gametogenesis (meiosis).
on of the causative agent by its mobility? The ovum was impregnated with a normal
spermatozoon. Which chromosomal di-
A. Phase-contrast microscopy sease can be found in her child?
B. Electron microscopy
C. Immune and electron microscopy A. Turner’s syndrome
D. Fluorescence microscopy B. Down’s syndrome
E. Immersion microscopy C. Patau’s syndrome
D. Edwards’ syndrome
189. Examination of a 6-month-old child E. Cat cry syndrome
revealed a delay in closure of the occipi-
tal fontanelle. When should it normally 194. After taking poor-quality food a pati-
close? ent developed repeated episodes of di-
arrhea. On the next day he presented with
A. Until 3 months decreased arterial pressure, tachycardia,
B. Before the child is born extrasystole. Blood pH is 7,18. These
C. Until 6 months abnormalities were caused by the
D. Until the end of the first year of life development of:
E. Until the end of the second year of life
A. Nongaseous acidosis
190. A 45 year old male died from di- B. Gaseous acidosis
sseminated tuberculosis. On autopsy the C. Nongaseous alkalosis
symptoms of tuberculosis were confirmed D. Gaseous alkalosis
by both microscopical and histological E. Metabolic alkalosis
analyses. All the affected organs had
epithelioid cell granulomas with caseous 195. A patient has been diagnosed with
necrosis in the centre. What kind of influenza. His condition became drasti-
hypersensitivity reaction underlies the cally worse after taking antipyretic drugs.
process of granuloma development? His consciousness is confused, AP is 80/50
mm Hg, Ps is 140/m, body temperature
A. Delayed droped down to 35, 8oC. What complicati-
B. Antibody-dependent cytotoxicity on developed in this patient?
C. Complement-dependent cytotoxicity
D. Anaphylactic A. Collapse
E. Immune complex B. Hyperthermia
C. Hypovolemia
191. A male patient has been diagnosed D. Acidosis
with acute radiation disease. Laboratory E. Alkalosis
examination revealed a considerable
reduction of platelet serotonin level. The 196. One of sections of central nervous
likely cause of platelet serotonin reducti- system has layerwise arrangement of
on is the disturbed metabolism of the neurocytes. Among them there are cells
following substance: of the following forms: stellate, fusiform,
horizontal, pyramidal. What section of
A. 5-oxytryptofane central nervous system is this structure
B. Tyrosine typical for?
C. Histidine
D. Phenylalanine A. Cortex of cerebrum
E. Serine B. Spinal cord
C. Cerebellum
192. In course of a conditional experiment D. Medulla oblongata
the development of mesenchyma cells E. Hypothalamus
was completely inhibited. Development
of the following muscular tissue will be di- 197. Pharmacological effects of anti-
sturbed: depressants are based upon blocking
(inhibiting) the enzyme that acts as a
A. Smooth muscular tissue catalyst for the breakdown of biogenic
B. Neural muscular tissue amines noradrenalin and serotonin in the
C. Epidermal muscular tissue mitochondria of cephalic neurons. What
D. Cardiac muscular tissue enzyme takes part in this process?
E. Skeletal muscular tissue
Krok 1 Medicine 2012 22

A. Monoamine oxidase In order to determine the diphtheria


B. Transaminase causative agent the material should be
C. Decarboxylase inoculated into the following differenti-
D. Peptidase al diagnostic medium:
E. Lyase
A. Blood tellurite agar
198. During examination of a pati- B. Endo’s agar
ent a dentist revealed a lot of "white C. Ploskyrev’s agar
spots zones of enamel demineralization. D. Sabouraud’s agar
What microorganisms take part in the E. Levenshtein-Yessen agar
development of this process?
200. A man with a stab wound in the regi-
A. Streptococcus mutans on of the quadrilateral foramen consulted
B. Streptococcus salivarius a doctor about it. Examination revealed
C. Streptococcus pyogenes that the injured couldn’t abduct his arm
D. Veilonella parvula from the body. What nerve is most likely
E. Staphylococcus epidermidis damaged?
199. A 4 year old child presents with A. n. axillaris
general weakness, sore throat and degluti- B. n. medianus
tive problem. After his examination a C. n. radialis
doctor suspected diphtheria and sent the D. n. ulnaris
material to the bacteriological laboratory. E. n. subclavius
Krok 1 Medicine 2013 1

1. A patient diagnosed with focal A. Mononuclear cells, T-lymphocytes and


tuberculosis of the upper lobe of the right lymphokines
lung had been taking isoniazid as a part B. Granulocytes, T-lymphocytes and IgG
of combination therapy. After some time, C. Plasma cells, T-lymphocytes and
the patient reported of muscle weakness, lymphokines
decreased skin sensitivity, blurred visi- D. B-lymphocytes, IgM
on, impaired motor coordination. Whi- E. Macrophages, B-lymphocytes and
ch vitamin preparation should be used to monocytes
address these phenomena?
5. Hemoglobin catabolism results in
A. Vitamin B6 release of iron which is transported to
B. Vitamin A the bone marrow by a certain transfer
C. Vitamin D protein and used again for the synthesis of
D. Vitamin B12 hemoglobin. Specify this transfer protein:
E. Vitamin C
A. Transferrin (siderophilin)
2. A 60-year-old male patient has a 9-year B. Transcobalamin
history of diabetes and takes insulin Semi- C. Haptoglobin
lente for the correction of hyperglycemia. D. Ceruloplasmin
10 days ago he began taking anaprilin E. Albumin
for hypertension. One hour after admini-
stration of the antihypertensive drug the 6. A 12-year-old boy has been hospitali-
patient developed hypoglycemic coma. zed for suspected food poisoning. The
What is the mechanism of hypoglycemia fecal samples were inoculated on the
in case of anaprilin use? Endo agar, which resulted in growth of
a large number of colorless colonies.
A. Inhibition of glycogenolysis What microorganism is most likely to be
B. Reduction of glucagon half-life EXCLUDED from the list of possible
C. Increase of insulin Semilente half-life causative agents of the disease?
D. Increase of bioavailability of insulin
Semilente A. Escherichia coli
E. Decrease in glucose absorption B. Salmonella enteritidis
C. Proteus vulgaris
3. Pterin derivatives (aminopterin and D. Pseudomonas aeruginosa
methotrexate) are the inhibitors of di- E. Yersinia enterocolitica
hydrofolate reductase, so that they inhibit
the regeneration of tetrahydrofolic acid 7. A 23-year-old patient has been admi-
from dihydrofolate. These drugs inhibit tted to a hospital with a craniocerebral
the intermolecular tranfer of monocarbon injury. The patient is in a grave condition.
groups, thus suppressing the synthesis of Respiration is characterized by prolonged
the following polymer: convulsive inspiration followed by a short
expiration. What kind of respiration is it
A. DNA typical for?
B. Protein
C. Homopolysaccharides A. Apneustic
D. Gangliosides B. Gasping breath
E. Glycosaminoglycans C. Kussmaul’s
D. Cheyne-Stokes
4. A child with suspected tuberculosis was E. Biot’s
given Mantoux test. After 24 hours the
site of the allergen injection got swollen, 8. It has been experimentally proven that
hyperemic and painful. What are the main the excitation of the motor neurons of
components that determine such response flexor muscles is accompanied by the inhi-
of the body? bition of the motor neurons of extensor
muscles. What type of inhibition underli-
es this phenomenon?
A. Reciprocal
B. Inhibition after excitation
C. Pessimal
D. Feedback
E. Lateral
Krok 1 Medicine 2013 2

9. A 3-year-old boy with pronounced A. Release of immature red blood cells


hemorrhagic syndrome doesn’t have anti- from the bone marrow
hemophilic globulin A (factor VIII) in B. Impaired iron absorption in the intesti-
the blood plasma. Hemostasis has been nes
impaired at the following stage: C. Increased destruction of red blood cells
in the spleen
A. Internal mechanism of prothrombinase D. Impaired globin synthesis
activation E. Increased excretion of body iron
B. External mechanism of prothrombinase
activation 14. A patient with diabetes developed
C. Conversion of prothrombin to thrombin a diabetic coma due to the acid-base
D. Conversion of fibrinogen to fibrin imbalance. Specify the kind of this
E. Blood clot retraction imbalance:

10. A patient got a gunshot wound of A. Metabolic acidosis


hip which damaged the sciatic nerve. Any B. Metabolic alkalosis
impact on the affected limb causes severe, C. Respiratory acidosis
excruciating pain. What mechanism of D. Gaseous alkalosis
pain is most likely in this case? E. Non-gaseous alkalosis
A. Causalgic 15. A girl receives antibiotics of the penici-
B. Reflex llin group for acute bronchitis. On the thi-
C. Phantom rd day of treatment she developed allergic
D. Endorphin hypofunction dermatitis. Which drug should be admini-
E. Enkephalin hypofunction stered?

11. A 60-year-old patient with a long hi- A. Loratadine


story of stenocardia takes coronarodilator B. Cromolyn sodium
agents. He has also been administered C. Beclomethasone
acetylsalicylic acid to reduce platelet D. Ephedrine hydrochloride
aggregation. What is the mechanism of E. Levamisole
antiplatelet action of acetylsalicylic acid?
16. A female patient has been di-
A. It reduces the activity of cyclooxygenase agnosed with cervical erosion, which is
B. It reduces the activity of phosphodi- a precancerous pathology. What defense
esterase mechanism can prevent the development
C. It enhances the activity of platelet of a tumor?
adenylate cyclase
D. It enhances the synthesis of prostacyclin A. Increase in natural killer level (NK-
E. It has membrane stabilizing effect cells)
B. High-dose immunological tolerance
12. A patient with bronchial asthma has C. Increase in the activity of lysosomal
developed acute respiratory failure. What enzymes
kind of respiratory failure occurs in this D. Simplification of the antigenic structure
case? of tissues
E. Low-dose immunological tolerance
A. Obstructive disturbance of alveolar
ventilation 17. Microscopy of the coronary artery of
B. Restrictive ventilatory defect a dead 53-year-old patient revealed lumi-
C. Perfusion nal occlusion due to a fibrous plaque wi-
D. Diffusion th some lipids. The most likely form of
E. Dysregulation of alveolar ventilation atherosclerosis in this case is:
13. On the fifth day after the acute A. Liposclerosis
blood loss a patient has been diagnosed B. Lipidosis
with hypochromic anemia. What is C. Prelipid stage
the main mechanism of hypochromia D. Atheromatosis
development? E. Ulceration
18. Autopsy of the patient revealed bone
marrow hyperplasia of tubular and flat
bones (pyoid marrow), splenomegaly
(6 kg) and hepatomegaly (5 kg),
Krok 1 Medicine 2013 3

enlargement of all lymph node groups.


What disease are the identified changes A. 0,5
typical for? B. 0,25
C. 0,125
A. Chronic myelogenous leukemia D. 0,625
B. Chronic lymphocytic leukemia E. 1
C. Multiple myeloma
D. Polycythemia vera 24. A casualty has a fracture in the region
E. Hodgkin’s disease of the inner surface of the left ankle. What
is the most likely site for the fracture?
19. As a result of an injury a patient
cannot extend his arm at the elbow. This A. Medial malleolus
may cause abnormal functioning of the B. Lower third of the fibula
following muscle: C. Astragalus
D. Lateral malleolus
A. Musculus triceps brachii E. Calcaneus
B. Musculus infraspinatus
C. Musculus levator scapulae 25. Some infectious diseases caused by
D. Musculus teres major bacteria are treated with sulfanilamides
E. Musculus subscapularis which block the synthesis of bacteria
growth factor. What is the mechanism of
20. A man sitting with his eyes closed, their action?
undergoes electroencephalography. What
rhythm will be recorded on the EEG if A. They are antivitamins of para-amino
there is an audible signal? benzoic acid
B. They inhibit the absorption of folic acid
A. Beta rhythm C. They are allosteric enzyme inhibitors
B. Theta rhythm D. They are involved in redox processes
C. Delta rhythm E. They are allosteric enzymes
D. Alpha rhythm
E. Gamma rhythm 26. A 42-year-old male patient with
gout has an increased blood uric acid
21. Electrophoretic study of a blood concentration. In order to reduce the level
serum sample, taken from the patient with of uric acid the doctor administered him
pneumonia, revealed an increase in one of allopurinol. Allopurinol is the competiti-
the protein fractions. Specify this fraction: ve inhibitor of the following enzyme:
A. γ-globulins A. Xanthine oxidase
B. Albumins B. Adenosine deaminase
C. α1 -globulins C. Adenine phosphoribosyltransferase
D. α2 -globulins D. Hypoxanthine-
E. β-globulins phosphoribosyltransferase
E. Guanine deaminase
22. Examination of an 18-year-old gi-
rl revealed the following features: 27. A 40-year-old female patient di-
hypoplasia of the ovaries, broad agnosed with acute pancreatitis has been
shoulders, narrow pelvis, shortening of the delivered to the admission department of
lower extremities, "sphinx neck". Mental a regional hospital. What drug should be
development is normal. The girl was di- administered the patient in the first place?
agnosed with Turner’s syndrome. What
kind of chromosome abnormality is it? A. Contrycal
B. Platyphyllin
A. Monosomy X C. Atropine
B. Trisomy X D. Metacin
C. Trisomy 13 E. Pirenzepine
D. Trisomy 18
E. Nullisomy X 28. A patient consulted a doctor about
being unable to abduct his right arm after
23. Hypertrichosis is the Y-linked a past trauma. Examination revealed that
character. The father has hypertrichosis, the passive movements were not limi-
and the mother is healthy. In this fami- ted. The patient was found to have the
ly, the probability of having a child with atrophy of the deltoid muscle. What nerve
hypertrichosis is: is damaged?
Krok 1 Medicine 2013 4

A. Respiratory
A. Axillary B. Hemic
B. Radial C. Circulatory
C. Ulnar D. Tissue
D. Median E. Histotoxic
E. Suprascapular
33. A female patient with bronchial
29. After a trauma of the upper third of asthma had taken prednisolone tablets (1
the anterior forearm a patient exhibits tablet 3 times a day) for 2 months. Due to
difficult pronation, weakening of palmar a significant improvement of her conditi-
flexor muscles and impaired skin sensiti- on the patient suddenly stopped taking it.
vity of 1-3 fingers. Which nerve has been What complication is likely to develop in
damaged? this case?
A. n. medianus A. Withdrawal syndrome
B. n. musculocutaneus B. Cushing’s syndrome
C. n. ulnaris C. Gastrorrhagia
D. n. cutaneus antebrachii medialis D. Upper body obesity
E. n. radialis E. Hypotension
30. A 38-year-old female patient complai- 34. A patient with suspected dysentery
ns of general weakness, cardiac pain, has been admitted to the infectious di-
increased appetite, no menstruation. seases hospital. Which basic method of
Objectively: the height is 166 cm, weight laboratory diagnosis must be applied in
108 kg, the patient has moon-shaped face, the first place?
subcutaneous fat is deposited mainly in
the upper body, torso and hips. There are A. Bacteriological
also blood-red streaks. Ps- 62/min, AP- B. Serological
160/105 mm Hg. Which of the following C. Allergic
diseases is the described pattern of obesi- D. Biological
ty most typical for? E. Microscopic
A. Cushing pituitary basophilism 35. During a surgery with the use of
B. Alimentary obesity hygronium the patient had an abrupt fall
C. Myxedema in blood pressure. Blood pressure can be
D. Insulinoma normalized by the representatives of the
E. Babinski-Frohlich syndrome following drug group:
31. A 60-year-old patient with a long A. α-adrenergic agonists
history of atherosclerosis and a previ- B. α-blockers
ous myocardial infarction developed an C. Ganglionic blockers
attack of retrosternal pain. 3 days later D. M-cholinergic agents
the patient was hospitalized and then di- E. N-cholinergic agents
ed of progressive cardiovascular insuffici-
ency. At autopsy a white fibrous depressed 36. A patient with respiratory failure has
area about 3 cm in diameter with clear blood pH of 7,35. pCO2 test revealed
boundaries was found in the posterior hypercapnia. Urine pH test revealed an
wall of the left ventricle and interventri- increase in the urine acidity. What form of
cular septum. The dissector evaluated acid-base imbalance is the case?
these changes as:
A. Compensated respiratory acidosis
A. Focal cardiosclerosis B. Compensated metabolic acidosis
B. Myocardial ischemia C. Decompensated metabolic acidosis
C. Myocardial infarction D. Compensated respiratory alkalosis
D. Myocarditis E. Decompensated respiratory alkalosis
E. Myocardial degeneration
37. On examination a patient was found
32. Measurements of the arterial pCO2 to have medial strabismus, the inward
and pO2 during an attack of bronchi- deviation of the eyeball and inability to
al asthma revealed hypercapnia and abduct the eyeball outwards. What nerve
hypoxemia respectively. What kind of is damaged?
hypoxia occurred in this case?
Krok 1 Medicine 2013 5

A. Abducent A. Gonorrhea
B. Oculomotor B. Syphilis
C. Ocular C. Venereal lymphogranulomatosis
D. Trochlear D. Chancroid
E. Visual E. Trichomoniasis
38. A patient with a dislocated shoulder 42. A male patient is 28 years old. Hi-
had been admitted to a hospital. With stological study of a cervical lymph node
the purpose of skeletal muscle relaxati- revealed a change of its pattern due to the
on he was given an injection of relaxant proliferation of epithelioid, lymphoid cells
dithylinum acting normally 5-7 minutes. and macrophages having nuclei in form of
However, the effect of dithylinum in this a horseshoe. In the center of some cell
patient lasted up to 8 hours. What is the clusters there were non-structured light-
most likely cause of the prolonged effect pink areas with fragments of nuclei. What
of dithylinum in this patient? disease are these changes typical for?
A. Genetic deficiency of blood choli- A. Tuberculosis
nesterase B. Hodgkin’s disease
B. Reduced activity of microsomal liver C. Actinomycosis
enzymes D. Tumor metastasis
C. Reduced drug excretion E. Syphilis
D. Material accumulation of the drug
E. Potentiation by another drug 43. A 35-year-old male patient has been
referred by an andrologist for the genetic
39. As a result of an injury of the knee joi- counselling for the deviations of physi-
nt a patient shows a drawer sign, that is cal and mental development. Objectively:
the anterior and posterior displacement the patient is tall, has asthenic constituti-
of the tibia relative to the femur. What li- on, gynecomastia, mental retardation. Mi-
gaments are damaged? croscopy of the oral mucosa cells revealed
sex chromatin (single Barr body) in 30%
A. Cruciate ligaments of cells. What is the most likely diagnosis?
B. Arcuate popliteal ligaments
C. Oblique popliteal ligament A. Klinefelter syndrome
D. Interosseous membrane B. DiGeorge syndrome
E. Collateral ligaments C. Down syndrome
D. Recklinghausen’s disease
40. The neurosurgical department has E. Cushing pituitary basophilism
admitted a 54-year-old male complaini-
ng of no sensitivity in the lower eyelid 44. A patient with jaundice has high
skin, lateral surface of nose, upper lip. total bilirubin that is mainly indirect
On examination the physician revealed (unconjugated), high concentration of
the inflammation of the second branch of stercobilin in the stool and urine. The
the trigeminal nerve. This branch comes level of direct (conjugated) bilirubin in
out of the skull through the following the blood plasma is normal. What kind of
foramen: jaundice can you think of?
A. Round foramen A. Hemolytic
B. Lacerated foramen B. Parenchymal (hepatic)
C. Oval foramen C. Mechanical
D. Spinous foramen D. Neonatal jaundice
E. Superior orbital fissure E. Gilbert’s disease
41. Bacteriological examination of 45. A male with a lesion of one of the CNS
purulent discharges from the urethra parts has asthenia, muscular dystonia,
revealed some gram-negative bean- balance disorder. Which CNS part has
shaped bacteria located in the leukocytes. been affected?
They can be identified as the causative
agent of the following disease: A. Cerebellum
B. Black substance
C. Reticular formation
D. Red nuclei
E. Vestibular nuclei
Krok 1 Medicine 2013 6

46. A 50-year-old patient has been admi- A. Persistent


nistered laevomycetin for the treatment of B. Hectic
typhoid fever, but on the next day the pati- C. Remittent
ent’s condition worsened, the temperature D. Hyperpyretic
rose to 39, 60C. The deterioration of the E. Recurrent
patient’s condition can be explained by:
51. The temperature in a production room
A. Effects of endotoxins of the causative is 36oC. Relative air humidity is 80%.
agent Under these conditions the human body
B. Allergic reaction transfers heat mainly through:
C. Insensitivity of the pathogen to
laevomycetin A. Sweat evaporation
D. Secondary infection B. Heat conduction
E. Re-infection C. Radiation
D. Convection
47. A 12-year-old patient has been admi- E. -
tted to a hospital for hemarthrosis of
the knee joint. From early childhood he 52. A hospitalized patient bitten by a rabid
suffers from frequent bleedings. Diagnose animal has an avulsive wound of shin.
the boy’s disease: What kind of vaccine must be given to
prevent rabies?
A. Hemophilia
B. Hemorrhagic vasculitis A. Anti-rabies vaccine
C. Hemolytic anemia B. DTaP
C. Td
D. B12 (folic acid)-deficiency anemia D. BCG
E. Thrombocytopenic purpura E. TABte
48. Examination of a patient with ischemic 53. At autopsy the occipital lobe of brain
heart disease revealed the impaired was found to have a cavity 2,5x1,5 cm
venous blood flow in the territory of large filled with a transparent liquid. The
the cardiac vein running in the anterior cavity had smooth brownish walls. What
interventricular sulcus of heart. What vein process had developed in the brain?
is it?
A. Cyst on the site of a hemorrhage
A. V. cordis magna B. Softening of the cerebrocortical grey
B. V. cordis media matter
C. V. cordis parva C. Brain abscess
D. V. posterior ventriculi sinistri D. Paracephalia
E. V. obliqua atrii sinistri E. A cyst on the site of the softening of the
49. For the direct injection of medi- cerebrocortical grey matter
cations into the liver surgeons use the 54. A child entering the school for the first
round ligament of liver. This manipulation time was given Mantoux test in order to
involves bougienage (lumen dilatation) of determine if there was a need for revacci-
the following vessel: nation. The reaction was negative. What
A. V. umbilicalis is the meaning of this test result?
B. A. umbilicalis A. No cell-mediated immunity to
C. Ductus venosus tuberculosis
D. V. porta B. Availability of cell-mediated immunity
E. A. hepatica propria to tuberculosis
50. A patient with lobar pneumonia C. No antibodies to the tuberculosis
has had body temperature of 39oC wi- bacteria
th daily temperature fluctuation of no D. No anti-toxic immunity to tuberculosis
more than 1o C for 9 days. This fever E. Presence of antibodies to the
can be characterized by the following tuberculosis bacteria
temperature curve: 55. Study of the biopsy material revealed
a granuloma consisting of lymphocytes,
plasma cells, macrophages with foamy
cytoplasm (Mikulicz cells), many hyaline
globules. What disease can you think of?
Krok 1 Medicine 2013 7

festation of adverse reactions to drugs is


A. Rhinoscleroma called:
B. Leprosy
C. Syphilis A. Idiosyncrasy
D. Tuberculosis B. Allergy
E. Actinomycosis C. Sensibilization
D. Tachyphylaxis
56. Autopsy of a 78-year-old patient E. Tolerance
revealed that retroperitoneal tissue was
soaked with blood, the abdominal aorta 61. A 40-year-old patient with the
had a sacciform protrusion including a progressing staphylococcal purulent peri-
defect with irregular edges. The wall of odontitis developed purulent inflammati-
the aorta was here and there of stone-like on of bone marrow spaces of the alveolar
density. This is the complication of the process, and then of the body of mandi-
following disease: ble. Microscopy revealed thinning of
bone trabeculae, foci of necrosis, bone
A. Atherosclerosis sequesters surrounded by the connecti-
B. Essential hypertension ve tissue capsule. What is the most likely
C. Systemic vasculitis diagnosis?
D. Visceral syphilis
E. Marfan syndrome A. Chronic osteomyelitis
B. Acute osteomyelitis
57. Glycogen polysaccharide is synthesi- C. Parodontome
zed from the active form of glucose. The D. Chronic fibrous periostitis
immediate donor of glucose residues duri- E. Purulent abscess
ng the glycogenesis is:
62. Curariform substances introduced
A. UDP-glucose into a human body cause the relaxati-
B. Glucose-1-phosphate on of all skeletal muscles. What changes
C. ADP-glucose in the neuromuscular synapse cause this
D. Glucose-6-phosphate phenomenon?
E. Glucose-3-phosphate
A. Blockade of N-cholinergic receptors of
58. After the diagnostic tests a 40-year- the synaptic membrane
old male has been referred for the B. Impaired acetylcholine release
lymphography of the thoracic cavity. The C. Blockade of Ca2+ channels of the
surgeon revealed that the tumor had presynaptic membrane
affected an organ whose lymphatic vessels D. Impaired cholinesterase synthesis
drain directly into the thoracic duct. Speci- E. Depolarization of the postsynaptic
fy this organ: membrane
A. Esophagus 63. A number of diseases can be di-
B. Trachea agnosed by evaluating activity of blood
C. Left main bronchus transaminases. What vitamin is one of
D. Heart cofactors of these enzymes?
E. Pericardium
A. B6
59. A patient with biliary dyskinesia B. B2
and constipations has been prescribed a C. B1
cholagogue having also a laxative effect. D. B8
What drug has been administered? E. B5
A. Magnesium sulfate 64. After a car accident a 23-year-old male
B. Allochol presented to the hospital with a cut wound
C. Cholosas of the anteromedial region of shoulder
D. Cholenzyme and arterial bleeding. Which artery was
E. Nicodinum damaged?
60. It is known that individuals with
genetically caused deficiency of glucose-
6-phosphate dehydrogenase may develop
RBC hemolysis in response to the admini-
stration of some antimalarial drugs. Mani-
Krok 1 Medicine 2013 8

A. A. brachialis A. Skeletal tissues


B. A. radialis B. Genito-urinary system
C. A. axillaris C. Skeletal striated muscle tissue
D. A. subscapularis D. Cardiac striated muscle tissue
E. A. profunda brachii E. Fibrous connective tissue of skin
65. During the operation on the small 69. A smear of sputum from the pati-
intestine the surgeon revealed an area ent with suspected lobar pneumonia was
of the mucous membrane with a single stained with the use of the following
longitudinal fold among the circular folds. stains and reagents: solution of genti-
Which portion of the small intestine is this an violet, Lugol’s solution, 96o alcohol,
structure typical for? water magenta. What staining method was
applied in this case?
A. Pars descendens duodeni
B. Pars horizontalis duodeni A. Gram
C. Pars ascendens duodeni B. Ziehl-Nielsen
D. jejunum C. Romanovsky
E. Distal ileum D. Neisser
E. Leffler
66. 14 days after quinsy a 15-year-old chi-
ld presented with morning facial swelling, 70. A patient has normally coloured stool
high blood pressure, "meat slops"urine. including a large amount of free fatty aci-
Immunohistological study of a renal ds. The reason for this is a disturbance of
biopsy sample revealed deposition of the following process:
immune complexes on the basement
membranes of the capillaries and in the A. Fat absorption
glomerular mesangium. What disease B. Fat hydrolysis
developed in the patient? C. Biliary excretion
D. Choleresis
A. Acute glomerulonephritis E. Lipase secretion
B. Acute interstitial nephritis
C. Lipoid nephrosis 71. Examination of the removed stomach
D. Acute pyelonephritis revealed a deep roundish defect with
E. Necrotizing nephrosis regular edges at the lesser curvature of the
antrum. The defect reached the muscular
67. A diseased child has a high fever, sore tunic and was 1,5 cm in diameter. Within
throat, swelling of submandibular lymph the defect floor there was a translucent
nodes. Objectively: pharyngeal mucosa dense area resembling of a hyaline carti-
is edematous, moderately hyperemic, the lage. What process had developed in the
tonsils are enlarged, covered with grayish floor of the stomach defect?
membrane tightly adhering to the tissues
above. Attempts to remove the membrane A. Local hyalinosis
produce the bleeding defects. What di- B. Amyloidosis
sease are these presentations typical for? C. Mucoid swelling
D. Fibrinoid alterations
A. Diphtheria E. General hyalinosis
B. Catarrhal tonsillitis
C. Scarlet fever 72. By the decarboxylation of glutamate in
D. Meningococcal disease the CNS an inhibitory mediator is formed.
E. Measles Name it:

68. Study of the biopsy material of an A. GABA


embryo revealed a zone of developmental B. Glutathione
abnormality in a somite. The zone was C. Histamine
located close to the endoderm and D. Serotonin
the notochord. What formations may E. Asparagine
have abnormal development in case of
pregnancy continuation? 73. Thermometry revealed that the
temperature of the exposed skin is by 1-
1,5o lower than the temperature of the
adjacent areas covered with clothing from
natural fabrics. The reason for this is that
the clothes reduce the heat loss through:
Krok 1 Medicine 2013 9

A. Convection A. Insulin
B. Radiation B. Adrenaline
C. Conduction C. Isadrinum
D. Evaporation D. Glibenclamide
E. - E. Furosemide
74. A specimen of pia mater includes a 79. In order to stimulate breathing in a chi-
vessel whose wall doesn’t have the tuni- ld born with asphyxia, the doctor gave him
ca media, the tunica externa is adherent a drug injection into the umbilical vein.
to the surrounding tissues, the intima is What drug might have been injected?
composed of a basement membrane and
endothelium. What vessel is it? A. Aethimizolum
B. Corazolum
A. Nonmuscular vein C. Cordiaminum
B. Muscular vein with underdeveloped D. Sulfocamphocainum
muscular elements E. Coffeinum
C. Muscular artery
D. Arteriole 80. A patient complains of pain in the ri-
E. Artery of mixed type ght lateral abdomen. Palpation revealed a
dense, immobile, tumor-like formation. A
75. A patient with extensive burns of torso tumor is likely to be found in the followi-
skin exhibits signs of severe intoxication. ng part of the digestive tube:
What stage of the burn disease is this typi-
cal for? A. Colon ascendens
B. Colon transversum
A. Burn toxemia C. Colon descendens
B. Burn shock D. Colon sigmoideum
C. Burn infection E. Caecum
D. Burn emaciation
E. Terminal 81. A patient underwent biopsy of the
soft palate arches for a suspected tumor
76. As a result of a craniocerebral injury (macroscopy revealed an ulcer with a
a patient has a decreased skin sensitivity. dense floor). Study of the biopsy materi-
What area of the cerebral cortex may be al revealed mucosal necrosis with infi-
damaged? ltration of lymphocytes, epithelioid cells,
plasma cells, single neutrophils in the
A. Posterior central gyrus submucosa. There were also apparent si-
B. Occipital region gns of endovasculitis and perivasculitis.
C. Cingulate gyrus The described changes are typical for:
D. Frontal cortex
E. Anterior central gyrus A. Primary syphilis
B. Aphthous stomatitis
77. A histological specimen of the eyeball C. Ulcerative stomatitis
shows a biconvex structure connected to D. Necrotizing ulcerative Vincent stomati-
the ciliary body by the fibers of the Zinn’s tis
zonule and covered with a transparent E. Pharyngeal diphtheria
capsule. Name this structure:
82. Healthy parents with unremarkable
A. Crystalline lens family history have the child with multi-
B. Vitreous body ple developmental defects. Cytogenetic
C. Ciliary body analysis revealed the trisomy 13 in the
D. Cornea somatic cells (Patau syndrome). What
E. Sclera phenomenon has caused the defects?
78. A comatose patient was taken to the A. Abnormal gametogenesis
hospital. He has a history of diabetes B. Somatic mutation
mellitus. Objectively: Kussmaul breathi- C. Recessive mutation
ng, low blood pressure, acetone odor of D. Dominant mutation
breath. After the emergency treatment E. Chromosomal mutation
the patient’s condition improved. What
drug had been administered to the pati- 83. A specimen shows an organ covered
ent? with the connective tissue capsule with
Krok 1 Medicine 2013 10

trabeculae radiating inward the organ. A. Normal (incomplete process of ossifi-


There is also cortex containing some cation)
lymph nodules, and medullary cords made B. Fracture of the femoral neck
of lymphoid cells. What organ is under C. Fissured fracture of the femoral neck
study? D. Dislocation of the femoral head
E. Radiographic film artifact
A. Lymph node
B. Thymus 88. A surgeon examined the patient and
C. Spleen found the injury of the upper third of the
D. Red bone marrow kidney. Considering the syntopy of the
E. Tonsils left kidney, the intactness of the following
organ should be checked at the same time:
84. A 25-year-old patient consulted a
doctor about dysmenorrhea and infertili- A. Stomach
ty. Examination revealed that the patient B. Liver
was 145 cm high and had underdeveloped C. Small intestine
secondary sex characteristics, alar folds on D. Transverse colon
the neck. Cytological study didn’t reveal E. Descending colon
any Barr bodies in the somatic cells. What
diagnosis was made? 89. A patient with urolithiasis has
unbearable spasmodic pain. To prevent
A. Turner’s syndrome pain shock, he has been given an injection
B. Klinefelter syndrome of atropine along with a narcotic analgesic
C. Morris syndrome having antispasmodic effect. What drug
D. Trisomy X syndrome was it?
E. -
A. Promedol
85. To prevent attacks of acute pancreati- B. Nalorphine
tis a doctor prescribed the patient trasylol C. Tramadol
(contrycal, gordox), which is an inhibitor D. Ethylmorphine hydrochloride
of: E. Morphine hydrochloride
A. Trypsin 90. Despite the administration of cardi-
B. Elastase otonics and a thiazide diuretic a pati-
C. Carboxypeptidase ent with chronic heart failure has persi-
D. Chymotrypsin stent edemata, there is a risk of ascites.
E. Gastricsin What medication should be administered
in order to enhance the diuretic effect of
86. A patient died from progressive heart the drugs used?
failure. Autopsy revealed that the heart
was enlarged in diameter, flabby. The A. Spironolactone
muscle section exhibited irregular blood B. Furosemide
supply. Histological study of myocardium C. Amiloride
revealed hyperemia, the stroma was found D. Clopamide
to have lymphohistiocytic infiltrates wi- E. Manithol
th degeneration of cardiomyocytes. The
revealed morphological changes are indi- 91. A patient with a pathology of
cative of: the cardiovascular system developed
edemata of the lower extremities. What
A. Non-purulent interstitial myocarditis is the mechanism of cardiac edema
B. Venous plethora development?
C. Cardiomyoliposis
D. Cardiosclerosis A. Increased hydrostatic pressure at the
E. Myocardial infarction venous end of the capillary
B. Increased oncotic pressure
87. A 13-year-old teenager underwent X- C. Increased hydrostatic pressure at the
ray examination of the hip joint. Exami- arterial end of the capillary
nation revealed a 3 mm wide radiolucent D. Reduced osmotic pressure
zone between the head and the shaft of E. Lymph efflux disorder
femur. This situation should be evaluated
as: 92. During the fight, a man had a cardi-
ac arrest due to the strong blow to the
upper region of the anterior abdominal
Krok 1 Medicine 2013 11

wall. Which of the following mechanisms of renal corpuscles was caused by the
has led to the cardiac arrest? following allergic reaction:
A. Parasympathetic unconditioned reflexes A. Immune complex
B. Sympathetic unconditioned reflexes B. Anaphylactic
C. Parasympathetic conditioned reflexes C. Cytotoxic
D. Sympathetic conditioned reflexes D. Delayed
E. Peripheral reflexes E. Stimulating
93. A pregnant woman underwent AB0 98. An unconscious patient was taken by
blood typing. Red blood cells were ambulance to the hospital. On objecti-
agglutinated with standard sera of the I ve examination the patient was found
and II blood groups, and were not aggluti- to have no reflexes, periodical convulsi-
nated with the III group serum. What is ons, irregular breathing. After laboratory
the patient’s blood group? examination the patient was diagnosed
with hepatic coma. Disorders of the
A. B(III) central nervous system develop due to the
B. 0(I) accumulation of the following metabolite:
C. A(II)
D. AB(IV) A. Ammonia
E. - B. Urea
C. Glutamine
94. Amniocentesis revealed two sex D. Bilirubin
chromatin bodies (Barr bodies) in each E. Histamine
cell of the sample. What disease is this
character typical for? 99. A 20-year-old male patient complai-
ns of general weakness, rapid fatigabi-
A. Trisomy X lity, irritability, decreased performance,
B. Klinefelter syndrome bleeding gums, petechiae on the skin.
C. Turner’s syndrome What vitamin deficiency may be a cause
D. Down’s syndrome of these changes?
E. Patau syndrome
A. Ascorbic acid
95. A hospital has admitetd a patient B. Riboflavin
complaining of abdominal bloating, di- C. Thiamine
arrhea, flatulence after eating protein D. Retinol
foods. These signs are indicative of the E. Folic acid
impaired digestion of proteins and their
increased degradation. Which of the 100. It is known that the monoami-
following compounds is the product of this ne oxidase (MAO) enzyme plays an
process? important part in the metabolism of
catecholamine neurotransmitters. In what
A. Indole way does the enzyme inactivate these
B. Bilirubin neurotransmitters (norepinephrine, epi-
C. Cadaverine nephrine, dopamine)?
D. Agmatine
E. Putrescine A. Oxidative deamination
B. Addition of an amino group
96. An attack of tachycardia that occurred C. Removal of a methyl group
in a patient was stopped by pressing on his D. Carboxylation
eyeballs. Which of the following reflexes E. Hydrolysis
underlies this phenomenon?
101. The cellular composition of exudate
A. Aschner largely depends on the etiological factor
B. Goltz of inflammation. What leukocytes are the
C. Bainbridge first to get into the focus of inflammation
D. Hering caused by pyogenic bacteria?
E. Bernard’s
97. A male patient has been di-
agnosed with acute post-streptococcal
glomerulonephritis. It is most likely that
the lesion of the basement membrane
Krok 1 Medicine 2013 12

A. Neutrophil granulocytes
B. Monocytes A. Monotony of work
C. Myelocytes B. State of "operating rest"
D. Eosinophilic granulocytes C. Increased intellectual component
E. Basophils D. Increased responsibility
E. Social inefficiency of labor
102. At the end of the working day a
worker of a hot work shop has been deli- 106. Mother of a boy who had recently
vered to a hospital. The patient complains returned from a summer camp found
of a headache, dizziness, nausea, general some small whitish insects up to 3 mm
weakness. Objectively: the patient is long on the child’s clothing. Specify the
conscious, his skin is hyperemic, dry, hot to parasite:
the touch. Heart rate is of 130/min. Respi-
ration is rapid, superficial. What disorder A. Pediculus humanus humanus
of thermoregulation is most likely to have B. Phtirus pubis
occurred in this patient? C. Pulex irritans
D. Cimex lectularius
A. Reduced heat transfer E. Blattella germanica
B. Increased heat transfer and reduced
heat production 107. Histological examination of the
C. Increased heat transfer and heat removed skin neoplasm revealed clusters
production and cords of atypical cells of stratified
D. Increased heat production with no squamous epithelium, growing into the
changes to the heat transfer underlying tissue. What diagnosis can be
E. Reduced heat production with no assumed?
changes to the heat transfer
A. Non-keratinizing squamous cell carci-
103. Alveolar space of the acinus noma
was invaded by some bacteria which B. Keratinizing squamous cell carcinoma
interacted with the surfactant. This led C. Carcinoma in situ
to the activation of the cells that are D. Papilloma
localized in the alveolar walls and on the E. Adenoma
surface. What cells are these?
108. An 18-year-old male has been di-
A. Alveolar macrophages agnosed with Marfan syndrome. Exami-
B. Alveolocytes type I nation revealed a developmental di-
C. Endothelial cells sorder of connective tissue and eye lens
D. Clara cells structure, abnormalities of the cardi-
E. Alveolocytes type II ovascular system, arachnodactylia. What
genetic phenomenon has caused the
104. A 35-year-old male developed acute development of this disease?
heart failure while running for a long ti-
me. What changes in the ionic compositi- A. Pleiotropy
on can be observed in the cardiac muscle? B. Complementarity
C. Codominance
A. Accumulation of Na+ and Ca2+ ions in D. Multiple allelism
the myocardium cells E. Incomplete dominance
B. Accumulation of K + and Mg 2+ ions in
the myocardium cells 109. A patient has severe catarrhal
symptoms. Material growth on Bordet-
C. Reduction of Na+ and Ca2+ ions in the Gengou agar showed mercury-drop-
myocardium cells like colonies. Examination of the blood
D. Reduction of K + and Mg 2+ ions in the smears revealed some small ovoid gram-
extracellular space positive bacilli sized 1-3 microns. What mi-
E. Reduction of Na+ and Ca2+ ions in the croorganisms were isolated?
extracellular space
A. Bordetella
105. Workers of a conveyor workshop B. Corynebacteria
received recommendations for the effecti- C. Mycobacteria
ve organization of working time and hi- D. Meningococcus
gher working efficiency. What peculiari- E. Brucella
ty of work in this workshop causes the
greatest stress for the workers? 110. A 66-year-old patient with Parki-
Krok 1 Medicine 2013 13

nson’s disease shows an improvement in revealed a dense well-circumscribed node


locomotor activity after prolonged use of 6 cm in diameter in the cranial cavity.
of a certain drug which is converted to The node was attached to the dura mater
dopamine by the decarboxylation. What and histologically consisted of clusters and
drug has the patient taken? micro-concentric structures of endotheli-
al cells, psammoma bodies. What kind of
A. Levodopa tumor was found at autopsy?
B. Naloxone
C. Celecoxib A. Meningioma
D. Droperidol B. Glioblastoma
E. Chlorpromazine C. Medulloblastoma
D. Melanoma
111. Enzymatic jaundices are accompani- E. Cancer metastasis
ed by abnormal activity of UDP-
glucuronyl transferase. What compound 116. Inherited diseases, such as
is accumulated in blood serum in case of mucopolysaccharidoses, are manifested
these pathologies? in metabolic disorders of connective ti-
ssue, bone and joint pathologies. The si-
A. Unconjugated bilirubin gn of this disease is the excessive urinary
B. Conjugated bilrubin excretion of the following substance:
C. Dehydrobilirubin
D. Hydrobilirubin A. Glycosaminoglycans
E. Choleglobin B. Amino acids
C. Glucose
112. For the study of serum proteins D. Lipids
various physical and physicochemical E. Urea
methods can be used. In particular, serum
albumins and globulins can be separated 117. An animal has an increased tonus
by this method: of extensor muscles. This is the result of
enhanced information transmission to the
A. Electrophoresis motoneurons of the spinal cord through
B. Polarography the following descending pathways:
C. Dialysis
D. Spectrography A. Vestibulospinal
E. Refractometry B. Medial corticospinal
C. Reticulospinal
113. Negative environmental factors have D. Rubrospinal
caused the dysfunction of myosatellite E. Lateral corticospinal
cells. What function of the whole muscle
fibre is likely to be changed in this case? 118. A specimen of a parenchymal
organ shows poorly delineated hexagonal
A. Regeneration lobules surrounding a central vein, and
B. Contraction the interlobular connective tissue contai-
C. Trophism ns embedded triads (an 0artery, a vein and
D. Contractile thermogenesis an excretory duct). What organ is it?
E. Relaxation
A. Liver
114. The laboratory for especially B. Pancreas
dangerous infections conducts mi- C. Thymus
croscopic examination of pathological D. Spleen
material from a patient with suspected E. Thyroid
plague. The sample was stained by Burri-
Gins technique. What property of the 119. A patient has been admitted to
causative agent can be identified by this the infectious diseases department for
technique? malaise, fever up to 38o C, jaundice. A
few months ago, the patient underwent
A. Capsule formation blood transfusion. The doctor suspected
B. Spore formation viral hepatitis B. What are the principal
C. Acid resistance methods of laboratory diagnosis of hepati-
D. Alkali resistance tis B?
E. Presence of volutin granules
115. Autopsy of a 62-year-old woman
Krok 1 Medicine 2013 14

A. Serological and gene diagnostics A. Translation


B. Virus isolation in cell culture and its B. Transcription
identification by the cytopathic effects C. Replication
C. Detection of virions in blood by electron D. Processing
microscopy E. Splicing
D. Isolation of the virus in laboratory
animals (neutralization reaction) 124. Diseases of the respiratory system
E. Isolation of the virus in chicken embryos and circulatory disorders impair the
transport of oxygen, thus leading to
120. After resection of the middle thi- hypoxia. Under these conditions the
rd of the femoral artery obliterated by a energy metabolism is carried out by
thrombus the limb is supplied with blood anaerobic glycolysis. As a result, the
through the bypasses. What artery plays following substance is generated and
the main part in the restoration of the accumulated in blood:
blood flow?
A. Lactic acid
A. Deep femoral artery B. Pyruvic acid
B. Superficial iliac circumflex artery C. Glutamic acid
C. Descending genicular artery D. Citric acid
D. Superficial epigastric artery E. Fumaric acid
E. External pudendal artery
125. A patient has been hospitalized for a
121. During the intravenous transfusi- suspected tumor of the prostate. During
on of the saline the patient’s conditi- the surgery, it was revealed that the tumor
on deteriorated dramatically, and the invaded the bladder. Which part of the
patient died from asphyxiation. Autopsy bladder was affected?
revealed acute venous congestion of
internal organs with the dramatic ri- A. Cervix
ght heart dilatation. When the right B. Apex
ventricle was punctured underwater, C. Bottom
the bubbles escaped. What pathological D. Body
process occurred in the patient? E. -

A. Air embolism 126. A casualty with an injury of the


B. Gaseous embolism temporal region has been diagnosed with
C. Adipose embolism epidural hematoma. Which of the arteries
D. Tissue embolism is most likely to be damaged?
E. Thromboembolism
A. Medial membranous artery
122. At the post-mortem examination the B. Medial cerebral artery
stomach of a patient with renal failure was C. Superficial temporal artery
found to have a yellow-brown coating on D. Anterior membranous artery
the thickened mucosa. The coating was E. Posterior auricular artery
firmly adhering to its surface and had si-
127. A 19-year-old male was found to
gnificant thickness. Microscopy revealed
congestion and necrosis of mucosal and have an elevated level of potassium in
the secondary urine. These changes mi-
submucosal layers, fibrin presence. What
ght have been caused by the increase in
is the most likely diagnosis?
the following hormone level:
A. Fibrinous gastritis
B. Croupous gastritis A. Aldosterone
B. Oxytocin
C. Gastric abscess
D. Esogastritis C. Adrenaline
D. Glucagon
E. Corrosive gastritis
E. Testosterone
123. Infectious diseases are treated with
128. Analysis of the ECG revealed the
antibiotics (streptomycin, erythromycin,
missing of several PQRST cycles. The
chloramphenicol). They inhibit the remaining waves and complexes are not
following stage of protein synthesis: changed. Specify the type of arrhythmia:
Krok 1 Medicine 2013 15

A. Sinoatrial block A. Expiratory reserve volume and residual


B. Atrial fibrillation volume
C. Atrioventricular block B. Inspiratory reserve volume and residual
D. Atrial premature beat volume
E. Intra-atrial block C. Inspiratory reserve volume, tidal
volume, residual volume
129. ECG of a patient displays an D. Expiratory reserve volume and tidal
abnormally long R wave (up to 0,18 volume
s). This is caused by a decrease in the E. Inspiratory reserve volume and tidal
conduction velocity of the following heart volume
structures:
134. It is required to evaluate the level of
A. Ventricles tissue excitability. For this purpose one
B. Atria should determine:
C. Atrio-ventricular node
D. Right ventricle A. Depolarization threshold
E. Left ventricle B. Resting potential
C. Critical level of depolarization
130. 6 hours after the myocardial infarcti- D. Action potential amplitude
on a patient was found to have elevated E. Action potential duration
level of lactate dehydrogenase in blood.
What isoenzyme should be expected in 135. Due to the use of poor-quality
this case? measles vaccine for preventive vaccinati-
on, a 1-year-old child developed an autoi-
A. LDH1 mmune renal injury. The urine was found
B. LDH2 to contain macromolecular proteins. What
C. LDH3 process of urine formation was disturbed?
D. LDH4
E. LDH5 A. Filtration
B. Reabsorption
131. A 46-year-old female is scheduled C. Secretion
for a maxillofacial surgery. It is known D. Reabsorption and secretion
that the patient is prone to high blood E. Secretion and filtration
coagulation. What natural anticoagulant
can be used to prevent blood clotting? 136. A patient has been administered an
anti-inflammatory drug that blocks the
A. Heparin action of cyclooxygenase. Specify this
B. Hirudin anti-inflammatory agent:
C. Sodium citrate
D. Fibrinolysin A. Aspirin
E. None of the above-listed substances B. Analgene
C. Allopurinol
132. A 50-year-old patient with food poi- D. Thiamin
soning is on a drip of 10% glucose soluti- E. Creatine
on. It not only provides the body with
necessary energy, but also performs the 137. A pneumonia patient has been
function of detoxification by the producti- administered acetylcysteine as a part
on of a metabolite that participates in the of complex therapy. What principle of
following conjugation reaction: therapy was taken into consideration
when applying this drug?
A. Glucuronidation
B. Sulfation A. Pathogenetic
C. Methylation B. Symptomatic
D. Glycosylation C. Etiotropic
E. Hydroxylation D. Antimicrobial
E. Immunomodulatory
133. To assess the effectiveness of breathi-
ng in patients, the indicator of functional 138. A 26-year-old female patient with
residual capacity is used. It includes the bronchitis has been administered a broad
following volumes: spectrum antibiotic as a causal treatment
drug. Specify this drug:
Krok 1 Medicine 2013 16

A. Doxycycline 142. One of the factors that cause obesity


B. Interferon is the inhibition of fatty acids oxidation
C. BCG vaccine due to:
D. Ambroxol
E. Dexamethasone A. Low level of carnitine
B. Impaired phospholipid synthesis
139. A 65-year-old male suddenly lost C. Excessive consumption of fatty foods
the vision in one eye due to the reti- D. Choline deficiency
nal detachment. The patient underwent E. Lack of carbohydrates in the diet
enucleation. Histological examination
of the removed eye retina and choroid 143. The genetic defect of pyruvate
revealed clusters of atypical cells wi- carboxylase deficiency is the cause of
th marked polymorphism of cells and delayed physical and mental development
nuclei, with a moderate number of mi- and early death in children. This defect
toses including the pathological ones. The is characterized by lacticemia, lactaci-
cell cytoplasm and intercellular medi- duria, disorder of a number of metabolic
um contained brown pigment giving a pathways. In particular, the following
positive DOPA reaction. Perls’ reaction process is inhibited:
was negative. What is the most likely di-
agnosis? A. Citric acid cycle and gluconeogenesis
B. Glycolysis and glycogenolysis
A. Melanoma C. Glycogenesis and glycogenolysis
B. Pigmented mole D. Lipolysis and lipogenesis
C. Hemorrhage E. Pentose phosphate pathway and
D. Cysticercosis glycolysis
E. Wilson’s disease
144. Deficiency of linoleic and linolenic
140. A child cut his leg with a piece of acids in the body leads to the skin
glass while playing and was brought to the damage, hair loss, delayed wound heali-
clinic for the injection of tetanus toxoid. ng, thrombocytopenia, low resistance to
In order to prevent the development infections. These changes are most likely
of anaphylactic shock the serum was to be caused by the impaired synthesis of
administered by Bezredka method. What the following substances:
mechanism underlies this method of
desensitization of the body? A. Eicosanoids
B. Interleukins
A. Binding of IgE fixed to the mast cells C. Interferons
B. Blocking the mediator synthesis in the D. Catecholamines
mast cells E. Corticosteroids
C. Stimulation of immune tolerance to the
antigen 145. During ventricular systole, the cardi-
D. Stimulation of the synthesis of antigen- ac muscle does not respond to additional
specific IgG stimulation because it is in the phase of:
E. Binding of IgE receptors to the mast
cells A. Absolute refractoriness
B. Relational refractoriness
141. Microscopy of the myocardium of C. Hyperexcitability
a patient who had died from heart fai- D. Subnormal excitability
lure revealed foci of fibrinoid necrosis E. There is no correct answer
located diffusely in the interstitial stroma,
and often around the vessels. Such 146. A mother had taken synthetic
foci were surrounded by lymphocytes, hormones during pregnancy. Her
macrophages, histiocytes. Pericardium daughter was born with hirsutism formally
was found to have signs of sero-fibrinous resembling of adrenal syndrome. Such
pericarditis. What is the most likely di- manifestation of variability is called:
agnosis? A. Phenocopy
A. Rheumatic heart disease B. Mutation
B. Myocardial infarction C. Recombination
C. Cardiomyopathy D. Heterosis
D. Cardiosclerosis E. Replication
E. - 147. Since a patient has had myocardial
Krok 1 Medicine 2013 17

infarction, atria and ventricles contract A. Corti’s organ - closer to helicotrema


independently from each other with a B. Corti’s organ - closer to the oval
frequency of 60-70 and 35-40 per minute. foramen
Specify the type of heart block in this case: C. Median part of the Corti’s organ
D. Muscles of the middle ear
A. Complete atrioventricular E. Eardrum
B. Partial atrioventricular
C. Sino-atrial 152. In our country, routine preventi-
D. Intra-atrial ve vaccinations against poliomyelitis
E. Intraventricular ivolve using live vaccine that is admini-
stered orally. What immunoglobulins are
148. A 67-year-old male patient consumes responsible for the development of local
eggs, pork fat, butter, milk and meat. post-vaccination immunity in this case?
Blood test results: cholesterol - 12,3
mmol/l, total lipids - 8,2 g/l, increased low- A. Secretory IgA
density lipoprotein fraction (LDL). What B. IgM
type of hyperlipoproteinemia is observed C. IgG
in the patient? D. Serum IgA
E. IgE
A. Hyperlipoproteinemia type IIa
B. Hyperlipoproteinemia type I 153. An experiment proved that UV-
C. Hyperlipoproteinemia type IIb irradiated skin cells of patients with
D. Hyperlipoproteinemia type IV xeroderma pigmentosum restore the nati-
E. Cholesterol, hyperlipoproteinemia ve structure of DNA slower than the cells
of healthy people due to the defect in
149. A 12-year-old child has a viral infecti- repair enzyme. What enzyme takes part
on complicated by obstructive bronchi- in this process?
tis. Bronchospasm can be eliminated by
inhalations of a drug from the following A. Endonuclease
pharmacological group: B. RNA ligase
C. Primase
A. β2 -agonists D. DNA polymerase
B. M-anticholinergics E. DNA gyrase
C. N-cholinomimetics
D. β2 -adrenergic blockers 154. A patient who has recently come
E. Analeptics from an endemic area presents with
elevated body temperature, headache,
150. In course of an experiment there has chills, malaise, that is with the symptoms
been an increase in the nerve conducti- which are typical for a common cold.
on velocity. This may be caused by an What laboratory tests are necessary to
increase in the concentration of the confirm or to refute the diagnosis of
following ions that are present in the malaria?
solution around the cell:
A. Microscopy of blood smears
A. Na+ B. Study of lymph node punctate
B. K + and Cl− C. Urinalysis
C. K + and Na+ D. Study of cerebrospinal fluid
D. Ca2+ and Cl− E. Microscopy of bone marrow punctate
E. Ca2+
155. What condition may develop 15-30
151. A male working as a blacksmith has minutes after re-administration of the
been tested for auditory acuity. The tests antigen as a result of the increased level of
revealed 50% hearing loss in the low- antibodies, mainly IgE, that are adsorbed
frequency range and a near-normal audi- on the surface of target cells, namely ti-
tory acuity in the high-frequency range. ssue basophils (mast cells) and blood
This condition has been caused by the basophils?
damage to the following structures of the
auditory system: A. Anaphylaxis
B. Antibody-dependent cytotoxicity
C. Delayed-type hypersensitivity
D. Immune complex hyperresponsiveness
E. Serum sickness
Krok 1 Medicine 2013 18

156. 10 days after having quinsy caused cemia accompanied by the formation of
by beta-hemolytic streptococcus a 6- tophi, urate calculi in the urinary tracts,
year-old child exhibited symptoms of as well as serious neuro-psychiatric di-
glomerulonephritis. What mechanism of sorders. The cause of this disease is the
glomerular lesion is most likely in this reduced activity of the following enzyme:
case?
A. Hypoxanthine-guanine phosphori-
A. Immunocomplex bosyltransferase
B. Cellular cytotoxicity B. Xanthine oxidase
C. Anaphylaxis C. Dihydrofolate reductase
D. Atopy D. Thymidylate synthase
E. Antibody-dependent cell-mediated E. Karbamoyl phosphate synthetase
cytolysis
161. In a car accident a man got injured
157. A 22-year-old woman ate some and lost a lot of blood. What changes in
seafood. 5 hours later the trunk and the peripheral blood are most likely to occur
distal parts of limbs got covered with on the 2nd day after the injury?
small itchy papules which were partially
fused together. After one day, the rash A. Erythropenia
disappeared spontaneously. Specify the B. Hypochromia
hypersensitivity mechanism underlying C. Anisocytosis
these changes: D. Microplania
E. Significant reticulocytosis
A. Atopy (local anaphylaxis)
B. Systemic anaphylaxis 162. In the surgical ward, the dressi-
C. Cellular cytotoxicity ng material was undergoing sterilizati-
D. Immune complex hypersensitivity on in an autoclave. Through an oversi-
E. Antibody-dependent cell-mediated ght of a nurse the mode of sterilization
cytolysis was changed and the temperature in the
autoclave reached only 100oC instead of
158. A hypertensive patient had been the due 120OC. What microorganisms can
keeping to a salt-free diet and taki- stay viable under these conditions?
ng antihypertensive drugs together with
hydrochlorothiazide for a long time. This A. Bacilli and clostridia
resulted in electrolyte imbalance. What B. Staphylococci and streptococci
disorder of the internal environment C. Mold and yeast fungi
occurred in the patient? D. Salmonella and klebsiella
E. Corynebacteria and mycobacteria
A. Hypochloremic alkalosis
B. Metabolic acidosis 163. As a result of a mechanical injury
C. Hyperkalemia an over 10 cm long portion of a peri-
D. Hypermagnesemia pheral nerve was damaged. This led to the
E. Increase in circulating blood volume impairment of the upper limb activity. The
patient was offered nerve transplantati-
159. A miner consulted a physician about on. What glial cells will participate in
the appearance of body rash followed by regeneration and provide the trophism of
a loss of appetite, bloating, duodenal pain, the injured limb?
frequent bowel movements, dizziness.
Ovoscopic probes of feces and duodenal A. Schwann cells
contents revealed some eggs covered with B. Fibrous cells
a transparent membrane through which C. Protoplasmic cells
4-8 germinal cells could be seen. What D. Microglia
disease is likely to have occurred in the E. Ependymal cells
patient?
164. A 26-year-old woman at 40 weeks
A. Ancylostomiasis pregnant has been delivered to the
B. Strongyloidiasis maternity ward. Objectively: the uterine
C. Trichocephaliasis cervix is opened, but the contractions are
D. Hymenolepiasis absent. The doctor has administered her
E. Enterobiasis a hormonal drug to stimulate the labor.
Name this drug:
160. Children with Lesch-Nyhan
syndrome have a severe form of hyperuri-
Krok 1 Medicine 2013 19

A. Oxytocin contents revealed some pear-shaped


B. Hydrocortisone protozoa with two nuclei and four pai-
C. Estrone rs of flagella. The organisms had also
D. Testosterone two axostyles between the nuclei and a
E. ACTH ventral adhesive disc. What protozoan
representative was found in the patient?
165. A patient has recurrent attacks of
epileptic seizures and stays unconscious A. Lamblia
between them. In order to stop convulsi- B. Toxoplasma
ons the drugs of the following group C. Leishmania
should be used in the first place: D. Intestinal trichomonad
E. Trypanosome
A. Tranquilizers
B. Neuroleptics 170. A specimen of an onion rootlet
C. Muscle relaxants includes a cell in which the fully
D. Sedatives condensed chromosomes are located in
E. Analeptics the equatorial plane making the monaster.
What phase of the mitotic cycle is the cell
166. A patient with arthritis and varicose in?
veins has been taking a non-steroidal anti-
inflammatory drug for a long time, which A. Metaphase
caused thrombosis of skin veins. Which B. Early telophase
of the following drugs might have caused C. Prophase
this complication? D. Interphase
E. Late telophase
A. Celecoxib
B. Indomethacin 171. When examining a patient, the doctor
C. Aspirin revealed a tumor of the bronchus which
D. Phenylbutazone borders on the aorta. Which bronchus is
E. Ibuprofen affected?
167. Students study the stages of A. Left principal
gametogenesis. They analyze a cell havi- B. Right principal
ng a haploid number of chromosomes, C. Right upper lobar
and each chromosome consists of two D. Left upper lobar
chromatids. The chromosomes are located E. Middle lobar
in the equatorial plane of the cell. Such si-
tuation is typical for the following stage of 172. A 54-year-old female was brought to
meiosis: the casualty department after a car acci-
dent. A traumatologist diagnosed her with
A. Metaphase of the second division multiple fractures of the lower extremiti-
B. Metaphase of the first division es. What kind of embolism is most likely
C. Anaphase of the first division to develop in this case?
D. Anaphase of the second division
E. Prophase of the first division A. Adipose
B. Tissue
168. A 35-year-old female patient C. Thromboembolism
underwent biopsy of the breast nodules. D. Gaseous
Histological examination revealed E. Air
enhanced proliferation of the small duct
epithelial cells and acini, accompanied 173. Microscopy of the bronchial
by the formation of glandular structures wall revealed atrophy of the mucosa,
of various shapes and sizes, which were metaplastic change from columnar to
located in the fibrous stroma. What is the squamous epithelium, an increase in
most likely diagnosis? the number of goblet cells, diffuse
infiltration of the bronchial wall wi-
A. Fibroadenoma th lymphoplasmacytic elements wi-
B. Adenocarcinoma th a large number of neutrophilic
C. Cystic breast granulocytes, pronounced sclerosis. Spesi-
D. Invasive ductal carcinoma fy the morphological form of bronchitis:
E. Mastitis
169. Examination of the duodenal
Krok 1 Medicine 2013 20

A. Chronic purulent bronchitis A. Intermittent administration of allergen


B. Acute bronchitis B. Antihistamines
C. Polypoid chronic bronchitis C. Glucocorticoids
D. Acute purulent bronchitis D. Administration of saline
E. Chronic bronchitis E. -
174. Due to the blockage of the common 179. A patient consulted a physici-
bile duct (which was radiographically an about chest pain, cough, fever.
confirmed), the biliary flow to the Roentgenography of lungs revealed eosi-
duodenum was stopped. We should expect nophilic infiltrates which were found to
the impairment of: contain the larvae. What kind of helmi-
nthiasis are these presentations typical
A. Fat emulsification for?
B. Protein absorption
C. Carbohydrate hydrolysis A. Ascariasis
D. Secretion of hydrochloric acid B. Echinococcosis
E. Salivation inhibition C. Fascioliasis
D. Cysticercosis
175. Typical manifestations of food poi- E. Trichinosis
soning caused by C. botulinum are
double vision, abnormal functioning of 180. A patient with signs of osteoporosis
the swallowing and breathing. These and urolithiasis has been admitted
symptoms develop as a result of: to the endocrinology department.
Blood test revealed hypercalcemia and
A. Exotoxin effects hypophosphatemia. These changes are
B. Enterotoxin effects associated with abnormal synthesis of the
C. Enterotoxic shock development following hormone:
D. Activation of adenylate cyclase
E. Pathogen adhesion to the enterocyte A. Parathyroid hormone
receptors B. Calcitonin
C. Cortisol
176. At the stage of translation in the D. Aldosterone
rough endoplasmic reticulum, the ri- E. Calcitriol
bosome moves along the mRNA. Amino
acids are joined together by peptide bonds 181. Histological examination of the bi-
in a specific sequence, and thus polypepti- opsy material obtained from the lower
de synthesis takes place. The sequence of third of the esophagus of a 57-year-old
amino acids in a polypeptide corresponds male with the symptoms of continuous
to the sequence of: reflux revealed the change of the strati-
fied squamous epithelium to the single-
A. mRNA codons layer columnar glandular epithelium wi-
B. tRNA nucleotides th signs of mucus production. Specify
C. tRNA anticodons the pathological process in the mucous
D. rRNA nucleotides membrane:
E. rRNA anticodons
A. Metaplasia
177. After the prolonged vomiting a B. Hyperplasia
pregnant 26-year-old woman was found C. Hypertrophy
to have the reduced volume of circulati- D. Organization
ng blood. What change in the total blood E. Regeneration
volume can be the case?
182. A 30-year-old female exhibits signs
A. Polycythemic hypovolemia of virilism (growth of body hair, balding
B. Simple hypovolemia temples, menstrual disorders). This condi-
C. Oligocythemic hypovolemia tion can be caused by the overproduction
D. Polycythemic hypervolemia of the following hormone:
E. Oligocythemic hypervolemia
A. Testosterone
178. On allergological examination a pati- B. Oestriol
ent has been diagnosed with pollinosis. C. Relaxin
Specific desensitization can be performed D. Oxytocin
by: E. Prolactin
Krok 1 Medicine 2013 21

183. A patient with bacterial periodontitis 188. As a result of a home injury, a patient
has been administered iontophoresis wi- suffered a significant blood loss, which led
th the use of iodine solution. Specify the to a fall in blood pressure. Rapid blood
mechanism of therapeutic action of this pressure recovery after the blood loss is
agent: provided by the following hormones:
A. Substitution of hydrogen atoms when A. Adrenaline, vasopressin
the protein amino group contains a ni- B. Cortisol
trogen atom C. Sex hormones
B. Reduction of the nitro group under the D. Oxytocin
effect of nitroreductase E. Aldosterone
C. Albumin formation
D. Changing the surface tension of the 189. A patient with constant headaches,
bacterial cell membrane pain in the occipital region, tinnitus, dizzi-
E. Inhibition of the cell wall formation ness has been admitted to the cardiology
department. Objectively: AP- 180/110 mm
184. A patient with extensive myocardi- Hg, heart rate - 95/min. Radiographically,
al infarction has developed heart failure. there is a stenosis of one of the renal
What pathogenetic mechanism contri- arteries. Hypertensive condition in this
buted to the development of heart failure patient has been caused by the activation
in the patient? of the following system:
A. Reduction in the mass of functioning A. Renin-angiotensin
myocardiocytes B. Hemostatic
B. Pressure overload C. Sympathoadrenal
C. Volume overload D. Kinin
D. Acute cardiac tamponade E. Immune
E. Myocardial reperfusion injury
190. A patient complains that at the
185. A patient who had been continuously bare mention of the tragic events that
taking drugs blocking the production of once occurred in his life he experiences
angiotensin II developed bradycardia and tachycardia, dyspnea and an abrupt rise
arrhythmia. A likely cause of these di- in blood pressure. What structures of the
sorders is: CNS are responsible for these cardiorespi-
ratory reactions in this patient?
A. Hyperkalemia
B. Hypokalemia A. Cerebral cortex
C. Hypernatremia B. Cerebellum
D. Hypocalcemia C. Lateral hypothalamic nuclei
E. Hypercalcemia D. Specific thalamic nuclei
E. Quadrigemina of mesencephalon
186. A patient has arterial hypertension.
What long-acting drug from the group 191. A patient consulted a dentist about
of calcium channel blockers should be limited mouth opening (trismus). He has
prescribed? a history of a stab wound of the lower
extremity. What infection may cause these
A. Amlodipine symptoms?
B. Octadine
C. Pyrroxanum A. Tetanus
D. Atenolol B. Brucellosis
E. Reserpine C. Whooping cough
D. Wound anaerobic infection
187. Human skin has a high breaking E. Tularemia
strength. It is known that the skin consi-
sts of epithelial tissue and two kinds of 192. Anatomical dead space is the portion
connective tissue. Which of the following of the air that is left in the airways after
tissues provides the skin strength? expiration. The reduction of the anatomi-
cal dead space is typical for the following
A. Unformed dense connective tissue situation:
B. Stratified squamous epithelium
C. Loose connective tissue
D. Single-layer epithelium
E. Transitional epithelium
Krok 1 Medicine 2013 22

A. Tracheostomy A. Microangiopathy
B. Forward flexion of head B. Macroangiopathy
C. Turning the lying patient on his left side C. Atherosclerosis
D. Turning the lying patient on his right D. Neuropathy
side E. Glomerulopathy
E. Breathing through the mouth
197. Analysis of the family history of
193. Analysis of the experimental spi- children with Van der Woude syndrome
rogram of a 55-year-old person revealed revealed that in their families one of the
a decrease in tidal volume and respiratory parents had the typical for this syndrome
amplitude compared to the situation of defects (cleft lip and palate, lip pits
ten years ago. The change in these indi- regardless of gender). What is the type
cators is caused by: of inheritance of this syndrome?
A. Decreased force of respiratory muscle A. Autosomal dominant
contraction B. X-linked recessive
B. Gas composition of the air C. X-linked dominant
C. Physical build of a person D. Autosomal recessive
D. Height of a person E. Multifactorial
E. Body mass of a person
198. Administration of doxycycline
194. A patient underwent a course of hydrochloride caused an imbalance of the
treatment for atherosclerosis. Laboratory symbiotic intestinal microflora. Specify
tests revealed an increase in the anti- the kind of imbalance caused by the anti-
atherogenic lipoprotein fraction in the biotic therapy:
blood plasma. The treatment efficacy is
confirmed by the increase in: A. Dysbacteriosis
B. Sensibilization
A. HDL C. Idiosyncrasy
B. VLDL D. Superimposed infection
C. IDL E. Bacteriosis
D. LDL
E. Chylomicrons 199. A 3-year-old child had eaten some
strawberries. Soon he developed a rash
195. A 65-year-old female patient has and itching. What was found in the child’s
chronic constipations due to the colon leukogram?
hypotonia. What drug should be chosen
in this case? A. Eosinophilia
B. Hypolymphemia
A. Bisacodyl C. Neutrophilic leukocytosis
B. Castor oil D. Monocytosis
C. Magnesium sulfate E. Lymphocytosis
D. Neostigmine methylsulfate
E. Metoclopramide 200. A 12-year-old patient was found to
have blood serum cholesterol at the rate
196. A female patient complains of of 25 mmol/l. The boy has a history of
vision impairment. On examination hereditary familial hypercholesterolemia,
she was found to have obesity, fasting which is caused by the impaired synthesis
hyperglycemia. What complication of di- of the following protein receptors:
abetes can cause vision impairment?
A. Low density lipoproteins
B. High density lipoproteins
C. Chylomicrons
D. Very low density lipoproteins
E. Intermediate density lipoproteins
Krok 1 Medicine 2014 1

1. The infectious diseases department A. Unconditioned sympathetic reflexes


of a hospital admitted a patient wi- B. Unconditioned parasympathetic
th nausea, liquid stool with mucus reflexes
and blood streaks, fever, weakness. C. Conditioned sympathetic reflexes
Dysentery was suspected. What D. Conditioned parasympathetic
method of laboratory diagnostics reflexes
should be applied to confirm the di- E. -
agnosis?
6. A 53-year-old male patient is di-
A. Bacteriological agnosed with Paget’s disease. The
B. Serological concentration of oxyproline in daily
C. Mycological urine is sharply increased, which pri-
D. Microscopic marily means intensified disintegration
E. Protozoological of:
2. Malaria is treated with structural A. Collagen
analogs of vitamin B2 (riboflavin). B. Keratin
These drugs disrupt the synthesis of C. Albumin
the following enzymes in plasmodium: D. Hemoglobin
E. Fibrinogen
A. FAD-dependent dehydrogenase
B. Cytochrome oxidase 7. A patient has hoarseness of voi-
C. Peptidase ce. During laryngoscopy a gray-white
D. NAD-dependent dehydrogenase larynx tumor with papillary surface has
E. Aminotransferase been detected. Microscopic investigati-
on has shown the following: growth of
3. A 6-year-old child with suspected connective tissue covered with multi-
active tuberculous process underwent layer, strongly keratinized pavement
the diagnostic Mantoux test. What epithelium, no cellular atypia. What is
immunobiological preparation was the most likely diagnosis?
injected?
A. Papilloma
A. Tuberculin B. Fibroma
B. BCG vaccine C. Polyp
C. DTP vaccine D. Angioma
D. Tularinum E. Angiofibroma
E. Td vaccine
8. During autopsy approximately 2,0
4. In a young man during exercise, the liters of pus have been found in the
minute oxygen uptake and carbon di- abdominal cavity of the corpse. Peri-
oxide emission equalled to 1000 ml. toneum is lustreless and has grayish
What substrates are oxidized in the shade, serous tunic of intestines has
cells of his body? grayish-colored coating that is easily
removable. Specify the most likely type
A. Carbohydrates of peritonitis in the patient:
B. Proteins
C. Fats A. Fibrinopurulent peritonitis
D. Carbohydrates and fats B. Hemorrhagic peritonitis
E. Carbohydrates and proteins C. Serous peritonitis
D. Tuberculous peritonitis
5. A sportsman spontaneously held E. -
breath for 40 seconds, which resulted in
an increase in heart rate and systemic 9. Autopsy of a dead patient
arterial pressure. Changes of these revealed bone marrow hyperplasia
indicators are due to activation of the of tubular and flat bones (pyoid
following regulatory mechanisms: marrow), splenomegaly (6 kg) and
hepatomegaly (5 kg), enlargement of
all lymph node groups. What disease
Krok 1 Medicine 2014 2

are the identified changes typical for? of a pesticide components is sodium


arsenate that blocks lipoic acid. Whi-
A. Chronic myelogenous leukemia ch enzyme activity is impaired by this
B. Chronic lymphocytic leukemia pesticide?
C. Multiple myeloma
D. Polycythemia vera A. Pyruvate dehydrogenase complex
E. Hodgkin’s disease B. Microsomal oxidation
C. Methemoglobin reductase
10. Autopsy of the dead patient who D. Glutathione peroxidase
died from pulmonary edema revealed E. Glutathione reductase
a large yellow-grey nidus in the
myocardium, and a fresh thrombus in 15. Stool culture test revealed in a 6-
the coronary artery. What is the most month-old bottle-fed baby the strain of
likely diagnosis? intestinal rod-shaped bacteria of anti-
gen structure 0-111. What diagnosis
A. Myocardial infarction can be made?
B. Cardiosclerosis
C. Myocarditis A. Colienteritis
D. Amyloidosis B. Gastroenteritis
E. Cardiomyopathy C. Choleriform disease
D. Food poisoning
11. An animal experiment is aimed at E. Dysentery-like disease
studying the cardiac cycle. All the heart
valves are closed. What phase of the 16. A boy referred to a genetics clinic
cycle is characterized by this status? was found to have 1 drumstick in blood
neutrophils. The boy is likely to have
A. Isometric contraction the following syndrome:
B. Asynchronous contraction
C. Protodiastolic period A. Klinefelter’s
D. Rapid filling B. Down’s
E. Reduced filling C. Turner’s
D. Edwards’
12. Cyanide is a poison that causes E. Trisomy X
instant death of the organism. What
enzymes found in mitochondria are 17. A drycleaner’s worker has been
affected by cyanide? found to have hepatic steatosis. This
pathology can be caused by the di-
A. Cytochrome oxidase (aa3) sruption of synthesis of the following
B. Flavin enzymes substance:
C. Cytochrome 5
D. NAD+-dependent dehydrogenase A. Phosphatidylcholine
E. Cytochrome P-450 B. Tristearin
C. Urea
13. Increased HDL levels decrease the D. Phosphatidic acid
risk of atherosclerosis. What is the E. Cholic acid
mechanism of HDL anti-atherogenic
action? 18. Ascarid eggs have been detected
during stool analysis. What drug
A. They remove cholesterol from ti- should be prescribed?
ssues
B. They supply tissues with cholesterol A. Mebendazole
C. They are involved in the breakdown B. Nystatin
of cholesterol C. Chloramphenicol
D. They activate the conversion of D. Tetracycline
cholesterol to bile acids E. Furazolidone
E. They promote absorption of
cholesterol in the intestine 19. Bacteriological examination of the
urine of the patient with pyelonephri-
14. It has been found out that one tis revealed microorganisms that
Krok 1 Medicine 2014 3

produced yellow-green pigment and ng nuclei in form of a horseshoe. In


a characteristic odor in meat-peptone the center of some cell clusters there
agar. What are they called? were non-structured light-pink areas
with fragments of nuclei. What disease
A. Pseudomonas are these changes typical for?
B. Escherichia
C. Proteas A. Tuberculosis
D. Klebsiella B. Hodgkin’s disease
E. Azotobacter C. Actinomycosis
D. Tumor metastasis
20. Feces of a patient contain high E. Syphilis
amount of undissociated fats and have
grayish-white color. Specify the cause 24. A biochemical urine analysis has
of this phenomenon: been performed for a patient with
progressive muscular dystrophy. In
A. Obturation of bile duct the given case muscle disease can be
B. Hypoactivation of pepsin by confirmed by the high content of the
hydrochloric acid following substance in urine:
C. Hypovitaminosis
D. Enteritis A. Creatine
E. Irritation of intestinal epithelium B. Porphyrin
C. Urea
21. A 46-year-old female patient D. Hippuric acid
consulted a doctor about pain in the E. Creatinine
small joints of the upper and lower
limbs. The joints are enlarged and 25. While examining foot blood supply
shaped like thickened nodes. Serum a doctor checks the pulsation of a
test revealed an increase in urate large artery running in the separate
concentration. This might be caused fibrous channel in front of articulatio
by a disorder in metabolism of: talocruralis between the tendons of
long extensor muscles of hallux and
A. Purines toes. What artery is it?
B. Carbohydrates
C. Lipids A. A. dorsalis pedis
D. Pyrimidines B. A. tibialis anterior
E. Amino acids C. A. tarsea medialis
D. A. tarsea lateralis
22. A 26-year-old female consulted a E. A. fibularis
doctor about having stool with whi-
te flat moving organisms resembli- 26. A patient with chronic heart fai-
ng noodles. Laboratory analysis lure with edema has increased level
revealed proglottids with the followi- of blood aldosterone. What diuretic
ng characteristics: long, narrow, with a would be most effective in this case?
longitudinal channel of the uterus wi-
th 17-35 lateral branches on each side. A. Spironolactone
What kind of intestinal parasite was B. Triamterene
found? C. Acetazolamide
D. Hydrochlorothiazide
A. Taeniarhynchus saginatus E. Furosemide
B. Taenia solium
C. Hymenolepis nana 27. During autopsy the following
D. Diphyllobothrium latum has been revealed: the meninges
E. Echinococcus granulosus of the upper cerebral hemispheres
are extremely plethoric, of yellow-
23. A male patient is 28 years old. Hi- green color and are soaked with
stological study of a cervical lymph purulent effluent. What kind of meni-
node revealed a change of its pattern ngitis is characterised by such clinical
due to the proliferation of epithelioid, presentations?
lymphoid cells and macrophages havi-
Krok 1 Medicine 2014 4

A. Meningococcal meningitis has got acute pain, redness, swelling


B. Tuberculous meningitis of her right index finger. A few mi-
C. Grippal meningitis nutes later, there appeared a blister fi-
D. Anthrax-induced lled with a transparent liquid of straw-
E. Epidemic typhus-induced yellow color. The described changes
are a manifestation of the following
28. A 41-year-old male patient pathological process:
has a history of recurrent attacks
of heartbeats (paroxysms), profuse A. Exudative inflammation
sweating, headaches. Examination B. Traumatic edema
revealed hypertension, hyperglycemia, C. Alternative inflammation
increased basal metabolic rate, and D. Proliferative inflammation
tachycardia. These clinical presentati- E. Vacuolar degeneration
ons are typical for the following
adrenal pathology: 33. A 42-year-old male with a lesion of
the ulnar nerve is unable to flex the
A. Hyperfunction of the medulla II and V fingers to the midline. Whi-
B. Hypofunction of the medulla ch muscle function is impaired in this
C. Hyperfunction of the adrenal cortex case?
D. Hypofunction of the adrenal cortex
E. Primary aldosteronism A. Palmar interosseous muscles
B. Dorsal interosseous muscle
29. A comminuted fracture of C. Fidicinales
infraglenoid tubercle caused by D. Short palmar muscle
shoulder joint injury has been detected E. Abductor muscle of little finger
during X-ray examination of a patient.
What muscle tendon attached at this 34. A smear from the tonsillar coating
site has been damaged? of a patient with suspected diphtheria
was found to contain blue bacilli with a
A. Long head of m. triceps brachii thickening at the poles. What method
B. Long head of m. biceps brachii of smear staining was used?
C. Medial head of m. triceps brachii
D. Lateral head of m. triceps brachii A. Leffler
E. Short head of m. biceps brachii B. Burri
C. Hins
30. A patient has increased thickness D. Gram
of alveolar-capillary membrane caused E. Neisser
by a pathologic process. The direct
consequence will be reduction of the 35. A child was born asphyxiated.
following value: What drug must be administered to the
newborn to stimulate breathing?
A. Diffusing lung capacity
B. Oxygen capacity of blood A. Aethimizolum
C. Respiratory minute volume B. Lobeline
D. Alveolar ventilation of lungs C. Prazosin
E. Expiratory reserve volume D. Atropine
E. Proserine
31. What drug will be most appropriate
for the patient who has chronic gastri- 36. A 40-year-old patient with the
tis with increased secretion? progressing staphylococcal purulent
periodontitis developed purulent
A. Pirenzepine inflammation of bone marrow spaces
B. Pancreatine of the alveolar process, and then of
C. Pepsin the body of mandible. Microscopy
D. Aprotinin revealed thinning of bone trabeculae,
E. Chlorphentermine foci of necrosis, bone sequesters
surrounded by the connective tissue
32. As a result of careless handling of capsule. What is the most likely di-
an iron, a 34-year-old female patient agnosis?
Krok 1 Medicine 2014 5

A. Immune-enzyme analysis
A. Chronic osteomyelitis B. Radioimmunoassay technique
B. Acute osteomyelitis C. Immunofluorescence test
C. Parodontome D. Bordet-Gengou test
D. Chronic fibrous periostitis E. Antigen-binding assay
E. Purulent abscess
41. A patient has been hospitalised wi-
37. Human red blood cells do not th provisional diagnosis of botulism.
contain mitochondria. What is the What serological reaction should be
main pathway for ATP production in used to reveal botulinum toxin?
these cells?
A. Neutralization reaction
A. Anaerobic glycolysis B. Agglutination reaction
B. Aerobic glycolysis C. Bordet-Gengou test
C. Oxidative phosphorylation D. Precipitation reaction
D. Creatine kinase reaction E. Immunofluorescence test
E. Cyclase reaction
42. Examination of a 52-year-old
38. A 40-year-old female patient has female patient has revealed a decrease
undergone thyroidectomy. Histological in the amount of red blood cells and
study of thyroid gland found the folli- an increase in free hemoglobin in
cles to be of different size and contain the blood plasma (hemoglobinemia).
foamy colloid, follicle epithelium is hi- Color index is 0,85. What type of
gh and forms papillae, there is focal anemia is being observed in the pati-
lymphocytic infiltration in stroma. Di- ent?
agnose the thyroid gland disease:
A. Acquired hemolytic
A. Basedow’s disease B. Hereditary hemolytic
B. Hashimoto’s thyroiditis C. Acute hemorrhagic
C. Riedel’s thyroiditis D. Chronic hemorrhagic
D. De Quervain’s disease E. Anemia due to diminished
E. Nodular goiter erythropoiesis
39. A 43-year-old-patient has arteri- 43. The processes of heat transfer in
al hypertension caused by an increase a naked person at room temperature
in cardiac output and general peri- have been studied. It was revealed that
pheral resistance. Specify the variant of under these conditions the greatest
hemodynamic development of arterial amount of heat is transferred by:
hypertension in the given case:
A. Heat radiation
A. Eukinetic B. Heat conduction
B. Hyperkinetic C. Convection
C. Hypokinetic D. Evaporation
D. Combined E. -
E. -
44. Urine analysis has shown high
40. A patient has been hospitalised levels of protein and erythrocytes
with provisional diagnosis of virus B in urine. This can be caused by the
hepatitis. Serological reaction based following:
on complementation of antigen with
antibody chemically bound to peroxi- A. Renal filter permeability
dase or alkaline phosphatase has been B. Effective filter pressure
used for disease diagnostics. What is C. Hydrostatic blood pressure in
the name of the applied serological glomerular capillaries
reaction? D. Hydrostatic primary urine pressure
in capsule
E. Oncotic pressure of blood plasma
45. The development of both immune
Krok 1 Medicine 2014 6

and allergic reactions is based upon colored. Tonsillar surface is covered


the same mechanisms of the immune with isolated grayish-colored necrosis
system response to an antigen. What nidi. What disease is it?
is the main difference between the
immune and allergic reactions? A. Scarlet fever
B. Meningococcal nasopharyngitis
A. Development of tissue lesion C. Diphtheria
B. Amount of released antigen D. Influenza
C. Antigen structure E. Measles
D. Routes by which antigens are deli-
vered into the body 50. In a patient elevation of body
E. Hereditary predisposition temperature takes turns with drops
down to normal levels during the day.
46. After a craniocerebral injury a pati- The rise in temperature is observed
ent is unable to recognize objects by periodically once in four days. Speci-
touch. What part of brain has been fy the type of temperature curve:
damaged?
A. Febris internuttens
A. Postcentral gyrus B. Febris continua
B. Occipital lobe C. Febris reccurens
C. Temporal lobe D. Febris hectica
D. Precentral gyrus E. Febris remitens
E. Cerebellum
51. As a result of a craniocerebral
47. A patient complaining of rapid injury, a patient has a decreased skin
pulse, dyspnea and bluish color of sensitivity. What area of the cerebral
mucosa has been admitted to the cardi- cortex is likely to be damaged?
ological department. The objective
symptoms are as follows: edema of A. Posterior central gyrus
lower extremities, ascites. Which of B. Occipital region
the given medicines should be prescri- C. Cingulate gyrus
bed for intravenous administration to D. Frontal cortex
improve the patient’s general state? E. Anterior central gyrus
A. Corglyconum 52. Diabetic nephropathy with uremia
B. Cordiamin has developed in a patient with
C. Adrenalin hydrochloride pancreatic diabetes. The velocity of
D. Digitoxin glomerular filtration is 9 ml/min. What
E. Drotaverine mechanism of a decrease in glomerular
filtration velocity and chronic renal fai-
48. As a result of a continuous lure development is most likely in the
chronic encephalopathy, a patient has case of this patient?
developed spontaneous motions and
a disorder of torso muscle tone. These A. Reduction of active nephron mass
are the symptoms of the disorder of the B. Decrease in systemic arterial
following conduction tract: pressure
C. Obstruction of nephron tubules with
A. Tractus rubrospinalis hyaline casts
B. Tractus corticospinalis D. Tissue acidosis
C. Tractus corticonuclearis E. Arteriolar spasm
D. Tractus spinothalamicus
E. Tractus tectospinalis 53. A 40-year-old patient has ulcer
perforation in the posterior wall of
49. A 10-year-old child has painful stomach. What anatomical structure
swallowing, neck edema, temperature will blood and stomach content leak
rise up to 39, 0oC , the whole body to?
is covered with bright-red petechial
rash. Back of the throat and tonsils
are hyperemic, the tongue is crimson-
Krok 1 Medicine 2014 7

A. Bursa omentalis 58. A patient complaining of pain in


B. Bursa praegastrica the left shoulder-blade region has been
C. Right lateral channel (canalis diagnosed with miocardial infarction.
lateralis dexter) What kind of pain does the patient
D. Left lateral channel (canalis lateralis have?
sinister)
E. Bursa hepatica A. Radiating
B. Visceral
54. A patient is diagnosed with heredi- C. Phantom
tary coagulopathy that is characterised D. Protopathic
by factor VIII deficiency. Specify the E. Epicritic
phase of blood clotting during which
coagulation will be disrupted in the gi- 59. A patient has a critical impairment
ven case: of protein, fat and hydrocarbon digesti-
on. Most likely it has been caused by
A. Thromboplastin formation low secretion of the following digesti-
B. Thrombin formation ve juice:
C. Fibrin formation
D. Clot retraction A. Pancreatic juice
E. - B. Saliva
C. Gastric juice
55. Angiocardiography of a 60-year- D. Bile
old male patient revealed constri- E. Intestinal juice
ction of a vessel located in the left
coronary sulcus of the heart. What is 60. A female patient has facial neuritis
the pathological vessel called? that has caused mimetic paralysis and
hearing impairment. Hearing impai-
A. Ramus circumflexus rment results from the paralysis of the
B. Ramus interventricularis posterior following muscle:
C. A. coronaria dextra
D. V.cordis parva A. Stapedius muscle
E. Ramus interventricularis anterior B. Anterior auricular muscle
C. Superior auricular muscle
56. Those organisms which in the D. Posterior auricular muscle
process of evolution failed to develop E. Nasal muscle
protection from H2 O2 can exist only
in anaerobic conditions. Which of the 61. A 22-year-old female student
following enzymes can break hydrogen consulted a physician about fever up to
38o C , weakness, sore throat. Objecti-
peroxide down?
vely: there is white coating of the
A. Peroxidase and catalase tongue. What histological structures of
B. Oxygenase and hydroxylase the tongue are involved in the formati-
C. Cytochrome oxidase, cytochrome B5 on of this coating?
D. Oxygenase and catalase
E. Flavin-dependent oxidase A. Epithelium of the filiform papillae
B. Epithelium of the foliate papillae
57. A patient complains of pain in C. Epithelium of the fungiform papillae
the right lateral abdomen. Palpation D. Epithelium of the circumvallate
revealed a dense, immobile, tumor- papillae
like formation. A tumor is likely to be E. Connective-tissue base of all the
found in the following part of the di- lingual papillae
gestive tube:
62. A patient has a traumatic injury of
A. Colon ascendens sternocleidomastoid muscle. This has
B. Colon transversum resulted in a decrease in the following
C. Colon descendens value:
D. Colon sigmoideum
E. Caecum
Krok 1 Medicine 2014 8

A. Inspiratory reserve volume surgical damage of certain brain


B. Expiratory reserve volume structures has caused deep prolonged
C. Respiratory volume sleep. What structure is most likely to
D. Residual volume cause such condition, if damaged?
E. Functional residual lung capacity
A. Reticular formation
63. The receptors under study provide B. Basal ganglion
transfer of information to the cortex C. Red nuclei
without thalamic involvement. Specify D. Hippocampus
these receptors: E. Cerebral cortex
A. Olfactory 68. A patient takes cholagogues. What
B. Tactile other process besides biliary excretion
C. Gustatory will be stimulated?
D. Visual
E. Auditory A. Intestinal motility
B. Gastric juice secretion
64. Prolonged fasting causes C. Pancreatic juice secretion
hypoglycemia which is amplified by D. Gastric motor activity
alcohol consumption, as the followi- E. Water absorption
ng process is inhibited:
69. A 3-year-old child with meni-
A. Gluconeogenesis ngeal symptoms died. Postmortem
B. Glycolysis macroscopy of the pia matter revealed
C. Glycogenolysis miliary nodules which were mi-
D. Lipolysis croscopically represented by a focus
E. Proteolysis of caseous necrosis with masses of epi-
thelioid and lymphoid cells with some
65. A 39-year-old female patient wi- crescent-shaped large cells inbetween
th a history of diabetes was hospitali- having peripheral nuclei. Specify the
zed in a precomatose state for diabetic type of meningitis in the child:
ketoacidosis. This condition had been
caused by an increase in the following A. Tuberculous
metabolite level: B. Syphilitic
C. Brucellar
A. Acetoacetate D. Grippal
B. Citrate E. Meningococcal
C. Alpha-ketoglutarate
D. Malonate 70. A patient with homogentisuria has
E. Aspartate signs of arthritis, ochronosis. In this
case, the pain in the joints is associ-
66. A 37-year-old female patient ated with the deposition of:
complains of headache, vertigo,
troubled sleep, numbness of limbs. For A. Homogentisates
the last 6 years she has been working B. Urates
at the gas-discharge lamp-producing C. Phosphates
factory in the lead-processing shop. D. Oxalates
Blood test findings: low hemoglobin E. Carbonates
and RBC level, serum iron concentrati-
on exceeds the norm by several times. 71. A patient with hereditary
Specify the type of anemia: hyperammonemia due to a disorder
of ornithine cycle has developed
A. Iron refractory anemia secondary orotaciduria. The increased
B. Iron-deficiency anemia synthesis of orotic acid is caused by an
C. Minkowsky-Shauffard disease increase in the following metabolite of
D. Hypoplastic anemia ornithine cycle:
E. Metaplastic anemia
67. During an animal experiment,
Krok 1 Medicine 2014 9

A. Carbamoyl phosphate A. Cholelithiasis


B. Citrulline B. Hemolytic jaundice
C. Ornithine C. Hepatitis
D. Urea D. Chronic colitis
E. Argininosuccinate E. Chronic gastritis
72. Amniocentesis revealed two sex 76. A child patient has dry cough. What
chromatin bodies (Barr bodies) in each non-narcotic antitussive drug will reli-
cell of the sample. What disease is this eve the patient’s condition?
character typical for?
A. Glaucine hydrochloride
A. Trisomy X B. Codeine phosphate
B. Klinefelter syndrome C. Morphine hydrochloride
C. Turner’s syndrome D. Potassium iodide
D. Down’s syndrome E. Althaea officinalis root extract
E. Patau syndrome
77. A patient complains of acute pain
73. A 49-year-old male patient wi- attacks in the right lumbar region.
th acute pancreatitis was likely to During examination the nephrolithic
develop pancreatic necrosis, while acti- obturation of the right ureter in the
ve pancreatic proteases were absorbed region between its abdominal and
into the blood stream and tissue protei- pelvic segments has been detected.
ns broke up. What protective factors of What anatomical boundary exists
the body can inhibit these processes? between those two segments?
A. α2 -macroglobulin, α1 -antitrypsin A. Linea terminalis
B. Immunoglobulin B. Linea semilunaris
C. Cryoglobulin, interferon C. Linea arcuata
D. Ceruloplasmin, transferrin D. Linea transversa
E. Hemoplexin, haptoglobin E. Linea inguinalis
74. A 16-year-old female patient has 78. When measuring total muscle acti-
fainted after quickly changing her body on potential it was revealed that it was
position from horizontal to vertical subject to the power-law relationship.
one. Which process from the ones The reason for this is that individual
listed below has caused the loss of muscle fibers differ in:
consciousness in the first place?
A. Depolarization threshold
A. Decreasing venous return B. Diameter
B. Increasing venous return C. Conduction velocity
C. Increasing central venous pressure D. Resting potential
D. Decreasing oncotic pressure of blood E. Critical level of depolarization
plasma
E. Increasing arterial pressure 79. A patient has insufficient blood
supply to the kidneys, which has
75. A 53-year-old male patient caused the development of pressor
complains of acute pain in the right effect due to the constriction of arterial
hypochondrium. Objective examinati- resistance vessels. This is the result of
on revealed scleral icterus. Laboratory the vessels being greately affected by
tests revealed increased ALT activity, the following substance:
and stercobilin was not detected in the
stool. What disease is characterized by A. Angiotensin II
these symptoms? B. Angiotensinogen
C. Renin
D. Catecholamines
E. Norepinephrine
80. A patient is diagnosed with iron-
deficiency sideroachrestic anemia,
Krok 1 Medicine 2014 10

progression of which is characteri- 84. As a result of past encephalitis, a


sed by skin hyperpigmentation, pi- male patient has developed an increase
gmentary cirrhosis, heart and pancreas in cerebrospinal fluid pressure in the
affection. Iron level in the blood serum right lateral ventricle. What can be the
is increased. What disorder of iron cause of this condition?
metabolism causes this disease?
A. Closure of right interventricular
A. Failure to assimilate iron leading to foramen
iron accumulation in tissues B. Closure of left interventricular
B. Excessive iron intake with food foramen
C. Disorder of iron absorption in C. Atresia of tubus medullaris
bowels D. Atresia of sylvian aqueduct
D. Increased iron assimilation by body E. Atresia of the fourth ventricle
E. - foramina
81. In a village, a case of anthrax 85. Parents of 5-year-old child report
had been registered. Medical servi- him o have frequent colds that develop
ces began epidemiologically indicated into pneumonias, presence of purulent
specific prophylaxis of population agai- rashes on the skin. Laboratory tests
nst anthrax. What preparation was have revealed the following: absence
used for this purpose? of immunoglobulins of any type, and
naked cells are absent from the lymph
A. Live vaccine nodes punctate. What kind of immune
B. Inactivated vaccine disorder is it?
C. Chemical vaccine
D. Genetically engineered vaccine A. X-linked hypogammaglobulinemia
E. Anatoxin (Bruton type agammaglobulinemia)
B. Autosomal recessive
82. Experimental stimulation of the agammaglobulinaemia (Swiss type)
sympathetic nerve branches that C. Hypoplastic anemia
innervate the heart caused an increase D. Agranulocytosis
in force of heart contractions because E. Louis-Barr syndrome
the membrane of typical cardi-
omyocytes permitted an increase in: 86. A 3-year-old child has conti-
nuous fever, lymph nodes are enlarged,
A. Calcium ion entry the amount of lymphocytes in blood
B. Calcium ion exit is significantly increased. Enzyme-
C. Potassium ion exit linked immunosorbent assay (ELISA)
D. Potassium ion entry revealed antigen of Epstein-Barr virus.
E. Calcium and potassium ion exit What diagnosis can be made based on
83. According to the results of glucose the information given above?
tolerance test, the patient has no A. Infectious mononucleosis
disorder of carbohydrate tolerance. B. Burkitt’s lymphoma
Despite that, glucose is detected in C. Herpetic lymphadenopathy
the patients’s urine (5 mmol/l). The D. Generalized infection caused by
patient has been diagnosed with renal herpes-zoster
diabetes. What renal changes cause E. Cytomegalovirus infection
glucosuria in this case?
87. A microslide contains the preparati-
A. Decreased activity of glucose on of a gland composed of several
reabsorption enzymes secretory saccule-shaped parts that
B. Increased activity of glucose open in the common excretory duct.
reabsorption enzymes What gland is it?
C. Exceeded glucose reabsorption
threshold
D. Increased glucose secretion
E. Increased glucose filtration
Krok 1 Medicine 2014 11

A. Simple branched alveolar gland go through?


B. Compound branched alveolar gland
C. Simple unbranched alveolar gland A. Supraorbital
D. Compound unbranched alveolar B. Lacerum
gland C. Superior orbital fissure
E. Simple branched tubular gland D. Spinosum
E. Oval
88. A patient intending to undergo
a gender reassignment surgery has 92. An patient with insomnia induced
been admitted to a specialised cli- by allergic rash and itch has been
nic. In the course of examination prescribed the drug that has antihi-
both male and female gonades have stamine and hypnotic effect. Specify
been revealed, with male structure of this drug:
external genitals. What kind of genital
maldevelopment has the patient? A. Benadryl
B. Loratadine
A. True hermaphroditism C. Prednisolone
B. Male pseudohermaphroditism D. Acetylsalicylic acid
C. Female pseudohermaphroditism E. Analgin
D. Accessory ovary
E. Ectopia of testis 93. In a cat with decerebrate rigidity
the muscle tone is to be decreased. This
89. An 18-year-old male has can be achieved by:
been diagnosed with Marfan
syndrome. Examination revealed a A. Destruction of the vestibular nuclei
developmental disorder of connecti- of Deiters
ve tissue and eye lens structure, B. Stimulation of the otolithic vesti-
abnormalities of the cardiovascular bular receptors
system, arachnodactylia. What C. Stimulation of the vestibular nuclei
genetic phenomenon has caused the of Deiters
development of this disease? D. Stimulation of the vestibulocochlear
nerve
A. Pleiotropy E. Stimulation of the ampullar vesti-
B. Complementarity bular receptors
C. Codominance
D. Multiple allelism 94. When studying the signs of
E. Incomplete dominance pulmonary ventilation, reduction of
forced expiratory volume has been
90. In case of alkaptonuria, homogenti- detected. What is the likely causeof this
sic acid is excreted in urine in large phenomenon?
amounts. The development of this di-
sease is associated with a disorder of A. Obstructive pulmonary disease
metabolism of the following amino B. Increase of respiratory volume
acid: C. Increase of inspiratory reserve
volume
A. Tyrosine D. Increase of pulmonary residual
B. Phenylalanine volume
C. Alanine E. Increase of functional residual lung
D. Methionine capacity
E. Asparagine
95. X-ray examination of a patient
91. A male patient complains of skin allowed to diagnose a tumor in the
insensitivity of inferior eyelid, external superior lobe of the right lung. There
lateral surface of nose and upper lip. is a probability of metastases spread to
A doctor in the course of examinati- the following lymph nodes:
on has revealed inflammation of the
second branch of trigeminal nerve.
What cranial foramen does this branch
Krok 1 Medicine 2014 12

A. Sternal
B. Inferior mediastinum A. Myelin
C. Anterior mediastinum B. Acetylcholine
D. Axillary C. Norepinephrine
E. Deep lateral cervical D. Dopamine
E. Serotonin
96. A 55-year-old patient with a
characteristic rash, fever, dizziness 100. A patient with suspected tumor
has been admitted to a hospital. He of lung had been admitted to the
has been provisionally diagnosed wi- oncological department. Examinati-
th typhus. No similar cases have been on revealed localised pathology in the
reported. In his youth (15 years old) inferior lobe of the left lung. How
the patient suffered typhus in a boardi- many bronchopulmonary segments
ng school. What disease is it? does this lobe have?

A. Brill’s disease A. 5
B. Typhoid fever B. 6
C. Measles C. 4
D. Rubella D. 3
E. Cholera E. 2

97. A young woman suddenly 101. During local anesthetization the


developed fever up to 39o C accompani- patient has gone into anaphylactic
ed by a strong headache. Examinati- shock. What drug must be admini-
on revealed marked nuchal rigidi- stered to the patient?
ty. Spinal puncture was performed.
Gram-stained smear of cerebrospi- A. Epinephrine hydrochloride
nal fluid contained many neutrophi- B. Diazepam
ls and Gram-positive diplococci. What C. Atropine sulfate
bacteria could be the cause of this di- D. Propranolol
sease? E. Nitroglycerin

A. Neisseria meningitidis 102. As a result of an injury, the integri-


B. Streptococcus pneumonia ty of the anterior spinal cord root was
C. Haemophilus influenzae broken. Specify the neurons and their
D. Staphylococcus aureus processes that had been damaged:
E. Pseudomonas aeruginosa A. Axons of motor neurons
98. In the course of an experiment B. Motor neuron dendrites
adenohypophysis of an animal has C. Axons of sensory neurons
been removed. The resulting atrophy D. Dendrites of sensory neurons
of thyroid gland and adrenal cortex E. Dendrites of association neurons
has been caused by deficiency of the 103. A patient who had been taking di-
following hormone: clofenac sodium for arthritis of mandi-
A. Tropic hormones bular joint developed an acute conditi-
B. Thyroid hormones on of gastric ulcer. Such side effect of
C. Somatotropin this medicine is caused by inhibition of
D. Cortisol the following enzyme:
E. Thyroxin A. Cyclooxygenase-1 (COX-1)
99. Degenerative changes in posteri- B. Cyclooxygenase-2 (COX-2)
or and lateral columns of spinal C. Lipoxygenase
cord (funicular myelosis) caused by D. Phosphodiesterase
methylmalonic acid accumulation E. Monoamine oxidase
occur in patients with B12 -deficiency 104. In one of Polessye regions there
anemia. This results in synthesis di- was an outbreak of helminthiasis mani-
sruption of the following substance: fested by cramps and facial edmata.
Krok 1 Medicine 2014 13

The developed preventive measures A. Left middle ear


in particular included ban for eating B. Right middle ear
infested pork even after heat processi- C. Left inner ear
ng. What helminthiasis was the case? D. Right inner ear
E. Right external ear
A. Trichinosis
B. Taeniarhynchosis 109. In course of invasive abdominal
C. Teniasis surgery a surgeon has to locate the ori-
D. Echinococcosis gin of the mesenteric root. Where is it
E. Alveococcosis normally localized?
105. A female patient saught medical- A. Duodenojejunal flexure
genetic consultation. Physical exami- B. Right flexure of colon
nation revealed pterygium colli C. Left flexure of colon
deformity (webbed neck), broad chest, D. Sigmoid colon
underdeveloped breasts. Study of E. Right mesenteric sinus
buccal epithelium cells revealed no
X-chromatin in the nuclei. This indi- 110. A 55-year-old male had been
cates that the patient has the following delivered to the resuscitation unit
syndrome: unconscious. Relatives reported him
to have mistakenly drunk an alcoholic
A. Turner’s solution of unknown origin. On exami-
B. Klinefelter’s nation the patient was diagnosed with
C. Patau’s methanol intoxication. What antidote
D. Down’s should be used in this case?
E. Edwards’
A. Ethanol
106. During blood transfusion a B. Teturamum
patient has developed intravascular C. Naloxone
erythrocyte hemolysis. What kind of D. Protamine sulfate
hypersensitivity does the patient have? E. Acetylcysteine
A. II type (antibody-dependent) 111. Due to the use of poor-
B. I type (anaphylactic) quality measles vaccine for preventi-
C. III type (immune complex) ve vaccination, a 1-year-old chi-
D. IV type (cellular cytotoxicity) ld has developed an autoimmune
renal injury. The urine was found
E. IV type (granulomatosis) to contain macromolecular proteins.
107. Patients with erythropoietic What process of urine formation has
porphyria (Gunther’s disease) have been disturbed?
teeth that fluoresce with bright red A. Filtration
color when subjected to ultraviolet B. Reabsorption
radiation; their skin is light-sensitive, C. Secretion
urine is red-colored. What enzyme can D. Reabsorption and secretion
cause this disease, when it is deficient? E. Secretion and filtration
A. Uroporphyrinogen III cosynthase 112. As a result of a road accident a 37-
B. Uroporphyrinogen I synthase year-old female victim developed uri-
C. Delta-aminolevulinate synthase nary incontinence. What segments of
D. Uroporphyrinogen decarboxylase the spinal cord had been damaged?
E. Ferrochelatase
A. S2 − S4
108. During the air and bone conducti- B. T h1 − T h5
on tests it was revealed that in the C. L1 − L2
left ear the tones were louder by bone D. T h2 − T h5
conduction. This might be associated E. T h1 − L1
with the disease of:
113. An 8-week-pregnant woman
Krok 1 Medicine 2014 14

with acute respiratory disease and development of aminoacidemia?


temperature rise up to 39, 0oC has
called in a doctor. The doctor insisted A. Increased proteolysis
on her avoiding taking paracetamol, B. Albuminosis
because in this period of pregnancy C. Decrease in the concentration of
there is a risk of its: amino acids in blood
D. Increase in the oncotic pressure in
A. Teratogenicity the blood plasma
B. Embryotoxicity E. Increase in low-density lipoprotein
C. Fetotoxicity level
D. Hepatotoxicity
E. Allergenicity 118. Symptoms of pellagra (vitamin PP
deficiency) is particularly pronounced
114. A pneumonia patient has been in patients with low protein diet,
administered acetylcysteine as a part because nicotinamide precursor in
of complex therapy. What principle humans is one of the essential amino
of therapy has been taken into consi- acids, namely:
deration when applying this drug?
A. Tryptophan
A. Pathogenetic B. Threonine
B. Symptomatic C. Arginine
C. Etiotropic D. Histidine
D. Antimicrobial E. Lysine
E. Immunomodulatory
119. A 68-year-old patient consults
115. A 26-year-old female patient wi- a cardiologist, complaining of high
th bronchitis has been administered a arterial blood pressure, pain in the
broad spectrum antibiotic as a causal heart egion, intermittent pulse. Prescri-
treatment drug. Specify this drug: be the β1 -adrenoreceptor blocker
for the treatment of the described
A. Doxycycline pathology:
B. Interferon
C. BCG vaccine A. Metoprolol
D. Ambroxol B. Morphine hydrochloride
E. Dexamethasone C. Nootropil
D. Fenoterol
116. When defining blood group E. Benzylpenicillin
according to the AB0 system, using
salt solutions of monoclonal antibodi- 120. A female suffered rubella duri-
es, agglutination didn’t occur with any ng pregnancy. The child was born
of the solutions. What blood group is with developmental abnormalities,
it? namely cleft lip and palate. The child’s
genotype is normal. These malformati-
A. 0 (I) ons are a manifestation of:
B. А (II)
C. В (III) A. Modification variability
D. АВ (IV) B. Polyploidy
E. - C. Combinative variability
D. Chromosomal mutation
117. Examination of a 56-year-old E. Aneuploidy
female patient with a history of type
1 diabetes revealed a disorder of 121. An electron micrograph shows
protein metabolism that is manifested a cell-to-cell adhesion consisting, in
by aminoacidemia in the laboratory each cell, of an attachment plaque.
blood test values, and clinically by the The intercellular space is filled wi-
delayed wound healing and decreased th electron-dense substance includi-
synthesis of antibodies. Which of ng transmembrane fibrillar structures.
the following mechanisms causes the Specify this adhesion:
Krok 1 Medicine 2014 15

A. Immune rejection
A. Desmosome B. Ischemic kidney failure
B. Synapse C. Glomerulonephritis
C. Tight junction D. Tubular necrosis
D. Nexus E. Pyelonephritis
E. Adherens junction
126. During cell division, DNA repli-
122. For biochemical diagnostics of cation occurs by a signal from the
myocardial infarction it is necessary cytoplasm, and a certain portion of the
to measure activity of a number of DNA helix unwinds and splits into two
enzymes and their isoenzymes. What individual strains. What enzyme facili-
enzymatic test is considered to be the tates this process?
best to prove or disprove the diagnosis
of infarction in the early period after A. Helicase
the chest pain is detected? B. RNA polymerase
C. Ligase
A. Creatine kinase isoenzyme CK-MB D. Restrictase
B. Creatine kinase isoenzyme CK-MM E. DNA polymerase
C. LDH1 lactate dehydrogenase
isoenzyme 127. A female patient, having visited
D. LDH2 lactate dehydrogenase the factory premises with lots of dust in
isoenzyme the air for the first time, has got cough
E. Aspartate aminotransferase and burning pain in the throat. What
cytoplasmic isoenzyme respiratory receptors, when irritated,
cause this kind of reaction?
123. A patient complains of pain in
the heart area during acute attack of A. Irritant receptors
gastric ulcer. What vegetative reflex B. Juxtacapillary (J) receptors
can cause this painful feeling? C. Stretch receptors of lungs
D. Proprioceptors of respiratory
A. Viscerovisceral reflex muscles
B. Viscerodermal reflex E. Thermoreceptors
C. Visceromotor reflex
D. Dermatovisceral reflex 128. Since a patient has had myocardi-
E. Motor-visceral reflex al infarction, his atria and ventri-
cles contract independently from each
124. A patient is diagnosed with acute other with a frequency of 60-70 and 35-
morphine hydrochloride intoxication. 40 per minute. Specify the type of heart
Prescribe an oxidizing agent for gastric block in this case:
lavage:
A. Complete atrioventricular
A. Potassium permanganate B. Partial atrioventricular
B. Chloramine C. Sino-atrial
C. Sulfocamphocainum (Procaine + D. Intra-atrial
Sulfocamphoric acid) E. Intraventricular
D. Cerigel
E. Chlorhexidine (bi)gluconate 129. A 35-year-old female patient has
been hospitalised with acute intoxi-
125. In the course of puncture biopsy cation caused by salts of high-density
of transplanted kidney the followi- metals (lead, most probably). As a part
ng has been revealed: diffuse infi- of complex therapy the antidote that
ltration of stroma by lymphocytes contains two active sulfhydric groups
and plasmocytes and necrotic arteri- has been prescribed. Specify this anti-
tis. What pathological process has dote:
developed in the transplant?
Krok 1 Medicine 2014 16

A. Dimercaprol hemorrhages. Blood coagulation tests


B. Metamizole reveal increased prothrombin time.
C. Mannitol The child is most likely to have a di-
D. Nalorphine hydrochloride sorder of the following biochemical
E. Calcium chloride process:
130. A 60-year-old male patient has A. Production of gamma-
type II diabetes. A doctor has prescri- carboxyglutamate
bed him synthetic hypoglycemic long- B. Conversion of homocysteine to
acting drug that is sulfonylurea deri- methionine
vative. What drug is it? C. Conversion of methylmalonyl CoA
to succinyl CoA
A. Glibenclamide D. Degradation of glutathione
B. Butamide E. Hydroxylation of proline
C. Metformin
D. Actrapid (soluble insulin) 135. A 63-year-old male patient with
E. Acarbose bladder atony had been prescribed a
medication, which he had been arbi-
131. A patient has been given atropi- trarily taking at a higher dose. The
ne sulfate for rapid relief of spastic patient developed hypehydration, sali-
colon symptoms. The use of this drug vation, diarrhea, muscle spasms. The
is contraindicated during the followng prescribed drug relates to the followi-
disease: ng group:
A. Glaucoma A. Cholinomimetics
B. Bronchial asthma B. Cholinesterase reactivators
C. Bradycardia C. Adrenergc blockers
D. Hypotension D. Tocolytics
E. Gastric ulcer E. Ganglionic blockers
132. As an example of specific human 136. A 67-year-old male patient
parasites one can name Plasmodium consumes eggs, pork fat, butter, mi-
falciparum, human pinworm and some lk and meat. Blood test results:
others. The source of parasite invasion cholesterol - 12,3 mmol/l, total lipi-
is always a human. Such specific human ds - 8,2 g/l, increased low-density li-
parasites cause the diseases that are poprotein fraction (LDL). What type
called: of hyperlipoproteinemia is observed in
A. Anthroponoses the patient?
B. Zoonoses A. Hyperlipoproteinemia type IIa
C. Anthropozoonoses B. Hyperlipoproteinemia type I
D. Infections C. Hyperlipoproteinemia type IIb
E. Multifactorial diseases D. Hyperlipoproteinemia type IV
133. In an experiment a dog had been E. Cholesterol, hyperlipoproteinemia
conditioned to salivate at the sight of 137. Tissue sampling of a 37-year-old
food and a flash of light. After conditi- male patient with chronic renal disease
oning the reflex, the light was then pai- has revealed the following: sclerosis,
red with the bell. The dog didn’t start lymphocytic and plasmocytic infiltrati-
to salivate. What type of inhibition was on of renal pelvis and calices walls,
observed? dystrophy and atrophy of tubules.
A. External Remaining tubules are enlarged and
B. Differential stretched with colloid masses, epitheli-
C. Extinctive um is flattened out ("scutiform"or
D. Persistent "shield-shaped"kidney). What is the
E. Protective most likely diagnosis?

134. A newborn baby has numerous


Krok 1 Medicine 2014 17

A. Chronic pyelonephritis laboratory tests that determine the


B. Tubular interstitial nephritis presence of HBA-DNA in blood serum
C. Acute pyelonephritis of the patient. What reference method
D. Glomerulonephritis is applied for this purpose?
E. Nephrosclerosis
A. Polymerase chain reaction
138. During the hystological study B. Hybridization method
of cortical shaft, basophilic cells wi- C. Hybridization signal amplification
th developed synthesis organelles can method
be seen on the bone surface under the D. Ligase chain reaction method
layer of fibers. These cells take part in E. ELISA diagnostic method
bone tissue regeneration. What shaft
layer are they located in? 142. On the 24th day since the onset
of disease, a male patient diagnosed
A. Periosteum with typhoid fever and undergoing
B. Bone treatment in an infectious diseases
C. Osteon layer hospital has suddenly developed cli-
D. Outer lamellae of compact bone nical presentations of acute abdomen
tissue leading to the death of the patient.
E. Inner lamellae of compact bone During autopsy peritonitis has been
tissue revealed, with numerous ulcers coveri-
ng the colon mucosa and reaching as
139. Autopsy of a 50-year-old male deep as muscular and, in places, serous
who had tuberculosis revealed a dense tunic. The ulcers have smooth edges
gray-white nidus in form of a nodule and even floor. The intestinal wall
2 cm in diameter in the subpleural is perforated. What stage of typhoid
portion of the upper right lobe. The fever has the lethal complication ari-
pleura in this region was thickened, sen at?
in the pleural cavity there was a
small amount of serous hemorrhagic A. Clean ulcer
fluid. Histological study of the region B. Medullary swelling
revealed some glandular structures wi- C. Necrosis
th signs of cellular atypia and abnormal D. Dirty ulcer
mitoses, which were found within the E. Regeneration
fibrous connective tissue. What other
pathology had developed in the lungs? 143. An HIV-positive patient’s cause
of death is acute pulmonary insuffi-
A. Adenocarcinoma ciency resulting from pneumonia.
B. Squamous cell carcinoma Pathohistological investigation of lungs
C. Adenoma has revealed interstitial pneumonia,
D. Fibrosarcoma alveolocyte desquamation and
E. Fibroma their methamorphoses: alveolocyte
enlargement, large intranuclear
140. In course of an experiment inclusions surrounded by lightly-
there has been an increase in the colored areas. Transformed cells
nerve conduction velocity. This may resemble owl’s eye. Name the
be caused by an increase in the pneumonia causative agent:
concentration of the following ions
that are present in the solution around A. Cytomegalovirus
the cell: B. Pneumococcus
C. Influenza virus
A. Na+ D. Candida fungi
B. K + and Cl− E. Toxoplasma
C. K + and Na+
D. Ca2+ and Cl− 144. The organisms to be identifi-
E. Ca2+ ed have a nucleus surrounded by a
nuclear membrane. Genetic material
141. Hepatitis B is diagnosed through is concentrated predominantly in the
Krok 1 Medicine 2014 18

chromosomes which consist of DNA A. Methionine, B6 , B12


strands and protein molecules. These B. Arginine, B2 , B3
cells divide mitotically. Identify these C. Alanine, B1 , P P
organisms: D. Valine, B3 , B2
E. Isoleucine, B1 , B2
A. Eukaryotes
B. Bacteriophages 149. A 2-year-old boy is diagnosed with
C. Prokaryotes Down syndrome. What chromosomal
D. Viruses changes may be the cause of this di-
E. Bacteria sease?
145. A 37-year-old male patient A. Trisomy 21
developed pseudoarthrosis after a B. Trisomy 13
closed fracture of the femur. Specify C. Trisomy X
the type of regeneration in the patient: D. Trisomy 18
E. Monosomy X
A. Pathological hypo-regeneration
B. Pathological hyper-regeneration 150. A patient who has recently arri-
C. Reparative ved from an endemic area presents
D. Physiological with elevated body temperature,
E. - headache, chills, malaise, that is with
the symptoms which are typical for a
146. Nucleolar organizers of the 13-15, common cold. What laboratory tests
21, 22 human chromosomes contain are necessary to prove or to disprove
about 200 cluster genes that synthesize the diagnosis of malaria?
RNA. These regions of chromosomes
bear the information on the following A. Microscopy of blood smears
type of RNA: B. Study of lymph node punctate
C. Urinalysis
A. rRNA D. Study of cerebrospinal fluid
B. tRNA E. Microscopy of bone marrow
C. mRNA punctate
D. snRNA
E. tRNA + rRNA 151. A patient has severe blood loss
caused by an injury. What kind of
147. The resuscitation unit has admi- dehydration will be observed in this
tted a patient in grave condition. particular case?
It is known that he had mistakenly
taken sodium fluoride which blocks A. Iso-osmolar
cytochrome oxidase. What type of B. Hyposmolar
hypoxia developed in the patient? C. Hyperosmolar
D. Normosmolar
A. Tissue E. -
B. Hemic
C. Cardiovascular 152. A 30-year-old patient has dyspnea
D. Hypoxic fits, mostly at night. He has been di-
E. Respiratory agnosed with bronchial asthma. What
type of allergic reaction according to
148. Steatosis is caused by the the Gell-Coombs classification is most
accumulation of triacylglycerols in likely in this case?
hepatocytes. One of the mechani-
sms of this disease development is a A. Anaphylactic
decrease in the utilization of VLDL B. Cytotoxic
neutral fat. What lipotropics prevent C. Stimulating
the development of steatosis? D. Immune complex
E. Delayed-type hypersensitivity
153. Decarboxylation of glutamate
induces production of gamma-
Krok 1 Medicine 2014 19

aminobutyric acid (GABA ) nned substance. Microscopic investi-


neurotransmitter. After breakdown, gation has shown the thickening of
GABA is converted into a metaboli- arteriole walls due to accumulation of
te of the citric acid cycle, that is: homogeneous anhistic pink-coloured
masses in them. Glomerules were
A. Succinate undersized, sclerotic, with atrophi-
B. Citric acid ed tubules. What disease are these
C. Malate changes characteristic of?
D. Fumarate
E. Oxaloacetate A. Essential hypertension
B. Pyelonephritis with kidney shrinkage
154. An outbreak of an intestinal C. Renal amyloidosis
infection occurred in a kindergarten on D. Acute glomerulonephritis
the eve of New Year holidays. Bacteri- E. Membranous nephropathy
ological examination of patients’
faeces didn’t reveal any pathogenic 158. A fixed-run taxi passenger has
bacteria. Electron microscopy revealed a sudden and expressed attack of
roundish structures with clear outer tachycardia. A doctor travelling by the
edges and a thick core resembling a same taxi has managed to slow down
wheel. Specify the most likely causati- his heart rate by pressing upon the
ve agent of this infection: eyeballs and thus causing the following
reflex:
A. Rotavirus
B. Adenovirus A. Dagnini-Aschner reflex
C. Coxsacki-virus B. Bainbridge reflex
D. E.coli C. Holtz’s reflex
E. P.vulgaris D. Hering-Breuer reflex
E. Frank-Starling mechanism
155. A smear of streptobacillus
preparation stained by Ozheshko 159. Histological examination of bi-
method has been studied microscopi- opsy samples taken from the thickened
cally with oil immersion. What edges of a gastric ulcer revealed small
structural feature of the bacteria has clusters of small, markedly atypical
been studied? hyperchromatic epithelial cells that
were localized in the overdeveloped
A. Spores stroma. Specify the tumor:
B. Capsule
C. Flagella A. Scirrhous undifferentiated carci-
D. Inclusions noma
E. Structure of cell wall B. Medullary carcinoma
C. Adenocarcinoma
156. After a road accident a victim D. Undifferentiated sarcoma
has tachycardia, arterial blood pressure E. Adenoma
130/90 mm Hg, tachypnoe, the skin
is pale and dry, excitation of central 160. A 10-year-old child was found to
nervous system is observed. What have a congenital hypoplasia of the
shock stage is the patient most likely left kidney. Ultrasound examination
in? revealed that the right kidney was
markedly enlarged and had regular
A. Erectile shape. No functional disorders were
B. Terminal revealed. Specify the process that
C. Torpid developed in the right kidney:
D. Preshock (compensation stage)
E. Agony A. Vicarious hypertrophy
B. Working hypertrophy
157. Autopsy has revealed shrunken C. Hypertrophic growth
kidneys weighing 50 mg, with fine- D. Pseudohypertrophy
grained surface and uniformly thi- E. Metaplasia
Krok 1 Medicine 2014 20

161. In allergic diseases, a dramaic temperature down to 37, 0oC . Specify


increase in basophilic leukocyte the leading mechanism in the 3rd stage
number in patients’ blood is observed. of fever:
This phenomenon is due to the followi-
ng basophil function: A. Peripheral vasodilation
B. Increased heat production
A. Participation of heparin and histami- C. Development of chill
ne in metabolism D. Increased diuresis
B. Phagocytosis of microorganisms and E. Increased respiratory rate
small particles
C. Immunoglobulin synthesis 166. A 35-year-old female patient
D. Phagocytosis of immune complexes has undergone biopsy of the breast
E. Participation in blood clotting nodules. Histological examination has
revealed enhanced proliferation of
162. A 47-year-old male patient the small duct and acini epithelial
developed intestinal colic against the cells, accompanied by the formation of
background of essential hypertension. glandular structures of various shapes
In this situation it would be most effi- and sizes, which were located in the fi-
cient to arrest the colic by using drugs brous stroma. What is the most likely
of the following group: diagnosis?
A. Myotropic antispasmodics A. Fibroadenoma
B. Anticholinesterase agents B. Adenocarcinoma
C. Sympathomimetics C. Cystic breast
D. M-cholinomimetics D. Invasive ductal carcinoma
E. Adrenomimetics E. Mastitis
163. A patient has been admitted to the 167. At a bacteriological laboratory
contagious isolation ward with signs animal skins are analyzed by means
of jaundice caused by hepatitis virus. of Ascoli precipitaion test. What is
Which of the symptoms given below detected if the reaction is positive?
is strictly specific for hepatocellular
jaundice? A. Anthrax agent antigens
B. Anaerobic infection toxin
A. Increase of ALT, AST level C. Brucellosis agent
B. Hyperbilirubinemia D. Yersinia surface antigen
C. Bilirubinuria E. Plague agent
D. Cholemia
E. Urobilinuria 168. Examination of the duodenal
contents revealed some pear-shaped
164. An underage patient has signs of protozoa with two nuclei and four pai-
achondroplasia (dwarfism). It is known rs of flagella. The organisms had also
that this is a monogenic disease and two axostyles between the nuclei and a
the gene that is responsible for the ventral adhesive disc. What protozoan
development of such abnormalities is representative was found in the pati-
a dominant one. The development of ent?
that child’s brother is normal. Specify
the genotype of the healthy child: A. Lamblia
B. Toxoplasma
A. aa C. Leishmania
B. AA D. Intestinal trichomonad
C. Aa E. Trypanosome
D. AaBb
E. AABB 169. A 36-year-old female patient
has a history of B2 -hypovitaminosis.
165. A patient has acute bronchitis. The most likely cause of speci-
The fever up to 38, 5o C had lasted for a fic symptoms (epithelial, mucosal,
week, presently there is a decrease in cutaneous, corneal lesions) is the defi-
Krok 1 Medicine 2014 21

ciency of: 174. An electron micrograph shows


a cell of neural origin. The terminal
A. Flavin coenzymes portion of the cell dendrite has cyli-
B. Cytochrome A1 ndrical shape and consists of 1000
C. Cytochrome oxidase closed membrane disks. What cell is
D. Cytochrome B represented by the micrograph?
E. Cytochrome C
A. Rod receptor cell
170. A 54-year-old female was brought B. Cone receptor cell
to the casualty department after a car C. Spinal node neuron
accident. A traumatologist diagnosed D. Neuron of the cerebral cortex
her with multiple fractures of the lower E. Neuron of the anterior horns of the
extremities. What kind of embolism is spinal cord
most likely to develop in this case?
175. Disruption of nerve fiber myeli-
A. Fat nogenesis causes neurological di-
B. Tissue sorders and mental retardation. These
C. Thromboembolism symptoms are typical for heredi-
D. Gaseous tary and acquired alterations in the
E. Air metabolism of:
171. A blood drop has been put into a A. Sphingolipids
test tube with 0,3% solution of NaCl. B. Neutral fats
What will happen to erythrocytes? C. Higher fatty acids
D. Cholesterol
A. Osmotic haemolysis E. Phosphatidic acid
B. Shrinkage
C. Mechanical haemolysis 176. A patient underwent surgical
D. Any changes will be observed removal of a cavitary liver lesion 2
E. Biological haemolysis cm in diameter. It was revealed that
the cavity wall was formed by dense
172. There are cortical and medullary fibrous connective tissue; the cavi-
substances separated by connective ty contained muddy, thick, yellowish-
tissue layer in the endocrine gland greenish fluid with an unpleasant
specimen. Parenchyma cells make up odor. Microscopically, the fluid consi-
three zones in cortical substance, wi- sted mainly of polymorphonuclear
th rounded masses in the superficial leukocytes. What pathological process
zone, parallel chords in the middle one, are these morphological changes typi-
reticular structure of cell chords in the cal for?
deep one. What gland is it?
A. Chronic abscess
A. Adrenal gland B. Acute abscess
B. Thyroid gland C. Phlegmon
C. Pituitary gland D. Empyema
D. Epiphysis E. -
E. Hypothalamus
177. A patient complains of
173. Histological specimen of a 10-day photoreception disorder and frequent
human embryo represents 2 contacting acute viral diseases. He has been
sacs (amniotic and yolk sacs). Specify prescribed a vitamin that affects
the structure that separates the amni- photoreception processes by produci-
otic cavity from the yolk sac: ng rhodopsin, the photosensitive pi-
A. Embryonic shield gment. What vitamin is it?
B. Amniotic stalk
C. Floor of the amniotic sac
D. Roof of the yolk sac
E. Extraembryonic mesoderm
Krok 1 Medicine 2014 22

A. Retinol acetate A. Genocopy


B. Tocopherol acetate B. Trisomy
C. Pyridoxine hydrochloride C. Phenocopy
D. Cyanocobalamin D. Monosomy
E. Thiamine E. -
178. A 7-year-old boy got ill with di- 182. A 28-year-old patient undergoi-
phtheria. On the third day he di- ng treatment in the pulmonological
ed of asphyxiation. At autopsy the department has been diagnosed wi-
mucosa of the larynx, trachea and th pulmonary emphysema caused by
bronchi had thickened, edematous, splitting of alveolar septum by tissular
lustreless appearance and was covered tripsin. The disease is cased by the
with gray films which could be easily congenital deficiency of the following
removed. Specify the type of laryngeal protein:
inflammation:
A. α1 -proteinase inhibitor
A. Croupous B. α2 -macroglobulin
B. Diphtheritic C. Cryoglobulin
C. Purulent D. Haptoglobin
D. Catarrhal E. Transferrin
E. Intermediate
183. A patient with signs of
179. A child has a history of osteoporosis and urolithiasis has
hepatomegaly, hypoglycemia, seizures, been admitted to the endocri-
especially on an empty stomach and nology department. Blood test
in stressful situations. The child is di- has revealed hypercalcemia and
agnosed with Gierke disease. This di- hypophosphatemia. These changes are
sease is caused by the genetic defect of associated with abnormal synthesis of
the following enzyme: the following hormone:
A. Glucose-6-phosphatase A. Parathyroid hormone
B. Amyloid-1,6-glycosidase B. Calcitonin
C. Phosphoglucomutase C. Cortisol
D. Glycogen phosphorylase D. Aldosterone
E. Glucokinase E. Calcitriol
180. A public utility specialist went 184. During a surgery for femoral
down into a sewer well without hernia a surgeon operates within the
protection and after a while lost boundaries of femoral trigone. What
consciousness. Ambulance doctors di- structure makes up its upper margin?
agnosed him with hydrogen sulfide
intoxication. What type of hypoxia A. Lig.inguinale
developed? B. Arcus iliopectineus
C. Lig.lacunare
A. Hemic D. Lig. pectinale
B. Overload E. Fascia lata
C. Tissue
D. Circulatory 185. A 19-year-old victim has been
E. Respiratory delivered to the casualty department
with a cut wound of the trapezius
181. A child with a normal karyotype muscle. Which of the cervical fasciae
is diagnosed with cleft lip and forms a sheath for this muscle?
hard palate, defects of the cardi-
ovascular system, microcephaly. The
child’s mother suffered rubella during
pregnancy. This pathology in the child
may be an example of:
Krok 1 Medicine 2014 23

A. Investing layer of cervical fascia cell volume has been observed (60%).
B. Muscular part of the pretacheal layer What other value will be affected by
of cervical fascia this change?
C. Visceral part of the pretracheal layer
of cervical fascia A. Increasing blood viscosity
D. Prevertebral layer of cervical fascia B. Increasing volume of blood circulati-
E. Carotid sheath of cervical fascia on
C. Leukopenia
186. A patient with acne has been D. Thrombocytopenia
prescribed doxycycline hydrochloride. E. Increasing erythrocyte sedimentati-
What recommendations should be gi- on rate (ESR)
ven to the patient, while he is taking
this drug? 191. A patient complains of palpi-
tation after stress. The pulse is 104
A. Avoid long stay in the sun bpm, P-Q=0,12 seconds, there are no
B. Take with large quantity of liquid, changes of QRS complex. What type
preferably milk of arrhythmia does the patient have?
C. Take before meal
D. The course of treatment should not A. Sinus tachycardia
exceed 1 day B. Sinus bradycardia
E. Do not take with vitamins C. Sinus arrhythmia
D. Ciliary arrhythmia
187. A 30-year-old patient with a past E. Extrasystole
history of virus B hepatitis complains
of prolonged nosebleeds. What drug 192. A 30-year-old patient has
will be most efficient in remedying this undergone keratoplasty in the
condition? transplantation center, cornea has been
taken fron a donor, who died in a road
A. Vicasolum accident. What kind of transplantation
B. Fraxiparine was performed?
C. Folic acid
D. Dipiridamol A. Allotransplantation
E. Asparcam B. Autotransplantation
C. Xenotransplantation
188. A patient has arterial hypertensi- D. Explantation
on. What long-acting drug from the E. Heterotransplantation
group of calcium channel blockers
should be prescribed? 193. A 29-year-old male with a knife
wound of neck presents with bleedi-
A. Amlodipine ng. During the initial d-bridement of
B. Octadine the wound the surgeon revealed the
C. Pyrroxanum injury of a vessel found along the
D. Atenolol lateral edge of the sternocleidomastoid
E. Reserpine muscle. Specify this vessel:
189. A patient has been diagnosed A. V. jugularis externa
with ARVI. Blood serum contains B. V. jugularis anterior
immunoglobulin M. What is the stage C. A. carotis externa
of infection in this case? D. A. carotis interna
E. V. jugularis interna
A. Acute
B. Prodromal 194. A histologic specimen represents
C. Incubation an organ with walls comprised
D. Reconvalescence of mucous, submucous, fibrous-
E. Carriage cartilaginous and adventitial membranes.
Epithelium is multirowed and ciliated,
190. In a dysentery patient undergoing muscular layer of mucous membrane is
treatment in the contagious isolation absent, submucous membrane contai-
ward, a significant increase in packed
Krok 1 Medicine 2014 24

ns serous-mucous glands, hyaline A. Alkalosis, hypocapnia, temperature


cartilage forms open circles. What drop
organ has the described morphologi- B. Acidosis, hypercapnia, temperature
cal features? rise
C. Acidosis, hypercapnia, temperature
A. Trachea drop
B. Tertiary bronchi (segmental bronchi) D. Acidosis, hypocapnia, temperature
C. Secondary bronchi (lobar bronchi) rise
D. Terminal bronchiole E. -
E. Larynx
198. Administration of doxycycline
195. In cancer patients who have been hydrochloride caused an imbalance
continuously receiving methotrexate, of the symbiotic intestinal microflora.
the target cells of tumor with time Specify the kind of imbalance caused
become insensitive to this drug. In this by the antibiotic therapy:
case, gene amplification of the followi-
ng enzyme is observed: A. Dysbacteriosis
B. Sensibilization
A. Dihydrofolate reductase C. Idiosyncrasy
B. Thiaminase D. Superimposed infection
C. Deaminase E. Bacteriosis
D. Thioredoxin reductase
E. - 199. A patient with signs of emotional
lability that result in troubled sleep has
196. Pancreas is known as a mixed been prescribed nitrazepam. Specify
gland. Endocrine functions include the sleep-inducing mechanism of this
production of insulin by beta cells. This drug:
hormone affects the metabolism of
carbohydrates. What is its effect upon A. GABA-ergic system activation
the activity of glycogen phosphorylase B. Blockade of opiate receptors
(GP) and glycogen synthase (GS)? C. Inhibition of stimulating amino acids
D. H1-histamine receptors stimulation
A. It inhibits GP and activates GS E. Supression of serotonergic
B. It activates both GP and GS neurotransmission
C. It inhibits both GP and GS
D. It activates GP and inhibits GS 200. A patient has been found to have
E. It does not affect the activity of GP a marked dilatation of saphenous vei-
and GS ns in the region of anterior abdomi-
nal wall around the navel. This is
197. A patient has the oxyhemoglobin symptomatic of pressure increase in
dissociation curve shifted to the left. the following vessel:
What blood changes induce this condi-
tion? A. V. portae hepatis
B. V. cava superior
C. V. cava inferior
D. V. mesenterica inferior
E. V. mesenterica superior
Крок 1 Medicine (англомовний варiант, iноземнi студенти) 2015 рiк 1

1. Characteristic sign of glycogenosis is 6. A 6-year-old child with suspected acti-


muscle pain during physical work. Blood ve tuberculous process has undergone di-
examination usually reveals hypoglycemia. agnostic Mantoux test. What immunobi-
This pathology is caused by congenital defi- ological preparation was injected?
ciency of the following enzyme:
A. Tuberculin
A. Glycogen phosphorylase B. BCG vaccine
B. Glucose 6-phosphate dehydrogenase C. DTP vaccine
C. α-amylase D. Tularinum
D. γ -amylase E. Td vaccine
E. Lysosomal glycosidase
7. During postembryonal haemopoiesis
2. Histologic specimen of a kidney in red bone marrow the cells of one of
demonstrates cells closely adjoined to the the cellular differons demonstrate gradual
renal corpuscle in the distal convoluted decrease in cytoplasmic basophilia as well
tubule. Their basement membrane is as increase in oxyphilia, the nucleus is being
extremely thin and has no folds. These cells forced out. Such morphological changes are
sense the changes in sodium content of uri- typical for the following haemopoiesis type:
ne and influence renin secretion occurring
in juxtaglomerular cells. Name these cells: A. Erythropoiesis
B. Lymphopoiesis
A. Macula densa cells C. Neutrophil cytopoiesis
B. Juxtaglomerular cells D. Eosinophil cytopoiesis
C. Mesangial cells E. Basophil cytopoiesis
D. Podocytes
E. Glomerular capillary endothelial cells 8. When taking exams students often have
dry mouth. The mechanism that causes this
3. A 46-year-old female patient has conti- state results from the following reflexes:
nuous history of progressive muscular
(Duchenne’s) dystrophy. Which blood A. Conditioned sympathetic
enzyme changes will be of diagnostic value B. Unconditioned parasympathetic
in this case? C. Conditioned parasympathetic
D. Unconditioned sympathetic
A. Creatine phosphokinase E. Unconditioned peripheral
B. Lactate dehydrogenase
C. Pyruvate dehydrogenase 9. A 67-year-old patient with clini-
D. Glutamate dehydrogenase cal diagnosis of chronic bronchitis,
E. Adenylate cyclase pneumosclerosis, and cardiopulmonary
decompensation has the biopsy material
4. A laboratory experiment on a dog taken from the suspicious area in his right
was used to study central parts of audi- bronchus mucosa. Cellular and tissue atypi-
tory system. One of the mesencephalon sm along with pearly bodies can be histologi-
structures was destroyed. The dog has lost cally detected. What pathologic process is
the orienting response to auditory signals. characterized by the described histological
What structure was destroyed? changes?
A. Inferior colliculi of corpora quadrigemina A. Squamous cell carcinoma of bronchus
B. Superior colliculi of corpora quadrigemina with keratinization
C. Substantia nigra B. Polypoid chronic bronchitis
D. Reticular formation nuclei C. Bronchiectasis
E. Red nucleus D. Acute bronchitis
E. Squamous cell metaplasia of bronchial
5. A patient has decreased concentrati- mucosa
on of magnesium ions that are requi-
red for ribosomes connection to granular 10. A microspecimen of heart shows
endoplasmic reticulum. This condition is rectangular cells from 50 to 120 micrometers
known to disrupt the process of protein bi- large with central position of nucleus
osynthesis. Disruption occurs at the followi- and developed myofibrils. The cells are
ng stage: connected by intercalated discs. These cells
are responsible for the following function:
A. Translation
B. Transcription A. Function of heart contractions
C. Replication B. Function of impulse conduction
D. Amino acids activation C. Endocrine
E. Processing D. Protective
E. Regeneratory
Крок 1 Medicine (англомовний варiант, iноземнi студенти) 2015 рiк 2

11. Untrained people often have muscle A. Gastrin


pain after sprints as a result of lactate B. Histamine
accumulation. This can be caused by C. Secretin
intensification of the following biochemical D. Cholecystokinin
process: E. Neurotensin
A. Glycolysis 17. Atretic bodies and developed yellow
B. Gluconeogenesis body can be observed along with follicles
C. Pentose phosphate pathway of various orders in an ovary specimen.
D. Lipogenesis What stage of ovarian and menstrual cycle is
E. Glycogenesis characterized by the described ovary condi-
tion?
12. Poisoning caused by botulinum toxin
that prevents calcium ions from entering A. Premenstrual
axone nerve endings of motoneurons is life- B. Menstrual
threatening because it can lead to: C. Postmenstrual
D. Regeneration
A. Respiratory arrest E. Follicle growth
B. Cardiac arrest
C. Vasotonic disorder 18. A 16-year-old adolescent is di-
D. Vomiting agnosed with hereditary UDP (uridine di-
E. Diarrhea phosphate) glucuronyltransferase defici-
ency. Laboratory tests revealed hyperbili-
13. Increased HDL levels decrease the risk rubinemia caused mostly by increased blood
of atherosclerosis. What is the mechanism of content of the following substance:
HDL anti-atherogenic action?
A. Unconjugated bilirubin
A. They remove cholesterol from tissues B. Conjugated bilirubin
B. They supply tissues with cholesterol C. Urobilinogen
C. They are involved in the breakdown of D. Stercobilinogen
cholesterol E. Biliverdine
D. They activate the conversion of
cholesterol to bile acids 19. Prior to glucose utilization in cells it is
E. They promote absorption of cholesterol in transported inside cells from extracellular
the intestine space through plasmatic membrane. This
process is stimulated by the following
14. It has been found out that one of a pesti- hormone:
cide components is sodium arsenate that
blocks lipoic acid. Which enzyme activity A. Insulin
is impaired by this pesticide? B. Glucagon
C. Thyroxin
A. Pyruvate dehydrogenase complex D. Aldosterone
B. Microsomal oxidation E. Adrenalin
C. Methemoglobin reductase
D. Glutathione peroxidase 20. After implantation of a cardiac valve
E. Glutathione reductase a young man systematically takes indirect
anticoagulants. His state was complicated
15. A drycleaner’s worker has been found to by hemorrhage. What substance content has
have hepatic steatosis. This pathology can decreased in blood?
be caused by the disruption of synthesis of
the following substance: A. Prothrombin
B. Haptoglobin
A. Phosphatidylcholine C. Heparin
B. Tristearin D. Creatin
C. Urea E. Ceruloplasmin
D. Phosphatidic acid
E. Cholic acid 21. A 12-year-old patient has white non-
pigmented spots on the skin. The spots
16. A 35-year-old man with peptic ulcer di- appeared after the patient became 10 years
sease has undergone antrectomy. After the old, and they constantly grow. This spots
surgery secretion of the following gastro- appeared due to the lack of the following
intestinal hormone will be disrupted the skin cells:
most:
Крок 1 Medicine (англомовний варiант, iноземнi студенти) 2015 рiк 3

A. Melanocytes A. Klinefelter’s syndrome


B. Adipocytes B. Down’s disease
C. Fibrocytes C. Turner’s syndrome
D. Plasmocytes D. Triple X syndrome
E. Labrocytes E. Hypophosphatemic rickets
22. A group of Ukrainian tourists returning 27. Pure culture of microorganisms was
from Samarqand was bringing with them obtained from pharynx of a child with
gerbils. During examination in customs offi- suspected diphtheria. Morphologic, tinctori-
ce ulcers were detected on the skin of the al, cultural, and biochemical properties
animals. What protozoa is the most likely to of the microorganisms were studied and
cause the disease in the animals, if mosqui- revealed to be characteristic of diphtheria
tos are the carriers? agents. What investigation should be addi-
tionally performed to make a conclusion,
A. Leishmania tropica major that these microorganisms are pathogenic
B. Balantidium coli diphtheria bacilli?
C. Plasmodium falciparum
D. Trypanosoma cruzi A. Determine toxigenic properties
E. Toxoplasma gondii B. Determine proteolytic properties
C. Determine urease activity
23. A 5-year-old child has been diagnosed D. Determine cystinase activity
with acute right distal pneumonia. Sputum E. Determine amylolytic activity
inoculation revealed that the causative
agent is resistant to penicillin and senstive 28. Autopsy of a man who had tuberculosis
to macrolides. What drug should be prescri- revealed a 3x2 cm large cavity in the superi-
bed? or lobe of the right lung. The cavity was
interconnected with a bronchus, its wall
A. Azithromycin was dense and consisted of three layers: the
B. Tetracycline internal layer was pyogenic, the middle layer
C. Gentamycin was made of tuberculous granulation tissue
D. Streptomycin and the external one was made of connecti-
E. Ampicillin ve tissue. What is the most likely diagnosis?
24. To an emergency ward a 7-year-old A. Fibrous cavernous tuberculosis
child was delivered in the condition of B. Fibrous focal tuberculosis
allergic shock caused by a bee sting. High C. Tuberculoma
concentration of histamine is observed in D. Acute focal tuberculosis
blood. Production of this amine is the result E. Acute cavernous tuberculosis
of the following reaction:
29. A 7-year-old child has acute onset of di-
A. Decarboxylation sease: temperature rise up to 38o C , rhinitis,
B. Hydroxylation cough, lacrimation, and large-spot rash on
C. Dehydrogenation the skin. Pharyngeal mucosa is edematous,
D. Deaminization hyperemic, with whitish spots in the buccal
E. Reduction area. What kind of inflammation causes the
changes in the buccal mucosa?
25. A 65-year-old man suffering from gout
complains of pain in his kidneys. Ultrasonic A. Catarrhal inflammation
examination revealed kidney stones. A B. Suppurative inflammation
certain substance in increased concentration C. Fibrinous inflammation
can cause kidney stones formation. Name D. Hemorrhagic inflammation
this substance: E. Serous inflammation
A. Uric acid 30. Analysis of sputum taken from a patient
B. Cholesterol with suspected pneumonia revealed slightly
C. Bilirubin elongated gram-positive diplococci with poi-
D. Urea nted opposite ends. What microorganisms
E. Cystine were revealed in the sputum?
26. Sex chromatin was detected during A. Streptococcus pneumoniae
examination of a man’s buccal epitheli- B. Staphylococcus aureus
um. It is characteristic of the following C. Klebsiella pneumoniae
chromosome disease: D. Neisseria meningitidis
E. Neisseria gonorrhoeae
31. Serological diagnostics of infectious di-
Крок 1 Medicine (англомовний варiант, iноземнi студенти) 2015 рiк 4

seases is based upon specific interaction with skin itching. Objectively the following can
antigenes. Specify the serological reaction be observed: skin hyperemia, tachycardia,
that underlies adhesion of microorganisms BP dropped down to 70/40 mm Hg. What
when they are affected by specific antibodi- kind of allergic reaction is this pathology?
es in presence of an electrolyte:
A. Anaphylactic
A. Agglutination reaction B. Cytotoxic
B. Precipitation reaction C. Stimulating
C. Complement-binding reaction D. Cell-mediated immune reaction
D. Hemadsorption reaction E. Immune complex
E. Neutralization reaction
37. A patient with probable liver abscess
32. A 4-year-old child was admitted to an was delivered to a surgical department. The
orthopaedic department with displaced shin patient for a long time had been on an
fracture. Bone fragments reposition requires assignment in an African country and had
analgesia. What drug should be chosen? recurrent cases of acute gastrointestinal di-
sturbance. What protozoan disease can it
A. Promedol be?
B. Analgin
C. Morphine hydrochloride A. Amebiasis
D. Panadol B. Trypanosomiasis
E. - C. Leishmaniasis
D. Malaria
33. Doctors make mostly radial incisions E. Toxoplasmosis
during mammary gland surgery. What parti-
culars of anatomical organization make such 38. Work in a mine is known to cause
surgical technique preferable? inhalation of large amounts of coal dust.
Inhaled coal dust can be detected in the
A. Lobe apexes converge towards nipples following pulmonary cells:
B. Lobe bases radiate from nipples
C. Transversal position of gland lobes A. Alveolar macrophages
D. Vertical position of gland lobes B. Respiratory epithelial cells
E. - C. Secretory epithelial cells
D. Capillary endothelial cells
34. A 41-year-old man has a history of E. Pericapillary cells
recurrent attacks of heartbeats (paroxysms),
profuse sweating, headaches. Examinati- 39. Electrical activity of neurons is being
on revealed hypertension, hyperglycemia, measured. They fire prior to and at the
increased basal metabolic rate, and beginning of inhalation. Where are these
tachycardia. These clinical presentations are neurons situated?
typical for the following adrenal pathology:
A. Medulla oblongata
A. Hyperfunction of the medulla B. Diencephalon
B. Hypofunction of the medulla C. Mesencephalon
C. Hyperfunction of the adrenal cortex D. Spinal cord
D. Hypofunction of the adrenal cortex E. Cerebral cortex
E. Primary aldosteronism
40. Electrocardiogram analysis demonstrates
35. A 12-year-old child has developed that cardiac cycle of a human equals 1
nephritic syndrome (proteinuria, hematuria, second. It means that heart rate per minute
cylindruria) 2 weeks after the case of tonsi- equals:
llitis, which is a sign of affected glomerular
basement membrane in the kidneys. What A. 60
mechanism is the most likely to cause the B. 50
basement membrane damage? C. 70
D. 80
A. Immune complex E. 100
B. Granulomatous
C. Antibody-mediated 41. Glomerular filtration of a person, who
D. Reaginic has been starving for a long time, has
E. Cytotoxic increased by 20%. The most likely cause of
filtration changes in the given conditions is:
36. Several minutes after a dentist admini-
stered novocaine for local anaesthesia of
a patient’s tooth, the following symptoms
sharply developed in the patient: fatigue,
Крок 1 Medicine (англомовний варiант, iноземнi студенти) 2015 рiк 5

A. Decrease of blood plasma oncotic A. Immune-enzyme analysis


pressure B. Radioimmunoassay technique
B. Increase of systemic blood pressure C. Immunofluorescence test
C. Increase of renal filter permeability D. Bordet-Gengou test
D. Increase of filtration factor E. Antigen-binding assay
E. Increase of renal plasma flow
47. A surgeon has to find the common
42. Cells of healthy liver actively synthesize hepatic duct during operative intervention
glycogen and proteins. What organelles are for treatment of concrements in the gall
the most developed in them? ducts. The common hepatic duct is located
between the leaves of:
A. Granular and agranular endoplasmic
reticulum A. Hepatoduodenal ligament
B. Cell center B. Hepatogastric ligament
C. Lysosomes C. Hepatorenal ligament
D. Mitochondria D. Round ligament of liver
E. Peroxisomes E. Venous ligament
43. Atria of an experimental animal were 48. It is known that the gene responsible for
superdistended with blood, which resulted development of blood groups according to
in decreased reabsorption of N a+ and water AB0 system has three allele variants. Exi-
in renal tubules. This can be explained by stence of the IV blood group can be explai-
the influence of the following factor on ki- ned by the following variability form:
dneys:
A. Combinative
A. Natriuretic hormone B. Mutational
B. Aldosterone C. Phenotypic
C. Renin D. Genocopy
D. Angiotensin E. Phenocopy
E. Vasopressin
49. When measuring power inputs of a
44. A patient with hypertensic crisis has person by the method of indirect calori-
increased content of angiotensin II in blood. metry the following results were obtai-
Angiotensin pressor effect is based on: ned: oxygen consumption is 1000 ml and
carbon dioxide production is 800 ml per mi-
A. Contraction of arteriole muscles nute. The person under examination has the
B. Activation of biogenic amine synthesis following respiratory coefficient:
C. Prostaglandin hyperproduction
D. Vasopressin production stimulation A. 0,8
E. Activation of kinin–kallikrein system B. 1,25
C. 0,9
45. A 43-year-old-patient has arterial D. 0,84
hypertension caused by increase in cardi- E. 1,0
ac output and general peripheral resi-
stance. Specify the variant of hemodynamic 50. On examination of a newborn boy’s
development of arterial hypertension in the external genitalia a fissure in the urethra
given case: opening on the inferior surface of his penis
is detected. What maldevelopment is it?
A. Eukinetic
B. Hyperkinetic A. Hypospadias
C. Hypokinetic B. Hermaphroditism
D. Combined C. Epispadia
E. - D. Monorchism
E. Cryptorchidism
46. A patient has been hospitalised
with provisional diagnosis of virus B 51. Poisoning caused by mercury (II) chlori-
hepatitis. Serological reaction based on de (corrosive sublimate) occurred in the
complementation of antigen with antibody result of safety rules violation. In 2 days
chemically bound to peroxidase or alkaline the patient’s diurnal diuresis became 620
phosphatase has been used for disease di- ml. The patient developed headache, vomiti-
agnostics. Name this serological reaction: ng, convulsions, dyspnea; moist crackles are
observed in the lungs. Name this pathology:
Крок 1 Medicine (англомовний варiант, iноземнi студенти) 2015 рiк 6

A. Acute renal failure and tonsils are sharply hyperemic ("flaming


B. Chronic renal failure pharynx"), "crimson tongue". On the tonsi-
C. Uremic coma ls surface there are isolated greyish necrosis
D. Glomerulonephritis focuses. What disease it might be?
E. Pyelonephritis
A. Scarlet fever
52. For people adapted to high external B. Meningococcal nasopharyngitis
temperatures profuse sweating is not C. Diphtheria
accompanied by loss of large volumes of D. Influenza
sodium chloride. This is caused by the effect E. Measles
the following hormone has on the perspi-
ratory glands: 58. A patient suffers from intermi-
ttent fevers and normalizations of body
A. Aldosterone temperature that occur during the day. The
B. Vasopressin temperature rise is observed regularly every
C. Cortisol fourth day. Specify the type of temperature
D. Tgyroxin curve:
E. Natriuretic
A. Febris internuttens
53. Along with normal hemoglobin types B. Febris continua
there can be pathological ones in the organi- C. Febris reccurens
sm of an adult. Name one of them: D. Febris hectica
E. Febris remitens
A. HbS
B. HbF 59. A woman with the III (В), Rh- blood
C. HbA1 group gave birth to a child with the II (А)
D. HbA2 blood group. The child is diagnosed with
E. HbO2 hemolytic disease of newborn caused by
54. Emotional stress causes activation of rhesus incompatibility. What blood group
hormon-sensitive triglyceride lipase in the and Rh can the father have?
adipocytes. What secondary mediator takes A. II (А), Rh+
part in this process?
B. I (0), Rh+
A. Cyclic adenosine monophosphate C. III (B), Rh+
B. Cyclic guanosine monophosphate D. I (0), Rh−
C. Adenosine monophosphate E. II (A), Rh−
D. Diacylglycerol
E. Ions of 2+ 60. A patient is diagnosed with hereditary
coagulopathy that is characterised by factor
55. A patient has been diagnosed with VIII deficiency. Specify the phase of blood
alkaptonuria. Choose an enzyme that can clotting during which coagulation will be di-
cause this pathology when deficient: srupted in the given case:
A. Homogentisic acid oxidase A. Thromboplastin formation
B. Phenylalanine hydroxylase B. Thrombin formation
C. Glutamate dehydrogenase C. Fibrin formation
D. Pyruvate dehydrogenase D. Clot retraction
E. Dioxyphenylalanine decarboxylase E. -
56. As a result of a continuous chronic 61. Angiocardiography of a 60-year-old man
encephalopathy a patient has developed revealed constriction of the vessel located in
spontaneous motions and disorder of torso the left coronary sulcus of his heart. Name
muscle tone. These are the symptoms of the this pathological vessel:
disorder of the following conduction tract:
A. Ramus circumflexus
A. Tractus rubrospinalis B. Ramus interventricularis posterior
B. Tractus corticospinalis C. A. coronaria dextra
C. Tractus corticonuclearis D. V. cordis parva
D. Tractus spinothalamicus E. Ramus interventricularis anterior
E. Tractus tectospinalis
62. A patient complains of pain in the ri-
57. A child is 10 years old. The followi- ght lateral abdomen. Palpation revealed a
ng presentations have developed: sharp dense, immobile, tumor-like formation. The
pain during swallowing, swollen neck, body tumor is likely to be found in the following
temperature rise up to 39, 0o C , bright-red fi- part of the digestive tube:
nely papular rash all over the body. Pharynx
Крок 1 Medicine (англомовний варiант, iноземнi студенти) 2015 рiк 7

A. Colon ascendens exception:


B. Colon transversum
C. Colon descendens A. Acetylcholine
D. Colon sigmoideum B. Noradrenaline
E. Caecum C. Adrenaline
D. Dopamine
63. During regular check-up a child is E. Serotonin
detected with interrupted mineralization of
the bones. What vitamin deficiency can be 68. A patient with urolithiasis has developed
the cause? severe pain attacks. For pain shock preventi-
on he was administered an antispasmodic
A. Calciferol narcotic analgesic along with atropine.
B. Riboflavin Name this drug:
C. Tocopherol
D. Folic acid A. Promedol
E. Cobalamin B. Nalorphine
C. Tramadol
64. During hystological analysis of the D. Ethylmorphine hydrochloride
lymph node situated in the posterior E. Morphine hydrochloride
neck triangle of an 18-year-old pati-
ent a morphologist detected a cluster 69. A patient with acute myocardial infarcti-
of cells including the following: isolated on has been administered heparin as a part
multinucleate Reed-Sternberg cells, large of complex therapy. Some time after heparin
and small Hodgkin’s cells and numerous injection the patient developed hematuria.
lymphocytes, isolated plasma cells, eosi- What heparin antagonist should be injected
nophils. What disease has developed in the to remove the complication?
patient?
A. Protamine sulfate
A. Lymphogranulomatosis B. Vicasol
B. Nodular lymphoma C. Aminocaproic acid
C. Burkitt’s lymphoma D. Neodicumarin
D. Lymphocytic lymphoma E. Fibrinogen
E. Chronic lymphocytic leukemia
70. A 37-year-old woman complains of
65. An infant has pylorospasm, weakness, headache, vertigo, troubled sleep, numbness
hypodynamia, convulsions as a result of of limbs. For the last 6 years she has been
frequent vomiting. What kind of acid-base working at a gas-discharge lamp-producing
disbalance is it? factory in a lead-processing shop. Blood test
findings: low hemoglobin and RBC level,
A. Excretory alkalosis serum iron concentration exceeds the norm
B. Excretory acidosis by several times. Specify the type of anemia:
C. Metabolic acidosis
D. Exogenous nongaseous acidosis A. Iron refractory anemia
E. Gaseous alkalosis B. Iron-deficiency anemia
C. Minkowsky-Shauffard disease
66. A 39-year-old man who had been D. Hypoplastic anemia
operated for the stomach ulcer died 7 E. Metaplastic anemia
days after the surgery. Autopsy revealed
that peritoneal leaves were dull, plephoric, 71. Despite the administration of cardi-
covered with massive yellow-greenish films, otonics and thiazide diuretic a patient with
the peritoneal cavity contained about 300 chronic heart failure has persistent edemas
ml of thick yellow-greenish liquid. What and the risk of ascites arose. What medicati-
pathologic process was revealed in the peri- on should be administered to enhance the
toneal cavity? diuretic effect of the administered drugs?
A. Fibrinous suppurative peritonitis A. Spironolactone
B. Serous peritonitis B. Furosemide
C. Fibrinous serous peritonitis C. Amiloride
D. Peritoneal commissures D. Clopamide
E. Fibrinous haemorrhagic peritonitis E. Manithol
67. Monoamine oxidase inhibitors are 72. Acute renal impairment caused death
widely used as psychopharmacological of a patient with hemorrhage. Autopsy
drugs. They change the level of nearly revealed enlarged kidneys with broad pale-
all neurotransmitters in synapses, with pink cortical layer expressively demarcated
the following neurotransmitter being the from dark-red renal pyramids. Macroscopic
examination revealed lack of epithelial
Крок 1 Medicine (англомовний варiант, iноземнi студенти) 2015 рiк 8

nuclei of convoluted tubules, tubulorrhexis, th hepatic coma. Disorders of the central


phlebostasis. The cell nuclei of choroid nervous system develop due to accumulati-
glomus and straight tubules were present. on of the following metabolite:
What pathology is it?
A. Ammonia
A. Necronephrosis B. Urea
B. Infarction C. Glutamine
C. Glomerulonephritis D. Bilirubin
D. Pyelonephritis E. Histamine
E. Nephrosis
77. When playing a child received a hit to the
73. A 3-year-old child with meningeal presternum region. As a result of this trauma
symptoms died. Postmortem macroscopy an organ located behind the presternum was
of the pia matter revealed miliary nodules damaged. Name this organ:
which were microscopically represented by
a focus of caseous necrosis with masses of A. Thymus
epithelioid and lymphoid cells with large B. Thyroid gland
cells containing crescent-shaped peripheral C. Heart
nuclei situated between them. Specify the D. Pericardium
type of meningitis in the child: E. Larynx

A. Tuberculous 78. A child suffers from dry cough. What


B. Syphilitic non-narcotic antitussive drug will relieve the
C. Brucellar patient’s condition?
D. Grippal
E. Meningococcal A. Glaucine hydrochloride
B. Codeine phosphate
74. A 66-year-old woman had intravenous C. Morphine hydrochloride
injection of magnesium sulfate solution D. Potassium iodide
to stop hypertensive crisis. However her E. Althaea officinalis root extract
arterial pressure did not decrease and
after repeated introduction of the same 79. A patient complains of acute pain attacks
preparation she developed sluggishness, in the right lumbar region. During exami-
slow response to stimuli; the patient is nation a nephrolithic obturation of the ri-
unconsciousness and her respiration is inhi- ght ureter in the region between its abdomi-
bited. What preparation is antagonist of nal and pelvic segments has been detected.
magnesium sulfate and can remove the What anatomical boundary exists between
symptoms of its overdose? those two segments?

A. Calcium chloride A. Linea terminalis


B. Potassium chloride B. Linea semilunaris
C. Sodium chloride C. Linea arcuata
D. Activated carbon D. Linea transversa
E. Potassium permanganate E. Linea inguinalis

75. A patient working at a pig farm complai- 80. A patient has insufficient blood supply
ns of paroxysmal abdominal pain, liquid to the kidneys, which has caused the
feces with mucus and blood, headache, development of pressor effect due to
weakness, fever. Examination of large constriction of arterial resistance vessels.
intestine revealed ulcers from 1 mm up to This condition results from the vessels being
several cm in diameter, feces contained oval strongly affected by the following substance:
unicellular organisms with cilia. What di- A. Angiotensin II
sease can be suspected? B. Angiotensinogen
A. Balantidiasis C. Renin
B. Amebiasis D. Catecholamines
C. Toxoplasmosis E. Norepinephrine
D. Lambliasis 81. In a village a case of anthrax has been
E. Trichomoniasis registered. Medical services began epidemi-
76. An unconscious patient was delivered ologically indicated specific prophylaxis of
by ambulance to the hospital. On objecti- population against anthrax. What preparati-
ve examination the patient was found to on was used for this purpose?
present no reflexes, periodical convulsi-
ons, irregular breathing. After laboratory
examination the patient was diagnosed wi-
Крок 1 Medicine (англомовний варiант, iноземнi студенти) 2015 рiк 9

A. Live vaccine Objectively: the patient’s weight is 117


B. Inactivated vaccine kg, he has moon-like hyperemic face, red-
C. Chemical vaccine blue striae of skin distension on his belly.
D. Genetically engineered vaccine Osteoporosis and muscle dystrophy are
E. Anatoxin present. AP is 210/140 mm Hg. What is the
most probable diagnosis?
82. Experimental stimulation of the
sympathetic nerve branches that innervate A. Cushing’s disease
the heart caused an increase in the force of B. Cushing’s syndrome
heart contractions because the membrane C. Conn’s disease
of typical cardiomyocytes permitted an D. Diabetes mellitus
increase in: E. Essential hypertension
A. Calcium ion entry 87. 2 days after labour a woman developed
B. Calcium ion exit shock along with DIC syndrome that
C. Potassium ion exit caused her death. Autopsy revealed
D. Potassium ion entry purulent endomyometritis, regional
E. Calcium and potassium ion exit purulent lymphangitis, lymphadenitis and
purulent thrombophlebitis. There were
83. According to the results of glucose also dystrophic alterations and interstitial
tolerance test a patient has no disorder inflammation of parenchymal organs. What
of carbohydrate tolerance. Despite that is the most likely diagnosis?
glucose is detected in the patients’s urine (5
mmol/l). The patient has been diagnosed wi- A. Septicemia
th renal diabetes. What renal changes cause B. Syphilis
glucosuria in this case? C. Tuberculosis of genital organs
D. Chorioadenoma destruens
A. Decreased activity of glucose reabsorpti- E. Hydatid mole
on enzymes
B. Increased activity of glucose reabsorption 88. In case of alkaptonuria, homogentisic
enzymes acid is excreted in urine in large amounts.
C. Exceeded glucose reabsorption threshold The development of this disease is associ-
D. Increased glucose secretion ated with metabolic disorder of the followi-
E. Increased glucose filtration ng amino acid:
84. Alveolar space of acinus was invaded by A. Tyrosine
bacteria that interacted with the surfactant. B. Phenylalanine
This led to the activation of the cells that C. Alanine
are localized in the alveolar walls and on the D. Methionine
surface. Name these cells: E. Asparagine
A. Alveolar macrophages 89. When blood circulation in the damaged
B. Alveolocytes type I tissue is restored, lactate accumulation stops
C. Endothelial cells and glucose consumption decelerates. These
D. Clara cells metabolic changes are caused by activation
E. Alveolocytes type II of the following process:
85. Parents of a 5-year-old boy report him A. Aerobic glycolysis
to have frequent colds that develop into B. Anaerobic glycolysis
pneumonias, presence of purulent rashes C. Lipolysis
on the skin. Laboratory tests have revealed D. Gluconeogenesis
the following: absence of immunoglobuli- E. Glycogen biosynthesis
ns of any type, and naked cells are absent
from the lymph nodes punctate. What kind 90. A doctor examined a patient, studied the
of immune disorder is it? blood analyses, and reached a conclusion,
that peripheral immunogenesis organs are
A. X-linked hypogammaglobulinemia affected. What organs are the most likely to
(Bruton type agammaglobulinemia) be affected?
B. Autosomal recessive agammaglobuli-
naemia (Swiss type) A. Tonsils
C. Hypoplastic anemia B. Thymus
D. Agranulocytosis C. Kidneys
E. Louis-Barr syndrome D. Red bone marrow
E. Yellow bone marrow
86. Examination of a 42-year-old patient
revealed a tumour of adenohypophysis. 91. A doctor asked a patient to make a deep
exhalation after a normal inhalation. What
Крок 1 Medicine (англомовний варiант, iноземнi студенти) 2015 рiк 10

muscles contract during such exhalation? skin. What is the most likely diagnosis?
A. Abdominal muscles A. Systemic lupus erythematosus
B. External intercostal muscles B. Nephrotic syndrome
C. Diaphragm C. Rheumatism
D. Trapezius muscles D. Dermatomyositis
E. Pectoral muscles E. Periarteritis nodosa
92. A 4-year-old child with hereditary 97. When studying the signs of pulmonary
renal lesion has signs of rickets; vitamin ventilation, reduction of forced expiratory
D concentration in blood is normal. What volume has been detected. What is the li-
is the most probable cause of rickets kely cause of this phenomenon?
development?
A. Obstructive pulmonary disease
A. Impaired synthesis of calcitriol B. Increase of respiratory volume
B. Increased excretion of calcium C. Increase of inspiratory reserve volume
C. Hyperfunction of parathyroid glands D. Increase of pulmonary residual volume
D. Hypofunction of parathyroid glands E. Increase of functional residual lung
E. Lack of calcium in food capacity
93. In a cat with decerebrate rigidity the 98. A specimen of a parenchymal organ
muscle tone is to be decreased. This can be shows poorly delineated hexagonal lobules
achieved by: surrounding a central vein, and the
interlobular connective tissue contains
A. Destruction of the vestibular nuclei of embedded triads (an artery, a vein and an
Deiters excretory duct). What organ is it?
B. Stimulation of the otolithic vestibular
receptors A. Liver
C. Stimulation of the vestibular nuclei of B. Pancreas
Deiters C. Thymus
D. Stimulation of the vestibulocochlear nerve D. Spleen
E. Stimulation of the ampullar vestibular E. Thyroid
receptors
99. A patient had a trauma that caused
94. Autopsy of a 5-year-old child revealed dysfunction of motor centres regulating
in the area of vermis of cerebellum a soft activity of head muscles. In what parts of
greyish-pink node 2 cm in diameter with cerebral cortex can the respective centre
blurred margins and areas of haemorrhage. normally be located?
Histologically this tumour consisted of
atypical monomorphous small round cells A. Inferior part of precentral gyrus
with large polymorphous nuclei. What B. Superior part of precentral gyrus
tumour is it? C. Supramarginal gyrus
D. Superior parietal lobule
A. Medulloblastoma E. Angular gyrus
B. Meningioma
C. Glioblastoma 100. During intravenous saline transfusion
D. Astrocytoma a patient’s condition deteriorated drasti-
E. Oligodendroglioma cally, and the patient died from asphyxiati-
on. Autopsy revealed acute venous congesti-
95. Surface with an intact toad on it was on of internal organs with sharp right heart
inclined to the right. Tone of extensor dilatation. When the right ventricle was
muscles became reflectory higher due to the punctured underwater, the bubbles escaped.
activation of the following receptors: What pathological process occurred in the
patient?
A. Vestibuloreceptors of utricle and saccule
B. Vestibuloreceptors of semicircular ducts A. Air embolism
C. Mechanoreceptors of foot skin B. Gaseous embolism
D. Photoreceptors of retina C. Adipose embolism
E. Proprioreceptors D. Tissue embolism
E. Thromboembolism
96. A patient with high-titer antinuclear
antibodies died from progressing renal 101. Diseases of respiratory system and ci-
impairment. Autopsy revealed mesangi- rculatory disorders impair the transport of
oproliferative glomerulonephritis and oxygen, thus causing hypoxia. Under these
abacterial polypous endocarditis. Periarteri- conditions the energy metabolism is carri-
al bulbar sclerosis was detected in spleen ed out by anaerobic glycolysis. As a result,
and productive proliferative vasculitis in
Крок 1 Medicine (англомовний варiант, iноземнi студенти) 2015 рiк 11

the following substance is generated and 107. An alcoholic has alcoholic psychosis
accumulated in blood: with evident psychomotor agitation. What
neuroleptic drug should be administered for
A. Lactic acid emergency aid?
B. Pyruvic acid
C. Glutamic acid A. Aminazine
D. Citric acid B. Diazepam
E. Fumaric acid C. Sodium bromide
D. Reserpine
102. Examination of a 6-month-old child E. Halothane
revealed a delay in closure of the occipital
fontanelle. When should it normally close? 108. During blood transfusion a patient
has developed intravascular erythrocyte
A. Before 3 months hemolysis. What kind of hypersensitivity
B. Before the child is born does the patient have?
C. Before 6 months
D. Before the end of the first year of life A. II type (antibody-dependent)
E. Before the end of the second year of life B. I type (anaphylactic)
C. III type (immune complex)
103. Sex chromosomes of a woman didn’t
separate and move to the opposite poles D. IV type (cellular cytotoxicity)
of a cell during gametogenesis (meiosis). E. IV type (granulomatosis)
The ovum was impregnated with a normal 109. In the course of an experiment
spermatozoon. Which chromosomal disease thalamocortical tracts of an animal were cut.
can be found in her child? What type of sensory perception remained
A. Turner’s syndrome intact?
B. Down’s syndrome A. Olfactory
C. Patau’s syndrome B. Auditory
D. Edwards’ syndrome C. Exteroreceptive
E. Cat cry syndrome D. Visual
104. A patient was prescribed loratadine to E. Nociceptive
treat allergic cheilitis. What is the mechani- 110. A 4-year-old child presents with
sm of action of this drug? general weakness, sore throat and deglutiti-
A. Blockade of H1 -histamine receptors ve problem. After his examination a doctor
B. Blockade of adrenergic receptors suspected diphtheria and sent the material
C. Increases activty of monoamine oxidase to a bacteriological laboratory. To determine
the diphtheria causative agent the material
D. Suppresses activity of N a+ /K + -ATPase should be inoculated into the following di-
E. Suppresses activity of choline esterase fferential diagnostic medium:
105. During introduction of local anesthesia A. Blood tellurite agar
a patient has gone into anaphylactic shock. B. Endo’s agar
What drug must be administered to the pati- C. Ploskyrev’s agar
ent? D. Sabouraud’s agar
A. Epinephrine hydrochloride E. Levenshtein-Yessen agar
B. Diazepam 111. When treating a patient with
C. Atropine sulfate chronic cardiac failure a doctor detected
D. Propranolol bradycardia and deterioration of the pati-
E. Nitroglycerin ent’s general state. Such condition is caused
106. A patient has been diagnosed wi- by cumulative effect of a drug. Which drug
th influenza. His condition drastically of those listed below has cumulative action?
worsened after taking antipyretic drugs. He A. Digoxin
is unconscious, AP is 80/50 mm Hg, Ps is
140/m, body temperature dropped down to B. Diphenhydramine (Dimedrol)
35, 8o C . What complication developed in C. Hydrochlorothiazide
this patient? D. Isosorbide
E. Retinol acetate
A. Collapse
B. Hyperthermia 112. A doctor was addressed by a 30-year-
C. Hypovolemia old man. There is a probability of the patient
D. Acidosis being HIV-positive. To clarify the diagnosis
E. Alkalosis the doctor proposed to perform polymerase
chain reaction. The basic process in this kind
Крок 1 Medicine (англомовний варiант, iноземнi студенти) 2015 рiк 12

of investigation is: A. Low level of carnitine


B. Impaired phospholipid synthesis
A. Gene amplification C. Excessive consumption of fatty foods
B. Transcription D. Choline deficiency
C. Genetic recombination E. Lack of carbohydrates in the diet
D. Genomic mutation
E. Chromosome mutation 118. During ventricular systole the cardiac
muscle does not respond to additional sti-
113. Due to the use of poor-quality measles mulation because it is in the phase of:
vaccine for preventive vaccination, a 1-
year-old child developed an autoimmune A. Absolute refractoriness
renal injury. The urine was found to contain B. Relational refractoriness
macromolecular proteins. What process of C. Hyperexcitability
urine formation was disturbed? D. Subnormal excitability
E. There is no correct answer
A. Filtration
B. Reabsorption 119. A patient is diagnosed with acute
C. Secretion morphine hydrochloride intoxication.
D. Reabsorption and secretion Prescribe the oxidizing agent for gastric
E. Secretion and filtration lavage:
114. A 26-year-old female patient with A. Potassium permanganate
bronchitis has been administered a broad B. Chloramine
spectrum antibiotic as a causal treatment C. Sulfocamphocainum (Procaine +
drug. Specify this drug: Sulfocamphoric acid)
D. Cerigel
A. Doxycycline E. Chlorhexidine (bi)gluconate
B. Interferon
C. BCG vaccine 120. During cell division DNA replication
D. Ambroxol occurs after a signal is received from the
E. Dexamethasone cytoplasm, then a certain portion of the
DNA helix unwinds and splits into two indi-
115. A 65-year-old man suddenly lost vision vidual strains. What enzyme facilitates this
in one eye due to the retinal detachment. process?
The patient underwent enucleation. Hi-
stological examination of the removed eye A. Helicase
retina and choroid revealed clusters of atypi- B. RNA polymerase
cal cells with marked polymorphism of cells C. Ligase
and nuclei, with a moderate number of mi- D. Restrictase
toses including the pathological ones. The E. DNA polymerase
cell cytoplasm and intercellular medium
contained brown pigment resulting in posi- 121. A patient has been given atropine
tive DOPA reaction. Perls’ reaction was sulfate for rapid relief of spastic colon
negative. What is the most likely diagnosis? symptoms. The use of this drug is contrai-
ndicated during the followng disease:
A. Melanoma
B. Pigmented mole A. Glaucoma
C. Hemorrhage B. Bronchial asthma
D. Cysticercosis C. Bradycardia
E. Wilson’s disease D. Hypotension
E. Gastric ulcer
116. During determining the blood group
according to the AB0 system with salt soluti- 122. As an example of specific human parasi-
ons of monoclonal antibodies agglutination tes one can name Plasmodium falciparum,
did not occur with any of the solutions. What human pinworm and some others. The
blood group is it? source of parasite invasion in these cases is
always a human. Such specific human parasi-
A. 0 (I) tes cause the diseases that are called:
B. А (II)
C. В (III) A. Anthroponoses
B. Zoonoses
D. АВ (IV) C. Anthropozoonoses
E. - D. Infections
117. One of the factors that cause obesity is E. Multifactorial diseases
inhibition of fatty acids oxidation due to: 123. In the course of an experiment there
has been increase in nerve conduction
Крок 1 Medicine (англомовний варiант, iноземнi студенти) 2015 рiк 13

velocity. This may be caused by increase in nerve was damaged. This caused an impai-
concentration of the following ions that are rment of the upper limb activity. The patient
present in the solution around the cell: was offered nerve transplantation. What gli-
al cells will participate in regeneration and
A. N a+ provide the trophism of the injured limb?
B. K + and Cl−
C. K + and N a+ A. Schwann cells
D. Ca2+ and Cl− B. Fibrous cells
E. Ca2+ C. Protoplasmic cells
D. Microglia
124. An HIV-positive patient’s cause of E. Ependymal cells
death is acute pulmonary insufficiency
resulting from pneumonia. Pathohistological 129. A 47-year-old man developed intesti-
investigation of lungs has revealed intersti- nal colic against the background of essential
tial pneumonia, alveolocyte desquamati- hypertension. In this situation it would be
on and methamorphoses: alveolocyte most efficient to arrest the colic by admini-
enlargement, large intranuclear inclusi- stering drugs of the following group:
ons surrounded by lightly-coloured areas.
Transformed cells resemble owl’s eye. Name A. Myotropic antispasmodics
the causative agent of pneumonia: B. Anticholinesterase agents
C. Sympathomimetics
A. Cytomegalovirus D. M-cholinomimetics
B. Pneumococcus E. Adrenomimetics
C. Influenza virus
D. Candida fungi 130. Microscopy of a female patient’s swabs
E. Toxoplasma made from vaginal secretion revealed gram-
negative bean-shaped diplococci. What
125. The organisms to be identified have a provisional diagnosis can be made?
nucleus surrounded by a nuclear membrane.
Genetic material is concentrated predomi- A. Gonorrhoea
nantly in the chromosomes that consist B. Syphilis
of DNA strands and protein molecules. C. Clamidiosis
These cells divide mitotically. Identify these D. Mycoplasmosis
organisms: E. Toxoplasmosis

A. Eukaryotes 131. A 54-year-old woman was brought to a


B. Bacteriophages casualty department after a car accident. A
C. Prokaryotes traumatologist diagnosed her with multiple
D. Viruses fractures of the lower extremities. What ki-
E. Bacteria nd of embolism is most likely to develop in
this case?
126. A 2-year-old boy is diagnosed wi-
th Down syndrome. What chromosomal A. Adipose
changes may be the cause of this disease? B. Tissue
C. Thromboembolism
A. Trisomy 21 D. Gaseous
B. Trisomy 13 E. Air
C. Trisomy X
D. Trisomy 18 132. A therapeutist has an appointment
E. Monosomy X with a 40-year-old patient complaining of
recurrent pain attacks in his hallux joints
127. After a road accident a victim has and their swelling. Urine analysis revealed
tachycardia, arterial blood pressure 130/90 its marked acidity and pink colour. What
mm Hg, tachypnoe, the skin is pale and substances can cause such changes in the
dry, excitation of central nervous system is urine?
observed. What shock stage is the patient
most likely in? A. Uric acid salt
B. Chlorides
A. Erectile C. Ammonium salts
B. Terminal D. Calcium phosphate
C. Torpid E. Magnesium sulfate
D. Preshock (compensation stage)
E. Agony 133. A 30-year-old man with diabetes
mellitus type I was hospitalised. The
128. As a result of a mechanical injury an patient is comatose. Laboratory tests
over 10 cm long portion of a peripheral revealed hyperglycemia and ketonemia.
What metabolic disorder can be detected
Крок 1 Medicine (англомовний варiант, iноземнi студенти) 2015 рiк 14

in this patient? following protein:


A. Metabolic acidosis A. α1 -proteinase inhibitor
B. Metabolic alkalosis B. α2 -macroglobulin
C. Respiratory acidosis C. Cryoglobulin
D. Respiratory alkalosis D. Haptoglobin
E. Normal acid-base balance E. Transferrin
134. A 15-year-old patient has fasting 139. A patient, who has been suffering for
plasma glucose level 4,8 mmol/l, one hour a long time from intestine disbacteriosis,
after glucose challenge it becomes 9,0 has increased hemorrhaging caused by di-
mmol/l, in 2 hours it is 7,0 mmol/l, in 3 sruption of posttranslational modification of
hours it is 4,8 mmol/l. Such parameters are blood-coagulation factors II, VII, IХ, and Х
characteristic of: in the liver. What vitamin deficiency is the
cause of this condition?
A. Subclinical diabetes mellitus
B. Diabetes mellitus type 1 A. К
C. Diabetes mellitus type 2 B. 12
D. Healthy person C. 9
E. Cushing’s disease D. С
E. Р
135. A patient has undergone surgical
removal of a cavitary liver lesion 2 cm 140. During a surgery for femoral hernia a
in diameter. It was revealed that the surgeon operates within the boundaries of
cavity wall was formed by dense fibrous femoral trigone. What structure makes up
connective tissue; the cavity contained its upper margin?
murky thick yellowish-green fluid with an
unpleasant odor. Microscopically the fluid A. Lig. inguinale
consisted mainly of polymorphonuclear B. Arcus iliopectineus
leukocytes. What pathological process are C. Lig. lacunare
these morphological changes typical for? D. Lig. pectinale
E. Fascia lata
A. Chronic abscess
B. Acute abscess 141. An obstetrician-gynecologist measures
C. Phlegmon pelvis size of a pregnant woman. A caliper
D. Empyema was used to measure the distance between
E. - the two iliac crests. What measurement of
large pelvis was made?
136. A patient consulted a physician about
chest pain, cough, fever. Roentgenography A. Distantia cristarum
of lungs revealed eosinophilic infiltrates B. Distantia throchanterica
which were found to contain the larvae. C. Distantia spinarum
What kind of helminthiasis are these D. Conjugata vera
presentations typical for? E. Conjugata anatomica

A. Ascariasis 142. A patient has arterial hypertension.


B. Echinococcosis What long-acting drug from the group of
C. Fascioliasis calcium channel blockers should be prescri-
D. Cysticercosis bed?
E. Trichinosis
A. Amlodipine
137. During appendectomy a patient had B. Octadine
the a. appendicularis ligated. This vessel C. Pyrroxanum
branches from the following artery: D. Atenolol
E. Reserpine
A. A. Ileocolica
B. A. colica dextra 143. A patient has been diagnosed
C. A. colica media with URTI. Blood serum contains
D. A. sigmoidea immunoglobulin M. What stage of infecti-
E. A. mesenterica inferior on is it?
138. A 28-year-old patient undergoing A. Acute
treatment in a pulmonological department B. Prodromal
has been diagnosed with pulmonary C. Incubation
emphysema caused by splitting of alveolar D. Reconvalescence
septum by tissular tripsin. The disease is E. Carriage
caused by the congenital deficiency of the
Крок 1 Medicine (англомовний варiант, iноземнi студенти) 2015 рiк 15

144. A 43-year-old patient suffers from acute A. Sphygmography


pancreatitis with disrupted common bile B. Plethysmography
duct patency. What condition can develop C. Rheography
in this case? D. Myography
E. Phlebography
A. Mechanical jaundice
B. Hemolytic jaundice 150. A patient has developed paroxysmal
C. Hepatocellular jaundice ventricular tachycardia against the
D. Hepatic coma background of cardiac infarction. What anti-
E. Portal hypertension arrhythmic drug should be chosen to avoid
lowering cardiac output?
145. A patient has a tumor of the eyesocket
tissues behind the eyeball. Disruption of A. Lidocaine hydrochloride
accomodation and pupil constriction is B. Procainamide
observed. What anatomical structure is C. Verapamil
damaged? D. Propranolol
E. Potassium chloride
A. Ganglion ciliare
B. N. nasociliaris 151. Electrocardiogram of a young man
C. N. lacrimalis reveals deviation of his electrical axis of
D. N. opticus heart to the left. This phenomenon can be
E. N. trochlearis caused by:
146. Lymphocytes and other cells of our A. Hypersthenic body type
body synthesize universal antiviral agents B. Asthenic body type
as a response to viral invasion. Name these C. Dilation of the right atrium
protein factors: D. Dilation of the right ventricle
E. Dilation of the left atrium
A. Interferon
B. Interleukin - 2 152. A 6-year-old child suffers from delayed
C. Cytokines growth, disrupted ossification processes,
D. Interleukin - 4 decalcification of the teeth. What can be the
E. Tumor necrosis factor cause?
147. A patient consulted a dentist about A. Vitamin D deficiency
restricted mouth opening (trismus). B. Decreased glucagon production
Anamnesis states a stab wound of the lower C. Insulin deficiency
extremity. What infection may cause these D. Hyperthyroidism
symptoms? E. Vitamin C deficiency

A. Tetanus 153. A 30-year-old patient’s blood test


B. Brucellosis revealed the following: erythrocyte count
C. Whooping cough is 6 · 1012 /l, hemoglobin is 10,55ммоль/l.
D. Wound anaerobic infection Vaquez’s disease was diagnosed. Name the
E. Tularemia leading part of pathogenesis:
148. A patient has damaged spinal cord whi- A. Neoplastic erythroid hyperplasia
te matter in the middle area of the posteri- B. Iron-deficiency
or white column, disrupted propriocepti- C. B12 -deficiency
ve sensibility of the lower limb joints and D. Hypoxia
muscles. What fibers are affected? E. Acidosis
A. Fasciculus gracilis 154. A 25-year-old patient complains of
B. Tr. spinothalamicus lateralis increasing pain in his leg muscles occurri-
C. Tr. spinocerebellaris anterior ng during walking and forcing him to make
D. Tr. spinocerebellaris posterior frequent stops. Objectively: skin of legs
E. Fasciculus cuneatus is pale, no hair-covering, toenails are wi-
th trophic changes, no pulsation of pedal
149. In an elderly person the change in heart arteries. The most probable cause of these
force and vessels physical properties were changes is:
detected; they can be clearly observed on
graphic recording of carotid pulse waves. A. Ischemia
What method was applied? B. Venous hyperemia
C. Arterial hyperemia
D. -
E. Embolism
Крок 1 Medicine (англомовний варiант, iноземнi студенти) 2015 рiк 16

155. This year influenza epidemic is A. Low-density lipoproteins


characterised by patients’ body temperature B. High-density lipoproteins
varying from 36, 9o C to 37, 9o C . Such fever C. Chylomicrons
is called: D. Very low-density lipoproteins
E. Middle-density lipoproteins
A. Subfebrile
B. High 161. Obesity is a common disease. The
C. Hyperpyretic aim of its treatment is to lower content
D. Apyretic of neutral fats in the body. What hormon-
E. Moderate sensitive enzyme is the most important for
intracellular lipolysis?
156. A woman complains of visual impai-
rment. Examination revealed obesity in the A. Triacylglycerol lipase
patient and her fasting plasma glucose level B. Protein kinase
is hyperglycemic. What diabetes complicati- C. Adenylate kinase
on can cause visual impairment/blindness? D. Diacylglycerol lipase
E. Monoacylglycerol lipase
A. Microangiopathy
B. Macroangiopathy 162. A 40-year-old woman was diagnosed
C. Atherosclerosis with glomerulonephritis based on her cli-
D. Neuropathy nical symptoms and the results of urine
E. Glomerulopathy analysis. Anamnesis states chronic tonsilli-
tis. What microorganisms are the most likely
157. Prolonged treatment of hypothyroidism cause for her kidney damage?
has caused general dystrophy, dental cari-
es, tachycardia, tremor of extremities. What A. Streptococci
drug is the cause of these side effects? B. Staphylococci
C. Escherichia
A. L-thyroxin D. Mycoplasma
B. Humulin (Human insulin) E. Meningococci
C. Parathyreoidinum
D. Thyrocalcitonin 163. A man is suffering from diarrhea. In
E. Prednisolone summer he spent his vacation in the south
at the sea coast. Bacteria with the following
158. A 3-year-old child has eaten some properties were detected in his feces: gram-
strawberries. Soon he developed a rash negative curved mobile monotrichous baci-
and itching. What was found in the child’s lli that do not produce spores or capsules.
leukogram? They are undemanding to nutrient medium
but require alkaline reaction (рН 8,5-09,5).
A. Eosinophilia Described are the agents of the following
B. Hypolymphemia enteric infection:
C. Neutrophilic leukocytosis
D. Monocytosis A. Cholera
E. Lymphocytosis B. Shigellosis
C. Typhoid fever
159. A patient has been found to have a D. Colienteritis
marked dilatation of the saphenous veins E. Pseudotuberculosis
in the region of anterior abdominal wall
around the navel. This is symptomatic of 164. A patient is diagnosed with chronic
pressure increase in the following vessel: atrophic gastritis attended by deficiency of
Castle’s intrinsic factor. What type of anemia
A. V. portae hepatis does the patient have?
B. V. cava superior
C. V. cava inferior A. B12 -deficiency anemia
D. V. mesenterica inferior B. Iron refractory anemia
E. V. mesenterica superior C. Hemolytic anemia
D. Iron-deficiency anemia
160. Cholesterol content in blood serum E. Protein-deficiency anemia
of a 12-year-old boy is 25 mmol/l.
Anamnesis states hereditary familial 165. A pregnant woman was detected wi-
hypercholesterolemia caused by synthesis th IgM to rubella virus. An obstetrician-
disruption of receptor-related proteins for: gynecologist recommended therapeutic
abortion due to the high risk of teratogenic
affection of the fetus. Detection of IgM was
of great importance as it is these specific
immunoglobulins that:
Крок 1 Medicine (англомовний варiант, iноземнi студенти) 2015 рiк 17

A. Hemolytic anemia
A. Indicate recent infection B. Pernicious anemia
B. Penetrate placental barrier C. Megaloblastic anemia
C. Have the largest molecular weight D. Sicklemia
D. Are associated with anaphylactic reacti- E. Iron-deficiency anemia
ons
E. Are the main factor of antiviral protection 171. In winter a 3-year-old child has
sharp rise of body temperature up to
166. A patient is diagnosed with cardi- 40o C . Hemorrhagic rash is observed on
ac infarction. Blood test for cardiospecific the skin and mucosa. Bean-shaped gram-
enzymes activity was performed. Which of negative microorganisms situated in pairs
the enzymes has three isoforms? are detected in the blood. What provisional
diagnosis can be made?
A. Creatine kinase
B. Lactate dehydrogenase A. Meningococcosis
C. Aspartate transaminase B. Gonorrhea
D. Alanine transaminase C. Scarlet fever
E. Pyruvate kinase D. Influenza
E. Diphtheria
167. A 50-year-old man, who has been
suffering from chronic hepatic failure for 172. Microelectrode analysis of nerve fi-
several years, has developed ascites. What ber biolectrical activity revealed, that its
is the main mechanism of this disorder membrane potential equals 90 mV. Its ini-
development? tial rest potential was 85 mV. What process
occurs in this case?
A. Increased pressure in portal vein system
B. Decrease of albumin and globulin A. Hyperpolarization
synthesis in liver B. Depolarization
C. Increased content of low-density and very C. Repolarization
low-density lipoproteins in blood D. Overshoot
D. Neurotoxins appearing in blood E. Supernormality
E. Increase of blood oncotic pressure
173. Parkinson’s disease is caused by di-
168. A 30-year-old man has sustained an sruption of dopamine synthesis. What brain
injury to his thorax in a traffic incident, whi- structure synthesizes this neurotransmitter?
ch caused disruption of his external respi-
ration. What type of ventilatory difficulty A. Substantia nigra
can be observed in the given case? B. Globus pallidus
C. Corpora quadrigemina
A. Restrictive extrapulmonary ventilatory D. Red nucleus
impairment E. Hypothalamus
B. Restrictive pulmonary ventilatory impai-
rment 174. Determining a patient’s blood group wi-
C. Obstructive ventilatory impairment th monoclonal test-reagents revealed positi-
D. Impaired ventilation regulation dysfuncti- ve agglutination reaction to anti-A and anti-
on B reagents, and negative reaction to anti-D.
E. Cardiovascular collapse What blood group does this patient have?
169. A 53-year-old man suffering from di- A. IV (АВ) Rh−
abetes mellitus has developed a painful B. II (А) Rh+
conical induration, bluish-red with yellow C. III (В) Rh−
center, on the skin of his neck. Such changes D. IV (АВ) Rh+
are characteristic of: E. I (0) Rh+
A. Furuncle 175. A patient visited a dentist to extract a
B. Abscess tooth. After the tooth had been extracted,
C. Carbuncle bleeding from the tooth socket continued
D. Phlegmon for 15 minutes. Anamnesis states that the
E. Empyema patient suffers from active chronic hepatitis.
What phenomenon can extend the time of
170. Biochemical analysis of an infant’s hemorrhage?
erythrocytes revealed evident glutathione
peroxidase deficiency and low concentrati-
on of reduced glutathione. What pathologi-
cal condition can develop in this infant?
Крок 1 Medicine (англомовний варiант, iноземнi студенти) 2015 рiк 18

A. Decrease of fibrinogen content in blood A. Prednisolone


B. Thrombocytopenia B. Thymus cytomedins (Thymalin)
C. Hypocalcemia C. Chloropyramine (Suprastin)
D. Increased activity of anticoagulation D. Riboflavin
system E. Fercovenum
E. Decrease of albumine content in blood
181. An experiment was aimed at testing
176. Name the halogen-containing antiseptic flexor reflex in a spinal frog, which was
with fungicidal properties, which is used to initiated by simultaneous stimulation with
treat dermatomycosis: isolated pre-threshold electrical impulses.
The frequency of those impulses was such,
A. Iodine solution that the reflex occurred. What process in the
B. Formalin solution nerve centers can be observed during this
C. Methylene blue experiment?
D. Brilliant green
E. Boric acid solution A. Temporal summation
B. Spatial summation
177. Due to severe pain syndrome a patient C. Presynaptic summation
was prescribed a narcotic analgesic. Name D. Postsynaptic summation
this drug: E. Threshold summation
A. Morphine 182. A patient diagnosed with acute
B. Metamizole (Analgin) dysentery has been treated for 3 days in an
C. Nimesulide infectious diseases hospital. On admission
D. Dimethyl sulfoxide there were complaints of high temperature,
E. Indometacin stomachache and fluid excrements with
mucus as often as 8-10 times a day. What
178. During pathomorphological kidney sample should be taken for analysis?
investigation of a patient, who for a long
time had been suffering from osteomyeli- A. Feces
tis and died from progressing renal fai- B. Urine
lure, the following was revealed: deposi- C. Bile
ts of homogeneous eosinophilic masses D. Liquor
in glomerular mesangium, arterial and E. Blood
arteriolar walls, and stroma, which became
red when stained with Congo red. What 183. An 18-year-old woman has body di-
pathological process is this? sproportion, wing-like folds on the skin of
her neck, underdeveloped ovaries, nuclei
A. Amyloidosis of her buccal epithelium cells have no Barr
B. Mucoid swelling bodies. Dermatoglyphics method revealed
C. Calcinosis that her adt angle is 66o . What provisional
D. Carbohydrate degeneration diagnosis can be made in this case?
E. Hyalinosis
A. Turner’s syndrome
179. A microslide presents a tissue with B. Cri du chat (cat cry) syndrome
spherical cells, each of them containing a C. Kleinfelter’s syndrome
large fat drop covered with thin cytoplasm D. Patau’s syndrome
layer in its center. Nucleus is compressed E. Edwards’ syndrome
and situated at the cell periphery. What ti-
ssue is it? 184. A 27-year-old patient with injury to
the neck has lost approximately 30% of
A. White adipose tissue the blood volume. The patient’s condition is
B. Brown adipose tissue severe: blood pressure is 60/40 mm Hg, heart
C. Mucous tissue rate is 140/min., respiratory rate is 30/min.,
D. Pigmented tissue conscious. Characterize the condition of the
E. Reticular tissue patient’s circulatory system:
180. To treat rheumatoid arthritis a A. Hypovolemic shock
65-year-old woman was prescribed an B. Cardiogenic shock
immunosuppressive hormonal drug as a part C. Collapse
of her complex therapy. Name this drug: D. Coma
E. Arterial hypertension
185. A soldier with explosion-caused trauma
was delivered to a hospital. Examinati-
on revealed his tympanic membrane to be
Крок 1 Medicine (англомовний варiант, iноземнi студенти) 2015 рiк 19

intact. What defense reflex prevented the has revealed inflammatory granulomatous
tympanic membrane from rupturing? reaction surrounding necrotizing masses in
the area of the first metatarsophalangeal joi-
A. Contraction of m. tensor tympani nt. Choose the most likely diagnosis:
B. Relaxation of m. tensor tympani
C. Contraction of m. auricularis arterior A. Gout (podagra)
D. Relaxation of m. auricularis arterior B. Pyrophosphate arthropathy
E. Relaxation of m. stapedins C. Rheumatoid arthritis
D. Hyperparathyroidism
186. A person with the fourth blood group E. Urolithiasis
(genotype IAIB) has in erythrocytes both
antigen A controlled by allele IA and 191. Granulomas containing lymphocytes
antigen B controlled by allele IB. This and macrophages were detected during
phenomenon is an example of the following analysis of skin biopsy material. Among
gene interation: macrophages there are large cells with fat
inclusions, which contain microorganisms
A. Codominance in spheric packages (Virchow’s cells). The
B. Complementarity following disease is based on the described
C. Semidominance type of hypersensitivity:
D. Polymery
E. Epistasis A. Leprosy
B. Syphilis
187. During narcosis a patient developed a C. Tuberculosis
risk of cerebral edema. What drug should be D. Rhinoscleroma
administered in this case? E. Epidemic typhus
A. Furosemide 192. A patient with suspected necrosis of
B. Dopamine the upper abdominal cavity organs was deli-
C. Phenazepam vered to a surgical department. This condi-
D. Triamterene tion is associated with acute circulatory di-
E. Sodium bromide sturbance of the following vessel:
188. During surgery performed in abdomi- A. Tuncus coeliacus
nal cavity a surgeon located ligament of li- B. A. mesenterica inferior
ver stretching from anterior abdominal wall C. A. mesenterica superior
(navel) to inferior surface of liver. What li- D. A. iliaca communis
gament is it? E. A. renalis
A. Round ligament of the liver 193. Name the drug that inhibits excretory
B. Falciform ligament of the liver function of pancreas during treatment of
C. Coronary ligament of the liver acute pancreatitis:
D. Venous ligament of the liver
E. Triangular ligament of the liver A. Contrykal (Aprotinin)
B. Allochol
189. A 9-year-old boy has acute onset of di- C. Раnzynorm
sease: sore throat, body temperature rise up D. Pancreatin (Mezym forte)
to 39, 5o C ; on the second day diffuse skin E. Festal
rash was detected all over his skin exept for
nasolabial triangle. On examination of oral 194. An 18-year-old patient has developed
cavity: crimson tongue, "flaming pharynx", candidiasis after the case of pneumonia
necrotic tonsillitis. What diagnosis is the treated with β -lactam antibiotic. What anti-
most likely? mycotic agent should be prescribed?
A. Scarlet fever A. Fluconazole
B. Measles B. Streptomycin
C. Diphtheria C. Ampicillin
D. Influenza D. Phthalylsulfathiazole
E. Meningococcemia E. Trimethoprim/sulfamethoxazole (Bi-
septol)
190. A 49-year-old man complains of
pain in his metatarsophalangeal joints 195. During autopsy of a 9-month-old girl’s
and joint deformation. In blood hyperuri- body, who died due to severe pneumonia
cemy can be observed. X-ray has revealed complicated with sepsis, lack of thymus is
metatarsophalangeal joint space narrowi- observed. In the lymph nodes the lymphoid
ng, erosion, periarticular calcification of follicles and cortical substance are absent;
the both joints, osteoporosis. Microscopy follicles of spleen are reduced in size with
Крок 1 Medicine (англомовний варiант, iноземнi студенти) 2015 рiк 20

no light zones and plasma cells. What is the 198. A patient after disrupted cerebral ci-
cause of such structural changes? rculation has developed paralysis. Choose
the anticholinesterase drug to be prescribed
A. Thymus agenesis in this case:
B. Accidental involution of thymus
C. Thymus hypoplasia A. Proserin
D. Thymus atrophy B. Cordiamin
E. Thymus aplasia C. Aceclidine
D. Methacin
196. A patient with femoral neck fracture, E. Hexamethonium (Benzohexonium)
who for a long time had to remain in bed
in a forced (supine) position, has developed 199. A 50-year-old woman is being treated
dark-brown lesions along the backbone; soft for shingles in a neurology unit. What reacti-
tissues are swollen, in the areas of macerati- vated virus causes this disease?
on there is a foul-smelling liquid. Name the
clinicopathologic type of necrosis: A. Varicella zoster virus (chickenpox virus)
B. Herpes simplex virus type 1
A. Bedsore C. Herpes simplex virus type 2
B. Infarction D. Measles virus
C. Sequestrum E. Cytomegalovirus
D. Coagulation necrosis
E. Dry gangrene 200. During examination of a patient a
doctor should use anatomical division of
197. A woman poisoned with unknown anterior abdominal wall into regions for
substance was hospitalised in a toxicologi- more precise diagnostics. How many such
cal department. What group of drugs can regions can abdomen be divided into?
be administered to decrease absorption and
introduction of the poison to her body? A. 9
B. 8
A. Adsorbents C. 6
B. Neuroleptics D. 5
C. Antioxidants E. 4
D. Organic nitrates
E. Cholinesterase inhibitors
Krok 1 Medicine (англомовний варiант, iноземнi студенти) 2016 рiк 1

1. A 35-year-old man has been deli-


vered into a surgical ward with a suppurati- A. Isometric
ng wound in the neck, anterior to trachea B. Isotonic
(previsceral space). If a surgical operation C. Auxotonic
is not performed urgently, there is a risk of D. Phasic
infection spreading to: E. Single

A. Thoracic cavity - anterior mediastinum 7. A patient suffers from high fever,


B. Thoracic cavity - middle mediastinum apnoea, pain in the thorax on the right.
C. Thoracic cavity - posterior mediastinum Pleurocentesis yielded 700 ml of yellow-
D. Retrovisceral space green viscous liquid. Make the diagnosis:
E. Interaponeurotic suprasternal space
A. Pleural empyema
2. Characteristic sign of glycogenosis is B. Bronchial pneumonia
muscle pain during physical work. Blood C. Serous pleurisy
examination usually reveals hypoglycemia. D. Hemorrhagic pleurisy
This pathology is caused by congenital defi- E. Pleural carcinomatosis
ciency of the following enzyme:
8. A patient suffers from disrupted patency
A. Glycogen phosphorylase of the airways at the level of small
B. Glucose 6-phosphate dehydrogenase and medium-sized bronchial tubes. What
C. α-amylase changes of acid-base balance can occur in
D. γ -amylase the patient?
E. Lysosomal glycosidase
A. Respiratory acidosis
3. Histologic specimen of a kidney B. Respiratory alkalosis
demonstrates cells closely adjoined to the C. Metabolic acidosis
renal corpuscle in the distal convoluted D. Metabolic alkalosis
tubule. Their basement membrane is E. Acid-base balance remains unchanged
extremely thin and has no folds. These cells
9. Upon toxic damage of hepatic cells
sense the changes in sodium content of uri-
ne and influence renin secretion occurring resulting in disruption of liver function the
in juxtaglomerular cells. Name these cells: patient developed edemas. What changes of
blood plasma are the main cause of edema
A. Macula densa cells development?
B. Juxtaglomerular cells
C. Mesangial cells A. Decrease of albumin content
B. Increase of globulin conten
D. Podocytes
E. Glomerular capillary endothelial cells C. Decrease of fibrinogen conten
D. Increase of albumin conten
4. Bacteriological analysis of tap water has E. Decrease of globulin conten
resulted in the following: total bacterial
count in 1,0 ml of water is 80, coli index 10. A 6-year-old child with suspected acti-
is 3. What would be the conclusion? ve tuberculous process has undergone di-
agnostic Mantoux test. What immunobi-
A. The water is safe for consumption ological preparation was injected?
B. The water quality is doubtful
C. The water quality is extremely doubtful A. Tuberculin
B. BCG vaccine
D. The water is polluted
E. The water is extremely polluted C. DTP vaccine
D. Tularinum
5. The process of metabolism in the human E. Td vaccine
body produces active forms of oxygen,
− 11. A 15-year-old boy has been diagnosed
including superoxide anion radical 2. with acute viral hepatitis. What blood value
This anion is inactivated by the following should be determined to confirm acute
enzyme: affection of hepatic cells?
A. Superoxide dismutase A. Aminotransferase activity (AST, ALT)
B. Catalase B. Unconjugated and conjugated bilirubin
C. Peroxidase content
D. Glutathione peroxidase C. Erythrocytes sedimentation rate (ESR)
E. Glutathione reductase D. Cholesterol content
6. What kind of muscle contraction occurs E. Protein fraction content
in an upper limb during an attempt to lift a 12. A 53-year-old man is diagnosed with
load beyond one’s strength?
Krok 1 Medicine (англомовний варiант, iноземнi студенти) 2016 рiк 2

Paget’s disease. Concentration of oxyproli- food poisoning. Microscopy of the culture


ne in daily urine is sharply increased, which inoculated in Kitt-Tarozzi medium revealed
primarily means intensified disintegration microorganisms resembling a tennis racket.
of: What diagnosis can be made?
A. Collagen A. Botulism
B. Keratin B. Salmonellosis
C. Albumin C. Cholera
D. Hemoglobin D. Dysentery
E. Fibrinogen E. Typhoid fever
13. When taking exams students often have 18. An infant has been diagnosed with mi-
dry mouth. The mechanism that causes this crocephaly. Doctors suspect that this brain
state results from the following reflexes: disorder developed due to the fact that
the mother had been taking actinomycin D
A. Conditioned sympathetic during her pregnancy. What germinal layers
B. Unconditioned parasympathetic have been affected by this teratogen?
C. Conditioned parasympathetic
D. Unconditioned sympathetic A. Ectoderm
E. Unconditioned peripheral B. Entoderm
C. Mesoderm
14. A patient has hoarseness of voice. Duri- D. Entoderm and mesoderm
ng laryngoscopy a gray-white larynx tumor E. All germinal layers
with papillary surface has been detected.
Microscopic investigation has shown the 19. A patient demonstrates sharp decrease
following: growth of connective tissue of pulmonary surfactant activity. This condi-
covered with multilayer, strongly kerati- tion can result in:
nized pavement epithelium, no cellular
atypia. What is the most likely diagnosis? A. Alveolar tendency to recede
B. Decreased airways resistance
A. Papilloma C. Decresed work of expiratory muscles
B. Fibroma D. Increased pulmonary ventilation
C. Polyp E. Hyperoxemia
D. Angioma
E. Angiofibroma 20. A patient is diagnosed with diabetic
coma. Blood sugar is 18,44 mmol/l. What
15. During autopsy approximately 2,0 liters glucose-regulating drug should be prescri-
of pus have been found in the abdominal bed in the given case?
cavity of the body. Peritoneum is dull and
of grayish shade, serous tunic of intestines A. Rapid-acting insulin
has grayish-colored coating that is easily B. Intermediate-acting insulin
removable. Specify the most likely type of C. Long-acting insulin
peritonitis in the patient: D. Biguanide
E. Sulfonylurea derivative
A. Fibrinopurulent peritonitis
B. Hemorrhagic peritonitis 21. Initial inoculation of water in 1%
C. Serous peritonitis peptone water resulted in growth of a thin
D. Tuberculous peritonitis film on the medium surface in 6 hours.
E. - Such cultural properties are characteristic
of causative agent of the following disesase:
16. Autopsy of a body revealed bone
marrow hyperplasia of tubular and flat A. Cholera
bones (pyoid marrow), splenomegaly (6 kg) B. Plague
and hepatomegaly (5 kg), enlargement of C. Tuberculosis
all lymph node groups. What disease are D. Dysentery
the identified changes typical of? E. Pseudotuberculosis

A. Chronic myelogenous leukemia 22. An infant born prematurely 2 days ago


B. Chronic lymphocytic leukemia presents with yellow coloring of skin and
C. Multiple myeloma mucosa. Such a condition in the infant
D. Polycythemia vera is caused by temporary deficiency of the
E. Hodgkin’s disease following enzyme:

17. A bacteriological laboratory has been


investigating a sample of homemade dri-
ed fish that was the cause of severe
Krok 1 Medicine (англомовний варiант, iноземнi студенти) 2016 рiк 3

A. UDP-glucuronyl transferase A. Bence Jones protein


B. Aminolevulinate synthase B. Bilirubin
C. Heme oxygenase C. Hemoglobin
D. Heme synthetase D. Ceruloplasmin
E. Biliverdine reductase E. Antitrypsin
23. It has been determined that one of a 28. A patient complaining of dizziness, thi-
pesticide components is sodium arsenate rst, difficult swallowing, and impaired visi-
that blocks lipoic acid. Enzyme activity can on of close objects has addressed a doctor.
be impaired by this pesticide. Name this Objectively: respiratory rate is increased,
enzyme: pupils are dilated, general agitation, talkati-
veness, though the speech is indistinct. BP is
A. Pyruvate dehydrogenase complex 110/70 mm Hg, heart rate is 110/min. Given
B. Microsomal oxidation symptoms can indicate overdosage of the
C. Methemoglobin reductase following drug:
D. Glutathione peroxidase
E. Glutathione reductase A. Atropine
B. Morphine
24. A 50-year-old woman diagnosed with C. Ephedrine
cardiac infarction has been delivered into D. Aminazine
an intensive care ward. What enzyme will E. Caffeine
be the most active during the first two days?
29. A dry-cleaner’s worker has been found
A. Aspartate aminotransferase to have hepatic steatosis. This pathology can
B. Alanine aminotransferase be caused by disruption of synthesis of the
C. Alanine aminopeptidase following substance:
D. LDH4
E. LDH5 A. Phosphatidylcholine
B. Tristearin
25. Stool culture test of a 6-month-old C. Urea
bottlefed baby revealed a strain of intesti- D. Phosphatidic acid
nal rod-shaped bacteria of antigen structure E. Cholic acid
0-111. What diagnosis can be made?
30. A 35-year-old man with peptic ulcer di-
A. Colienteritis sease has undergone antrectomy. After the
B. Gastroenteritis surgery secretion of the following gastro-
C. Choleriform disease intestinal hormone will be disrupted the
D. Food poisoning most:
E. Dysentery-like disease
A. Gastrin
26. Parents of a sick 5-year-old girl vi- B. Histamine
sited a genetic consultation. Karyotype C. Secretin
investigation revealed 46 chromosomes. D. Cholecystokinin
One chromosome of the 15th pair was E. Neurotensin
abnormally long, having a part of the
chromosome belonging to the 21st pair 31. A 16-year-old adolescent is di-
attached to it. What mutation occurred in agnosed with hereditary UDP (uridine di-
this girl? phosphate) glucuronyltransferase defici-
ency. Laboratory tests revealed hyperbi-
A. Translocation lirubinemia caused mostly by increased
B. Deletion blood content of the following substance:
C. Inversion
D. Deficiency A. Unconjugated bilirubin
E. Duplication B. Conjugated bilirubin
C. Urobilinogen
27. A patient consulted a doctor with D. Stercobilinogen
complaints of dyspnea occurring after E. Biliverdine
physical exertion. Physical examination
revealed anemia, paraprotein was detected 32. A 60-year-old patient with a long hi-
among gamma globulins. What value should story of atherosclerosis and a previous
be determined in the patient’s urine to myocardial infarction developed an attack
confirm the diagnosis of myeloma? of retrosternal pain. 3 days later the patient
was hospitalized and then died of progressi-
ve cardiovascular insufficiency. During
autopsy a white fibrous depressed area
about 3 cm in diameter with clear margi-
Krok 1 Medicine (англомовний варiант, iноземнi студенти) 2016 рiк 4

ns was found within the area of posterior A. Tuberculosis


wall of the left ventricle and interventricular B. Hodgkin’s disease
septum. The dissector considered these C. Actinomycosis
changes to be: D. Tumor metastasis
E. Syphilis
A. Focal cardiosclerosis
B. Myocardial ischemia 37. Sex chromatin was detected during
C. Myocardial infarction examination of a man’s buccal epitheli-
D. Myocarditis um. It is characteristic of the following
E. Myocardial degeneration chromosome disease:
33. A patient with hypertension has A. Klinefelter’s syndrome
developed headache, tinnitus, vomiting, hi- B. Down’s disease
gh BP up to 220/160 mm Hg. On exami- C. Turner’s syndrome
nation: facial asymmetry on the right, voli- D. Triple X syndrome
tional mobility is absent, increased tendon E. Hypophosphatemic rickets
reflexes and muscle tone of extremities on
the right. What motor disorder of nervous 38. A patient, having suffered a thermal
system occurred in this case? burn, developed painful boils filled with
turbid liquid in the skin. What morphologi-
A. Hemiplegia cal type of inflammation has developed in
B. Paraplegia the patient?
C. Tetraplegia
D. Hyperkinesis A. Serous
E. Monoplegia B. Proliferative
C. Croupous
34. A 7-year-old child in the state of allergic D. Granulomatous
shock caused by a bee sting has been E. Diphtheritic
delivered into an emergency ward. High
concentration of histamine was observed 39. Autopsy of a man with tuberculosis
in blood. Production of this amine was the revealed a 3x2 cm large cavity in the superi-
result of the following reaction: or lobe of the right lung. The cavity was
interconnected with a bronchus, its wall
A. Decarboxylation was dense and consisted of three layers:
B. Hydroxylation the internal layer was pyogenic, the middle
C. Dehydrogenation layer was made of tuberculous granulation
D. Deaminization tissue and the external one was made of
E. Reduction connective tissue. What is the most likely
diagnosis?
35. A 26-year-old woman consulted a doctor
about having stool with white flat moving A. Fibrous cavernous tuberculosis
organisms resembling noodles. Laboratory B. Fibrous focal tuberculosis
analysis revealed proglottids with the C. Tuberculoma
following characteristics: long, narrow, wi- D. Acute focal tuberculosis
th a longitudinal channel of the uterus with E. Acute cavernous tuberculosis
17-35 lateral branches on each side. What
kind of intestinal parasite was found? 40. A 7-year-old child has acute onset of di-
sease: temperature rise up to 38o C , rhinitis,
A. Taeniarhynchus saginatus cough, lacrimation, and large-spot rash on
B. Taenia solium the skin. Pharyngeal mucosa is edematous,
C. Hymenolepis nana hyperemic, with whitish spots in the buccal
D. Diphyllobothrium latum area. What kind of inflammation caused the
E. Echinococcus granulosus changes in the buccal mucosa?
36. A man is 28 years old. Histological A. Catarrhal inflammation
investigation of the cervical lymph node B. Suppurative inflammation
revealed a change of its pattern due to proli- C. Fibrinous inflammation
feration of epithelioid, lymphoid cells and D. Hemorrhagic inflammation
macrophages with horseshoe-shaped nuclei. E. Serous inflammation
In the center of some cell clusters there
were non-structured light-pink areas with 41. After a traffic accident a 36-year-old
fragments of nuclei. What disease are these patient has developed muscle paralysis of
changes typical of? the extremitis on the right, lost pain and
thermal sensitivity on the left, and partially
lost tactile sensitivity on both sides. What
part of the brain is the most likely to be
Krok 1 Medicine (англомовний варiант, iноземнi студенти) 2016 рiк 5

damaged? (paroxysms), profuse sweating, headaches.


Examination revealed hypertension,
A. Right-hand side of the spinal cord hyperglycemia, increased basal metabolic
B. Motor cortex on the left rate, and tachycardia. These clinical
C. Left-hand side of the spinal cord presentations are typical of the following
D. Anterior horn of the spinal cord adrenal pathology:
E. Posterior horn of the spinal cord
A. Hyperfunction of the medulla
42. A 4-year-old child has been admitted to B. Hypofunction of the medulla
an orthopaedic department with displaced C. Hyperfunction of the adrenal cortex
shin fracture. Bone fragments reposition D. Hypofunction of the adrenal cortex
requires analgesia. What drug should be E. Primary aldosteronism
chosen?
47. During autopsy of a man, who di-
A. Promedol ed of acute transmural cardiac infarction,
B. Analgin the following has been detected on the
C. Morphine hydrochloride pericardium surface: fibrous whitish-brown
D. Panadol deposit connecting parietal and visceral
E. - pericardial layers. What kind of inflammati-
on occurred in the pericardium?
43. While examining foot blood supply a
doctor checks the pulsation of a large artery A. Croupous
running in the separate fibrous channel in B. Diphtheritic
front of articulatio talocruralis between the C. Serous
tendons of long extensor muscles of hallux D. Suppurative
and toes. What artery is it? E. Granulomatous
A. A. dorsalis pedis 48. A 12-year-old child developed nephri-
B. A. tibialis anterior tic syndrome (proteinuria, hematuria, cyli-
C. A. tarsea medialis ndruria) 2 weeks after a case of tonsilli-
D. A. tarsea lateralis tis, which is a sign of affected glomerular
E. A. fibularis basement membrane in the kidneys. What
44. Representatives of a certain human mechanism is the most likely to cause the
population can be characterized by basement membrane damage?
elongated body, height variability, decreased A. Immune complex
volume of muscle mass, increased length B. Granulomatous
of limbs, decreased size and volume of rib C. Antibody-mediated
cage, increased perspiration, decreased indi- D. Reaginic
ces of base metabolism and fat synthesis. E. Cytotoxic
What type of adaptive evolution is it?
49. A man arrived into a traumatologi-
A. Tropical cal department with a trauma of the right
B. Arctic shoulder. Examination revealed a displaced
C. Moderate humeral shaft fracture on the right in the
D. Intermediate middle one-third of the humerus; the pati-
E. Mountain ent cannot extend the fingers of his right
45. A 59-year-old woman has been hospiali- hand. What nerve is damaged?
zed in a surgical ward due to exacerbation of A. Radial
chronic osteomyelitis of the left shin. Blood B. Ulnar
test: leukocytes - 15, 0 · 109 /l. Leukogram: C. Median
myelocytes - 0%, metamyelocytes - D. Musculocutaneous
8%, stab neutrophils - 28%, segmented E. Axillary
neutrophils - 32%, lymphocytes - 29%,
monocytes - 3%. Such blood count would 50. Work in a mine is known to cause
be called: inhalation of large amounts of coal dust.
Inhaled coal dust can be detected in the
A. Regenerative left shift following pulmonary cells:
B. Right shift
C. Hyperregenerative left shift A. Alveolar macrophages
D. Degenerative left shift B. Respiratory epithelial cells
E. Regenerative-degenerative left shift C. Secretory epithelial cells
D. Capillary endothelial cells
46. A 41-year-old man has a hi- E. Pericapillary cells
story of recurrent attacks of heartbeats
Krok 1 Medicine (англомовний варiант, iноземнi студенти) 2016 рiк 6

51. What drug will be the most appropriate A. Malaria


for a patient suffering from chronic gastritis B. AIDS
with increased secretion? C. Epidemic typhus
D. Sepsis
A. Pirenzepine E. Hepatitis
B. Pancreatine
C. Pepsin 57. Human red blood cells contain no mi-
D. Aprotinin tochondria. What is the main pathway for
E. Chlorphentermine ATP production in these cells?
52. A 63-year-old man, who has been A. Anaerobic glycolysis
suffering from chronic fibrous-cavernous B. Aerobic glycolysis
pulmonary tuberculosis for 24 years, has C. Oxidative phosphorylation
been delivered to a nephrology department D. Creatine kinase reaction
with uremia. Intravital diagnostic test for E. Cyclase reaction
amyloid in the kidneys was positive. What
amyloidosis is it in this case? 58. Atria of an experimental animal were
superdistended with blood, which resulted
A. Secondary systemic in decreased reabsorption of Na+ and water
B. Primary systemic in renal tubules. This can be explained by
C. Localized (focal) the influence of the following factor on ki-
D. Hereditary (genetic) dneys:
E. Senile
A. Natriuretic hormone
53. Cells of a healthy liver actively synthesi- B. Aldosterone
ze glycogen and proteins. What organelles C. Renin
are the most developed in them? D. Angiotensin
E. Vasopressin
A. Granular and agranular endoplasmic
reticulum 59. A woman gave birth to a stillborn
B. Cell center baby with numerous malformations. What
C. Lysosomes protozoan disease could cause intrauterine
D. Mitochondria death?
E. Peroxisomes
A. Toxoplasmosis
54. Immune-enzyme assay has detected B. Leishmaniasis
HBs antigen in blood serum. What disease C. Malaria
is it characteristic of? D. Amebiasis
E. Lambliasis
A. Viral hepatitis type B
B. Viral hepatitis type A 60. A 42-year-old patient complains of
C. AIDS pain in the epigastral area, vomiting; vomit
D. Tuberculosis masses have the color of coffee-grounds;
E. Syphilis the patient suffers from melena. Anamnesis
records gastric ulcer disease. Blood formula:
55. A patient has been diagnosed with erythrocytes - 2, 8 · 1012 /l, leukocytes -
gonorrhea. As fluoroquinolones are the 8 · 109 /l, Hb- 90 g/l. What complication is
drugs of choce for treatment of gonorrhea it?
the patient should be prescribed:
A. Hemorrhage
A. Ciprofloxacin B. Penetration
B. Furazolidone C. Perforation
C. Fluorouracil D. Canceration
D. Sulfacarbamide (Urosulfanum) E. Pyloric stenosis
E. Cefazolin
61. A patient has been hospitalised
56. Autopsy of a Middle-Eastern woman, with provisional diagnosis of virus B
who had been suffering from wasting hepatitis. Serological reaction based on
fever for a long time, revealed enlarged complementation of antigen with antibody
blackened liver and spleen. Bone marrow chemically bound to peroxidase or alkaline
was hyperplastic and black-colored as well. phosphatase was used for disease diagnosti-
Cerebral cortex was smoky grey. What di- cs. What is the name of the applied serologi-
sease is it characteristic of? cal reaction?
Krok 1 Medicine (англомовний варiант, iноземнi студенти) 2016 рiк 7

A. Immune-enzyme analysis
B. Radioimmunoassay A. Heat radiation
C. Immunofluorescence test B. Heat conduction
D. Bordet-Gengou test C. Convection
E. Antigen-binding assay D. Evaporation
E. -
62. A patient with insulin-dependent di-
abetes mellitus has been administered 67. Due to destruction of certain structures
insulin. After a certain period of time of the brainstem an animal has lost its ori-
the patient developed fatigue, irritabili- entation reflexes in response to strong light
ty, excessive sweating. What is the main stimuli. What structures were destroyed?
mechanism of such presentations developi-
ng? A. Anterior quadrigeminal bodies
B. Posterior quadrigeminal bodies
A. Carbohydrate starvation of the brain C. Red nuclei
B. Increased glycogenolysis D. Vestibular nuclei
C. Increased ketogenesis E. Substantia nigra
D. Increased lipogenesis
E. Decreased glyconeogenesis 68. Urine analysis has shown high levels of
protein and erythrocytes in urine. This can
63. Examination of a 52-year-old woman be caused by the following:
has revealed a decrease in the amount
of red blood cells and an increase in A. Renal filter permeability
free hemoglobin in the blood plasma B. Effective filter pressure
(hemoglobinemia). Color index is 0,85. C. Hydrostatic blood pressure in glomerular
What type of anemia is being observed in capillaries
the patient? D. Hydrostatic primary urine pressure in
capsule
A. Acquired hemolytic E. Oncotic pressure of blood plasma
B. Hereditary hemolytic
C. Acute hemorrhagic 69. Along with normal hemoglobin types
D. Chronic hemorrhagic there can be pathological ones in the organi-
E. Anemia due to diminished erythropoiesis sm of an adult. Name one of them:

64. Poisoning caused by mercury (II) chlori- A. HbS


de (corrosive sublimate) occurred in the B. HbF
result of safety rules violation. In 2 days C. HbA1
the patient’s diurnal diuresis was 620 ml. D. HbA2
The patient developed headache, vomiting, E. HbO2
convulsions, dyspnea; moist crackles were 70. Development of both immune and
observed in the lungs. Name this pathology: allergic reactions is based upon the same
A. Acute renal failure mechanisms of immune system response
B. Chronic renal failure to an antigen. What is the main difference
C. Uremic coma between immune and allergic reactions?
D. Glomerulonephritis A. Development of tissue lesion
E. Pyelonephritis B. Amount of released antigen
65. For people adapted to high external C. Antigen structure
temperatures profuse sweating is not D. Routes by which antigens are delivered
accompanied by loss of large volumes of into the body
sodium chloride. This is caused by the effect E. Hereditary predisposition
the following hormone has on perspiratory 71. Histologic preparation stained wi-
glands: th orcein demonstrates from 40 to 60
A. Aldosterone fenestrated elastic membranes within the
B. Vasopressin middle coat of vessel. Name this vessel:
C. Cortisol A. Elastic artery
D. Tgyroxin B. Muscular artery
E. Natriuretic C. Mixed type artery
66. The processes of heat transfer in a naked D. Muscular vein
person at room temperature have been E. Nonmuscular vein
studied. It was revealed that under these 72. Angiocardiography of a 60-year-old man
conditions the greatest amount of heat is revealed constriction of a vessel located in
transferred by:
Krok 1 Medicine (англомовний варiант, iноземнi студенти) 2016 рiк 8

the left coronary sulcus of the heart. Name ration is slow and shallow due to suppressi-
this pathological vessel: on of the respiratory center. What kind of
respiratory failure occurred in this case?
A. Ramus circumflexus
B. Ramus interventricularis posterior A. Ventilatory disregulation
C. A. coronaria dextra B. Ventilatory obstruction
D. V.cordis parva C. Ventilatory restriction
E. Ramus interventricularis anterior D. Perfusion
E. Diffusion
73. A comatose patient was taken to the
hospital. He has a history of diabetes melli- 78. On histological examination of uterine
tus. Objectively: Kussmaul breathing, low mucosa the following is detected: sinuous
blood pressure, acetone odor of breath. glands, serratiform and corkscrew-shaped
After the emergency treatment the pati- elongated growths of stroma with cell proli-
ent’s condition improved. What drug had feration. Make the diagnosis:
been administered?
A. Glandular endometrial hyperplasia
A. Insulin B. Acute endometritis
B. Adrenaline C. Leiomyoma
C. Isadrinum D. Vesicular mole
D. Glibenclamide E. Placental polyp
E. Furosemide
79. 10 minutes after the beginning of
74. A patient complains of pain in the ri- heavy physical work a person demonstrates
ght lateral abdomen. Palpation revealed a increase of erythrocyte number in blood
dense, immobile, tumor-like formation. The from 4, 0 · 1012 /l to 4, 5 · 1012 /l. What is the
tumor is likely to be found in the following cause of this phenomenon?
part of the digestive tube:
A. Erythrocytes exit from depot
A. Colon ascendens B. Suppression of erythrocyte destruction
B. Colon transversum C. Erythropoiesis activation
C. Colon descendens D. Increase of cardiac output
D. Colon sigmoideum E. Water loss
E. Caecum
80. A patient has a traumatic injury
75. A patient hospitalized due to mercury of sternocleidomastoid muscle. This has
intoxication presents with the followi- resulted in a decrease of the following value:
ng processes in the kidneys: focal
necrotic changes of tubules of major A. Inspiratory reserve volume
renal regions, edema, leukocyte infiltrati- B. Expiratory reserve volume
on and hemorrhages in the interstitial ti- C. Respiratory volume
ssue, venous congestion. What condition D. Residual volume
developed in the patient? E. Functional residual lung capacity
A. Acute necrotic nephrosis 81. Autopsy of a 40-year-old woman,
B. Acute glomerulonephritis who died of cerebral hemorrhage duri-
C. Chronic renal failure ng hypertensic crisis, revealed: upper-
D. Acute pyelonephritis body obesity, hypertrichosis, hirsutism,
E. Chronic pyelonephritis stretchmarks on the skin of thighs and
abdomen. Pituitary basophil adenoma is
76. According to phenotypic diagnosis a detected in the anterior lobe. What di-
female patient has been provisionally di- agnosis is the most likely?
agnosed with X-chromosome polysomia.
This diagnosis can be confirmed by A. Cushing’s disease
cytogenetic method. What karyotype will B. Essential hypertension
confirm the diagnosis? C. Alimentary obesity
D. Simmonds’ disease
A. 47(ХХХ) E. Hypothalamic obesity
B. 48(XXXY)
C. 48(XXYY) 82. A specimen shows an organ covered
D. 47(XXY) with the connective tissue capsule with
E. 46(XX) radiating trabeculae. There is also cortex
containing lymph nodules, and medullary
77. An uconscious young man in the state of cords made of lymphoid cells. What organ
morphine intoxication has been delivered is under study?
into an admission room. The patient’s respi-
Krok 1 Medicine (англомовний варiант, iноземнi студенти) 2016 рiк 9

A. Lymph node one-third. Flexing function was disrupted


B. Thymus in the shoulder and elbow joints, which is
C. Spleen caused by the damage to the:
D. Red bone marrow
E. Tonsils A. Biceps muscle of the arm
B. Triceps muscle of the arm
83. After a craniocerebral injury a patient C. Anconeus muscle
has lost the ability to recognize shapes of D. Deltoid muscle
objects by touch (stereognosis). What area E. Coracobrachial muscle
of cerebral cortex normally contains the
relevant center? 88. A 40-year-old patient suffers
from bronchial asthma and prolonged
A. Superior parietal lobule tachycardia. Choose the optimal drug for
B. Inferior parietal lobule rapid relief of bronchial spasm in the given
C. Supramarginal gyrus case:
D. Postcentral gyrus
E. Angular gyrus A. Salbutamol
B. Adrenalin hydrochloride
84. Monoamine oxidase inhibitors are C. Ephedrine hydrochloride
widely used as psychopharmacological D. Orciprenaline
drugs. They change the level of nearly E. Isoprenaline (Isadrinum)
all neurotransmitters in synapses, with
the following neurotransmitter being the 89. A patient with urolithiasis has
exception: developed severe pain attacks. For pain
shock prevention he was administered an
A. Acetylcholine antispasmodic narcotic analgesic along with
B. Noradrenaline atropine. Name this drug:
C. Adrenaline
D. Dopamine A. Promedol
E. Serotonin B. Nalorphine
C. Tramadol
85. A worker of an agricultural enterprise D. Ethylmorphine hydrochloride
had been suffering from an acute disease E. Morphine hydrochloride
with aggravating intoxication signs, which
resulted in his death. On autopsy: the spleen 90. A patient suffers from acute cardi-
is enlarged, flaccid, dark cherry-red in the opulmonary failure with pulmonary edema.
section, yields excessive pulp scrape. Soft What diuretic should be prescribed in the
meninges of fornix and base of the brain given case?
are edematous and saturated with blood
(”cardinal’s cap”). Microscopically: serous- A. Furosemide
hemorrhagic inflammation of meninges and B. Triamterene
cerebral tissues. Make the diagnosis: C. Spironolactone
D. Hydrochlorothiazide (Dichlothiazidum)
A. Anthrax E. Acetazolamide (Diacarb)
B. Tularemia
C. Plague 91. A patient with acute myocardial infarcti-
D. Cholera on has been administered heparin as a
E. Brucellosis part of complex therapy. Some time after
heparin injection the patient developed
86. Autopsy of an 8-month-old boy, who di- hematuria. What heparin antagonist should
ed of severe pneumonia complicated with be injected to remove the complication?
sepsis, revealed absence of thymus. Lymph
nodes have no lymphoid follicles and corti- A. Protamine sulfate
cal substance. In the spleen the follicles are B. Vicasol
decreased in size and have no light centers. C. Aminocaproic acid
What is the cause of such changes? D. Neodicumarin
E. Fibrinogen
A. Thymus agenesis
B. Thymus aplasia 92. The key reaction of fatty acid synthesis
C. Thymus atrophy is production of malonyl-CoA. What
D. Thymus hypoplasia metabolite is the source of malonyl-CoA
E. Accidental thymic involution synthesis?
87. A patient has been delivered into a
surgical ward with an incised wound of the
anterior surface of the shoulder in its lower
Krok 1 Medicine (англомовний варiант, iноземнi студенти) 2016 рiк 10

A. Acetyl-CoA A. Balantidiasis
B. Succinyl-CoA B. Amebiasis
C. Acyl-CoA C. Toxoplasmosis
D. Malonate D. Lambliasis
E. Citrate E. Trichomoniasis
93. A family of healthy students, who have 98. Blood group of a 30-year-old man has
arrived from Africa, gave birth to a chi- been determined before a surgery. The
ld with signs of anemia. The child has di- blood was Rhesus-positive. Agglutination
ed shortly after. Examination has revealed did not occur with standard 0 (I), А (II),
that the child’s erythrocytes are abnormally and В (III) serums. The blood belongs to
crescent-shaped. The disease is characreri- the following group:
zed by autosomal recessive inheritance.
Determine the genotype of the child’s A. 0 (I)
parents: B. А (II)
C. В (III)
A. Аа х Аа
B. Аа х аа D. АВ (IV)
C. АА х АА E. -
D. аа х аа 99. Histological specimen of a hemopoietic
E. Аа х АА organ shows clusters of node- and band-
94. At a certain stage of cell cycle shaped lymphocytes that along with stroma
chromosomes reach cellular poles, undergo elements compose cortical and medullar
despiralization; nuclear membranes are substances. Name this organ:
being formed around them; nucleolus is A. Lymph node
restored. What stage of mitosis is it? B. Spleen
A. Telophase C. Red bone marrow
B. Prophase D. Thymus
C. Prometaphase E. Palatine tonsil
D. Metaphase 100. It is known that in catecholami-
E. Anaphase ne metabolism a special role belongs
95. Cardiac arrest occurred in a patient to monoamine oxidase (MAO). This
during a surgery of the small intestine. enzyme inactivates mediators (noadrenalin,
What regulatory mechamisms resulted in adrenalin, dopamine) by:
the cardiac arrest in this case?
A. Oxidative deamination
A. Unconditioned parasympathetic reflexes B. Adjoining amino groups
B. Unconditioned sympathetic reflexes C. Removing methyl groups
C. Conditioned parasympathetic reflexes D. Carboxylation
D. Conditioned sympathetic reflexes E. Hydrolysis
E. Metasympathetic reflexes
101. Cellular composition of exudate
96. Vestibular receptors of semicircular largely depends on the etiological factor of
canals of an animal have been destroyed. inflammation. What leukocytes are the first
What reflexes will disappear as a result? to be involved in the focus of inflammation
caused by pyogenic bacteria?
A. Statokinetic reflex during movements
with angular acceleration A. Neutrophil granulocytes
B. Statokinetic reflex during movements B. Monocytes
with linear acceleration C. Myelocytes
C. Head-righting reflex D. Eosinophilic granulocytes
D. Body-righting reflex E. Basophils
E. Primary orienting reflex
102. A surgeon has detected inflammation
97. A patient working at a pig farm complai- of the Meckel’s diverticulum in a patient.
ns of paroxysmal abdominal pain, liquid During surgical invasion it can be located in
feces with mucus and blood, headache, the:
weakness, fever. Examination of large
intestine revealed ulcers from 1 mm up to A. Ileum
several cm in diameter, feces contained oval B. Jejunum
unicellular organisms with cilia. What di- C. Colon
sease can be suspected? D. Duodenum
E. Sigmoid colon
Krok 1 Medicine (англомовний варiант, iноземнi студенти) 2016 рiк 11

103. A patient complains of acute pain A. X-linked hypogammaglobulinemia


attacks in the right lumbar region. Duri- (Bruton type agammaglobulinemia)
ng examination the nephrolithic obturation B. Autosomal recessive agammaglobuli-
of the right ureter in the region between naemia (Swiss type)
its abdominal and pelvic segments was C. Hypoplastic anemia
detected. What anatomical boundary exi- D. Agranulocytosis
sts between those two segments? E. Louis-Barr syndrome
A. Linea terminalis 108. A microslide contains the specimen
B. Linea semilunaris of a gland composed of several secretory
C. Linea arcuata saccule-shaped parts that open in the
D. Linea transversa common excretory duct. What gland is it?
E. Linea inguinalis
A. Simple branched alveolar gland
104. A patient died of cancerous cachexia B. Compound branched alveolar gland
with primary localization of cancer in the C. Simple unbranched alveolar gland
stomach. Autopsy revealed acutely enlarged D. Compound unbranched alveolar gland
liver with uneven surface and numerous E. Simple branched tubular gland
protruding nodes; the nodes had clear
margins in the section, rounded shape, 109. Microelectrode technique allowed to
gray-pink color, varying density, someti- register a potential following ”all-or-none”
mes contained necrotic foci. Histologically: law and capable of undecremental spreadi-
there are atypical cells in the nodes. What ng. Specify this potential:
pathologic process occurred in the liver?
A. Action potential
A. Cancer metastases B. Excitatory postsynaptic potential
B. Abscesses C. Rest potential
C. Regeneratory nodes D. Inhibitory postsynaptic potential
D. Infarction E. Receptor potential
E. Hepatic cancer
110. Examination of a patient revealed
105. A patient has insufficient blood hypertrophy and inflammation of lymphoid
supply to the kidneys, which caused tissue, edema of mucous membrane
the development of pressor effect due between palatine arches (acute tonsillitis).
to the constriction of arterial resistance What tonsil is normally situated in this area?
vessels. This is the result of the vessels
being greately affected by the following A. Tonsilla palatina
substance: B. Tonsilla pharyngealis
C. Tonsilla tubaria
A. Angiotensin II D. Tonsilla lingualis
B. Angiotensinogen E. -
C. Renin
D. Catecholamines 111. Histological specimen of an ovary
E. Norepinephrine demonstrates a spherical structure
composed of large glandular cells contai-
106. Experimental stimulation of ning lutein. What hormone is produced by
sympathetic nerve branches that innervate the cells of this structure?
heart caused an increase in force of heart
contractions because membrane of typical A. Progesterone
cardiomyocytes permitted an increase in: B. Estrogens
C. Testosterone
A. Calcium ion entry D. Corticosterone
B. Calcium ion exit E. Aldosterone
C. Potassium ion exit
D. Potassium ion entry 112. A patient, who has been subsisting
E. Calcium and potassium ion exit exclusively on polished rice, has developed
polyneuritis due to thiamine deficiency.
107. Parents of a 5-year-old child report him What substance is an indicator of such avi-
to have frequent colds that develop into taminosis, when it is excreted with urine?
pneumonias, presence of purulent rashes
on the skin. Laboratory tests have revealed A. Pyruvic acid
the following: absence of immunoglobuli- B. Malate
ns of any type; naked cells are absent from C. Methylmalonic acid
the lymph nodes punctate. What kind of D. Uric acid
immune disorder is it? E. Phenyl pyruvate
Krok 1 Medicine (англомовний варiант, iноземнi студенти) 2016 рiк 12

113. When blood circulation in the damaged with large polymorphous nuclei. What
tissue is restored, lactate accumulation tumour is it?
stops and glucose consumption decelerates.
These metabolic changes are caused by acti- A. Medulloblastoma
vation of the following process: B. Meningioma
C. Glioblastoma
A. Aerobic glycolysis D. Astrocytoma
B. Anaerobic glycolysis E. Oligodendroglioma
C. Lipolysis
D. Gluconeogenesis 119. When studying the signs of pulmonary
E. Glycogen biosynthesis ventilation, reduction of forced expiratory
volume has been detected. What is the li-
114. A 67-year-old patient complains of kely cause of this phenomenon?
periodic heartache, dyspnea during light
physical activities. ECG reveals extraordi- A. Obstructive pulmonary disease
nary contractions of heart ventricles. Such B. Increase of respiratory volume
arrhythmia is called: C. Increase of inspiratory reserve volume
D. Increase of pulmonary residual volume
A. Extrasystole E. Increase of functional residual lung
B. Bradycardia capacity
C. Tachycardia
D. Flutter 120. A specimen of a parenchymal organ
E. Fibrillation shows poorly delineated hexagonal lobules
surrounding a central vein, and the
115. In investigation of serum proteins vari- interlobular connective tissue contains
ous physical and physicochemical methods embedded triads (an artery, a vein and an
can be used. In particular, serum albumi- excretory duct). What organ is it?
ns and globulins can be separated by the
method of: A. Liver
B. Pancreas
A. Electrophoresis C. Thymus
B. Polarography D. Spleen
C. Dialysis E. Thyroid
D. Spectrography
E. Refractometry 121. A patient had a trauma that caused
dysfunction of motor centers regulating
116. Pupil dilation occurs when a person activity of head muscles. In what parts of
steps from a light room into a dark one. cerebral cortex can these centers normally
What reflex causes such a reaction? be located?
A. Sympathetic unconditioned reflex A. Inferior part of precentral gyrus
B. Sympathetic conditioned reflex B. Superior part of precentral gyrus
C. Metasympathetic reflex C. Supramarginal gyrus
D. Parasympathetic unconditioned reflex D. Superior parietal lobule
E. Parasympathetic conditioned reflex E. Angular gyrus
117. Cells of a person working in the 122. At the post-mortem examination the
Chornobyl Exclusion Zone have undergone stomach of a patient with renal failure was
a mutation in DNA molecule. However, found to have a yellow-brown coating on
with time the damaged interval of DNA the thickened mucosa. The coating was fi-
molecule has been restored to its initial rmly adhering to its surface and had si-
structure with a specific enzyme. In this case gnificant thickness. Microscopy revealed
the following occurred: congestion and necrosis of mucosal and
submucosal layers, fibrin presence. What is
A. Repair the most likely diagnosis?
B. Replication
C. Transcription A. Fibrinous gastritis
D. Reverse transcription B. Croupous gastritis
E. Translation C. Gastric abscess
D. Esogastritis
118. Autopsy of a 5-year-old child revealed E. Corrosive gastritis
in the area of the vermis of cerebellum a soft
grayish-pink node 2 cm in diameter with 123. A 60-year-old man suffering from
blurred margins and areas of haemorrhage. chronic hepatitis frequently observes nasal
Histologically this tumour consisted of and gingival hemorrhages, spontaneous
atypical monomorphous small round cells hemorrhagic rashes on the skin and mucosa.
Krok 1 Medicine (англомовний варiант, iноземнi студенти) 2016 рiк 13

Such presentations result from: case?


A. Decreased synthesis of prothrombin and A. Trichinosis
fibrinogen B. Taeniarhynchosis
B. Increased blood content of ami- C. Teniasis
notransferases D. Echinococcosis
C. Decreased synthesis of serum albumins E. Alveococcosis
D. Increased blood content of macroglobuli-
ns and cryoglobulins 129. A patient demonstrates functional loss
E. Decreased blood content of choli- of nasal halves of the retinas. What area of
nesterase visual pathways is affected?

124. Leading symptoms of primary A. Optic chiasm


hyperparathyroidism are osteoporosis and B. Left optic tract
renal damage resulting in urolithiasis C. Right optic tract
development. What substances are the basis D. Left optic nerve
of uroliths in such cases? E. Right optic nerve

A. Calcium phosphate 130. A 26-year-old woman with bronchitis


B. Uric acid has been administered a broad spectrum
C. Cystine antibiotic as a causal treatment drug. Speci-
D. Bilirubin fy this drug:
E. Cholesterol
A. Doxycycline
125. An oncology department has admitted B. Interferon
a patient with suspected pulmonary tumor. C. BCG vaccine
On examination a pathology localized wi- D. Ambroxol
thin the lower lobe of the right lung was E. Dexamethasone
detected. How many bronchopulmonary
segments are there in this lobe? 131. A 16-year-old young man sufferi-
ng from seasonal allergic rhinitis has
A. 5 been prescribed a highly active second-
B. 6 generation H1 blocker, which can be
C. 4 characterized by absence of marked sedati-
D. 3 ve action. Name this drug:
E. 2
A. Loratadine
126. A 45-year-old woman suffers B. Pipolphen
from arterial hypertension with high C. Chloropyramine (Suprastin)
blood concentration of angiotensin II. D. Indometacin
What antihypertensive drug is the most E. Erythromycin
recommended in the given case?
132. Examination of a 56-year-old woman
A. Lisinopril with a history of type 1 diabetes mellitus
B. Prazosin revealed a disorder of protein metaboli-
C. Metoprolol sm that is manifested by aminoacidemia in
D. Reserpine the laboratory blood test values, and cli-
E. Verapamil nically by the delayed wound healing and
decreased synthesis of antibodies. Which
127. An alcoholic suffers from alcoholic of the following mechanisms causes the
psychosis with evident psychomotor agitati- development of aminoacidemia?
on. What neuroleptic drug should be admi-
nistered for emergency aid? A. Increased proteolysis
B. Albuminosis
A. Aminazine C. Decrease in the concentration of amino
B. Diazepam acids in blood
C. Sodium bromide D. Increase in the oncotic pressure in the
D. Reserpine blood plasma
E. Halothane E. Increase in low-density lipoprotein level
128. In one of Polessia regions there was 133. A patient with injury sustained to a part
an outbreak of helminthiasis manifested by of the central nervous system demonstrates
cramps and facial edemas. The developed disrupted coordination and movement
preventive measures in particular included amplitude, muscle tremor during volitional
ban for eating infested pork even after movements, poor muscle tone. What part of
heat processing. What helminthiasis was the the central nervous system was injured?
Krok 1 Medicine (англомовний варiант, iноземнi студенти) 2016 рiк 14

A. Histamine
A. Cerebellum B. Hageman’s factor
B. Medulla oblongata C. Lysosomal enzymes
C. Oliencephalon D. Noradrenaline
D. Mesencephalon E. Adrenaline
E. Prosencephalon
139. Autopsy of a 28-year-old patient, who
134. A 36-year-old patient has been admini- had been suffering from rheumatism and
stered a depolarizing muscle relaxant duri- died of heart failure, revealed pancardi-
ng a surgery. Name this drug: tis. Histological investigation of myocardi-
um of the left ventricle posterior wall
A. Dithylinum and interventricular septum detected peri-
B. Proserin vascular cellular focal infiltrates composed
C. Pipecuronium bromide (Arduan) of macrophages and creating palisade
D. Diazepam structures surrounding areas of fibrinoid
E. Aminazine necrosis. Determine the type of myocarditis:
135. A man has suffered multiple bone A. Granulomatous
fractures of his lower extremities duri- B. Diffuse interstitial productive
ng a traffic accident. During transportati- C. Diffuse interstitial exudative
on to a hospital his condition was further D. Focal interstitial exudative
aggravated: blood pressure decreased, there E. -
were signs of pulmonary artery embolism.
What kind of embolism is the most likely in 140. When ascending to the top of Elbrus,
the given case? a mountain climber experiences oxygen
starvation, dyspnea, palpitations, and
A. Fat embolism numbness of the extremities. What kind of
B. Air embolism hypoxia has developed in the mountain cli-
C. Gas embolism mber?
D. Tissue embolism
E. Thromboembolism A. Hypoxic
B. Circulatory
136. UN volunteers have arrived in Ni- C. Hemic
geria to assist the locals in aftermath D. Tissue
of earthquakes. What drug should they E. Cardiac
prescribe for individual chemoprophylaxis
of malaria? 141. A 2-year-old boy is diagnosed wi-
th Down syndrome. What chromosomal
A. Chingamin changes can cause this disease?
B. Pyrantel
C. Pyrimethamine (Chloridinum) A. Trisomy 21
D. Primaquine B. Trisomy 13
E. Interferon (Laferon) C. Trisomy X
D. Trisomy 18
137. After an extended treatment wi- E. Monosomy X
th sulfanamides a patient has developed
macrocytic anemia. Production of active 142. A 62-year-old patient has been hospi-
forms of the following vitamin is disrupted talized due to massive cerebral hemorrhage.
in such a condition: Blood pressure is 70/30 mm Hg, heart
rate is 120/min., respiratory rate is 4/min.,
A. Folic acid unconscious, no response to external sti-
B. Thiamine muli. Such condition can be determined as:
C. Riboflavin
D. Pyridoxine A. Coma
E. Cyanocobalamin B. Shock
C. Collapse
138. A woman resting in the countrysi- D. Stress
de has been stung by a bee. Immediately E. Agony
after she developed pain in the stung area.
In a few minutes there developed a vesi- 143. A 3-year-old girl with mental retardati-
cle, erythema and intense itch; later - urti- on has been diagnosed with sphingomyelin
caria and expiratory dyspnea. What factors lipidosis (Niemann-Pick disease). In this
resulted in the patient developing expi- condition synthesis of the following
ratory dyspnea? substance is disrupted:
Krok 1 Medicine (англомовний варiант, iноземнi студенти) 2016 рiк 15

A. Sphingomyelinase the following reflex:


B. Glycosyltransferase
C. Sphingosine A. Dagnini-Aschner reflex
D. Ceramides B. Bainbridge reflex
E. Gangliosides C. Holtz’s reflex
D. Hering-Breuer reflex
144. What condition may develop 15-30 mi- E. Frank-Starling mechanism
nutes after re-administration of an antigen
as a result of the increased level of anti- 149. A 47-year-old man developed intesti-
bodies, mainly IgE, that are adsorbed on nal colic against the background of essenti-
the surface of target cells, namely tissue al hypertension. In this situation it would
basophils (mast cells) and blood basophi- be the most efficient to arrest the colic by
ls? administering drugs of the following group:
A. Anaphylaxis A. Myotropic antispasmodics
B. Antibody-dependent cytotoxicity B. Anticholinesterase agents
C. Delayed-type hypersensitivity C. Sympathomimetics
D. Immune complex hyperresponsiveness D. M-cholinomimetics
E. Serum sickness E. Adrenomimetics
145. Local anesthetics (novocaine, lidocaine 150. A therapeutist has an appointment
and others) decreases pain sensitivity of ti- with a 40-year-old patient complaining of
ssues by blocking Na+ and K + ions from recurrent pain attacks in his hallux joints
permeating membranes of nerve fibers and and their swelling. Urine analysis revealed
endings. Such mechanism of drug action is its marked acidity and pink color. What
called: substances can cause such changes in uri-
ne?
A. Membrane ionic
B. Receptor A. Uric acid salt
C. Enzyme B. Chlorides
D. Antienzyme C. Ammonium salts
E. Direct chemical D. Calcium phosphate
E. Magnesium sulfate
146. After a road accident a victim has
tachycardia, arterial blood pressure 130/90 151. Exophthalmus observed during
mm Hg, tachypnoe, the skin is pale and thyrotoxicosis is caused by accumulation of
dry, excitation of central nervous system is highly water-binding substances within the
observed. What shock stage is the patient retrobulbar tissues. Name these substances:
most likely in? A. Glycosaminoglycans
A. Erectile B. Cholesterol
B. Terminal C. ATP
C. Torpid D. Kreatine
E. Phospholipids
D. Preshock (compensation stage)
E. Agony 152. A patient presents with dry peeli-
147. A woman complaining of sharp pain ng skin, frequent cases of acute respi-
ratory diseases, xerophthalmia. What vi-
in her lower abdomen has been delivered
into an admission room. A gynecologist on tamin preparation should be prescribed in
this case?
examination makes a provisional diagnosis
of extrauterine pregnancy. What anatomical A. Retinol acetate
structure should be punctated to confirm di- B. Thiamine
agnosis? C. Cyanocobalamin
A. Rectouterine pouch D. Menadione (Vikasolum)
B. Utriculosaccular chamber E. Ergocalciferol
C. Recto-vesical pouch 153. Parenchyma of an organ is composed of
D. Retropubic space pseudounipolar neurons localized under the
E. Intersigmoidal recess capsule of connective tissue. Central place
148. A passenger of a fixed-run taxi has a belongs to nerve fibers. Name this organ:
sudden and expressed attack of tachycardia.
A doctor travelling by the same taxi has
managed to slow down his heart rate by
pressing upon the eyeballs and thus causing
Krok 1 Medicine (англомовний варiант, iноземнi студенти) 2016 рiк 16

A. Spinal ganglion A. Testosterone


B. Sympathetic ganglion B. Oestriol
C. Intramural ganglion C. Relaxin
D. Nerve trunk D. Oxytocin
E. Spinal cord E. Prolactin
154. A patient consulted a physician about 159. During a surgery for femoral hernia a
chest pain, cough, fever. Roentgenography surgeon operates within the boundaries of
of lungs revealed eosinophilic infiltrates femoral trigone. What structure makes up
that were found to contain larvae. What ki- its upper margin?
nd of helminthiasis are these presentations
typical of? A. Lig. inguinale
B. Arcus iliopectineus
A. Ascariasis C. Lig. lacunare
B. Echinococcosis D. Lig. pectinale
C. Fascioliasis E. Fascia lata
D. Cysticercosis
E. Trichinosis 160. Activation of a number of hemostatic
factors occurs through their joining with
155. During appendectomy a patient had calcium ions. What structural component
the a. appendicularis ligated. This vessel allows for adjoining of calcium ions?
branches from the following artery:
A. Gamma-carboxyglutamic acid
A. A. ileocolica B. Gamma-aminobutyric acid
B. A. colica dextra C. Gamma-oxybutyric acid
C. A. colica media D. Hydroxyproline
D. A. sigmoidea E. Monoamine-dicarboxylic acids
E. A. mesenterica inferior
161. A patient has arterial hypertension.
156. A patient with signs of osteoporosis What long-acting calcium channel blocker
and urolithiasis has been admitted should be prescribed?
to an endocrinology department.
Blood test revealed hypercalcemia and A. Amlodipine
hypophosphatemia. These changes are B. Octadine
associated with abnormal synthesis of the C. Pyrroxanum
following hormone: D. Atenolol
E. Reserpine
A. Parathyroid hormone
B. Calcitonin 162. Material obtained from a patient
C. Cortisol contains several types of microorganisms
D. Aldosterone (staphylococci and streptococci) causative
E. Calcitriol of the patient’s disease. Name this type of
infection:
157. Prescription of penicillin G sodium
salt has caused development of neurotoxic A. Mixed infection
effects (hallucinations, convulsions). Such B. Superinfection
reaction is the result of antagonism with the C. Reinfection
following neurotransmitter: D. Consecutive infection
E. Coinfection
A. GABA
B. Dopamine 163. A laboratory has been investigati-
C. Serotonin ng virulence of a diphtheria agent. In the
D. Adenosine process of the experiment the infection was
E. Acetylcholine introduced intraperitoneally into test ani-
mals. The dosage of bacteria resulting in
158. A 30-year-old woman exhibits signs 95% mortality of test animals was found.
of virilism (growth of body hair, balding What unit of virulence measurement was
temples, menstrual disorders). This condi- determined?
tion can be caused by overproduction of the
following hormone: A. DLM
B. DCL
C. LD50
D. ID
E. LD5
164. A patient complains of palpitations
Krok 1 Medicine (англомовний варiант, iноземнi студенти) 2016 рiк 17

after stress. Pulse is 104/min., P-Q=0,12 A. Ulnar


seconds, there are no changes in QRS B. Median
complex. What type of arrhythmia does the C. Radial
patient have? D. Musculocutaneous
E. Medial cutaneous nerve of the forearm
A. Sinus tachycardia
B. Sinus bradycardia 170. A 30-year-old patient’s blood test has
C. Sinus arrhythmia revealed the following: erythrocyte count
D. Ciliary arrhythmia is 6 · 1012 /l, hemoglobin is 10,55 mmol/l.
E. Extrasystole Vaquez’s disease was diagnosed. Name the
leading part of pathogenesis:
165. A patient consulted a dentist about
restricted mouth opening (trismus). He has A. Neoplastic erythroid hyperplasia
a history of a stab wound of the lower B. Iron-deficiency
extremity. What infection can cause these C. B12 -deficiency
symptoms? D. Hypoxia
E. Acidosis
A. Tetanus
B. Brucellosis 171. Pancreas is known as a mixed gland.
C. Whooping cough Endocrine functions include production
D. Wound anaerobic infection of insulin by beta cells. This hormone
E. Tularemia affects metabolism of carbohydrates. What
is its effect on the activity of glycogen
166. Patient’s systolic blood pressure is phosphorylase (GP) and glycogen synthase
90 mm Hg, diastolic - 70 mm Hg. Such (GS)?
blood pressure is caused by decrease of the
following factor: A. It inhibits GP and activates GS
B. It activates both GP and GS
A. Pumping ability of the left heart C. It inhibits both GP and GS
B. Pumping ability of the right heart D. It activates GP and inhibits GS
C. Aortic compliance E. It does not affect the activity of GP and
D. Total peripheral resistance GS
E. Vascular tone
172. This year influenza epidemic is
167. A 29-year-old man with a knife wound characterised by patients’ body temperature
of the neck presents with bleeding. Duri- varying from 36, 9oC to 37, 9oC . Such fever
ng the initial d-bridement of the wound is called:
a surgeon revealed the injury of a vessel
situated along the lateral edge of the A. Subfebrile
sternocleidomastoid muscle. Specify this B. High
vessel: C. Hyperpyretic
D. Apyretic
A. V. jugularis externa E. Moderate
B. V. jugularis anterior
C. A. carotis externa 173. Fructosuria is known to be connected
D. A. carotis interna with inherited deficiency of fructose 1-
E. V. jugularis interna phosphate aldolase. What product of
fructose metabolism will accumulate in the
168. A 6-year-old child suffers from delayed organism resulting in toxic action?
growth, disrupted ossification processes,
decalcification of the teeth. What can be A. Fructose 1-phosphate
the cause? B. Glucose 1-phosphate
C. Glucose 6-phosphate
A. Vitamin D deficiency D. Fructose 1,6-biphosphate
B. Decreased glucagon production E. Fructose 6-phosphate
C. Insulin deficiency
D. Hyperthyroidism 174. A woman complains of visual impai-
E. Vitamin C deficiency rment. Examination revealed obesity in the
patient and her fasting plasma glucose level
169. A patient addressed a hospital with is hyperglycemic. What diabetes complicati-
complaints of lost sensitivity of the skin of on can cause visual impairment/blindness?
the little finger. What nerve is the most li-
kely to be damaged?
Krok 1 Medicine (англомовний варiант, iноземнi студенти) 2016 рiк 18

A. Microangiopathy A. Cholera
B. Macroangiopathy B. Shigellosis
C. Atherosclerosis C. Typhoid fever
D. Neuropathy D. Colienteritis
E. Glomerulopathy E. Pseudotuberculosis
175. Administration of doxycycline 180. An athlete (long-distance runner) duri-
hydrochloride has caused an imbalance of ng a contest developed a case of acute cardi-
the symbiotic intestinal microflora. Specify ac insufficiency. This pathology resulted
the kind of imbalance caused by the antibi- from:
otic therapy:
A. Cardiac volume overload
A. Dysbacteriosis B. Disrupted coronary circulation
B. Sensibilization C. Direct damage to myocardium
C. Idiosyncrasy D. Pericardium pathology
D. Superimposed infection E. Cardiac pressure overload
E. Bacteriosis
181. Parkinson’s disease is caused by di-
176. Cholesterol content in blood serum sruption of dopamine synthesis. What brain
of a 12-year-old boy is 25 mmol/l. structure synthesizes this neurotransmitter?
Anamnesis states hereditary familial
hypercholesterolemia caused by synthesis A. Substantia nigra
disruption of receptor-related proteins for: B. Globus pallidus
C. Corpora quadrigemina
A. Low-density lipoproteins D. Red nucleus
B. High-density lipoproteins E. Hypothalamus
C. Chylomicrons
D. Very low-density lipoproteins 182. Name the halogen-containing anti-
E. Middle-density lipoproteins septic with fungicidal properties, which is
used to treat dermatomycosis:
177. During recording of a spirogram a pati-
ent calmly exhaled. How do we call the A. Iodine solution
volume of air remaining in the lungs? B. Formalin solution
C. Methylene blue
A. Functional residual capacity D. Brilliant green
B. Pulmonary residual volume E. Boric acid solution
C. Expiratory reserve volume
D. Tidal volume 183. Due to severe pain syndrome a patient
E. Vital capacity of lungs was prescribed a narcotic analgesic. Name
this drug:
178. A 40-year-old woman was diagnosed
with glomerulonephritis based on her cli- A. Morphine
nical symptoms and the results of urine B. Metamizole (Analgin)
analysis. Anamnesis states chronic tonsilli- C. Nimesulide
tis. What microorganisms are the most likely D. Dimethyl sulfoxide
cause for the kidney damage in this case? E. Indometacin
A. Streptococci 184. During pathomorphological renal
B. Staphylococci investigation of a patient, who for a long
C. Escherichia time had been suffering from osteomyeli-
D. Mycoplasma tis and died of progressing renal fai-
E. Meningococci lure, the following was revealed: deposi-
ts of homogeneous eosinophilic masses
179. A man is suffering from diarrhea. In in glomerular mesangium, arterial and
summer he spent his vacation in the south arteriolar walls, and stroma, which colored
at the sea coast. Bacteria with the following red when stained with Congo red. What
properties were detected in his feces: gram- pathological process is this?
negative curved mobile monotrichous baci-
lli that do not produce spores or capsules. A. Amyloidosis
Bacilli are undemanding to nutrient medi- B. Mucoid swelling
um but require alkaline reaction (рН 8,5- C. Calcinosis
9,5). Described are the agents of the followi- D. Carbohydrate degeneration
ng enteric infection: E. Hyalinosis
185. During experiment a dog has
developed conditioned digestive reflex in
Krok 1 Medicine (англомовний варiант, iноземнi студенти) 2016 рiк 19

response to a sound stimulus. This conditi- A. Disruption of protein metabolism


oned reflex will not be exhibited anymore regulation
after the extirpation of the following areas B. Hypoglycemia
of the cerebral hemispheres: C. Ketonemia
D. Increased lipid catabolism
A. Temporal lobe on both sides E. Anemia
B. Occipital lobe on one side
C. Parietal lobe on both sides 191. A patient with signs of intestinal
D. Temporal lobe on one side infection (vomiting, diarrhea, abdominal
E. Occipital lobe on both sides pain) has been presenting with increasing
symptoms of intoxication for three days.
186. Coenzym A participates in numerous Papular rash appeared on the uncovered
important metabolic reactions. It is a deri- skin areas and spread to the torso. A
vative of the following vitamin: doctor suspected pseudotuberculosis. What
laboratory test allows confirming this di-
A. Pantothenic acid agnosis within the first week from the onset
B. Thiamine of disease?
C. Niacin
D. Calciferol A. Bacteriological
E. Ubiquinone B. Microscopic
C. Serological
187. A patient diagnosed with acute D. Allergic
dysentery has been treated for 3 days in an E. Biological
infectious diseases hospital. On admission
there were complaints of high temperature, 192. A microslide demonstrates an organ
stomachache and fluid excrements with with its wall consisting of three membranes.
mucus up to 8-10 times a day. What sample The inner membrane has tubular glands and
should be taken for analysis? undergoes cyclic changes. Name this organ:
A. Feces A. Uterus
B. Urine B. Esophagus
C. Bile C. Vagina
D. Liquor D. Ureter
E. Blood E. Urinary bladder
188. A patient with arthritis has been 193. A patient with femoral neck fracture,
prescribed an anti-inflammatory selective who for a long time had to remain in bed
COX-2 inhibitor. Select this drug among in a forced (supine) position, has developed
those given below: dark-brown lesions along the backbone; soft
A. Celecoxib tissues are swollen, in the areas of macerati-
B. Phenylbutazone (Butadion) on there is a foul-smelling liquid. Name the
clinicopathologic type of necrosis:
C. Dimethylsulfoxide (Dimexid)
D. Indometacin A. Bedsore
E. Metamizole (Analgin) B. Infarction
C. Sequestrum
189. A person with the fourth blood group D. Coagulation necrosis
(genotype IAIB) has in erythrocytes both E. Dry gangrene
antigen A controlled by allele IA and
antigen B controlled by allele IB. This 194. A patient is 20 years old, an athlete. He
phenomenon is an example of the following addressed a doctor with complaints of fati-
gene interation: gue, fever up to 38o C - 40o C . Objectively:
the liver and spleen are enlarged, lymph
A. Codominance nodes on palpation are slightly enlarged,
B. Complementarity dense, painless. Blood test: Нb- 100 g/l;
C. Semidominance erythrocytes - 2, 9 · 1012 /l; leukocytes -
D. Polymery
E. Epistasis 4, 4 · 109 /l. Leukogram: 68% of blast cells.
Cytochemical investigation of blast cells
190. A patient with diabetes mellitus revealed negative reactions to glycogen,
suffers from persistently nonhealing surgi- peroxidase, non-specific esterase, lipids.
cal wound, which is a sign of disrupted ti- Name this disease:
ssue trophism. What is the cause of such
disorder?
Krok 1 Medicine (англомовний варiант, iноземнi студенти) 2016 рiк 20

A. Acute undifferentiated leukemia edema has developed in the child?


B. Acute myeloid leukemia
C. Acute monoblastic leukemia A. Allergic
D. Acute lymphoblastic leukemia B. Inflammatory
E. Acute megakaryoblastic leukemia C. Cardiac
D. Alimentary
195. Impression smear of mucosa biopsy E. Hepatic
material has been obtained from a patient
with peptic ulcer disease of the stomach. 198. During examination of a patient a
Gram-negative arcuate bent microorgani- doctor should use anatomical division of
sms were detected, urease activity test anterior abdominal wall into regions for
was positive. What microorganisms were more precise diagnostics. How many regi-
detected in the patient? ons can abdomen be divided into?
A. Helicobacter A. 9
B. Spirochete B. 8
C. Spirilla C. 6
D. Leptospira D. 5
E. Treponema E. 4
196. A patient after disrupted cerebral ci- 199. A child has a wound located posterior
rculation has developed paralysis. Choose to the mastoid process. Bright red blood
the anticholinesterase drug to be prescribed flows from the wound. Damaged are the
in this case: branches of the following artery:
A. Proserin A. A. occipitalis
B. Cordiamin B. A. temporalis superior
C. Aceclidine C. A. maxillaris
D. Methacin D. A. carotis externa
E. Hexamethonium (Benzohexonium) E. A. carotis interna
197. 30 minutes after drinking mango juice 200. When investigating human saliva it is
a child suddenly developed a local swelli- necessary to assess its hydrolytic properti-
ng in the area of the soft palate, which es. What substance should be used as a
impeded swallowing and, eventually, respi- substrate in the process?
ration. Mucosa of the swollen area was
hyperemic and painless. Blood test revealed A. Starch
moderate eosinophilia. Body temperature B. Proteins
was normal. Anamnesis states that the elder C. Fats
sister of the child has been suffering from D. Fiber
bronchial asthma attacks. What kind of E. Amino acids
MINISTRY OF PUBLIC HEALTH OF UKRAINE

Department of human resources policy, education and science

Testing Board

Student ID Last name

Variant ___________________

Test items for licensing examination

Krok 1
MEDICINE
General Instruction
Every one of these numbered questions or unfinished statements in
this chapter corresponds to answers or statements endings. Choose the
answer (finished statements) that fits best and fill in the circle with the
corresponding Latin letter on the answer sheet.
ББК 54.1я73
УДК 61

Authors of items: Ananko S.Ya., Antonenko V.P., Bielan S.M., Bielienichev I.F., Blinder O.O.,
Bohatyrova O.V., Boikiv D.P., Bondarenko N.M., Bordiakivska L.H., Brazhnikov A.M., Chekman I.S.,
Cherednyk S.A., Cherkas L.P., Cherkashyna L.P., Danylchuk V.V., Datsko T.V., Deineka S.Ye.,
Demianenko I.A., Diadyk O.O., Duhadko L.M., Dzevulska I.V., Fartushok N.V., Fedoniuk L.Ya.,
Fedorchenko O.V., Fedorchenko O.Ye., Fomenko I.S., Haidash I.S., Hanziy T.V., Herasymovych E.V.,
Hliebova L.Yu., Holod B.V., Horhol N.I., Hrechyn A.B., Hryha I.V., Hrytsyna I.V., Huzik N.M.,
Ivanova A.Y., Ivanova S.A., Ivchenko R.A., Kaptiukh R.P., Karvatsky I.M., Kava T.V., Kazak L.I.,
Khara M.R., Khripkov I.S., Kikhtenko O.V., Kiriakulov H.S., Klishch I.M., Klopotsky H.A.,
Kolesnikova S.V., Kolot E.H., Korolenko H.S., Kratenko H.S., Kratinova M.A., Kriuk Yu.Ya.,
Krushynska T.Yu., Kryshtal M.V., Kukurychkin Ye.R., Kulakov A.Yu., Kulikova N.A., Kyriukhin I.F.,
Linchevska L.P., Lisniak O.I., Luchko I.M., Makovetsky O.V., Maksymchuk T.P., Malyshev S.L.,
Matiushenko P.M., Matveiko L.M., Melnychuk I.B., Meshchyshen I.F., Mikhieieva N.H., Mishchenko A.V.,
Mishchenko A.F., Mishchenko K.M., Mishchenko N.P., Mishyn V.V., Moseichuk I.P., Mozghunov O.V.,
Mruh V.M., Mykula M.M., Naumova O.V., Nikolenko O.H., Obraztsova O.H., Oleshchuk O.M.,
Oliynyk I.Yu., Omelchenko O.D., Opryshko V.I., Ovchynnikov S.O., Papinko I.Ya., Pavliy S.Y.,
Piskunova N.V., Pobihun A.A., Polevyk I.V., Popova L.D., Redko R.P., Romanenko M.I., Romanenko O.V.,
Rosola T.F., Rozghoniuk Yu.D., Rukavyshnikova S.M., Rudnytska O.H., Saiuk N.P., Sas L.M.,
Savchenko N.V., Savenko L.D., Selsky P.R., Semenets P.F., Semeniuk T.M., Serdiuchenko I.Ya.,
Servetnyk M.I., Shanko V.M., Shapoval O.M., Shenderiuk O.P., Shevtsova A.I., Shliakhovenko O.O.,
Shutak V.I., Siedov V.I., Sikoryn Ya.Ya., Skorobohata T.H., Skorobohatova Z.M., Snehir A.H.,
Stebliuk M.V., Stetsenko S.O., Siusarev A.A., Suprunov K.V., Svirsky O.O., Sydorenko I.I., Tarasova K.V.,
Terekhovska O.I., Tkachenko V.P., Tomashova S.A., Tsehelsky A.A., Tuchak O.I., Tusyk O.T.,
Tvorko M.S., Vasylieva A.H., Verbytsky V.V., Verkhova O.O., Vernyhor O.O., Vilkhova I.V.,
Vinnikov Yu.M., Voitsekhovsky V.H., Volobuiev M.A., Vorobets Z.D., Yashchyshyn Z.M.,
Yastremska S.O., Yelsky V.M., Zdykhovsky I.O., Zelenina N.M., Zhulinsky V.O., Zhuravel T.O.,
Zviaholska I.M. and Committees of professional expertise.

Item reviewers. Ananko S.Ya., Basy R.V., Bilash S.M., Deineka S. Ye., Deltsova O.I., Demydova K.Yu.,
Fomina L.V., Hahrin V.V., Haidash I.S., Holovatiuk O.L., Horhol N.I., Kava T.V., Koldunov V.V.,
Korolenko H.S., Kovalchuk L.Ye., Linchevska L.P., Maly K.D., Melnikova O.V., Melnyk N.O.,
Neporada K.S., Nikolenko O. H., Ovchynnikov S. O., Prokofieva N. V., Pushkar M. S., Pykaliuk V. S.,
Salata O.V., Shevelenkova A.V., Shumeiko O.V., Sikora V.Z., Skliarov O.Ya., Skorobohatova Z.M.,
Synytska A.M., Tananakina T.P., Teleshova O.V., Tertyshny S.I., Tkachuk S.S., Trzhetsynsky S.D,
Vinnykov Yu.M., Vorobets Z.D., Yoltukhivsky M.V., Zelenina N.M., Zhadinsky M.V., Zhylinsky V.O.,
Zinkovska L.Ya.

The book includes test items for use at licensing integrated examination “Krok 1. Medicine” and
further use in teaching.

The book has been developed for students of medical, pediatric and medical-and-prophylactic
faculties and academic staff of higher medical educational establishments.

Approved by Ministry of Public Health of Ukraine as examination and teaching


publication based on expert conclusions (Orders of MPH of Ukraine of 14.08.1998 №251,
of 27.12.1999 №303, of 18.06.2002 №221, of 16.10.2002 №374, of 16.04.2003 №239, of
29.05.2003 №233).

© Copyright Testing Board.


Krok 1 Medicine (англомовний варiант, iноземнi студенти) 2017 рiк 1

1. A 35-year-old man has been delivered


into a surgical ward with a suppurating A. Isometric
wound in the neck, anterior to the trachea B. Isotonic
(previsceral space). If a surgical operation C. Auxotonic
is not performed urgently, there is a risk of D. Phasic
infection spreading to the: E. Single

A. Thoracic cavity - anterior mediastinum 7. The patient’s pyramids of the medulla


B. Thoracic cavity - middle mediastinum oblongata are damaged by tumor growth.
C. Thoracic cavity - posterior mediastinum As a result the conduction of nervous
D. Retrovisceral space impulses will be impaired in the following
E. Interaponeurotic suprasternal space pathway:

2. In the life cycle of a cell during mitosis A. Tr. corticospinalis


a natural change in the amount of genetic B. Tr. corticonuclearis
material occurs. The DNA doubles at the C. Tr. corticopontinus
following stage: D. Tr. dentatorubralis
E. Tr. spinocerebellaris
A. Interphase
B. Prophase 8. A patient complains of pain in the upper
C. Metaphase umbilical region. On palpation there is a
D. Anaphase mobile painful intestine. What intestine is
E. Telophase being palpated by the doctor?

3. A woman with seasonal vasomotor rhini- A. Transverse colon


tis, who works as a train dispatcher and is an B. Jejunum
outpatient, should be prescribed an antihi- C. Duodenum
staminic drug that does not suppress central D. Ileum
nervous system. Name this drug: E. Sigmoid colon

A. Loratadine 9. After inoculation of investigated material


B. Dimedrol (Diphenhydramine) (feces) on 1% alkaline peptone water and
C. Diprazine (Promethazine) 8-hour-long incubation in the thermostat
D. Suprastin (Chloropyramine) under 37o C there is growth of pale bluish
E. Tavegyl (Clemastine) film observed. Such cultural properties are
characteristic of the agent of the following
4. During sanitary and bacteriological testi- disease:
ng of water with the membrane filter techni-
que there were revealed two red coloni- A. Cholera
es on the membrane filter (Endo medium) B. Plague
through which 500 ml of water was filtered. C. Typhoid fever
Calculate the coli index and coli titer of this D. Paratyphoid A fever
water: E. Dysentery

A. 4 and 250 10. Histological investigation of the uterine


B. 2 and 500 scrape of the 45-year-old woman with di-
C. 250 and 4 sturbed ovarian menstrual cycle revealed
D. 500 and 2 increased number of endometrial glands,
E. 250 and 2 some of which are serrated, while others are
dilated and cyst-like. Make the diagnosis:
5. Protective function of saliva is based
on several mechanisms, including the A. Endometrial cystic glandular hyperplasia
presence of enzyme that has bacterici- B. Placental polyp
dal action and causes lysis of complex C. Atypical endometrial hyperplasia
capsular polysaccharides of staphylococci D. Glandular endometrial polyp
and streptococci. Name this enzyme: E. Endometrial adenocarcinoma

A. Lysozyme 11. A pregnant woman with several mi-


B. Alpha-amylase scarriages in anamnesis is prescribed a
C. Oligo-1,6-glucosidase therapy that includes vitamin preparati-
D. Collagenase ons. What vitamin facilitates carrying of a
E. Beta-glucuronidase pregnancy?

6. What kind of muscle contraction occurs


in an upper limb during an attempt to lift a
load beyond one’s strength?
Krok 1 Medicine (англомовний варiант, iноземнi студенти) 2017 рiк 2

A. Alpha-tocopherol A. Ramus dexter a. hepatica propria


B. Folic acid B. А. hepatica communis
C. Cyanocobalamin C. А. gasro-duodenalis
D. Pyridoxal phosphate D. Ramus sinister a. hepatica propria
E. Rutin E. А. pancreato-duodenalis sup
12. A patient has decreased concentrati- 17. During treatment with bismuth
on of magnesium ions that are requi- preparations a patient with syphilis
red for ribosomes connection to granular developed gray spots on his oral mucosa
endoplasmic reticulum. This condition is and nephropathy symptoms. What drug is
known to disturb the process of protein used as an antidote to bismuth preparations
biosynthesis. Disturbance occurs at the poisoning?
following stage:
A. Unithiol
A. Translation B. Nalorphine
B. Transcription C. Bemegride
C. Replication D. Naloxone
D. Amino acids activation E. Methylene blue
E. Processing
18. A patient with periodontitis of the lower
13. A patient with chronic heart failure molar came to the doctor. It was determi-
presents with increased blood viscosity. ned that the inflammatory process spread to
Capillaroscopy detected damage to the the lymph nodes. What lymph nodes were
vessel walls of the microcirculation system. the first to be affected by the inflammatory
What disorder is possible in the given case? process?
A. Blood ”sludge” phenomenon A. Submandibular
B. Thrombosis B. Lateral cervical
C. Embolism C. Anterior cervical
D. Arterial hyperemia D. Submental
E. Venous hyperemia E. Facial
14. A 3-year-old boy with pronounced 19. A 63-year-old man suffers from
hemorrhagic syndrome has no anti- esophageal carcinoma, presents with
hemophilic globulin A (factor VIII) in the metastases into the mediastinal lymph
blood plasma. Hemostasis has been impai- nodes and cancerous cachexia. What
red at the following stage: pathogenetic stage of neoplastic process is
observed in the patient?
A. Internal mechanism of prothrombinase
activation A. Progression
B. External mechanism of prothrombinase B. Promotion
activation C. Transformation
C. Conversion of prothrombin to thrombin D. Initiation
D. Conversion of fibrinogen to fibrin E. -
E. Blood clot retraction
20. A person is in a room with air
15. Paronychia of the patient’s little finger temperature of 38o C and relative air humi-
was complicated with phlegmon of the hand dity of 50%. What type of heat transfer
and forearm. In this case the suppuration ensures maintenance of constant body core
had spread through the: temperature under these conditions?
A. Vagina synovialis communis mm. A. Evaporation
flexorum B. Radiation
B. Vagina tendinis m. flexor pollicis longi C. Conduction and convection
C. Canalis carpalis D. Convection
D. Vagina tendinis m. flexor carpi radialis E. -
E. Interfascial compartments
21. A patient has hoarseness of voice. Duri-
16. During cholecystectomy besides a. ng laryngoscopy a gray-white larynx tumor
cystyca another artery was pulled into the with papillary surface has been detected.
ligature. Ligation of this artery resulted in Microscopic investigation has shown the
right-sided necrosis of the liver which led to following: growth of connective tissue
the death of the patient. What artery was covered with multilayer, strongly kerati-
mistakenly ligated along with a. cystyca? nized pavement epithelium, no cellular
atypia. What is the most likely diagnosis?
Krok 1 Medicine (англомовний варiант, iноземнi студенти) 2017 рiк 3

A. Papilloma A. Proserin
B. Fibroma B. Platyphyllin
C. Polyp C. Cyclodol (Trihexyphenidyl)
D. Angioma D. Atropine
E. Angiofibroma E. Dithyline (Suxamethonium chloride)
22. A shepherd, who tended to the flock of 26. Examination of the patient with
sheep with his dogs, gradually developed traumatic brain injury revealed that he has
pain in the chest and bloody expectorati- lost the ability to discern the movement
ons. X-ray revealed spheric helminth larvae of an object on the skin. What part of the
in the patient’s lungs. Specify the helminth cerebral cortex is damaged?
that could be the causative agent of this di-
sease: A. Posterior central gyrus
B. Occipital lobe
A. Echinococcus C. Parietal lobe
B. Hymenolepis nana D. Frontal lobe
C. Diphylobotrium latum E. Anterior central gyrus
D. Fasciola hepatica
E. Taenia solium 27. Blood test of the patient revealed
albumine content of 20 g/l and increased
23. A 67-year-old patient with cli- activity of lactate dehydrogenase isoenzyme
nical diagnosis of chronic bronchitis, 5 (LDH5). These results indicate disorder of
pneumosclerosis, and cardiopulmonary the following organ:
decompensation has the biopsy material
taken from the suspicious area in his ri- A. Liver
ght bronchus mucosa. Cellular and tissue B. Kidneys
atypism along with pearly bodies can be C. Heart
histologically detected. What pathologic D. Lungs
process is characterized by the described E. Spleen
histological changes?
28. Section shows significant enlargement
A. Squamous cell carcinoma of bronchus of the patient’s right kidney. There is a
with keratinization nephrolith at the place of incision. Renal
B. Polypoid chronic bronchitis pelvic lumen is distended with accumulating
C. Bronchiectasis urine. Renal parenchyma is acutely thinned
D. Acute bronchitis out. What is the most correct diagnosis?
E. Squamous cell metaplasia of bronchial
mucosa A. Hydronephrosis
B. Pyelectasis
24. A child with point mutation presents C. Hydroureteronephrosis
with absence of glucose 6-phosphatase, D. Renal cyst
hypoglycemia, and hepatomegaly. What E. Nephroblastoma
pathology are these signs characteristic of?
29. A patient demonstrates sharp decrease
A. Von Gierke’s disease (Glycogen storage of pulmonary surfactant activity. This condi-
disease type I) tion can result in:
B. Cori’s disease (Glycogen storage disease
type III) A. Alveolar tendency to recede
B. Decreased airways resistance
C. Addison’s disease (Primary adrenal C. Decreased work of expiratory muscles
insufficiency) D. Increased pulmonary ventilation
D. Parkinson’s disease E. Hyperoxemia
E. McArdle’s disease (Glycogen storage
disease type V) 30. After a case of cold the patient
developed a lacrimation disorder. This di-
25. At the 2-3 day after the gastric resection sorder was caused by functional disturbance
the patient’s intestinal peristalsis failed to of the following autonomic ganglion:
restore. What should the patient be prescri-
bed to stimulate the function of his gastroi- A. Pterygopalatine
ntestinal tract? B. Ciliary
C. Otic
D. Submandibular
E. Sublingual
31. A patient is diagnosed with diabetic
coma. Blood sugar is 18,44 mmol/l. What
Krok 1 Medicine (англомовний варiант, iноземнi студенти) 2017 рiк 4

glucose-regulating drug should be prescri- A. Creatinine


bed in the given case? B. Total lipids
C. Glucose
A. Rapid-acting insulin D. Mineral salts
B. Intermediate-acting insulin E. Uric acid
C. Long-acting insulin
D. Biguanide 37. A 30-year-old woman first developed
E. Sulfonylurea derivative pain, swelling, and skin redness in the area
of joints about a year ago. Provisional di-
32. A man came into the admission room agnosis is rheumatoid arthritis. One of the
with complaints of edemas, rapid heart rate, likely causes of this disease is change in the
dyspnea, and cyanotic mucosal tunics. He structure of the following connective tissue
was diagnosed with chronic heart failure. protein:
What drug should be prescribed to improve
the patient’s general state? A. Collagen
B. Mucin
A. Digoxin C. Myosin
B. Papaverine hydrochloride D. Ovalbumin
C. Mesaton (Phenylephrine) E. Troponin
D. Cordiamin
E. Nitroglycerine 38. A 15-year-old teenager complains of
lack of air, general weakness, palpitati-
33. Exposure to colchicine resulted in ons. Heart rate is 130/min., BP is 100/60
metaphase plate of a human containing 23 mm Hg. ECG: QRS complex has normal
chromosomes more than it is normal. Name shape and duration. The number of P waves
this mutation: and ventricular complexes is equal, T wave
merges with P wave. What type of cardiac
A. Polyploidy arrhythmia is observed in the teenager?
B. Aneuploidy
C. Polyteny A. Sinus tachycardia
D. Inversion B. Sinus extrasystole
E. Translocation C. Atrial fibrillation
D. Atrial thrill
34. A 30-year-old man complains of E. Paroxysmal atrial tachycardia
suffocation, heaviness in the chest on the
right, general weakness. Body temperature 39. A patient complaining of dizziness, thi-
is 38,9o C . Objectively the right side of rst, difficult swallowing, and impaired visi-
the chest lags behind the left side during on of close objects has addressed a doctor.
respiration. Pleurocentesis yielded exudate. Objectively: respiratory rate is increased,
What is the leading factor of exudation in pupils are dilated, general agitation, talkati-
the patient? veness, though the speech is indistinct. BP is
110/70 mm Hg, heart rate is 110/min. Given
A. Increased permeability of the vessel wall symptoms can indicate overdosage of the
B. Increased blood pressure following drug:
C. Hypoproteinemia
D. Erythrocyte aggregation A. Atropine
E. Decreased resorption of pleural fluid B. Morphine
C. Ephedrine
35. A 46-year-old woman suffering from D. Aminazine
cholelithiasis developed jaundice. Her urine E. Caffeine
became dark yellow, while feces are light-
colored. What substance will be the most 40. On autopsy of a 40-year-old woman,
increased in concentration in the blood who had been suffering from rheumatoid
serum in this case? arthritis, her liver is found to be dense
and enlarged. On dissection its tissue is
A. Conjugated bilirubin red-brown colored, with enlarged follicles
B. Unconjugated bilirubin resembling semi-transparent grayish-white
C. Biliverdine granules. What is the most likely pathologi-
D. Mesobilirubin cal process?
E. Urobilinogen
A. Sago spleen
36. A traumatology unit received a pati- B. Sugar-coated spleen
ent with crushed muscular tissue. What bi- C. Lardaceous spleen
ochemical indicator of urine will be raised D. Splenic hyalinosis
in this case? E. Porphyry spleen
Krok 1 Medicine (англомовний варiант, iноземнi студенти) 2017 рiк 5

41. On autopsy the dissector determined cells and venous sinusoids. What organ can
that the lungs are enlarged, pale, soft, do not be characterized by these morphological
deflate, crunch when cut. Microscopically features?
there are dilated alveolar ducts, alveolar
septa are thin, and signs of intracapillary A. Adrenal gland
sclerosis are observed. What pulmonary di- B. Kidney
sorder are these presentations characteristic C. Lymph node
of? D. Thymus
E. Thyroid gland
A. Emphysema
B. Pneumosclerosis 46. A child with suspected colienteritis was
C. Pneumothorax delivered to the infectious diseases hospital.
D. Atelectasis Colibacillus was obtained from the child’s
E. Pneumonia feces. How to determine whether this baci-
llus is of pathogenic variety?
42. During removal of the hyperplastic
thyroid gland of a 47-year-old woman, A. Agglutination reaction with serum 0
the parathyroid gland was damaged. B. Based on its biochemical properties
One month after the surgery the pati- C. By means of bacteriophage typing
ent developed signs of hypoparathyroi- D. Microscopy of stained smears
dism: frequent convulsions, hyperreflexia, E. Based on the nature of its growth in Endo
laryngospasm. What is the most likely cause medium
of the patient’s condition?
47. A patient ungergoes right-sided
A. Hypocalcemia pulmonectomy due to lung cancer. Name
B. Hyponatremia the anatomical structures of the right lung
C. Hyperchlorhydria radix (downward order):
D. Hypophosphatemia
E. Hyperkalemia A. Bronchus, artery, veins
B. Artery, bronchus, veins
43. On examination the patient presents C. Artery, veins, bronchus
with hirsutism, moon-shaped face, stretch D. Veins, artery, bronchus
marks on the abdomen. BP is 190/100 mm E. Veins, bronchus, artery
Hg, blood glucose is 17,6 mmol/l. What
pathology is such clinical presentation 48. In a township there was registered
characteristic of? an outbreak of hepatitis, which was attri-
buted to water supply. What hepatitis virus
A. Adrenocortical hyperfunction could be the cause of the outbreak in this
B. Hyperthyroidism township?
C. Hypothyroidism
D. Gonadal hypofunction A. Hepatitis E virus
E. Hyperfunction of the insular apparatus B. Hepatitis C virus
C. Hepatitis D virus
44. Histological specimen of the ovary D. Hepatitis G virus
shows large hollow structures. Primary E. Hepatitis B virus
oocyte within these structures is surrounded
with transparent membrane and radiati- 49. A 64-year-old woman presents with di-
ng crown and is situated in the cumulus sturbed fine motor function of her fingers,
oophorus, the wall is made of follicular cell marked muscle rigidity, and tremor. The
layer and theca. What ovarian structure can neurologist diagnosed her with Parkinson’s
be characterized by these morphological disease. What brain structures are damaged
features? resulting in this disease?

A. Mature (tertiary) follicle A. Substantia nigra


B. Primordial follicle B. Thalamus
C. Primary follicle C. Red nuclei
D. Сorpus luteum D. Cerebellum
E. Corpus atreticum E. Reticular formation

45. Histological specimen demonstrates 50. Due to prolonged taking of a drug the
a parenchymal organ with cortical patient can develop osteoporosis, gastric
and medullary substances. The cortical mucosal erosions, hypokalemia, sodium
substance is composed of bands of epitheli- and water retention, and decreased blood
al cells with capillary blood vessels between content of corticotropine. Specify this drug:
them. The bands form three zones. The
medullary substance consists of сhromaffin
Krok 1 Medicine (англомовний варiант, iноземнi студенти) 2017 рiк 6

A. Prednisolone impression smear revealed gram-negative


B. Hydrochlorothiazide spiral-shaped microorganisms, urease acti-
C. Digoxin vity test is positive. What bacteria were
D. Indometacin detected?
E. Reserpine
A. Helycobacter pylori
51. Nitrogen is being excreted from the B. Spirilla minor
body mainly as urea. When activity of a C. Shigella flexneri
certain enzyme in the liver is low, it results D. Treponema pallidum
in inhibition of urea synthesis and nitrogen E. Campylobacter jeuni
accumulation in blood and tissues. Name
this enzyme: 56. A patient suffering from gout was
prescribed allopurinol. What pharmacologi-
A. Carbamoyl phosphate synthetase cal property of allopurinol provides
B. Aspartate aminotransferase therapeutic effect in this case?
C. Urease
D. Amylase A. Competitive inhibition of xanthine oxi-
E. Pepsin dase
B. Acceleration of nitrogen-containing
52. After pancreatic surgery the patient substances excretion
developed hemorrhagic syndrome with di- C. Acceleration of pyrimidine nucleotides
sturbed 3rd stage of blood clotting. What catabolism
will be the most likely mechanism of the D. Deceleration of pyrimidine nucleotides
hemostatic disorder? salvage
E. Acceleration of nucleic acids synthesis
A. Fibrinolysis activation
B. Decrease of prothrombin synthesis 57. A woman, who has been suffering from
C. Decrease of fibrinogen synthesis marked hypertension for 15 years, has lately
D. Qualitative abnormalities of fibri- developed dyspnea, palpitations, slightly
nogenesis decreased systolic pressure, while diastolic
E. Fibrin-stabilizing factor deficiency pressure remains the same. What is the main
mechanism of heart failure development in
53. A patient with jaundice has high this case?
total bilirubin that is mainly indirect
(unconjugated), high concentration of A. Cardiac overload due to increased
stercobilin in the feces and urine. The level vascular resistance
of direct (conjugated) bilirubin in the blood B. Cardiac overload due to increased blood
plasma is normal. What type of jaundice can volume
be suspected? C. Damage to the myocardium
D. Disorder of impulse conduction in the
A. Hemolytic myocardium
B. Parenchymal (hepatic) E. Dysregulation of cardiac function
C. Mechanical
D. Neonatal 58. Old burial ground for animal refuse,
E. Gilbert’s disease which has not been in use for the last 50
years, is planned to be given for housing
54. Histological specimen shows organ development. However, the soil analysis
parenchyma to consist of lymphoid tissue detected viable spores of a causative agent
that forms lymph nodules; the nodules of an extremely dangerous disease. What
are located diffusely and have a central microorganism is the most likely to remain
artery. What anatomical structure has such in the soil for such a long period of time?
morphological characteristics?
A. Bacillus anthracis
A. Spleen B. Francisella tularensis
B. Tonsil C. Brucella abortus
C. Lymph node D. Yersinia pestis
D. Thymus E. Mycobacterium bovis
E. Red bone marrow
59. Blood of the patients with diabetes
55. During fibergastroscopy of a patient wi- mellitus shows increased content of free
th ulcer disease of the stomach, the mucosal fatty acids. Name the most likely cause of
biopsy material is taken from the area of this:
an ulcer. Impression smear is prepared from
the biopsy material and stained by Gram
method; the rest of the biopsy material is
tested for urease activity. Microscopy of the
Krok 1 Medicine (англомовний варiант, iноземнi студенти) 2017 рiк 7

A. Increased activity of adipose triglyceride


lipase A. Medulla oblongata
B. Accumulation of palmitoyl-CoA in B. Diencephalon
cytosol C. Mesencephalon
C. Activation of ketone bodies utilization D. Spinal cord
D. Activation of apoА1, apoА2, and apoА4 E. Cerebral cortex
apolipoprotein synthesis
E. Decreased activity of plasma 65. Investigation of an isolated cardi-
phosphatidylcholine-cholesterol- ac myocyte determined that it does not
acyltransferase generate excitation impulses automati-
cally, which means this cardiac myocyte
60. During autopsy of a man, who had been was obtained from the following cardiac
suffering from mitral stenosis, the lungs are structure:
revealed to be dense and brown-colored.
What pathologic process had occurred in A. Ventricles
the lungs? B. Sinoatrial node
C. Atrioventricular node
A. Hemosiderosis D. His’ bundle
B. Hemochromatosis E. Purkinje’s fibers
C. Jaundice
D. Hemomelanosis 66. To lose some weight a woman has been
E. Lipofuscinosis limiting the amount of products in her di-
et. 3 months later she developed edemas
61. Examination of the coronary arteries and her diuresis increased. What dietary
revealed atherosclerotic plaques with calci- component deficiency is the cause of this?
nosis that close the arterial opening by 1/3.
In the muscle there are numerous whiti- A. Proteins
sh layers of connective tissue. Name the B. Fats
process detected in the myocardium: C. Carbohydrates
D. Vitamins
A. Diffuse cardiosclerosis E. Minerals
B. Tiger heart
C. Postinfarction cardiosclerosis 67. The patients with organic brain disorder
D. Myocarditis can take the following drug to improve their
E. Myocardial infarction memory:

62. T-lymphocytes are determined to be A. Piracetam


affected with HIV. In this case viral enzyme B. Nitrazepam
reverse transcriptase (RNA-dependent C. Medazepam
DNA-polymerase) catalyzes the synthesis D. Diazepam
of: E. Caffeine

A. DNA based on the viral RNA matrix 68. A 40-year-old man developed skin
B. Viral RNA based on the DNA matrix redness and swelling in the neck area,
C. Viral protein based on the viral RNA where eventually a small abscess appeared.
matrix On section the focus is dense and yellow-
D. Viral DNA based on the DNA matrix green colored. In the purulent masses there
E. Informational RNA based on the viral are white granules. Histologically there
protein matrix are fungal druses, plasma and xanthome
cells, and macrophages detected. Specify
63. A woman with dense area in her the most correct etiological name of this
mammary gland came to a surgeon. To mi- pathological process:
nimize the trauma to the mammary gland
lobule during the operation the surgeon A. Actinomycosis
should make the incision: B. Furuncle
C. Carbuncle
A. Radially D. Syphilis
B. Vertically E. Leprosy
C. Transversely
D. Arcuately 69. In preparation for business trip abroad
E. - the doctor was prescribed a histoschi-
zontocidal antimalarial drug as a personal
64. Electrical activity of neurons is being means of disease prevention. What drug was
measured. They fire prior to and at the given to the doctor?
beginning of inhalation. Where are these
neurons situated?
Krok 1 Medicine (англомовний варiант, iноземнi студенти) 2017 рiк 8

A. Chingamin antitoxin serum was introduced into the test


B. Biseptol (Co-trimoxazole) group of mice; the control group of mice
C. Doxycycline received the extract without antibotulinic
D. Mefloquine serum. What serological reaction was used?
E. Quinine
A. Neutralization
70. A patient, who has been suffering from B. Precipitation
bronchial asthma for a long time, developed C. Complement binding
acute respiratory failure. What is the main D. Opsonophagocytic
mechanism of pathology development in E. Double immunodiffusion
this case?
75. A 50-year-old man is diagnosed with
A. Obstructive disorders of pulmonary ischemic heart disease and cardiosclerosis
ventilation with hypertensive syndrome. What drug
B. Restrictive disorders of pulmonary venti- should be prescribed in this case?
lation
C. Pulmonary blood supply disturbance A. Metoprolol
D. Pulmonary enzyme system disturbance B. Acetylsalicylic acid
E. Decreased elasticity of the pulmonary C. Corglycon
tissue D. Strophanthine
E. Potassium chloride
71. A 40-year-old woman has undergone
thyroidectomy. Histological study of thyroid 76. Poisoning caused by mercury (II) chlori-
gland found the follicles to be of different de (corrosive sublimate) occurred in the
size and contain foamy colloid, follicle epi- result of safety rules violation. In 2 days the
thelium is high and forms papillae, there is patient’s diurnal diuresis became 620 ml.
focal lymphocytic infiltration in the stroma. The patient developed headache, vomiti-
Diagnose the thyroid gland disease: ng, convulsions, dyspnea; moist crackles are
observed in the lungs. Name this pathology:
A. Basedow’s disease
B. Hashimoto’s thyroiditis A. Acute renal failure
C. Riedel’s thyroiditis B. Chronic renal failure
D. De Quervain’s disease C. Uremic coma
E. Nodular goiter D. Glomerulonephritis
E. Pyelonephritis
72. A patient has been hospitalized
with provisional diagnosis of virus B 77. Autopsy of a 9-year-old child shows
hepatitis. Serological reaction based on numerous irregular defects of varying depth
complementation of antigen with antibody with uneven margins and gray-white films
chemically bound to peroxidase or alkaline tightly attached to the underlying tissue on
phosphatase has been used for disease di- the rectal mucosa of the body. What disease
agnostics. What is the name of the applied can be suspected?
serological reaction?
A. Dysentery
A. Enzyme-linked immunosorbent assay B. Salmonellosis
B. Radioimmunoassay technique C. Cholera
C. Immunofluorescence test D. Typhoid fever
D. Complement fixation test E. Amebiasis
E. Immobilization test
78. A man presents with glomerular fi-
73. Due to blood loss the circulating blood ltration rate of 180 ml/min., while norm is
volume of a patient decreased. How will it 125±25 ml/min. The likely cause of it is the
affect the blood pressure in this patient? decreased:
A. Systolic and diastolic pressure will A. Plasma oncotic pressure
decrease B. Effective filtration pressure
B. Only systolic pressure will decrease C. Hydrostatic blood pressure in the
C. Only diastolic pressure will decrease glomerular capillaries
D. Systolic pressure will decrease, while D. Renal blood flow
diastolic will increase E. Permeability of the renal filter
E. Diastolic pressure will decrease, while
systolic will increase 79. A young man came to a hospital with
complaints of disturbed urination. Exami-
74. A bacteriological laboratory tests nation of his external genitalia revealed
canned meat for botulinum toxin. Extract the urethra to be split on the top, with uri-
of the tested material and ABE botulinum ne flowing out of this opening. What type
Krok 1 Medicine (англомовний варiант, iноземнi студенти) 2017 рiк 9

of external genitalia maldevelopment is A. Ulcer disease


observed in this case? B. Ischemic heart disease
C. Diabetes mellitus
A. Epispadia D. Essential hypertension
B. Phimosis E. Bronchial asthma
C. Hermaphroditism
D. Paraphimosis 84. Histologic preparation stained wi-
E. Hypospadias th orcein demonstrates from 40 to 60
fenestrated elastic membranes within the
80. A 10-year-old child had cut his leg wi- middle coat of the vessel. Name this vessel:
th a glass shard, when playing, and was
delivered to the outpatient department A. Elastic artery
to receive antitetanus serum. To prevent B. Muscular artery
development of anaphylactic shock the C. Mixed type artery
serum was introduced by Bezredka method. D. Muscular vein
This method of organism hyposensitization E. Nonmuscular vein
is based on the following mechanism:
85. A woman with the III (В), Rh (-) blood
A. Binding of mast cell-fixed IgE group gave birth to a child with the II (А)
B. Blocking of mast cell mediators synthesis blood group. The child is diagnosed with
C. Stimulation of immune tolerance to hemolytic disease of newborn caused by
antigen rhesus incompatibility. What blood group
D. Stimulation of antigen-specific IgG2 and Rh are likely in the father?
E. Stabilization of mast cell membranes
A. II (А), Rh (+)
81. A 38-year-old man, who has B. I (0), Rh (+)
been suffering from systemic lupus C. III (B), Rh (+)
erythematosus for 3 years, developed di- D. I (0), Rh (-)
ffuse renal lesions accompanied by massi- E. II (A), Rh (-)
ve edemas, marked proteinuria, hyperli-
pidemia, and dysproteinemia. What is 86. A 40-year-old woman with Cushi-
the most likely mechanism of proteinuria ng’s disease presents with steroid di-
develoment in this case? abetes. On biochemical examination she has
hyperglycemia and hypochloremia. What
A. Autoimmune damage to the nephrons process activates in the first place in such
B. Inflammatory damage to the nephrons patients?
C. Ischemic damage to the tubules
D. Increased blood proteins A. Gluconeogenesis
E. Morbid affection of the urinary tracts B. Glycogenolysis
C. Glucose reabsorption
82. During experiment a part of the brain D. Glucose transportation into a cell
was extracted, which resulted in asynergy E. Glycolysis
and dysmetria development in the test ani-
mal. What part of the brain was extracted in 87. A 40-year-old prisoner died of
the animal? tuberculosis in the corrective labor camp.
Autopsy of the body revealed deformation
A. Cerebellum and diminishing of both lung apices; in the
B. Frontal lobe both upper lobes there are multiple cavi-
C. Parietal lobe ties with dense walls 2-3 mm thick; in the
D. Mesencephalon lower lung lobes there are disseminated
E. Reticulum foci of caseous necrosis varying from 5 mm
83. A woman with polyarticular rheumatoid to 2 cm in diameter. Diagnose the type of
arthritis was prescribed a non-steroidal tuberculosis:
antiinflammatory drug - diclofenac sodium. A. Secondary fibro-cavitary tuberculosis
After the patient has been taking it for some B. Secondary fibrous-focal tuberculosis
time, her concomitant disease exacerbated, C. Hematogenous macrofocal pulmonary
which forced the doctor to cancel the tuberculosis
prescription of this drug. What concomi- D. Primary tuberculosis, primary affect
tant disease could necessitate cancellation development
of this drug prescription? E. Secondary cirrhotic tuberculosis
88. A 7-year-old boy died of acute
posthemorrhagic anemia caused by profuse
hemorrhage in the gastrointestinal tract.
Krok 1 Medicine (англомовний варiант, iноземнi студенти) 2017 рiк 10

Postmortem study revealed the followi-


ng: macroscopically there were acutely A. Radiating
enlarged various groups of the lymph nodes, B. Visceral
thymomegaly, hepatosplenomegaly, and C. Phantom
bright red bone marrow; microscopically D. Protopathic
there was hypercellular bone marrow wi- E. Epicritic
th monomorphic infiltrations composed of
blasts and diffuse-focal tumor infiltrations 93. During regular check-up a child is
in the liver, spleen, lymph nodes, brain determined to have interrupted minerali-
substance and tunics. Make the diagnosis: zation of the bones. What vitamin defici-
ency can be the cause?
A. Acute lymphoblastic leukemia
B. Acute myeloblastic leukemia A. Calciferol
C. Acute undifferentiated leukemia B. Riboflavin
D. Acute monoblastic leukemia C. Tocopherol
E. Acute plasmablastic leukemia D. Folic acid
E. Cobalamin
89. Autopsy of a man, who served on a
nuclear submarine, revealed the followi- 94. On examination the patient was
ng pathologies: bone marrow atrophy determined to have strong, balanced, inert
(panmyelophthisis), anemia, leukopenia, type of higher nervous activity according to
Pavlov’s classification. What temperament
thrombocytopenia, lymphocytes disi-
ntegration in the lymph nodes, spleen, according to Hippocrates is it?
gastrointestinal lymphatic system, and A. Phlegmatic
hemorrhages into the adrenal glands. What B. Sanguine
disease had developed in this case? C. Choleric
A. Acute radiation sickness D. Melancholic
B. Decompression sickness E. -
C. Acute leukemia 95. A specimen shows an organ covered wi-
D. Acute anemia th connective tissue capsule with trabeculae
E. Vibration disease radiating inward the organ. The organ’s
90. After sensitization a test animal recei- cortex contains lymph nodules; there are
ved subcutaneously a dose of antigen. At medullary cords made of lymphoid cells.
the site of injection a fibrinous inflammati- What organ is under study?
on developed with alteration of vessel walls, A. Lymph node
basal substance, and fibrous structures of B. Thymus
connective tissue. The inflammation took C. Spleen
form of mucoid and fibrinoid degeneration, D. Red bone marrow
fibrinoid necrosis. What immune response E. Tonsils
occurred in the test animal?
96. Brain autopsy revealed an edema,
A. Immediate hypersensitivity hyperemia, and small hemorrhages in
B. Delayed hypersensitivity the medulla oblongata. Microscopically
C. Transplantation immune reaction chromatolysis, hydropia and nerve cell
D. Normergic reaction necrosis are observed; within the cytoplasm
E. Granulomatosis of hippocampal nerve cells there are eosi-
91. Preoperative examination revealed nophilic structures (Negri bodies) detected.
prothrombin deficiency in the blood of the What diagnosis corresponds with the descri-
patient. What drug should be preliminarily bed morphological signs?
prescribed to mitigate blood loss in the pati- A. Rabies
ent during the surgery? B. Meningococcal meningitis
A. Vicasol (Menadione) C. Encephalitis
B. Thrombin D. Encephalomyelitis
C. Aminocapronic acid E. Brucellosis
D. Phenylin (Phenindione) 97. A married couple came for a genetic
E. Contrykal (Aprotinin) counseling. The husband suffers from
insulin-independent diabetes mellitus, while
92. A patient complaining of pain in the left the wife is healthy. What is the probability of
shoulder-blade region has been diagnosed their child developing insulin-independent
with miocardial infarction. What kind of diabetes mellitus?
pain does the patient have?
Krok 1 Medicine (англомовний варiант, iноземнi студенти) 2017 рiк 11

A. Higher than in the population A. Ambroxol


B. The same as in the population B. Anaprilin (Propranolol)
C. Lower than in the population C. Atropine sulfate
D. 100% D. Magnesium sulfate
E. 50% E. Paracetamol
98. A 46-year-old patient suffering from 103. A patient suffers from acute cardi-
ulcer disease of the stomach is diagnosed opulmonary failure with pulmonary edema.
with rheumatoid arthritis. What antii- What diuretic should be prescribed in the
nflammatory drug would be the most advi- given case?
sable in this case?
A. Furosemide
A. Celecoxib B. Triamterene
B. Prednisolone C. Spironolactone
C. Analgin (Metamizole) D. Dichlothiazidum (Hydrochlorothiazide)
D. Promedol (Trimeperidine) E. Diacarb (Acetazolamide)
E. Paracetamol
104. Therapeutics unit of a hospital recei-
99. Electron micrograph of the kidney ved a man suffering from ulcer disease of
shows fenestrated endothelium lying on the the stomach with hyperacidity. Which of the
basement membrane; the external surface listed group of drugs must be used as a part
of the membrane has adjacent dendritic epi- of the complex therapy of this patient?
thelial cells. What do these structures form
in the kidney? A. Histamine H2 -receptor antagonists
B. Calcium channel blockers
A. Filtration barrier C. Nonsteroidal antiinflammatory drugs
B. Juxtaglomerular apparatus D. Steroidal antiinflammatory drugs
C. Distal nephron E. Histamine H1 -receptor antagonists
D. Henle’s loop
E. Proximal nephron 105. A 45-year-old woman gave birth to
a boy with cleft maxilla (cleft lip and
100. A worker of an agricultural enterprise palate). On additional examination there
had been suffering from an acute disease are significant disturbances of the boy’s
with aggravating intoxication signs, whi- nervous, cardiovascular, and visual systems.
ch resulted in his death. On autopsy: the Karyotype investigation allowed diagnosing
spleen is enlarged, flaccid, dark cherry-red the patient with trisomy 13. What syndrome
on section, yields excessive pulp scrape. Soft is present in the boy?
meninges of the fornix and base of the brain
are edematous and saturated with blood A. Patau
(”cardinal’s cap”). Microscopically: serous- B. Down
hemorrhagic inflammation of meninges and C. Kleinfelter
cerebral tissues. Make the diagnosis: D. Turner
E. DiGeorge
A. Anthrax
B. Tularemia 106. Pathologic material (mucosal excreti-
C. Plague on from the nasal passages) obtained from
D. Cholera a patient provisionally diagnosed with
E. Brucellosis influenza was delivered to the virological
laboratory. What quick test allows detecti-
101. A woman suffers from tonsillitis ng specific viral antigen in the investigated
complicated with retropharyngeal abscess material?
that is localized in the spatium retrovi-
scerale. In this case the suppurative process A. Direct and indirect immunofluorescence
can spread to the: (IF)
B. Direct and indirect enzyme-linked
A. Mediastinum posterius
B. Spatium interaponeuroticum suprasternale immunosorbent assay (ELISA)
C. Mediastinum anterius C. Hemagglutination inhibition assay (HAI)
D. Spatium pretracheale D. Reverse indirect haemagglutination (RI-
E. Spatium interscalenum HA)
E. Radioimmunoassay (RIA)
102. A patient with chronic bronchitis was
prescribed a drug with mucolytic action. 107. A 5-year-old child is diagnosed
Name this drug: with Bruton syndrome (X-linked
agammaglobulinemia) that manifests itself
in severe clinical course of bacterial
Krok 1 Medicine (англомовний варiант, iноземнi студенти) 2017 рiк 12

infections and absence of B lymphocytes A. Monoamine oxidase


and plasma cells. What changes of B. Diamine oxidase
immunoglobulin content can be observed in C. L-amino acids oxidase
blood serum of the child with immunodefi- D. D-amino acid oxidase
ciency? E. Phenylalanine 4-monooxygenase
A. Decreased IgA, IgM 112. A 2-year-old child presents with acute
B. Increased IgA, IgM psychomotor retardation, vision and heari-
C. Decreased IgD, IgE ng impairment, sharp enlargement of the
D. Increased IgD, IgE liver and spleen. The child is diagnosed wi-
E. No changes th hereditary Niemann-Pick disease. What
genetic defect is the cause of this disease?
108. A 37-year-old man, who was worki-
ng in a caisson, after being lifted to the A. Sphingomyelinase deficiency
surface suddenly developed signs of acute B. Glucose 6-phosphatase deficiency
cerebral circulation disturbance and loss of C. Amylo-1,6-glucosidase deficiency
consciousness. Several days later he died. D. Acid lipase deficiency
On autopsy in the left cerebral hemisphere E. Xanthine oxidase deficiency
there was detected a gray soft irregular
focus 5х6х3,5 cm in size. What process had 113. Clinical presentations of a woman
occurred in the brain? allowed provisionally diagnosing her with
X polysomy. Cytogenetic method is applied
A. Ischemic stroke to clarify the diagnosis. The diagnosis will
B. Hemorrhagic infarction be confirmed if the patient’s karyotype is:
C. Abscess
D. Cyst A. 47, ХХХ
E. Tumor B. 48, XXXY
C. 48, XXYY
109. A short-term physical load resulted in D. 47, XXY
reflex increase of heart rate and systemic E. 46, XX
arterial pressure in a person. What receptor
activation was the most contributory to 114. During dehelmintization there was a
inducing the pressor reflex? 3,5-meter-long tapeworm produced from
the patient’s intestine.There are 4 suckers
A. Proprioceptors of the working muscles and hooks on the tapeworm’s scolex.
B. Vascular chemoreceptors Mature segments of the tapeworm are
C. Vascular volume receptors immobile and have up to 12 uterine
D. Vascular baroreceptors branches. What disease is it?
E. Hypothalamic thermoreceptors
A. Teniasis
110. During experiment a skeletal muscle is B. Echinococcosis
being stimulated with a series of electrical C. Beef tapeworm infection
impulses. What type of muscular contracti- D. Diphyllobothriasis
on will develop, if each following impulse E. Opisthorchiasis
occurs within the relaxation period after the
previous single contraction of the muscle? 115. A patient presents with steatorrhea.
This disorder can be linked to disturbed
A. Incomplete tetanus supply of the intestine with the following
B. Smooth tetanus substances:
C. Series of single contractions
D. Muscle contracture A. Bile acids
E. Asynchronous tetanus B. Carbohydrates
C. Tripsin
111. Depression and emotional di- D. Chymotrypsin
sturbances result from the lack of E. Amylase
noradrenaline, serotonin, and other bi-
ogenic amines in the brain. Their content 116. Cytochrome oxidase is a hemeprotein
in the synapses can be increased through that is an end component of the mi-
administration of antidepressants that inhi- tochondrial respiratory chain. What reacti-
bit the following enzyme: on is catalyzed with this enzyme?
Krok 1 Medicine (англомовний варiант, iноземнi студенти) 2017 рiк 13

A. Transfer of reduced equivalents to following: erythrocytes - 5, 5 · 1012 /l,


molecular oxygen hemoglobin - 180 g/l, leukocytes - 7 ·
B. Cytochrome synthesis 109 /l, neutrophils - 64%, basophils - 0,5%,
C. Transfer of reduced equivalents to ubiqui- eosinophils - 0,5%, monocytes - 8%,
none lymphocytes - 27%. These values primari-
D. Cytochrome splicing ly indicate the stimulation of:
E. Adenosine triphosphate synthesis
A. Erythropoiesis
117. A 16-year-old girl fainted when she tri- B. Leukopoiesis
ed to quickly change her position from hori- C. Lymphopoiesis
sontal to vertical. What caused the loss of D. Granulocytopoiesis
consciousness in the girl? E. Immunogenesis
A. Decreased venous return 122. Experimental stimulation of the
B. Increased venous return sympathetic nerve branches that innervate
C. Increased central venous pressure the heart caused an increase in the force of
D. Decreased oncotic plasma pressure heart contractions because the membrane
E. Increased arterial pressure of typical cardiomyocytes permitted an
118. An unconscious patient was delivered increase in:
by ambulance to the hospital. On objective A. Calcium ion entry
examination the patient was found to have B. Calcium ion exit
no reflexes, periodical convulsions, irregular C. Potassium ion exit
breathing. After laboratory examination the D. Potassium ion entry
patient was diagnosed with hepatic coma. E. Calcium and potassium ion exit
Disorders of the central nervous system
develop due to the accumulation of the 123. As a result of past encephalitis, a man
following metabolite: has developed an increase in cerebrospinal
fluid pressure in the right lateral ventricle.
A. Ammonia What can be the cause of this condition?
B. Urea
C. Glutamine A. Closure of the right interventricular
D. Bilirubin foramen
E. Histamine B. Closure of the left interventricular
foramen
119. A patient complains of acute pain C. Atresia of the tubus medullaris
attacks in the right lumbar region. During D. Atresia of the sylvian aqueduct
examination the nephrolithic obturation of E. Atresia of the fourth ventricle foramina
the right ureter in the region between its
abdominal and pelvic segments has been 124. A patient with pneumonia has body
detected. What anatomical boundary exists temperature of 39,2oC . What cells are the
between those two segments? main producers of endogenous pyrogen that
had caused such temperature rise?
A. Linea terminalis
B. Linea semilunaris A. Monocytes
C. Linea arcuata B. Eosinophils
D. Linea transversa C. Neutrophils
E. Linea inguinalis D. Endotheliocytes
E. Fibroblasts
120. After mushroom poisoning the pati-
ent developed signs of acute hepatic fai- 125. A patient is diagnosed with compressi-
lure leading to his death. On autopsy the on fracture of the lumbar vertebra. The pati-
liver is diminished, flaccid; the capsule ent presents with acutely increased lumbar
is wrinkled; the tissue is ochre-yellow on lordosis. What ligament was damaged in
section. Microscopically: fatty degeneration this patient resulting in such deformation
of hepatocytes, necrotic central segments of vertebral column curvature?
of the hepatic lobes. These changes are
characteristic of: A. Anterior longitudinal ligament
B. Posterior longitudinal ligament
A. Massive progressive necrosis C. Yellow ligament
B. Fatty hepatosis D. Iliolumbar ligament
C. Acute exudative hepatitis E. Interspinal ligament
D. Acute productive hepatitis
E. Hepatolenticular degeneration 126. Gram-positive spore-forming baci-
lli were extracted in anoxic environment
121. Blood test of an athlete shows the from the patient’s wound contaminated
Krok 1 Medicine (англомовний варiант, iноземнi студенти) 2017 рiк 14

with soil. Cultivation on a blood-glucose A. Sympathetic unconditioned reflex


agar resulted in growth of the colonies B. Sympathetic conditioned reflex
surrounded with hemolysis zone. What C. Metasympathetic reflex
agent was extracted from the wound? D. Parasympathetic unconditioned reflex
E. Parasympathetic conditioned reflex
A. Сlostridium perfringens
B. Staphylococcus aureus 132. A patient complaining of heartburn has
C. Clostridium botulinum undergone biopsy of the gastric mucosa. In
D. Escherichia coli the sample there are numerous cells with
E. Pseudomonas aeruginosa oxyphilic cytoplasm in the glandular epi-
thelium. Name these cells:
127. The patient’s ECG shows that in the
second standard lead from the extremities A. Exocrine parietal cells
the P waves are positive, their amplitude is B. Exocrine chief cells
0,1 mV (norm is 0,05-0,25 mV), duration - C. Mucous cells
0,1 seconds (norm is 0,07-0,10 seconds). It D. Epithelial cells
can be concluded that the following process E. Endocrine cells
occurs normally in the cardiac atria: 133. Inherited diseases, such as
A. Depolarization mucopolysaccharidoses, manifest in
B. Repolarization metabolic disorders of connective tissue,
C. Activation bone and joint pathologies. The sign of this
D. Contraction disease is the excessive urinary excretion of
E. Relaxation the following substance:

128. Autopsy of a patient, who died of bi- A. Glycosaminoglycans


lateral bronchopneumonia, shows in the B. Amino acids
left lung lower lobe a cavity 5 cm in di- C. Glucose
ameter, filled with liquid yellowish-white D. Lipids
substance. What complication of the pati- E. Urea
ent’s pneumonia had developed? 134. A woman was delivered to a
A. Abscess gynecological unit with signs of acute
B. Gangrene abdomen and suspected extrauterine
C. Granuloma pregnancy with oviduct rupture. Where will
D. Sequestrum the blood accumulate in this case?
E. Tuberculoma A. Rectouterine pouch
129. A patient, who has been subsisting B. Vesicouterine pouch
exclusively on polished rice, has developed C. Retrovesical pouch
polyneuritis due to thiamine deficiency. D. Right lateral canal
What substance is an indicator of such avi- E. Left lateral canal
taminosis, when it is excreted with urine? 135. A patient had a trauma that caused
A. Pyruvic acid dysfunction of motor centers regulating
B. Malate activity of head muscles. These centers can
C. Methylmalonic acid normally be located in the following area of
D. Uric acid the cerebral cortex:
E. Phenyl pyruvate A. Inferior part of the precentral gyrus
130. It is known that pentose-phosphate B. Superior part of the precentral gyrus
pathway actively functions in the C. Supramarginal gyrus
erythrocytes. What is the main function of D. Superior parietal lobule
this metabolic pathway in the erythrocytes? E. Angular gyrus

A. Counteraction to lipid peroxidation 136. At the post-mortem examination the


B. Activation of microsomal oxidation stomach of a patient with renal failure was
C. Neutralization of xenobiotics found to have a yellow-brown coating on
D. Oxidation of glucose into lactate the thickened mucosa. The coating was fi-
E. Increase of lipid peroxidation rmly adhering to its surface and had si-
gnificant thickness. Microscopy revealed
131. Pupil dilation occurs when a person congestion and necrosis of mucosal and
steps from a light room into a dark one. submucosal layers, fibrin presence. What is
What reflex causes such a reaction? the most likely diagnosis?
Krok 1 Medicine (англомовний варiант, iноземнi студенти) 2017 рiк 15

A. Diphtheritic gastritis heat processing. What helminthiasis was the


B. Croupous gastritis case?
C. Phlegmonous gastritis
D. Catarrhal gastritis A. Trichinosis
E. Corrosive gastritis B. Taeniarhynchosis
C. Teniasis
137. Cystinuria manifests itself in a D. Echinococcosis
human as cystine stones in the kidneys E. Alveococcosis
(homozygous individuals) or increased
cystine content in the urine (heterozygous 142. A person with vitamin A deficiency
individuals). Cystinuria is a monogenic di- develops twilight vision disturbance. Name
sorder. Determine the type of interaction the cells that fulfill this photoreceptor
between the genes of cystinuria and normal function:
urine cystine excretion: A. Rod cells
A. Semidominance B. Horizontal cells of retina
B. Epistasis C. Bipolar neurons
C. Complete dominance D. Cone cells
D. Complementarity E. Ganglionic nerve cells
E. Codominance 143. Presented is the biopsy material of an
138. A patient with obliterating organ consisting of saccule-shaped rounded
atherosclerosis has undergone sympathectomy structures of varying size. Inside these
of the femoral artery in the area of femoral structures there is a gel-like non-cellular
triangle. What type of arterial hyperemia substance - colloid; structure walls are
was developed in the patient due to this composed of one layer of cuboidal cells that
surgery? lay on the basement membrane. Between
the saccules there is connective tissue with
A. Neuroparalytic vessels. Name this organ:
B. Reactive
C. Metabolic A. Thyroid gland
D. Neurotonic B. Pancreas
E. Functional C. Parotid gland
D. Thymus
139. On bronchoscopy there is a polypoid E. Parathyroid gland
growth 1,0 cm in diameter with ulcer in
its center in the upper lobe of the right 144. A 26-year-old woman with bronchitis
lung. Histological investigation revealed a has been administered a broad spectrum
tumor composed of lymphocyte-like cells antibiotic as a causal treatment drug. Speci-
with hyperchromic nuclei, the cells form fy this drug:
layers and bands. What is the most likely A. Doxycycline
tumor type? B. Interferon
A. Undifferentiated small cell carcinoma C. BCG vaccine
B. Undifferentiated large cell carcinoma D. Ambroxol
C. Squamous cell carcinoma E. Dexamethasone
D. Adenocarcinoma 145. Water affects the mucosa of lower nasal
E. Glandular squamous cell carcinoma passages resulting in diving reflex. This
140. The brain trauma unit received a pati- responce manifests itself as:
ent with damaged greater wing of the A. Reflex apnea
sphenoid bone. The fracture line crosses B. Reflex dyspnea
the spinous foramen of the sphenoid. What C. Reflex hyperpnea
vessel was damaged? D. Cough
A. Middle meningeal artery E. Bronchial spasm
B. Superficial temporal artery 146. Examination of a 56-year-old woman
C. Lateral pterygoid artery with a history of type 1 diabetes revealed
D. Anterior deep temporal artery a disorder of protein metabolism that
E. Posterior deep temporal artery is manifested by aminoacidemia in the
141. In one of Polessye regions there was laboratory blood test values, and clini-
an outbreak of helminthiasis manifested by cally by the delayed wound healing and
cramps and facial edemas. The developed decreased synthesis of antibodies. Which
preventive measures in particular included of the following mechanisms causes the
ban for eating infested pork even after development of aminoacidemia?
Krok 1 Medicine (англомовний варiант, iноземнi студенти) 2017 рiк 16

152. To treat bronchitis the patient was


A. Increased proteolysis prescribed a beta-lactam antibiotic. Its
B. Albuminosis mechanism of action is based on inhibition
C. Decrease in concentration of blood amino of murein production, which results in death
acids of the causative agent. Name this drug:
D. Increase in plasma oncotic pressure
E. Increase in low-density lipoproteins level A. Penicillin G Sodium Salt
B. Bijochinol (Quinine bismuth iodide)
147. One of the factors that cause obesity is C. Ciprofloxacin
the inhibition of fatty acids oxidation due to: D. Azithromycin
E. Streptomycin
A. Low carnitine content
B. Impaired phospholipid synthesis 153. Cell membrane rest potential changed
C. Excessive consumption of fatty foods from -85 to -90 mV. It can be caused by
D. Choline deficiency activation of the following cell membrane
E. Lack of carbohydrates in the diet channels:
148. Streptomycin and other aminoglycosi- A. Potassium
des prevent the joining of formyl- B. Sodium
methionyl-tRNA by bonding with the 30S C. Potassium and sodium
ribosomal subunit. This effect leads to di- D. Calcium
sruption of the following process: E. Potassium and calcium
A. Translation initiation in procaryotes 154. Ionizing radiation or vitamin E defici-
B. Translation initiation in eucaryotes ency affect the cell by increasing lysosome
C. Transcription initiation in procaryotes membrane permeability. What are the possi-
D. Transcription initiation in eucaryotes ble consequences of this pathology?
E. Replication initiation in procaryotes
A. Partial or complete cell destruction
149. In hot weather ventilators are often B. Intensive protein synthesis
used to normalize the microclimate in the C. Intensive energy production
heated rooms. It leads to intensified heat D. Restoration of cytoplasmic membrane
transfer from the human body by means of: E. Formation of maturation spindle
A. Convection 155. A 22-year-old woman ate some
B. Conduction and convection seafood. 5 hours later her torso and distal
C. Conduction parts of her limbs developed small itchy
D. Radiation papules which were partially fused together.
E. Evaporation One day later the rash disappeared
spontaneously. Specify the hypersensitivi-
150. During gastric resection the patient ty mechanism underlying these changes:
received mixed anesthesia with tubocurarin
chloride muscle relaxant; to restore A. Atopy (local anaphylaxis)
spontaneous respiration the patient recei- B. Systemic anaphylaxis
ved proserin. What pharmacological group C. Cellular cytotoxicity
does this drug belong to? D. Immune complex hypersensitivity
E. Antibody-dependent cell-mediated
A. Cholinesterase inhibitors cytolysis
B. Angiotensin-converting-enzyme inhibi-
tors 156. During acute hemorrhage the body
C. Calcium channel blockers loses not only fluid but also electrolytes.
D. Muscarinic antagonists What substance solution can be used as a
E. Muscarinic agonists simple blood substitute?
151. A 67-year-old man consumes eggs, pork A. Sodium chloride
fat, butter, milk and meat. Blood test results: B. Sodium bromide
cholesterol - 12,3 mmol/l, total lipids - 8,2 C. Albumin
g/l, increased low-density lipoprotein fracti- D. Sodium nucleotide
on (LDL). What type of hyperlipoprotei- E. Calcium chloride
nemia is observed in the patient?
157. A patient has been admitted to the
A. Hyperlipoproteinemia type IIa contagious isolation ward with signs of
B. Hyperlipoproteinemia type I jaundice caused by hepatitis virus. Which of
C. Hyperlipoproteinemia type IIb the symptoms given below is strictly specific
D. Hyperlipoproteinemia type IV for hepatocellular jaundice?
E. Cholesterol, hyperlipoproteinemia
Krok 1 Medicine (англомовний варiант, iноземнi студенти) 2017 рiк 17

A. Increase of ALT, AST level


B. Hyperbilirubinemia A. Functional residual capacity
C. Bilirubinuria B. Vital capacity
D. Cholemia C. Respiratory minute volume
E. Urobilinuria D. Respiration rate
E. Maximal breathing capacity
158. In a body of a 37-year-old woman,
who died with signs of pulmonary edema, 163. In the South and Central Ameri-
there was detected acute deformation of ca there can be found a species of
the aortic valve: it is shortened, thickened, trypanosomes that is the causative agent of
ulcerated, has areas of stone-like density. Chagas disease. What animal is the infecti-
On its external surface there are large, up on carrier specific to this disease?
to 2 cm in diameter, thrombotic plaques.
Left ventricle wall is 2,2 cm thick. Cardiac A. Triatomine bug
muscle is dull, matt, and flaccid. What type B. Cockroach
of endocarditis corresponds with described C. Tsetse fly
alterations of the aortic valve? D. Mosquito
E. Gnat
A. Ulcerative polypoid endocarditis
B. Diffuse endocarditis 164. A 54-year-old woman was brought to
C. Acute verrucous endocarditis the emergency department after a car acci-
D. Recurrent verrucous endocarditis dent. A traumatologist diagnosed her with
E. Fibroplastic endocarditis multiple fractures of the lower extremities.
What kind of embolism is the most likely to
159. Students study the stages of develop in this case?
gametogenesis. They analyze a cell wi-
th haploid number of chromosomes, whi- A. Adipose
th each chromosome consisting of two B. Tissue
chromatids. The chromosomes are located C. Thromboembolism
in the equatorial plane of the cell. Such si- D. Gaseous
tuation is typical of the following stage of E. Air
meiosis:
165. A 30-year-old man with diabetes
A. Metaphase of the second division mellitus type I was hospitalized. The
B. Metaphase of the first division patient is comatose. Laboratory tests
C. Anaphase of the first division revealed hyperglycemia and ketonemia.
D. Anaphase of the second division What metabolic disorder can be detected
E. Prophase of the first division in this patient?

160. A 38-year-old woman developed a A. Metabolic acidosis


bronchial asthma attack. Which of the li- B. Metabolic alkalosis
sted bronchial spasmolytics is effective C. Respiratory acidosis
for emergency aid and belongs to beta-2- D. Respiratory alkalosis
adrenergic agonists? E. Acid-base balance is normal

A. Salbutamol 166. Typical manifestations of food poisoni-


B. Adrenaline ng caused by C. botulinum are double visi-
C. Ipratropium bromide on, abnormal functioning of the swallowing
D. Platyphyllin and breathing. These symptoms develop as
E. Atropine the result of:

161. After emotional upset a woman has A. Exotoxin action


been suffering from disturbed sleep for B. Enterotoxin action
several days. What soporific drug would be C. Enterotoxic shock development
preferable for this type of insomnia? D. Activation of adenylate cyclase
E. Pathogen adhesion to the enterocyte
A. Nitrazepam receptors
B. Phenobarbital
C. Ethaminal sodium (Pentobarbital) 167. An infant, who was on synthetic
D. Barbamylum (Amobarbital) formula feeding, developed signs of vitamin
E. Chloral hydrate B1 deficiency. What reactions does this vi-
tamin take part in?
162. During training session in the
laboratory the students were performing
spirography on themselves. What indicator
CANNOT be measured with this method?
Krok 1 Medicine (англомовний варiант, iноземнi студенти) 2017 рiк 18

A. Keto acids oxidative decarboxylation A. Glucocorticoids


B. Amino acids transamination B. Adrenaline
C. Amino acids decarboxylation C. Insulin
D. Proline hydroxylation D. Mineralocorticoids
E. Redox reactions E. Vasopressin
168. A 30-year-old woman developed the si- 173. A 30-year-old patient’s blood test
gns of virilism (body hair growth, balding revealed the following: erythrocyte count
temples, disturbed menstrual cycle). What is 6 · 1012 /l, hemoglobin is 10,55 mmol/l.
hormone can cause this condition when Vaquez’s disease was diagnosed. Name the
hyperproduced? leading part of pathogenesis in this case:
A. Testosterone A. Neoplastic erythroid hyperplasia
B. Estriol B. Iron-deficiency
C. Relaxin C. B12 -deficiency
D. Oxytocin D. Hypoxia
E. Prolactin E. Acidosis
169. A histological specimen shows signifi- 174. Deaf parents with genotypes DDee and
cant amount of mucous connective tissue ddEE gave birth to a child with normal
(Wharton’s jelly), vessels, as well as residual hearing. Specify the interaction of D and
yolk and allantois. Name this organ: E genes:
A. Umbilical cord A. Complementary interaction
B. Esophagus B. Complete dominance
C. Ureter C. Epistasis
D. Urethra D. Polymery
E. Vermiform appendix E. Overdominance
170. On examination of a 6-year-old chi- 175. Corticosteroid hormones regulate the
ld the doctor noticed grayish film on adaptation processes of the body as a
the child’s tonsils. Microscopy of the whole to environmental changes and ensure
smears stained by Neisser method detected the maintenance of internal homeostasis.
there Corynebacterium diphtheriae. What What hormone activates the hypothalamo-
morphologic feature was the most indicati- pituitary-adrenal axis?
ve for determining the type of the agent?
A. Corticoliberin
A. Polar placement of volutin granules B. Somatoliberin
B. Localization of the causative agent within C. Somatostatin
macrophages D. Corticostatin
C. Spores that exceed cells in diameter E. Thyroliberin
D. Fence-like position of the agent’s cells
E. Presence of the capsule 176. A patient with signs of emotional labi-
lity that result in troubled sleep has been
171. During the sports competition a boxer prescribed nitrazepam. Specify the sleep-
received a strong blow to the abdomen, inducing mechanism of this drug:
which caused a knockout due to a brief
drop in blood pressure. What physiological A. GABA-ergic system activation
mechanisms are the cause of this condition? B. Blockade of opiate receptors
C. Inhibition of stimulating amino acids
A. Stimulation of parasympathetic nerves D. H1 -histamine receptors stimulation
B. Alteration of transcapillary exchange E. Supression of serotonergic neurotransmi-
C. Ischemia of the central nervous system ssion
D. Abrupt change in body fluid volume
E. Stimulation of sympathetic nerves 177. A 50-year-old inpatient during exami-
nation presents with glucosuria and blood
172. After a severe stress the patient glucose of 3,0 mmol/l, which are the most
presents with eosinopenia in the blood test. likely to be caused by:
In this case the decreased number of eosi-
nophils can explain changes in the level of A. Renal disorder
the following hormones: B. Diabetes insipidus
C. Myxedema
D. Essential hypertension
E. Pellagra
178. A man is suffering from diarrhea. In
Krok 1 Medicine (англомовний варiант, iноземнi студенти) 2017 рiк 19

summer he spent his vacation in the south A. Substantia nigra


at the sea coast. Bacteria with the following B. Globus pallidus
properties were detected in his feces: gram- C. Corpora quadrigemina
negative curved mobile monotrichous baci- D. Red nuclei
lli that do not produce spores or capsules. E. Hypothalamus
They are undemanding to nutrient medium
but require alkaline reaction (рН - 8,5-9,5). 183. Determining a patient’s blood group
Described are the agents of the following with monoclonal test-reagents revealed
enteric infection: positive agglutination reaction to anti-A
and anti-B reagents, and negative reacti-
A. Cholera on to anti-D. What blood group does this
B. Shigellosis patient have?
C. Typhoid fever
D. Colienteritis A. IV (АВ) Rh (-)
E. Pseudotuberculosis B. II (А) Rh (+)
C. III (В) Rh (-)
179. A pregnant woman was detected to D. IV (АВ) Rh (+)
have IgM to rubella virus. An obstetrician- E. I (0) Rh (+)
gynecologist recommended therapeutic
abortion due to the high risk of teratogenic 184. A patient visited a dentist to extract a
affection of the fetus. Detection of IgM was tooth. After the tooth had been extracted,
of great importance as it is these specific bleeding from the tooth socket continued
immunoglobulins that: for 15 minutes. Anamnesis states that the
patient suffers from active chronic hepatitis.
A. Indicate recent infection What phenomenon can extend the time of
B. Penetrate placental barrier hemorrhage?
C. Have the largest molecular weight
D. Are associated with anaphylactic reacti- A. Decrease of fibrinogen content in blood
ons B. Thrombocytopenia
E. Are the main factor of antiviral protection C. Hypocalcemia
D. Increased activity of anticoagulation
180. During examination of a teenager system
with xanthomatosis the family history of E. Decrease of albumine content in blood
hypercholesterolemia is revealed. What
transportable lipids are increased in 185. During ascent into mountains a person
concentration in case of such a disease? develops increased respiration rate and
rapid heart rate. What is the cause of these
A. Low-density lipoproteins changes?
B. Chylomicrons
C. Very low-density lipoproteins A. Decrease of O2 partial pressure
D. High-density lipoproteins B. Increase of CO2 partial pressure
E. Intermediate-density lipoproteins C. Increase of blood pH
D. Increase of nitrogen content in air
181. On examination the patient is found to E. Increase of air humidity
have low production of adrenocorticotropic
hormone. How would this affect production 186. To stop the bleeding the patient
of the other hormones? was prescribed a direct coagulant. Duri-
ng introduction of the solution the patient
A. Decrease adrenocorticotropic hormones was complaining of pain along the vein, hot
synthesis sensation, and palpitations. Name the drug
B. Decrease hormone synthesis in the that causes such symptoms:
adrenal medulla
C. Decrease insulin synthesis A. Calcium chloride
D. Increase sex hormones synthesis B. Hirudine
E. Increase thyroid hormones synthesis C. Ergocalciferol
D. Pentoxyl
182. Parkinson’s disease is caused by E. Streptokinase
disturbance of dopamine synthesis.
What brain structure synthesizes this 187. A student, whose educational achi-
neurotransmitter? evements throughout the semester were
poor, feels emotionally tense during the
final test. What is the primary cause that
induced leading mechanism of emotional
tension in this case?
Krok 1 Medicine (англомовний варiант, iноземнi студенти) 2017 рiк 20

A. Lack of information A. Rifampicin


B. Tight time B. Tetracycline
C. Ttight time and lack of energy C. Streptocide
D. Lack of energy D. Furasolidone
E. Lack of energy and information E. Bactrim (Co-trimoxazole)
188. A young family came for a genetic 193. During surgery performed in the
counseling to identify the father of their chi- abdominal cavity a surgeon located li-
ld. The husband insists that the child does gament of liver stretching from anterior
not resemble him at all and cannot possibly abdominal wall (navel) to inferior surface
be his. Polymerase chain reaction method of liver. What ligament is it?
for person identification is based on the
following: A. Round ligament of the liver
B. Falciform ligament of the liver
A. Gene amplification C. Coronary ligament of the liver
B. Nucleotide deletion D. Venous ligament of the liver
C. Genetic recombination E. Triangular ligament of the liver
D. Missense mutation
E. Transduction 194. On examination of a patient with di-
sease onset 5 days ago the doctor suspected
189. A 52-year-old man presents with fever tularemia and prescribed the patient tularin
and pain in the joints. Both of his fi- intracutaneously. What is the purpose of this
rst metatarsophalangeal articulations are drug administration in the patient?
deformed, swollen, and reddened. Blood
urea is high. The patient is diagnosed wi- A. Allergy diagnostics
th gout. What is the main developmental B. Prevention
factor in the pathogenesis of this disease? C. Treatment
D. Treatment evaluation
A. Hyperuricemy E. Prognosis for the disease
B. Argininosuccinic aciduria
C. Hyperazotemia 195. A patient developed increased blood
D. Hyperaminoacidemia content of HCO3− against the background
E. Citrullinuria of repeated and uncontrollable vomiti-
ng. What will be the leading mechanism
190. A 67-year-old man was delivered to the in compensation of developed acid-base
cardiology unit with complaints of periodi- imbalance?
cal pain in the heart, dyspnea after even
insignificant physical exertion, cyanosis, and A. Decreased pulmonary ventilation
edemas. ECG revealed additional contracti- B. Increased pulmonary ventilation
ons of the heart ventricles. Name this type C. Increased renal reabsorption of bi-
of rhythm disturbance: carbonate
D. Increased renal reabsorption of ammonia
A. Extrasystole E. -
B. Bradycardia
C. Tachycardia 196. A 13-year-old boy presents wi-
D. Flutter th eczematous rashes on his shins and
E. Fibrillation torso. Anamnesis states cases of otitis,
pneumonia, and furuncles in the patient.
191. During narcosis the patient developed Blood test: platelets - 70 · 109 /l, low acti-
a risk of cerebral edema. What drug should vity of T helper and T suppressor cells,
be administered in this case? low IgM, with normal IgA and IgG. What
A. Furosemide immunodeficient disease does this boy
B. Dopamine have?
C. Phenazepam A. Wiskott-Aldrich syndrome
D. Triamterene
E. Sodium bromide B. Louis-Bar syndrome (Ataxia-
telangiectasia)
192. A patient with pulmonary tuberculosis C. Severe combined immunodeficiency
is prescribed the most effective anti- (Swiss type)
tuberculous antibiotic. Name this drug: D. DiGeorge syndrome
E. Chediak-Higashi syndrome
197. During the exam a student was unable
to correctly answer all the questions in his
question card, which was accompanied by
Krok 1 Medicine (англомовний варiант, iноземнi студенти) 2017 рiк 21

the reddening of his face and hot sensati- 199. Due to trauma the patient presents wi-
on. What type of arterial hyperemia did the th disturbed function of the parotid gland.
student develop in this case? What nerve ensures its secretion function?
A. Neurotonic A. N. petrosus minor
B. Metabolic B. N. petrosus major
C. Postischemic C. N. petrosus profundus
D. Pathologic D. N. auricularis minor
E. Neuroparalytic E. N. auricularis major
198. An injured person with wound of 200. A patient developed pyoinflammatory
the anterior cervical region presents with process of periodontal tissues caused by
hemorrhage. The outflowing blood is dark. activation of the microorganisms inherent
What vessel is damaged? in the body, which are a part of oral mucosal
microflora. What type of infection is it?
A. V. jugularis anterior
B. V. jugularis externa A. Autoinfection
C. V. jugularis interna B. Exogenous infection
D. A. carotis externa C. Reinfection
E. A. thyroidea superior D. Superinfection
E. Relapse
INSTRUCTIONAL BOOK
Testing Board

TEST ITEMS FOR LICENSING EXAMINATION: KROK 1. MEDICINE.

Kyiv. Testing Board.


(English language).

Approved to print 04.05/№256 Paper size 60х84 1/8


Offset paper. Typeface. Times New Roman Cyr. Offset print.
Conditional print pages 20. Accounting publishing pages 24.
Issue. 2673 copies.
List of abbreviations

A/G Albumin/globulin ratio HR Heart rate


A-ANON Alcoholics anonymous IDDM Insulin dependent diabetes mellitus
ACT Abdominal computed tomography IFA Immunofluorescence assay
ADP Adenosine diphosphate IHD Ischemic heart disease
ALT Alanin aminotranspherase IU International unit
AMP Adenosine monophosphate LDH Lactate dehydrogenase
AP Action potential MSEC Medical and sanitary expert committee
ARF Acute renal failure NAD Nicotine amide adenine dinucleotide
AST Aspartat aminotranspherase NADPH Nicotine amide adenine dinucleotide
ATP Adenosine triphosphate phosphate restored
BP Blood pressure NIDDM Non-Insulin dependent diabetes mellitus
bpm Beats per minute PAC Polyunsaturated aromatic carbohydrates
C.I. Color Index PAS Periodic acid & shiff reaction
CBC Complete blood count pCO2 CO2 partial pressure
CHF Chronic heart failure pO2 CO2 partial pressure
CT Computer tomography pm Per minute
DIC Disseminated intravascular coagualtion Ps Pulse rate
DCC Doctoral controlling committee r roentgen
DM-2 Non-Insulin dependent diabetes mellitus RBC Red blood count
DTP Anti diphtheria-tetanus vaccine RDHA Reverse direct hemagglutination assay
ECG Electrocardiogram Rh Rhesus
ESR Erythrocyte sedimentation rate (R)CFT Reiter's complement fixation test
FC Function class RIHA Reverse indirect hemagglutination assay
FAD Flavin adenine dinucleotide RNA Ribonucleic acid
FADH2 Flavin adenine dinucleotide restored RR Respiratory rate
FEGDS Fibro-esphago-gastro-duodenoscopy S1 Heart sound 1
FMNH2 Flavin mononucleotide restored S2 Heart sound 2
GIT Gastrointestinal tract TU Tuberculin unit
GMP Guanosine monophosphate U Unit
Hb Hemoglobin USI Ultrasound investigation
HbA1c Glycosylated hemoglobin V/f Vision field
Hct Hematocrit WBC White blood count
HIV Human immunodeficiency virus X-ray Roentgenogram
MINISTRY OF PUBLIC HEALTH OF UKRAINE

Department of human resources policy, education and science

Testing Board

Student ID Last name

Variant ___________________

Test items for licensing examination

Krok 1
MEDICINE
General Instruction
Every one of these numbered questions or unfinished statements in
this chapter corresponds to answers or statements endings. Choose the
answer (finished statements) that fits best and fill in the circle with the
corresponding Latin letter on the answer sheet.
54.1 73
61

Authors of items: Andrieieva V.F., Andriishyn O.P., Arnautova L.V., Basiy R.V., Batyshcheva N.Yu.,
Bevziuk D.O., Bielan S.M., Bielienichev I.F., Bilets M.V., Blinder O.O., Bodnar Ya.Ya., Boiko O.V.,
Bondarenko Yu.I., Bordiakivska L.H., Chehodar A.Ya., Cherkashyna L.P., Cherkashyna O.Ye.,
Chernenko H.P., Chernobai L.V., Chernovska N.V., Chystolinova L.I., Danylchuk V.V., Datsko T.V.,
Davydenko I.S., Deineka S.Ye., Demianenko I.A., Diachyshyna L.V., Diadyk O.O., Dieltsova O.I.,
Dutchak U.M., Dzevulska I.V., Fartushok N.V., Hanziy T.V., Harets V.I., Herasymovych E.V., Herush I.V.,
Hetsko O.I., Holieva N.V., Holota L.H., Holotiuk L.Ye., Holovatiuk O.L., Honcharova N.H.,
Horianova N.O., Hryhorenko V.K., Hrytsenko L.Z., Hrytsyna I.V., Hubina-Vakulyk H.I., Hurmak I.S.,
Husakova L.V., Ivanova N.I., Karvatsky I.M., Khara M.R., Khodorovsky H.I., Komissarov I.V.,
Korolenko H.S., Kratinova M.A., Kryshtal M.V., Kryvetsky V.V., Kucheriavchenko M.O., Kukovska I.L.,
Kukurychkin Ye.R., Kulish A.S., Kulitka E.F., Ladutko S.V., Lemke M.O., Linchevska L.P., Loitra A.O.,
Lozynska R.V., Lutsiuk M.B., Luzin V.I., Lyzyn D.V., Maksymchuk T.P., Minukhin V.V.,
Mishchenko A.V., Mishchenko K.M., Mozghunov O.V., Mruh V.M., Nakonechna O.A., Navnyko T.S.,
Nikolaienko I.V., Nikolenko O.H., Nykytiuk H.P., Oblyvach A.V., Obraztsova O.H., Oleshchuk O.M.,
Oliynyk I.Yu., Opryshko V.I., Ovchynnikov S.O., Pasholok S.P., Pelykh V.Ye., Piddubniak Yu.H.,
Piskunova N.V., Prokofieva N.V., Rodynsky O.H., Romanenko O.V., Rukavyshnikova S.M., Saiuk N.P.,
Salata O.V., Samoilovych I.M., Savchenko N.V., Selsky P.R., Semeniuk T.M., Senchiy V.M.,
Serdiuchenko I.Ya., Servetnyk M.I., Shanko V.M., Shcherbak L.F., Shcherbakov S.M., Shenderiuk O.P.,
Shevtsova A.I., Skorobohatova Z.M., Skrebkova O.Yu., Smirnova T.V., Snehir A.H., Stebliuk M.V.,
Stetsenko S.O., Stklianina L.V., Svirsky O.O., Sydorchuk I.Y., Symachova A.V., Sytnikova V.O.,
Taldykin P.Ye., Tarasenko L.M., Tertyshna O.V., Tertyshny S.I., Tkachenko V.P., Tumansky V.O.,
Tusyk O.T., Tymoshenko S.H., Vasiuta V.S., Vasylchenko V.M., Vasylenko I.V., Vasylieva A.H.,
Vasylyk V.M., Vernyhor O.O., Voitsekhovsky V.H., Voloshyn M.A., Vorobets Z.D., Vysotsky I.Yu.,
Yashchenko A.M., Yasinsky V.I., Yastremska S.O., Yeshchenko V.Yu., Yoltukhivsky M.M.,
Yushkova V.V., Zakharchuk O.I., Zdykhovsky I.O., Zelenina N.M., Zhadinsky M.V., Zhulinsky V.O.,
Ziatkovska N.Ye. and Committees of professional expertise.

Item reviewers. Ananko S.Ya., Basy R.V., Bilash S.M., Deineka S. Ye., Deltsova O.I.,
Demydova K.Yu., Fomina L.V., Hahrin V.V., Haidash I.S., Holovatiuk O.L., Horhol N.I., Kava T.V.,
Koldunov V.V., Korolenko H.S., Kovalchuk L.Ye., Linchevska L.P., Maly K.D., Melnikova O.V.,
Melnyk N.O., Neporada K.S., Nikolenko O.H., Ovchynnikov S.O., Prokofieva N.V., Pushkar M.S.,
Pykaliuk V.S., Salata O.V., Shevelenkova A.V., Shumeiko O.V., Sikora V.Z., Skliarov O.Ya.,
Skorobohatova Z.M., Synytska A.M., Tananakina T.P., Teleshova O.V., Tertyshny S.I., Tkachuk S.S.,
Trzhetsynsky S.D, Vinnykov Yu.M., Vorobets Z.D., Yoltukhivsky M.V., Zelenina N.M., Zhadinsky M.V.,
Zhylinsky V.O., Zinkovska L.Ya.

The b k i cl de e i em f e a lice i g i eg a ed e ami a i K k 1. Medici e a d


further use in teaching.

The book has been developed for students of medical, pediatric and medical-and-prophylactic
faculties and academic staff of higher medical educational establishments.

Approved by Ministry of Public Health of Ukraine as examination and teaching


publication based on expert conclusions (Orders of MPH of Ukraine of 14.08.1998 №251,
of 27.12.1999 №303, of 18.06.2002 №221, of 16.10.2002 №374, of 16.04.2003 №239, of
29.05.2003 №233).

C igh Te i g B a d.
Krok 1 Medicine (англомовний варiант, iноземнi студенти) 2018 рiк 1

1. A 40-year-old woman on examinati- onal diseases hospital on the 8th day since
on presents with intensified basal metabolic the disease onset. The patient complains of
rate. What hormone present in excess leads headache, malaise, and weakness. A sample
to such condition? of blood was taken for the serological test.
Widal agglutination test results with blood
A. Triiodothyronine sample diluted 1:200 and typhoid fever O-
B. Thyrocalcitonin diagnosticum were positive. What diagnosis
C. Glucagon can be made based on the results of this
D. Aldosterone test?
E. Somatostatin
A. Typhoid fever
2. Autopsy of a woman revealed the followi- B. Dysentery
ng morphologic changes: stenosis of the C. Cholera
atrioventricular opening, mitral insuffici- D. Leptospirosis
ency. Histologically there are focal cardi- E. Tuberculosis
osclerosis and ”blooming” Aschoff nodules
in the myocardium. What is the most likely 7. Protective function of saliva is based
diagnosis? on several mechanisms, including the
presence of enzyme that has bacterici-
A. Rheumatism dal action and causes lysis of complex
B. Scleroderma capsular polysaccharides of staphylococci
C. Dermatomyositis and streptococci. Name this enzyme:
D. Polyarteritis nodosa
E. Systemic lupus erythematosus A. Lysozyme
B. α-amylase
3. After a case of sepsis a 27-year-old C. Oligo-1,6-glucosidase
woman developed ”bronzed” skin di- D. Collagenase
scoloration characteristic of Addison’s di- E. β -glucuronidase
sease. Hyperpigmentation mechanism in
this case is based on increased secretion of: 8. In the process of hemoglobin catabolism
iron is released and then as a part of speci-
A. Melanocyte-stimulating hormone al transport protein is returned to the bone
B. Somatotropin marrow, to be used again for hemoglobin
C. Gonadotropin synthesis. Name this transport protein:
D. β -lipotropin
E. Thyroid-stimulating hormone A. Transferrin
B. Transcobalamin
4. A 16-year-old girl presents with no hair on C. Haptoglobin
the pubis and in the armpits, her mammary D. Ceruloplasmin
glands are underdeveloped, no menstruati- E. Albumin
ons. What hormone imbalance can it be
indicative of? 9. The first-aid center has received a vi-
ctim of a traffic accident diagnosed with
A. Ovarian failure closed displaced fracture of the middle thi-
B. Hyperthyroidism rd of the thigh. For repositioning of bone
C. Hypothyroidism fragments the patient received 10 ml of
D. Pancreatic islet failure 2% dithylinum solution intravenously, whi-
E. Adrenal medulla hyperfunction ch resulted in prolonged period of apnoea
and muscle relaxation. What enzyme is defi-
5. From the feces of a patient with cient, resulting in such pharmacogenetic
acute gastroenteritis a pure culture of enzymopathy?
microorganisms was obtained. The mi-
croorganisms are small mobile slightly A. Pseudocholinesterase
curved gram-negative bacilli that within 6 B. Uridine diphosphate
hours grow into a light blue film on the 1% glucuronyltransferase
alkaline peptone water. Such properties are C. Glucose 6-phosphate dehydrogenase
characteristic of the following microorgani- D. Methemoglobin reductase
sm: E. N-acetyltransferase
A. Vibrio 10. An 18-year-old student presents wi-
B. Spirochaete th enlarged thyroid gland accompanied by
C. Clostridium accelerated metabolism and increased heart
D. Bacillus rate. These signs can be observed during
E. Spirilla hypersecretion of thyroxin. What organelles
of thyroid cells are primarily responsible for
6. A patient was brought into the infecti- hormone production and secretion?
Krok 1 Medicine (англомовний варiант, iноземнi студенти) 2018 рiк 2

A. Golgi apparatus A. Ramus dexter a. hepatica propria


B. Mitochondria B. А. hepatica communis
C. Ribosomes C. А. gasro-duodenalis
D. Centrosomes D. Ramus sinister a. hepatica propria
E. Lysosomes E. А. pancreato-duodenalis sup
11. A patient complains of pain in the upper 16. Microscopy of the puncture sample
umbilical region. On palpation there is a obtained from the inflammation focus of
mobile painful intestine. What intestine is the patient with cutaneous abscess revealed
being palpated by the doctor? numerous blood cells of different types.
What cells are the first to transfer from
A. Transverse colon vessels to tissues during inflammation?
B. Jejunum
C. Duodenum A. Neutrophils
D. Ileum B. Monocytes
E. Sigmoid colon C. Basocytes
D. Eosinophils
12. A laboratory experiment on a dog E. Lymphocytes
was used to study central parts of audi-
tory system. One of the mesencephalon 17. During the first year of life an infant
structures was destroyed. The dog has lost presents with disturbed process of breast
the orienting response to auditory signals. milk curdling. What cells of the proper
What structure was destroyed? gastric glands are functionally disturbed?
A. Inferior colliculi of corpora quadrigemina A. Main exocrinocytes
B. Superior colliculi of corpora quadrigemi- B. Parietal exocrinocytes
na C. Cervical mucous cells
C. Substantia nigra D. Accessory mucous cells
D. Reticular formation nuclei E. Exocrinocytes
E. Red nucleus
18. A lab rat has subcutaneously received
13. Histological investigation of the uterine mercury(II) chloride in the amount of 5
scrape of the 45-year-old woman with di- mg/kg. 24 hours later the plasma creatini-
sturbed ovarian menstrual cycle revealed ne concentration increased several times.
increased number of endometrial glands, What mechanism of retention azotemia is
some of which are serrated, while others are observed in this case?
dilated and cyst-like. Make the diagnosis:
A. Decreased glomerular filtration
A. Endometrial cystic glandular hyperplasia B. Increased creatinine production in the
B. Placental polyp muscles
C. Atypical endometrial hyperplasia C. Increased creatinine reabsorption
D. Glandular endometrial polyp D. Increased glomerular filtration
E. Endometrial adenocarcinoma E. Increased creatinine production in the
renal tubules
14. A 45-year-old man diagnosed with
hepatic cirrhosis and ascites underwent 19. Collagenosis patients typically present
drainage of 5 liters of fluid from his abdomi- with connective tissue destruction
nal cavity, which resulted in development processes. The presence of these processes
of syncopal state due to insufficient blood can be confirmed by the increase in:
supply to the brain. What circulatory di-
sorder occurred in the abdominal cavity in A. Blood oxyproline and oxylysine
this case? B. Blood creatine and creatinine
C. LDH-isoenzyme activity in the blood
A. Arterial hyperemia D. Transaminase activity in the blood
B. Ischemia E. Blood urates
C. Venous hyperemia
D. Thrombosis 20. A 63-year-old man suffers from
E. Embolism esophageal carcinoma, presents with
metastases into the mediastinal lymph
15. During cholecystectomy besides a. nodes and cancerous cachexia. What
cystyca another artery was pulled into the pathogenetic stage of neoplastic process is
ligature. Ligation of this artery resulted in observed in the patient?
right-sided necrosis of the liver which led to
the death of the patient. What artery was
mistakenly ligated along with a. cystyca?
Krok 1 Medicine (англомовний варiант, iноземнi студенти) 2018 рiк 3

A. Progression A. Interstitial fluid inflow to the vessels


B. Promotion B. Restoration of blood protein composition
C. Transformation C. Increase of reticulocyte number
D. Initiation D. Restoration of erythrocyte number
E. - E. Erythropoiesis activation
21. After an X-ray examination of the 26. ECG analysis of the patient shows that
tuberculosis clinic patient, he was diagnosed the T waves are positive in the second
with tumor of the right lung. During operati- standard limb lead and their amplitude
on the surgeon removed the middle lobe of and duration is normal. The conclusion can
the patient’s right lung. This lobe includes: be made that the following process occurs
normally in the patient’s ventricles:
A. Segmentum laterale et segmentum mediale
B. Segmentum basale anterius et posterius A. Repolarization
C. Segmentum anterius et segmentum apicale B. Depolarization
D. Segmentum lingualare superius et inferius C. Excitation
E. Segmentum apicale (superius) et D. Contraction
segmentum basale mediale E. Relaxation
22. A 2-year-old child presents with physical 27. A patient demonstrates sharp decrease
retardation and frequent pneumonias. The of pulmonary surfactant activity. This condi-
child was diagnosed with non-closure of the tion can result in:
arterial canal. Hemodynamics disturbance
in this case is caused by communication A. Alveolar tendency to recede
between the: B. Decreased airways resistance
C. Decreased work of expiratory muscles
A. Aorta and pulmonary trunk D. Increased pulmonary ventilation
B. Pulmonary trunk and pulmonary veins E. Hyperoxemia
C. Superior vena cava and aorta
D. Superior vena cava and pulmonary trunk 28. After a case of common cold the patient
E. Aorta and pulmonary veins developed numbness of the right side of the
face. Examination revealed disturbed pain
23. A child diagnosed with purulent and thermal sensitivity in the right half of
inflammation of the middle ear was brought the face. What nerve was damaged?
to the otolaryngology department. The di-
sease started with the inflammation of the A. Trigeminal
nasopharynx. It was determined that the B. Facial
infection had reached the tympanic cavity C. Glossopharyngeal
through the eustachian tube that is located D. Vagus
in the: E. Hypoglossal

A. Canalis musculotubarius 29. X-ray exmination of a 57-year-old man


B. Сanaliculus tympanicus indicates local areas of hard bone tissue
C. Сanalis caroticus resorption in some of the patient’s bones.
D. Сanaliculus chordae tympani These changes can be associated with
E. Сanaliculi caroticо tympanici increased activity of:

24. During the prestart period an athlete A. Osteoclasts


develops increased frequency and force B. Chondroblasts
of cardiac contractions. These changes are C. Osteocytes
caused by intensification of the following D. Osteoblasts
reflex responces: E. Chondrocytes

A. Sympathetic conditioned 30. A patient presents with acute attack of


B. Sympathetic unconditioned cholelithiasis. Laboratory examination of
C. Parasympathetic conditioned the patient’s feces will show the following in
D. Parasympathetic unconditioned this case:
E. Peripheral
A. Negative reaction to stercobilin
25. Due to trauma the patient has lost B. Positive reaction to stercobilin
25% of circulating blood volume. Name the C. Connective tissue
emergency compensatory mechanism agai- D. Partially digested cellulose
nst blood loss: E. Starch granules
31. During autopsy of a 34-year-old man,
who died of chronic kidney failure due
Krok 1 Medicine (англомовний варiант, iноземнi студенти) 2018 рiк 4

to renal amyloidosis, in the lungs (mai- A. Hemorrhagic infarction


nly in the lower lobes) the pathologist B. Carneous degeneration
detected multiple bronchial dilations filled C. Gangrene
with purulent masses accumulated in the D. Hemorrhage
bronchial lumen. Surface of the lungs secti- E. Atelectasis
on has fine-meshed pattern and resembles a
honeycomb. Histologically there is a chronic 36. A patient complains of dizziness, thirst,
inflammation detected in the bronchial wall, difficult swallowing, and impaired vision of
muscle fibers are replaced with connective close objects. Objectively: respiratory rate
tissue. These changes in the lungs can be is increased, pupils are dilated, general agi-
defined as: tation, talkativeness, though the speech is
indistinct. BP is 110/70 mm Hg, heart rate
A. Bronchiectases is 110/min. Given symptoms can indicate
B. Bronchopneumonia overdosage of the following drug:
C. Chronic bronchitis
D. Chronic pneumonia A. Atropine
E. Lung abscesses B. Morphine
C. Ephedrine
32. Initial inoculation of water in 1% D. Aminazine
peptone water resulted in growth of a thin E. Caffeine
film on the medium surface in 6 hours.
Such cultural properties are characteristic 37. A patient presents with indigestion,
of causative agent of the following disease: stomachaches, and excessive salivation. Si-
milar simptoms had already been observed
A. Cholera in this patient previously. Laboratory
B. Plague analysis detected oval eggs covered with
C. Tuberculosis lumpy capsules in the patient’s feces. What
D. Dysentery is the most likely cause of the patient’s di-
E. Pseudotuberculosis sorder?

33. A 30-year-old man complains of A. Ascariasis


suffocation, heaviness in the chest on the B. Trichocephaliasis
right, general weakness. Body temperature C. Diphyllobothriasis
is 38.9o C . Objectively the right side of D. Enterobiasis
the chest lags behind the left side during E. Fascioliasis
respiration. Pleurocentesis yielded exudate.
What is the leading factor of exudation in 38. Regional lymph nodes surrounding an
the patient? infected wound are enlarged. Histological
examination shows increased number of
A. Increased permeability of the vessel wall macrophages, lymphocytes, and lymphatic
B. Increased blood pressure follicles, as well as a large amount of plasma
C. Hypoproteinemia cells, in the cortical layer of the lymph
D. Erythrocyte aggregation nodes. What process in the lymph nodes
E. Decreased resorption of pleural fluid is indicated by these histologic changes?

34. A 42-year-old man with gout presents A. Antigen stimulation


with high content of uric acid in blood. The B. Acquired deficiency of lymphoid tissue
patient was prescribed allopurinol to lower C. Congenital deficiency of lymphoid tissue
the concentration of uric acid. Allopurinol D. Neoplastic aberration
is a competitive inhibitor of the following E. Transplant rejection
enzyme:
39. During removal of the hyperplastic
A. Xanthine oxidase thyroid gland of a 47-year-old woman,
B. Adenosine deaminase the parathyroid gland was damaged.
C. Adenine phosphoribosyltransferase One month after the surgery the pati-
D. Hypoxanthine phosphoribosyltransferase ent developed signs of hypoparathyroi-
E. Guanine deaminase dism: frequent convulsions, hyperreflexia,
laryngospasm. What is the most likely cause
35. Autopsy revealed a large wedge-shaped of the patient’s condition?
patch of a dense dark red tissue with clear
margins in the upper lobe of the right lung. A. Hypocalcemia
Histological examination detected there B. Hyponatremia
necrosis of the alveolar walls; the alveolar C. Hyperchlorhydria
lumen is tightly packed with erythrocytes. D. Hypophosphatemia
What process occurred in the lungs? E. Hyperkalemia
Krok 1 Medicine (англомовний варiант, iноземнi студенти) 2018 рiк 5

40. On examination the patient presents


with hirsutism, moon-shaped face, stretch A. IgM antibodies to Epstein-Barr virus
marks on the abdomen. BP is 190/100 were detected
mm Hg, blood glucose is 17.6 mmol/L. B. IgM antibodies to herpes simplex virus
What pathology is such clinical presentation were detected
characteristic of? C. Fourfold increase in number of antibodies
to Epstein-Barr virus was detected
A. Adrenocortical hyperfunction D. Herpesvirus was isolated
B. Hyperthyroidism E. Cytomegalovirus antibodies were
C. Hypothyroidism detected
D. Gonadal hypofunction
E. Hyperfunction of the insular apparatus 46. A 64-year-old woman presents with di-
sturbed fine motor function of her fingers,
41. A 45-year-old woman presents with marked muscle rigidity, and tremor. The
breast cancer. Metastases can spread in this neurologist diagnosed her with Parkinson’s
case to the following regional lymph nodes: disease. What brain structures are damaged
resulting in this disease?
A. Axillary, parasternal
B. Abdominal, cervical A. Substantia nigra
C. Cervical, parasternal B. Thalamus
D. Parasternal, mediastinal C. Red nuclei
E. Aortic, mediastinal D. Cerebellum
E. Reticular formation
42. A 40-year-old pregnant woman
underwent amniocentesis. Examination 47. A 20-year-old young man with tall
determined the fetal karyotype to be 47, stature, asthenic body type, signs of
XY+21. What fetal pathology was detected? hypogonadism and gynecomastia, and low
sperm count (azoospermia) has karyotype
A. Down syndrome 47, XXY. What hereditary syndrome can
B. Klinefelter syndrome be characterized by this chromosomal
C. Turner syndrome anomaly?
D. Phenylketonuria
E. Patau syndrome A. Klinefelter syndrome
B. Wiskott-Aldrich syndrome
43. A patient used an indirect-acting C. Turner syndrome
adrenergic agonist to treat rhinitis. After the D. Louis-Bar syndrome (ataxia-
patient has been putting in the nose drops telangiectasia)
for several days, the vasoconstrictive effect E. Down syndrome
of the drug gradually diminished. Name this
phenomenon: 48. Pathogenic staphylococcus was obtained
from the purulent wound of the patient. Its
A. Tachyphylaxis antibiotic sensitivity was determined to be
B. Idiosyncrasy as follows: penicillin growth inhibition zone
C. Teratogenicity - 8 mm; oxacillin - 9 mm, ampicillin - 10
D. Allergy mm, gentamicin - 22 mm, lincomycin - 11
E. Cumulation mm. What antibiotic should be chosen for
44. In a township there was registered an treatment in this case?
outbreak of hepatitis, which had supposedly A. Gentamicin
spread through the water supply. What B. Oxacillin
hepatitis virus could be the cause of the C. Ampicillin
outbreak in this township? D. Penicillin
A. Hepatitis E virus E. Lincomycin
B. Hepatitis C virus 49. A patient presents with dilated blood
C. Hepatitis D virus vessels of the anterior medial surface of the
D. Hepatitis G virus lower leg. This condition resulted from the
E. Hepatitis B virus dilation of the following blood vessel:
45. A medical student was hospitalized A. V. saphena magna
into the infectious diseases unit on the 2nd B. А. tibialis anterior
day after the disease onset; the patient is C. V. saphena parva
suspected to have infectious mononucleosis. D. А. tibialis posterior
What results of laboratory analysis can E. V. poplitea
confirm this diagnosis immediately on the
day of the hospitalization? 50. A patient, who had received a thermal
Krok 1 Medicine (англомовний варiант, iноземнi студенти) 2018 рiк 6

burn, developed painful boils filled with A. Iris sphincter muscle, ciliary muscle
turbid liquid on the skin. What morphologi- B. Iris dilator muscle, ciliary muscle
cal type of inflammation has developed in C. Superior oblique muscle, ciliary muscle
the patient? D. Lateral rectus muscle, iris sphincter
muscle
A. Serous E. Iris sphincter and iris dilator muscles
B. Proliferative
C. Croupous 56. A patient with hypochromic anemia has
D. Granulomatous hair with split ends and suffers from hair
E. Diphtheritic loss. The nails are brittle. Gustatory sensati-
ons are affected. What is the mechanism of
51. Nitrogen is being excreted from the development of these symptoms?
body mainly as urea. When activity of a
certain enzyme in the liver is low, it results A. Iron enzymes deficiency
in inhibition of urea synthesis and nitrogen B. Vitamin B12 deficiency
accumulation in blood and tissues. Name C. Low production of parathyroid hormone
this enzyme: D. Vitamin A deficiency
E. Low production of thyroid hormones
A. Carbamoyl phosphate synthetase
B. Aspartate aminotransferase 57. A patient suffers from hepatic cirrhosis.
C. Urease What substance excreted in urine should
D. Amylase be analyzed to characterize the antitoxic
E. Pepsin function of liver?
52. Coronary artery thrombosis resulted A. Hippuric acid
in development of myocardial infarction. B. Ammonium salts
What mechanisms of cell damage are leadi- C. Creatinine
ng in this disease? D. Uric acid
E. Amino acids
A. Calcium
B. Lipid 58. A 65-year-old woman, who had been
C. Acidotic suffering from deep vein thrombophlebi-
D. Electroosmotic tis of the lower leg, suddenly died when
E. Protein awaiting her appointment with the doctor.
Autopsy revealed loose friable red masses
53. People, who for a long time remained in with corrugated dull surface in the main
hypodynamic state, develop intense pain in pulmonary artery and its bifurcation. What
the muscles after a physical exertion. What pathologic process was discovered by the
is the most likely cause of this pain? pathologist in the pulmonary artery?
A. Accumulation of lactic acid in muscles A. Thromboembolism
B. Intensive breakdown of muscle proteins B. Thrombosis
C. Accumulation of creatinine in muscles C. Tissue embolism
D. Decreased content of lipids in muscles D. Foreign body embolism
E. Increased content of ADP in muscles E. Fat embolism
54. Histological specimen shows organ 59. Blood of the patients with diabetes
parenchyma to consist of lymphoid tissue mellitus shows increased content of free
that forms lymph nodules; the nodules fatty acids. Name the most likely cause of
are located diffusely and have a central this:
artery. What anatomical structure has such
morphological characteristics? A. Increased activity of adipose triglyceride
lipase
A. Spleen B. Accumulation of palmitoyl-CoA in
B. Tonsil cytosol
C. Lymph node C. Activation of ketone bodies utilization
D. Thymus D. Activation of apoА1, apoА2, and apoА4
E. Red bone marrow apolipoprotein synthesis
E. Decreased activity of plasma
55. A 25-year-old woman complains of vi- phosphatidylcholine-cholesterol-
sual impairment. Examination revealed di- acyltransferase
sturbed eye accommodation, the pupil is
dilated and unresponsive to light. What 60. Representatives of a certain human
muscles are functionally disturbed in this population can be characterized by
case? elongated body, height variability, decreased
volume of muscle mass, increased length
Krok 1 Medicine (англомовний варiант, iноземнi студенти) 2018 рiк 7

of limbs, decreased size and volume of rib ng state?


cage, increased perspiration, decreased indi-
ces of base metabolism and fat synthesis. A. Respiratory volume
What type of adaptive evolution is it? B. Vital lung capacity
C. Inspiratory reserve volume
A. Tropical D. Expiratory reserve volume
B. Arctic E. Total lung capacity
C. Moderate
D. Intermediate 66. The carotid bodies on both sides were
E. Mountain removed in a test animal. Which of the li-
sted factors WILL NOT be able to cause
61. A 56-year-old man complains of thirst hyperventilation in the test animal?
and frequent urination. The endocrinologist
diagnosed this patient with diabetes melli- A. Hypoxemia
tus and prescribed him glibenclamide. What B. Physical exertion
mechanism of action does this drug have? C. Hypercapnia
D. Acidosis
A. Stimulation of β -cells of islets of E. Increase of core body temperature
Langerhans
B. Facilitates glucose uptake by the tissues 67. Due to prolonged stay in the mountai-
C. Facilitates glucose transport through cell ns at the altitude of 3000 m above the sea
membranes level, a person developed increased oxygen
D. Suppression of α-cells of islets of capacity of blood, which was directly caused
Langerhans by intensified production of:
E. Inhibits glucose absorption in the intestine
A. Erythropoietins
62. Patients with bile duct obstruction typi- B. Leukopoietins
cally present with inhibited blood clotting C. Carbaminohemoglobin
and develop hemorrhages due to insuffici- D. Catecholamines
ent assimilation of vitamin: E. 2,3-bisphosphoglycerate

A. K 68. A woman has been limiting the amount


B. A of products in her diet to lose some weight. 3
C. D months later she developed edemas and her
D. E diuresis increased. What dietary component
E. C deficiency is the cause of this?

63. A test animal receives electrical A. Proteins


impulses that irritate the sympathetic nerve B. Fats
that innervates blood vessels of the skin. C. Carbohydrates
What reaction will it cause in the blood D. Vitamins
vessels? E. Minerals

A. Arterial and venous constriction 69. A microslide of the lung tissue sample
B. No reaction taken from a patient with pneumonia shows
C. Arterial dilation damage to the cells that carry out respi-
D. Arterial and venous dilation ratory function. What cells of the alveolar
E. Venous dilation wall are damaged?

64. Examination of the coronary arteries A. Type 2 alveolar cells


revealed atherosclerotic plaques with calci- B. Type 1 alveolar cells
nosis that narrow the arterial opening by C. Macrophages
1/3. In the muscle there are numerous whi- D. Club cells
tish layers of connective tissue. Name the E. Lymphocytes
process detected in the myocardium:
70. After a psychic trauma a woman
A. Diffuse cardiosclerosis developed periodical increases in her
B. Tiger heart blood pressure accompanied by headache,
C. Postinfarction cardiosclerosis palpitations, and general weakness. What
D. Myocarditis mechanism of hypertension development
E. Myocardial infarction does this woman have?

65. A person has increased pulmonary


ventilation due to physical exertion. What
indicator of external respiration will be si-
gnificantly increased compared to the resti-
Krok 1 Medicine (англомовний варiант, iноземнi студенти) 2018 рiк 8

A. Increased arteriolar tone A. Alterative


B. Increased circulating blood volume B. Diffuse exudative
C. Decreased cardiac output C. Focal exudative
D. Tachycardia D. Interstitial
E. Venoconstriction E. Granulomatous
71. An electron micrograph of a nephron 76. A schizophrenia patient was prescribed
segment shows cuboidal cells with cili- aminazine. What pharmacodynamic action
ated lining on their apical surfaces; their of this drug is the grounds for its prescripti-
basal surfaces have basal striation with mi- on in this case?
tochondria located between the cytolemma
invaginations. Name the described nephron A. Antipsychotic
segment: B. Antiemetic
C. Hypothermic
A. Proximal tubule D. Muscle relaxant
B. Collecting ducts E. Hypotensive
C. Distal tubule
D. Thin limbs of Henle’s loop 77. A 40-year-old man developed skin
E. Glomerular capsule redness and swelling in the neck area,
where eventually a small abscess appeared.
72. A patient has been taking bisacodyl for On section the focus is dense and yellow-
a long time to treat chronic constipation. green colored. In the purulent masses there
However, several weeks later the aperient are white granules. Histologically there
effect of the drug diminished. What is the are fungal druses, plasma and xanthome
possible cause of this? cells, and macrophages detected. Specify
the most correct etiological name of this
A. Acquired tolerance pathological process:
B. Drug dependence
C. Material cumulation A. Actinomycosis
D. Functional cumulation B. Furuncle
E. Sensitization C. Carbuncle
D. Syphilis
73. A force generated by the muscle is not E. Leprosy
enough to lift a load. What type of muscle
contraction occurs in this case? 78. A patient was hospitalized into the
infectious diseases unit on the 11th day si-
A. Isometric nce the disease onset and provisionally di-
B. Tetanic agnosed with typhoid fever. What biological
C. Isotonic material should be collected from the pati-
D. Eccentric ent for the analyzes at this stage?
E. Concentric
A. Blood serum
74. Condition of a patient with diabetes B. Urine
mellitus sharply deteriorated after a regular C. Feces
injection of insulin. The patient became D. Bile
anxious and broke out in cold sweat; tremor E. Roseola secretion
of the extremities, general weakness, and
dizziness appeared. What medicine can 79. A man with ischemic heart disease
remove these symptoms? has been taking his medicine too often
throughout a day, which resulted in poisoni-
A. Adrenaline ng. Examination detects cyanosis of skin
B. Tolbutamide and mucosa, sharp drop of blood pressure,
C. Caffeine tachycardia, and respiratory depression.
D. Noradrenaline Blood methemoglobin is high. What type
E. Glibutid (Buformin) of medicine did the patient overdose on?
75. Microscopy of the myocardium of a girl, A. Organic nitrates
who died of diphtheria due to heart failure, B. α-adrenergic blockers
shows fatty degeneration, multiple foci of C. Calcium channel blockers
cardiomyocytes necrosis, and small foci of D. Adenosine-based drugs
cellular infiltrates in the interstitium. What E. Myotropic antispasmodics
type of myocarditis is it?
80. General structure of eukaryotic genes is
as follows: exon-intron-exon. Such functi-
onal structure of a gene leads to certain
specifics of the transcription process. What
Krok 1 Medicine (англомовний варiант, iноземнi студенти) 2018 рiк 9

sequence will correspond with precursor sease can be characterized by such results
mRNA (immature)? of macroscopic examination?
A. Exon-intron-exon A. Acute pyelonephritis
B. Exon-exon-intron B. Acute glomerulonephritis
C. Exon-exon C. Renal amyloidosis
D. Intron-exon D. Nephrolithiasis
E. Exon-intron E. Polycystic kidney
81. Atria of a test animal were superdi- 86. A 54-year-old man complains of general
stended with blood, which resulted in weakness, frequent colds, and bruises
decreased reabsorption of Na+ and water constantly appearing on his body. Blood
in renal tubules. This can be explained by test: erythrocytes - 2.5 · 1012 /L; Hb- 80 g/L;
the effect of the following factor on the ki- color index - 0.9; reticulocytes - absent;
dneys: platelets - 50 · 109 /L; leukocytes - 58 · 109 /L;
leukogram: basocytes - 5%, eosinophils -
A. Natriuretic hormone 15%, myeloblasts - 6%, myelocytes - 10%,
B. Aldosterone juvenile - 18%, stab neutrophils - 26%,
C. Renin segmented neutrophils - 10%, lymphocytes
D. Angiotensin - 8%, monocytes - 2%, ESR - 40 mm/hour.
E. Vasopressin What hematologic conclusion can be made?
82. A patient with clinical signs of a pri- A. Chronic myelogenous leukemia
mary immunodeficiency has functionally di- B. Leukemoid response
sturbed mechanism of antigen-presentation C. Myeloblastic leukemia
to the immunocompetent cells. What cells D. Chronic lymphocytic leukemia
are likely to have structural defects? E. Basophilic eosinophilic leukocytosis
A. Macrophages, monocytes 87. A mutation has occurred in a cell in the
B. T-lymphocyte first exon of the structural gene. The number
C. B-lymphocyte of nucleotide pairs changed from 290 to 250.
D. Fibroblasts Name this type of mutation:
E. O-lymphocytes
A. Deletion
83. During intensive muscle work there is a B. Inversion
large amount of ammonia produced in the C. Duplication
muscles. What amino acid plays the main D. Translocation
role in the transportation of ammonia to E. Nullisomy
the liver and participates in gluconeogenesis
reactions? 88. The dorsal root of the spinal nerve of a
test animal was severed. What changes will
A. Alanine occur in the innervation area?
B. Arginine
C. Lysine A. Loss of sensitivity
D. Ornithine B. Loss of motor function
E. Aspartate C. Decreased muscle tone
D. Increased muscle tone
84. Encephalopathy has developed in a chi- E. Loss of sensitivity and motor function
ld with hemolytic disease of the newborn.
What substance had increased in the child’s 89. Due to destruction of certain structures
blood, resulting in damage to the CNS? of the brainstem a test animal has lost its ori-
entation reflexes in response to strong light
A. Unconjugated bilirubin stimuli. What structures were destroyed?
B. Bilirubin-albumin complex
C. Bilirubin glucuronide A. Anterior quadrigeminal bodies
D. Verdohemoglobin B. Posterior quadrigeminal bodies
E. Bile acids C. Red nuclei
D. Vestibular nuclei
85. Renal examination shows the kidney E. Substantia nigra
to be swollen and plethoric, with easily
removable capsule. Renal pelvis and calyces 90. A toxin neutralized with 0.4%
are dilated and filled with turbid urine; their formaldehyde under 37-40o C for 4 weeks is
mucosa is dull and has hemorrhagic foci. used for vaccination. This preparation was
On section the renal tissue is variegated, first used by Gaston Ramon for diphtheria
there are yellow-gray areas surrounded with prevention. Name this preparation:
plethoric and hemorrhagic zone. What di-
Krok 1 Medicine (англомовний варiант, iноземнi студенти) 2018 рiк 10

A. Anatoxin pregnancy with impending miscarriage was


B. Immunoglobulin brought to the hospital. What combination
C. Antitoxic serum of Rh-factor of the mother and the fetus can
D. Adjuvant be the cause of this condition?
E. Inactivated vaccine
A. Mother Rh (-), fetus Rh (+)
91. 24 hours after an appendectomy the B. Mother Rh (-), fetus Rh (-)
patient’s blood test shows neutrophilic C. Mother Rh (+), fetus Rh (-)
leukocytosis with a regenerative shift. What D. Mother Rh (+), fetus Rh (+)
is the most likely mechanism of absolute E. -
leukocytosis development in the patient’s
peripheral blood? 96. A specimen of a 10-day-old human
embryo shows two interconnected sacs
A. Intensification of leukopoiesis
B. Leukocyte redistribution (amniotic and yolk sacs). Name the
C. Decreased leukocyte disintegration structure located in the place where these
D. Deceleration of leukocyte migration to two sacs connect:
the tissues A. Embryonic shield
E. Immunity activation B. Floor of the amniotic sac
92. The right leg of a 40-year-old woman C. Roof of the amniotic sac
measured at the shin level is by 2 cm smaller D. Amniotic stalk
in the diameter than the left leg. Ankle- E. Extraembryonic mesoderm
jerk (Achilles) and knee-jerk reflexes are 97. A woman with the III (В), Rh (-) blood
absent on the right. What is the most likely group gave birth to a child with the II (А)
mechanism of hyporeflexia development blood group. The child is diagnosed with
during peripheral paralysis? hemolytic disease of newborn caused by
A. Disturbed conduction of stimulation rhesus incompatibility. What blood group
B. Inhibition of pyramidal motoneuron and Rh does the father have?
C. Disturbed synaptic impulse transmission A. II (А), Rh (+)
D. Activation of excitatory impulses from
the CNS B. I (0), Rh (+)
E. Disturbed perception of stimulation C. III (B), Rh (+)
D. I (0), Rh (-)
93. A 38-year-old man, who has E. II (A), Rh (-)
been suffering from systemic lupus
erythematosus for 3 years, developed di- 98. A 40-year-old woman suffers from
ffuse renal lesions accompanied by massi- Cushing’s disease - steroid diabetes.
ve edemas, marked proteinuria, hyperli- On biochemical examination she has
pidemia, and dysproteinemia. What is hyperglycemia and hypochloremia. What
the most likely mechanism of proteinuria process activates in the first place in such
develoment in this case? patients?
A. Autoimmune damage to the nephrons A. Gluconeogenesis
B. Inflammatory damage to the nephrons B. Glycogenolysis
C. Ischemic damage to the tubules C. Glucose reabsorption
D. Increased blood proteins D. Glucose transport to the cells
E. Morbid affection of the urinary tracts E. Glycolysis
94. A 59-year-old man, a business manager, 99. A patient with a chemical burn has
developed intense burning retrosternal pain developed esophageal stenosis. The pati-
that irradiates to the left arm. The pain ent presents with acute weight loss due
occurred in the evening after the tax audit. to problematic food intake. Blood test:
15 minutes later the patient’s condition erythrocytes - 3.0 · 1012/L, Hb- 106 g/L, total
normalized. What mechanism of angina protein - 57 g/L. What type of starvation
pectoris development is leading in this pati- does this patient suffer from?
ent?
A. Incomplete starvation
A. Increased level of blood catecholamines B. Protein starvation
B. Coronary atherosclerosis C. Complete starvation
C. Intravascular aggregation of blood cells D. Water starvation
D. Coronary artery thrombosis E. Absolute starvation
E. Functional cardiac overload
100. Autopsy of the body of a man, who
95. A 25-year-old woman at her third for a long time had been working at the
Krok 1 Medicine (англомовний варiант, iноземнi студенти) 2018 рiк 11

factory with high content of silicon dioxide A. Benzylpenicillin


in the air, revealed enlarged dense lungs wi- B. Rifampicin
th numerous round and oval sclerotic foci. C. Erythromycin
The foci were miliary and larger in size and D. Streptomycin sulfate
colored gray or gray-black. What is the most E. Levomycetin (Chloramphenicol)
likely diagnosis?
105. A microslide of the skin sample taken
A. Nodular silicosis from the finger of a child shows that epi-
B. Diffuse sclerotic silicosis dermis is insufficiently developed. What
C. Anthracosilicosis germ layer was damaged in the process of
D. Silicoanthracosis embryo development?
E. Asbestosis
A. Ectoderm
101. Autopsy of the body of a 50-year- B. Mesoderm
old man, who had been suffering from C. Endoderm
tuberculosis and died of cardiopulmonary D. Mesenchyme
decompensation, shows lobar affection of E. Ectomesenchyme
the lungs: the upper lobe of the right lung is
enlarged, dense, yellow on section, crumbli- 106. Neutralization of xenobiotics and acti-
ng, with pleural fibrin deposition. What type ve endogenous metabolites often occurs via
of secondary tuberculosis can be characteri- introduction of an oxygen atom into the
zed by this pathology? substrate molecule. What process occurs as
the result?
A. Caseous pneumonia
B. Fibrous focal tuberculosis A. Hydroxylation
C. Infiltrative tuberculosis B. Decarboxylation
D. Tuberculoma C. Transamination
E. Acute focal tuberculosis D. Phosphorilation
E. Deaminization
102. A woman has come to the hospital wi-
th complaints of general weakness, dizzi- 107. In the process of an experiment,
ness, and dyspnea. Resently she has been vascular resistance to the blood flow was
taking levomycetin (chloramphenicol) for measured in the different areas of ci-
prevention of enteric infection. Blood test: rculatory system. The highest resistance was
detected in the:
erythrocytes - 1.9 · 1012 /L, hemoglobin - 58
g/L, color index - 0.9, leukocytes - 2.2 G/L, A. Arterioles
reticulocytes - 0.3%. What type of anemia is B. Arteries
it indicative of? C. Capillaries
D. Venules
A. Hypoplastic E. Veins
B. Metaplastic
C. Aplastic 108. A 34-year-old man died in a comatose
D. Hemolytic state. According to his family after a busi-
E. Iron-deficiency ness trip to an African country he developed
periodical jaundice attacks. Autopsy shows
103. Preoperative examination revealed the following: dense enlarged spleen with
prothrombin deficiency in the blood of the slate-black pulp; enlarged plethoric liver,
patient. What drug should be preliminarily gray-black on section; cerebral gray matter
prescribed to mitigate blood loss in the pati- is brown-gray; cerebral white matter contai-
ent during the surgery? ns numerous small hemorrhages. What
A. Vicasol (Menadione) infectious disease can be suspected?
B. Thrombin A. Malaria
C. Aminocapronic acid B. Meningococcemia
D. Phenylin (Phenindione) C. Prion infection
E. Contrykal (Aprotinin) D. Generalized herpetic infection
E. Generalized cryptococcosis
104. A patient with streptococcal infecti-
on of the gingiva was prescribed a drug 109. What diagnostic method should be
with β -lactam ring in its structure. What used in industry to test the raw leather for
drug of those listed below belongs to this presence of B. antracis?
pharmacological group?
Krok 1 Medicine (англомовний варiант, iноземнi студенти) 2018 рiк 12

A. Ascoli’s thermo precipitation test A. Acetylcholine


B. Microscopy with Burry-Gins stain B. Noradrenaline
C. Microscopy with Aujeszky stain C. Adrenaline
D. Bacteriological analysis D. Dopamine
E. Serological test E. Serotonin
110. A 46-year-old patient suffering from 115. A 17-year-old girl suffers from periodi-
ulcer disease of the stomach is diagnosed cal palpitations that last several minutes.
with rheumatoid arthritis. What antii- Her heart rate is 200/min., rhythmic. What
nflammatory drug would be the most advi- heart rhythm disorder developed in this
sable in this case? patient?
A. Celecoxib A. Paroxysmal tachycardia
B. Prednisolone B. Sinus tachycardia
C. Analgin (Metamizole) C. Sinus bradycardia
D. Promedol (Trimeperidine) D. Extrasystole
E. Paracetamol E. Atrioventricular block
111. A patient with myocardial infarction 116. Kidney X-ray image obtained by means
has acute heart failure. Among the drugs of pyelography shows that the minor calyces
that increase the force of heart contractions converge to form 2 major calyces that adjoin
the least dangerous in this case will be: to the renal pelvis, from which the ureter
exits. What type of urinary tract is it?
A. Dobutamine
B. Adrenaline A. Mature
C. Isadrinum (Isoprenaline) B. Embryonic
D. Euphyllin (Aminophylline) C. Bicornuate
E. Caffeine D. Cystic
E. Fetal
112. During intensive physical exertion,
one of the energy sources for the working 117. A patient has been brought into a
muscles is glucose produced as the result of surgical ward with an incised wound of the
gluconeogenesis. This process is the most anterior surface of the shoulder in its lower
intensive in the following organ: one-third. Flexing function was disturbed
in the shoulder and elbow joints, which is
A. Liver caused by the damage to the:
B. Brain
C. Lungs A. Biceps muscle of the arm
D. Muscles B. Triceps muscle of the arm
E. Stomach C. Anconeus muscle
D. Deltoid muscle
113. During inflienza epidemic, morbidity in E. Coracobrachial muscle
the schoolchildren, who did not participate
in sports, was 40%, while in the schoolchi- 118. A patient suffers from acute cardi-
ldren, who engaged in regular physical aci- opulmonary failure with pulmonary edema.
vities, morbidity did not exceed 20%. What What diuretic should be prescribed in the
adaptation mechanism ensured low morbi- given case?
dity in the physically active schoolchildren?
A. Furosemide
A. Cross-adaptation B. Triamterene
B. Specific adaptation C. Spironolactone
C. Physiological adaptation D. Hydrochlorothiazide (Dichlothiazidum)
D. Biochemical adaptation E. Acetazolamide (Diacarb)
E. Genetic adaptation
119. Ketosis develops in the patients with di-
114. Monoamine oxidase inhibitors are abetes mellitus, as the result of activation of
widely used as psychopharmacological fatty acids oxidation processes. What acid-
drugs. They change the level of nearly base imbalance can result from accumulati-
all neurotransmitters in synapses, with on of excessive ketone bodies in the blood?
the following neurotransmitter being the
exception: A. Metabolic acidosis
B. Metabolic alkalosis
C. No imbalance occurs
D. Respiratory acidosis
E. Respiratory alkalosis
Krok 1 Medicine (англомовний варiант, iноземнi студенти) 2018 рiк 13

120. A 5-year-old child is diagnosed canals of a test animal have been destroyed.
with Bruton syndrome (X-linked What reflexes will disappear as a result?
agammaglobulinemia) that manifests itself
as severe clinical course of bacterial A. Statokinetic reflex during movements
infections and absence of B lymphocytes with angular acceleration
and plasma cells. What changes of B. Statokinetic reflex during movements
immunoglobulin content can be observed in with linear acceleration
blood serum of the child with immunodefi- C. Head-righting reflex
ciency? D. Body-righting reflex
E. Primary orienting reflex
A. Decreased IgA, IgM
B. Increased IgA, IgM 126. Acute renal impairment caused death
C. Decreased IgD, IgE of a patient with hemorrhage. Autopsy
D. Increased IgD, IgE revealed enlarged kidneys with broad pale
E. No changes pink cortical layer expressively demarcated
from dark red renal pyramids. Macroscopic
121. During the skill-building session in mi- examination revealed lack of epitheli-
crobiology the students need to stain the al nuclei of the convoluted tubules,
prepared and fixed sputum smears obtained tubulorrhexis, phlebostasis. The cell nuclei
form a tuberculosis patient. What staining of the choroid glomus and straight tubules
technique should be used in this case? were present. What pathology is it?
A. Ziehl-Neelsen A. Necronephrosis
B. Burry B. Infarction
C. Giemsa C. Glomerulonephritis
D. Gins D. Pyelonephritis
E. Gram E. Nephrosis
122. A woman with hypophyseal di- 127. A boxer had sustained a blow to the
abetes insipidus developed a water-mineral left parotid area, after which he developed
imbalance. What type of water-mineral paralysis of the facial muscles on this side.
imbalance develops in such cases? What nerve had been damaged?
A. Hyperosmolar dehydration A. Facial
B. Hypoosmolar dehydration B. Ophthalmic
C. Isoosmolar dehydration C. Maxillary
D. Hypoosmolar hyperhydration D. Mandibular
E. Hyperosmolar hyperhydration E. Lesser petrosal
123. First-year schoolchildren have received 128. A patient has an open facial wound wi-
tuberculin skin test (Mantoux test) at the th undermined edges; tissue necrosis with
school nurse’s office. The purpose of this gradually developing partial gangrene that
test was: nearly reaches the bone tissue is observed.
On close examination the wound contains
A. To determine the children that need to live larvae. The patient is diagnosed with
receive BCG vaccination tissue myiasis. What Diptera larvae are the
B. To preventively vaccinate against causative agents of this disease?
tuberculosis
C. To measure immunity stress toward A. Wohlfahrtia magnifica
diphtheria B. Glossina palpalis
D. To measure allergization rate toward C. Musca domestica
rickettsia D. Phlebotomus pappatachi
E. To detect parotitis in the schoolchildren E. Stomoxys calcitrans

124. In an experiment it is necessary to 129. A hereditary disease - homocysti-


assess neuromotor and muscle excitability. nuria - is caused by disturbed transformati-
What value shoud be measured to make the on of homocysteine into methionine.
assessment? Accumulated homocysteine forms its di-
mer (homocystine) that can be found in uri-
A. Sensory threshold ne. What vitamin preparation can decrease
B. Action potential amplitude homocysteine production?
C. Resting potential
D. Threshold potential
E. Action potential duration
125. Vestibular receptors of semicircular
Krok 1 Medicine (англомовний варiант, iноземнi студенти) 2018 рiк 14

A. Vitamin B12
B. Vitamin C A. H2 antagonists
C. Vitamin B1 B. H1 antagonists
D. Vitamin B2 C. α-adrenergic antagonists
E. Vitamin P P D. β -adrenergic antagonists
E. α-adrenergic agonists
130. During hypersensitivity skin test a pati-
ent received an allergen subcutaneously, 135. Autopsy of the body revealed waxy
after which the patient developed skin degeneration of the rectus abdominis
redness, edema, and pain due to histami- muscles. In the terminal segment of the
ne action. This biogenic amine is produced small intestine there are ulcers 3-5 cm in
as the result of the following transformation diameter. The ulcer walls are covered in
of histidine amino acid: a crumbling grayish-white substance. The
ulcer edges are markedly raised above the
A. Decarboxylation mucosa. Widal test is positive. Make the di-
B. Methylation agnosis:
C. Phosphorilation
D. Isomerization A. Typhoid fever
E. Deaminization B. Nonspecific ulcerative colitis
C. Relapsing fever
131. During the fight a man has received a D. Crohn’s disease
strong blow to the upper anterior abdomi- E. Dysentery
nal wall, which resulted in the cardiac arrest.
What mechanism has led to the cardiac 136. Genealogical analysis of a child with
arrest in this case? myotonic dystrophy determined that this di-
sease is present in every generation, equally
A. Parasympathetic unconditioned presented by the relatives of both genders,
B. Sympathetic unconditioned the risk of inheriting this disease is equal no
C. Parasympathetic conditioned matter which parent is affected. If one of the
D. Sympathetic conditioned parents is heterozygous for this disease and
E. Peripheral the other parent is healthy, the risk of them
giving birth to a sick child is 50%. What type
132. A woman has lost a lot of blood duri- of disease inheritance is it?
ng the childbirth. Her blood group needs to
be determined. Erythrocyte agglutination A. Autosomal dominant
occurred with standard serums 0 (I) and А B. Autosomal recessive
(II) and did not occur with standard serum C. X-linked dominant
B (III). What blood group does this woman D. X-linked recessive
have? E. Y-linked
A. B (III) 137. A 30-year-old man has undergone
B. 0 (I) surgical removal of a thyroid tumor. Hi-
C. А (II) stologically the tumor is made up of papi-
D. АВ (IV) llary structures varying in size that emerge
E. - from the inner surface of dilated cystic folli-
cles and are covered with atypical epitheli-
133. A 60-year-old man diagnosed with um. What is the most likely diagnosis?
chronic heart failure was brought to the
hospital. After a long course of treatment A. Papillary carcinoma
the patient developed signs of intoxicati- B. Colloid nodular goiter
on: dyspnea, extrasystole, nausea, and di- C. Macrofollicular adenoma
sturbed perception of colors. What medici- D. Follicular carcinoma
ne has caused such side-effects? E. Medullary carcinoma

A. Digoxin 138. A young woman, a foreign student


from Tehran, has made an appointment with
B. Anaprilin (Propranolol) the urologist. She complains of the sensati-
C. Nitroglycerine on of heaviness in her lower abdomen and
D. Drotaverine a small amount of blood being excreted
E. Furosemide with urine at the end of each urination.
134. A patient with exacerbated peptic ulcer Microscopy of urine detects the presence
disease of the stomach has made an appoi- of parasite eggs, approximately 140х70 mi-
ntment with the doctor. What type of drugs cron in size, with a terminal spike. What
should be included in the complex therapy diagnosis can be made by the infectious di-
of this patient? seases specialist?
Krok 1 Medicine (англомовний варiант, iноземнi студенти) 2018 рiк 15

A. Schistosomiasis
B. Opisthorchiasis A. Phenylketonuria
C. Dicroceliasis B. Albinism
D. Paragonimiasis C. Tyrosinosis
E. Fascioliasis D. Alkaptonuria
E. Gout
139. Various biological preparations can be
used for poliomyelitis prevention. What 144. An oncology patient is to undergo a
drug induces the type of local intestinal surgery on the descending colon. Name the
mucosal immunity that lasts the longest? main source of blood supply to this organ:
A. Oral vaccination with live vaccine A. Inferior mesenteric artery
B. Parenteral vaccination with inactivated B. Superior mesenteric artery
vaccine C. Celiac trunk
C. Oral introduction of poliovirus-specific D. Middle colic artery
immunoglobulin E. Splenic artery
D. Parenteral vaccination with live vaccine
E. Parenteral introduction of normal human 145. A patient presents with enlarged cervi-
immunoglobulin cal lymph nodes. Other lymph nodes and
internal organs are without changes. Peri-
140. Histological analysis of a removed skin pheral blood test results are normal. Hi-
tumor shows clusters and bands composed stological examination of biopsy material
of atypical stratified squamous epithelium taken from the cervical lymph node shows
cells that penetrate into the underlying ti- smoothed-out pattern, absent follicles,
ssue. What preliminary diagnosis can be homogeneous cell composition represented
made? by lymphoblasts. The cells penetrate into
the lymph node capsule. What diagnosis can
A. Non-keratinizing squamous cell carci- be made?
noma
B. Solid cancer A. Lymphoblastic leukemia
C. Carcinoma in situ B. Myeloblastic leukemia
D. Papilloma C. Erythroblastic leukemia
E. Adenoma D. Sezary disease
E. Burkitt lymphoma
141. Autopsy of a patient, who died of bi-
lateral bronchopneumonia, shows in the left 146. When studying the pulmonary venti-
lung lower lobe a cavity 5 cm in diameter, lation values, the reduction of forced expi-
filled with yellowish-white liquid. What ratory volume has been detected. What is
complication of the patient’s pneumonia the likely cause of this phenomenon?
had developed?
A. Obstructive pulmonary disease
A. Abscess B. Increase of respiratory volume
B. Gangrene C. Increase of inspiratory reserve volume
C. Granuloma D. Increase of pulmonary residual volume
D. Sequestrum E. Increase of functional residual lung
E. Tuberculoma capacity
142. A patient with parkinsonism was 147. A man is a carrier of HIV that is
prescribed levodopa, which led to rapid an RNA virus. The cells of this patient
improvement of the patient’s condition. synthesize viral DNA. This process is based
What is the mechanism of action of this on:
drug?
A. Reverse transcription
A. Stimulation of dopamine synthesis B. Replication
B. Muscarinic acetylcholine receptor C. Transcription
blockade D. Repair
C. Stimulation of dopamine receptors E. Translation
D. Anticholinesterase action
E. Muscarinic acetylcholine receptor sti- 148. Sanitary assessment of a pond, where
mulation the children from a recreation summer
camp take their swims, detected there oval
143. A sick child presents with high content cysts 50-60 micron in diameter, with 2 nuclei
of phenyl pyruvate in urine (normally it visible in their cytoplasm (macronucleus
is practically absent). Blood phenylalanine and micronucleus). What protozoa do these
level is 350 mg/L (norm - 15 mg/L). What di- cysts belong to?
sease are these symptoms characteristic of?
Krok 1 Medicine (англомовний варiант, iноземнi студенти) 2018 рiк 16

A. Balantidium
B. Lamblia A. Neurinoma
C. Toxoplasma B. Malignant neurinoma
D. Amoeba C. Ganglioneuroma
E. Euglena D. Neuroblastoma
E. Ganglioneuroblastoma
149. A hospital has received a 24-year-old
man, who had received a penetrating wound 154. A patient has undergone surgical
to the eye, which has caused the vitreous removal of the pylorus. Decreased secretion
body to run out. As the result of this, reti- of the following hormone can be expected:
nal detachment occurred. What retinal layer
was tightly adherent to the vascular tunic of A. Gastrin
the eye and did not detach? B. Histamine
C. Secretin
A. Retinal pigment epithelium D. Cholecystokinin
B. Layer of rods and cones E. Gastric inhibitory polypeptide
C. Ganglion cell layer
D. Outer nuclear layer 155. An electron micrograph shows a
E. Inner nuclear layer small vessel with endothelial layer but wi-
thout basement membrane and pericytes;
150. A newborn presents with weak suckli- anchoring fibrils are present. Name this
ng, frequent vomiting, and hypotonia. vessel:
Blood and urine citrulline are very high.
What metabolic process is disturbed? A. Lymph capillary
B. Arteriole
A. Ornithine cycle C. Venule
B. Tricarboxylic acid cycle D. Sinusoid hemocapillary
C. Glycolysis E. Visceral hemocapillary
D. Gluconeogenesis
E. Cori cycle 156. One of the causes of pernicious anemia
is disturbed synthesis of transcorrin -
151. Stool test detects in the patients feces a Castle’s intrinsic factor - by the parietal cells
large amount of undigested fats.This patient of the stomach. What substance is called
is the most likely to have disturbed secreti- Castle’s extrinsic factor?
on of the following enzymes:
A. Cobalamin
A. Pancreatic lipases B. Folic acid
B. Pancreatic amylase C. Pyridoxine
C. Pancreatic proteases D. Riboflavin
D. Bile lipase E. Biotin
E. Gastric protease
157. A patient with contact dermatitis needs
152. A 19-year-old young man was exami- to be prescribed an antihistamine drug wi-
ned in the nephrology clinic. High calcium thout somnolescent effect. Select this drug
was detected in his secondary urine. What from the list:
hormone is likely to cause such change, if it
is produced in excess? A. Loratadine
B. Dimedrol (Benadryl)
A. Aldosterone C. Suprastin (Chloropyramine)
B. Oxytocin D. Diprazine (Promethazine)
C. Adrenaline E. Ranitidine
D. Glucagon
E. Testosterone 158. Congenital pyruvate carboxylase defi-
ciency causes physical and mental retardati-
153. A young woman presents with a on in children and leads to early death. It
tumor along the auditory nerve. The tumor is characterized by lactic acidemia, lactaci-
is node-shaped, 3 cm in diameter, soft duria, and a number of metabolic disorders.
and elastic, pink-white colored, and has Among others, inhibition of the following
homogeneous structure. Microscopically occurs:
the tumor contains bundles of cells with
oval nuclei. These cellular fibrous bundles A. Citric acid cycle and gluconeogenesis
form regular structures made up of parallel B. Glycolysis and glycogenolysis
rows of regularly oriented cells arranged C. Glycogenesis and glycogenolysis
in the form of a palisade with acellular D. Lipolysis and lipogenesis
homogeneous area in between (Verocay E. Pentose-phosphate pathway and
bodies). Name this type of tumor: glycolysis
Krok 1 Medicine (англомовний варiант, iноземнi студенти) 2018 рiк 17

159. A woman with enteritis accompani- substance is disturbed:


ed by severe diarrhea presents with loss of
water in the extracellular space, increased A. Sphingomyelinase
water content in the cells, and decreasing B. Glycosyltransferase
blood osmolarity. Name this type of water- C. Sphingosine
electrolyte imbalance: D. Ceramides
E. Gangliosides
A. Hypoosmolar hypohydration
B. Hyperosmolar hyperhydration 165. A traumatology unit has received a
C. Hyperosmolar hypohydration patient with a wrist trauma and a clinical
D. Hypoosmolar hyperhydration presentation of the damage to the nerve
E. Isoosmolar hypohydration that passes through the carpal tunnel. Name
this nerve:
160. To stimulate the labor activity a parturi-
ent woman was prescribed a drug - a A. N. medianus
posterior pituitary hormone that does not B. N. ulnaris
affect the blood pressure. As the pregnancy C. N. radialis
progresses, the sensitivity to this hormone D. N. axillaris
increases. Name the prescribed drug: E. N. musculocutaneus

A. Oxytocin 166. A patient, who has been taking β -


B. Dinoprostone adrenergic blockers, developed a bronchi-
C. Dinoprost al spasm. What group of bronchodilators
D. Pituitrin should be chosen to stop the bronchial
E. Ergotal spasm?
161. Cell membrane rest potential changed A. Myotropic antispasmodics
from -85 to -90 mV. It can be caused by B. β -adrenergic blockers
activation of the following cell membrane C. Indirect adrenergic agonists
channels: D. Muscarinic cholinomimetic agents
E. Cholinesterase inhibitors
A. Potassium
B. Sodium 167. A man has an inguinal hernia. The
C. Potassium and sodium hernial sac exits through the following
D. Calcium opening:
E. Potassium and calcium
A. Anulus superficialis canalis inguinalis
162. Ionizing radiation or vitamin E defici- B. Foramen suprapiriforme
ency affect the cell by increasing lysosome C. Linea alba
membrane permeability. What are the possi- D. Anulus profundus canalis inguinalis
ble consequences of this pathology? E. Anulus femoralis
A. Partial or complete cell destruction 168. After a nephrectomy the patient
B. Intensive protein synthesis developed enteroparesis. What cholinergic
C. Intensive energy production agent with anticholinesterase action should
D. Restoration of cytoplasmic membrane be prescribed in this case?
E. Formation of maturation spindle
A. Proserin
163. A patient is diagnosed with B. Carbacholine
glucocerebroside lipidosis (Gaucher’s di- C. Aceclidine
sease) that manifests as splenomegaly, li- D. Pilocarpine
ver enlargement, affected bone tissue, and E. Acetylcholine
neuropathies. What enzyme of complex li- 169. Parenchyma of an organ is composed of
pid catabolism is deficient, thus causing this pseudounipolar neurons localized under the
disease? capsule of connective tissue. Central place
A. Glucocerebrosidase belongs to nerve fibers. Name this organ:
B. Hexosaminidase A. Spinal ganglion
C. Sphingomyelinase B. Sympathetic ganglion
D. β -galactosidase C. Intramural ganglion
E. Hyaluronidase D. Nerve trunk
164. A 3-year-old girl with mental retardati- E. Spinal cord
on has been diagnosed with sphingomyelin 170. An obstetrician-gynecologist measures
lipidosis (Niemann-Pick disease). In this pelvis size of a pregnant woman. A caliper
condition the synthesis of the following
Krok 1 Medicine (англомовний варiант, iноземнi студенти) 2018 рiк 18

was used to measure the distance between A. Glucose-6-phosphate dehydrogenase


the two iliac crests. What measurement of B. Hexokinase
large pelvis was made? C. Fructokinase
D. Pyruvate kinase
A. Distantia cristarum E. Lactate dehydrogenase
B. Distantia throchanterica
C. Distantia spinarum 176. When examining a biopsy materi-
D. Conjugata vera al obtained from the thyroid gland, the
E. Conjugata anatomica pathologist discovered lymphocyte infi-
ltration of the thyroid tissues and destructi-
171. Genetic recombination is achieved via on of the parenchymal elements. Diffuse
several mechanisms. One such mechanism lymphocyte infiltration with lymphoid folli-
is crossingover. It occurs at the following cles was detected in the stroma. What is the
stage of prophase in the first meiotic divisi- most likely diagnosis?
on:
A. Hashimoto’s thyroiditis (chronic
A. Pachynema lymphocytic thyroiditis)
B. Leptonema B. Papillary thyroid cancer
C. Zygonema C. Undifferentiated thyroid carcinoma
D. Diplonema D. Solid adenoma of the thyroid
E. Diakinesis E. Graves’ disease (toxic diffuse goiter)
172. ECG of the patient shows increased 177. An 8-year-old girl presents with signs of
duration of the QRS complex. What is the disturbed twilight vision. This condition is
most likely cause? caused by the deficiency of vitamin:
A. Increased period of ventricular excitation A. A
B. Disturbed conduction in the atrioventri- B. E
cular node C. D
C. Increased atrial excitability D. K
D. Increased atrial and ventricular excitabili- E. F
ty
E. Increased period of atrial excitation 178. A 25-year-old young man came to the
doctor complaining of general weakness,
173. A 50-year-old inpatient during exami- rapid fatigability, irritability, reduced worki-
nation presents with glucosuria and blood ng ability, and bleeding gums. What vitamin
glucose of 3.0 mmol/L, which are the most is likely to be deficient in this case?
likely to be caused by:
A. Ascorbic acid
A. Renal disorder B. Riboflavin
B. Diabetes insipidus C. Thiamine
C. Myxedema D. Retinol
D. Essential hypertension E. Folic acid
E. Pellagra
179. A 52-year-old man presents with fever
174. A 40-year-old woman was diagnosed and pain in the joints. Both of his fi-
with glomerulonephritis based on her cli- rst metatarsophalangeal articulations are
nical symptoms and the results of urine deformed, swollen, and reddened. Blood
analysis. Anamnesis states chronic tonsilli- urea is high. The patient is diagnosed wi-
tis. What microorganisms are the most likely th gout. What is the main developmental
cause for her kidney damage? factor in the pathogenesis of this disease?
A. Streptococci A. Hyperuricemy
B. Staphylococci B. Argininosuccinic aciduria
C. Escherichia C. Hyperazotemia
D. Mycoplasma D. Hyperaminoacidemia
E. Meningococci E. Citrullinuria
175. A 7-year-old child presents with 180. A person with the fourth blood group
marked signs of hemolytic anemia. Bi- (genotype IAIB) has in erythrocytes both
ochemical analysis of erythrocytes determi- antigen A controlled by allele IA and
ned low concentration of NADPH and antigen B controlled by allele IB. This
reduced glutathione. What enzyme is defi- phenomenon is an example of the following
cient in this case leading to the biochemical gene interaction:
changes and their clinical manifestations?
Krok 1 Medicine (англомовний варiант, iноземнi студенти) 2018 рiк 19

A. Codominance 186. During surgery performed in abdomi-


B. Complementarity nal cavity a surgeon located the ligament of
C. Semidominance liver stretching from the anterior abdominal
D. Polymery wall (navel) to the inferior surface of liver.
E. Epistasis What ligament is it?
181. Vascular endothelium is characterized A. Round ligament of the liver
by high metabolic activity and synthesi- B. Falciform ligament of the liver
zes vasoactive substances. Among these C. Coronary ligament of the liver
substances there is a potent vasodilator D. Venous ligament of the liver
synthesized from L-arginine. Name this E. Triangular ligament of the liver
vasodilator:
187. A patient has disturbed blood supply
A. Nitrogen oxide to the anterior papillary muscle of the left
B. Histamine ventricle. What cardiac artery supplies this
C. Bradykinin muscle with blood?
D. Acetylcholine
E. Adrenaline A. Ramus interventricularis anterior
B. Ramus interventricularis posterior
182. A victim of a traffic accident was recei- C. A. coronaria dextra
ved by the intensive care unit. The patient is D. Ramus circumflexus
in a grave condition that can be characteri- E. Ramus marginalis
zed as a severe pathologic process that leads
to exhaustion of vital functions and puts the 188. Autopsy of the body of an elderly man
patient into the marginal state between life revealed yellow spots and streaks in the
and death due to critical reduction of capi- aortic intima and white-yellow protruding
llary circulation in the affected organs. The plaques in the area of aortic bifurcation.
patient is in the state of: Microscopy (with hematoxylin and eosin
staining) shows round cavities in the thi-
A. Shock ckened aortic intima. The cavities color
B. Collapse orange when stained with sudan 3 and are
C. Coma surrounded with overgrown connective ti-
D. Agony ssue. What process developed in the aortic
E. Preagony intima?
183. A person becomes less receptive to A. Metabolic disorder of cholesterol and
pain in physically or emotionally straining cholesterol ethers
situations due to activation of: B. Local hyalinosis
A. Antinociceptive system C. Metabolic disorder of neutral fat
B. Thyroid gland functions D. Systemic hyalinosis
C. Nociceptive system E. Secondary amyloidosis
D. Adrenal glands functions 189. An 18-year-old patient has developed
E. Parasympathetic nervous system candidiasis after the case of pneumonia
184. A patient suffers from posttraumatic treated with β -lactam antibiotic. What anti-
hemorrhage that resulted in development mycotic agent should be prescribed?
of hemorrhagic shock. What volume of ci- A. Fluconazole
rculating blood was lost by the patient? B. Streptomycin
A. 25-40% C. Ampicillin
B. 40-50% D. Phthalylsulfathiazole
C. 12-25% E. Trimethoprim/sulfamethoxazole (Bi-
D. 50-75% septol)
E. 3-20%
190. Preventive vaccination against poli-
185. A patient with pulmonary tuberculosis omyelitis is made with inactivated
is prescribed the most effective anti- vaccine introduced parenterally. What
tuberculous antibiotic. Name this drug: immunoglobulins create the postvaccinal
immunity in this case?
A. Rifampicin
B. Tetracycline A. IgM, IgG
C. Streptocide B. IgG, secretory IgA
D. Furasolidone C. IgM, secretory IgA
E. Bactrim (Co-trimoxazole) D. Serum IgА, IgM
E. IgЕ, IgM
Krok 1 Medicine (англомовний варiант, iноземнi студенти) 2018 рiк 20

191. During pregnancy a woman has been provides sensitivity of the affected area?
taking an antiepileptic drug - sodium
valproate. It resulted in her child developing A. N. musculocutaneus
a vertebral column malformation - spina bi- B. N. medianus
fida. Name the described effect of the drug: C. N. ulnaris
D. N. axillaris
A. Teratogenic effect E. N. radialis
B. Immunosuppressive effect
C. Acquired tolerance 197. Autopsy of a 50-year-old woman,
D. Dependence who had been suffering from systemic
E. Sensitizing effect connective tissue disease, reveals small
dense kidneys with lumpy surface. Mi-
192. A patient was prescribed atropi- croscopy of the renal glomeruli detects
ne sulfate to treat intestinal colic. What there foci of fibrinoid necrosis, hyali-
concomitant disease can be a contraindi- ne thrombi, hematoxylin bodies, and
cation for this drug? so-called ”wire loop thickening” of the
basement membranes. What disease can
A. Glaucoma be characterized by such changes in the ki-
B. Bronchial asthma dneys?
C. Sinus bradycardia
D. Hypotension A. Systemic lupus erythematosus
E. Vertigo B. Rheumatism
C. Scleroderma
193. A patient has developed status epilepti- D. Rheumatoid arthritis
cus. What medicine should be used in this E. Dermatomyositis
case to stop the seizures?
198. A 4-year-old child presents with
A. Diazepam numerous carious cavities and yellow-
B. Cyclodol (Trihexyphenidyl) colored teeth. The mother has a history of
C. Diprazine (Promethazine) antibiotic treatment during her pregnancy.
D. Sodium bromide What antibiotic was the most likely taken
E. Valerian extract by the child’s mother?
194. A 48-year-old man is unconscious. A. Doxycycline
He has a history of several syncopal B. Streptomycin sulfate
episodes with convulsions. ECG shows C. Ampicillin
deformed QRS complexes unconnected wi- D. Erythromycin
th P waves, atrial contractions are approxi- E. Cefazolin
mately 70/min., ventricular contractions -
25-30/min. Name the type of arrhythmia in 199. A patient developed pyoinflammatory
this case: process of periodontal tissues caused by
activation of the microorganisms inherent
A. Complete atrioventricular block in the body, which are a part of oral mucosal
B. First-degree atrioventricular block microflora. What type of infection is it?
C. Second-degree atrioventricular block
D. Intraatrial block A. Autoinfection
E. Intraventricular block B. Exogenous infection
C. Reinfection
195. In human organism significant blood D. Superinfection
loss leads to decreased blood pressure, E. Relapse
tachycardia, and weakness. Eventually the
sensation of thirst appears. What hormone 200. After a hand fracture the man has lost
participates in the development of this the ability to touch his little finger with the
sensation? thumb on the affected hand. Examination of
the palmar surface shows one of the palmar
A. Angiotensin 2 muscles to be atrophied. Name the affected
B. Cortisol muscle:
C. Serotonin
D. Dopamine A. M. opponens policis
E. Adrenalin B. Mm. lumbricales
C. Mm. interossei
196. After a trauma the man has lost skin D. M. flexor digiti minimi
sensitivity on the lateral surface of his E. M. palmaris brevis
forearm. What nerve of the brachial plexus
INSTRUCTIONAL BOOK
Testing Board

TEST ITEMS FOR LICENSING EXAMINATION: KROK 1. MEDICINE.

Kyiv. Testing Board.


(English language).

A ed i 03.05/ 249 Pa e i e 60 84 1/8


Offset paper. Typeface. Times New Roman Cyr. Offset print.
Conditional print pages 20. Accounting publishing pages 24.
Issue. 3285 copies.
List of abbreviations

A/G Albumin/globulin ratio HR Heart rate


A-ANON Alcoholics anonymous IDDM Insulin dependent diabetes mellitus
ACT Abdominal computed tomography IFA Immunofluorescence assay
ADP Adenosine diphosphate IHD Ischemic heart disease
ALT Alanin aminotranspherase IU International unit
AMP Adenosine monophosphate LDH Lactate dehydrogenase
AP Action potential MSEC Medical and sanitary expert committee
ARF Acute renal failure NAD Nicotine amide adenine dinucleotide
AST Aspartat aminotranspherase NADPH Nicotine amide adenine dinucleotide
ATP Adenosine triphosphate phosphate restored
BP Blood pressure NIDDM Non-Insulin dependent diabetes mellitus
bpm Beats per minute PAC Polyunsaturated aromatic carbohydrates
C.I. Color Index PAS Periodic acid & shiff reaction
CBC Complete blood count pCO2 CO2 partial pressure
CHF Chronic heart failure pO2 CO2 partial pressure
CT Computer tomography pm Per minute
DIC Disseminated intravascular coagualtion Ps Pulse rate
DCC Doctoral controlling committee r roentgen
DM-2 Non-Insulin dependent diabetes mellitus RBC Red blood count
DTP Anti diphtheria-tetanus vaccine RDHA Reverse direct hemagglutination assay
ECG Electrocardiogram Rh Rhesus
ESR Erythrocyte sedimentation rate (R)CFT Reiter's complement fixation test
FC Function class RIHA Reverse indirect hemagglutination assay
FAD Flavin adenine dinucleotide RNA Ribonucleic acid
FADH2 Flavin adenine dinucleotide restored RR Respiratory rate
FEGDS Fibro-esphago-gastro-duodenoscopy S1 Heart sound 1
FMNH2 Flavin mononucleotide restored S2 Heart sound 2
GIT Gastrointestinal tract TU Tuberculin unit
GMP Guanosine monophosphate U Unit
Hb Hemoglobin USI Ultrasound investigation
HbA1c Glycosylated hemoglobin V/f Vision field
Hct Hematocrit WBC White blood count
HIV Human immunodeficiency virus X-ray Roentgenogram
Ashiq Tutorials

Krok-1 Question Paper

2019
Бiологiя 2

1. A young man has come to A. Repair


the genetic consultation. He complains B. Replication
of abnormalities in his physical and C. Transcription
reproductive development. Microscopy of his D. Reverse transcription
oral mucosa cells shows one Barr body. What E. Translation
karyotype is the most likely in this young
man? 6. A man is a carrier of HIV that is an RNA
virus. The cells of this patient synthesize viral
A. 47, ХХY DNA. This process is based on:
B. 45, Х0
C. 47, XY, +21 A. Reverse transcription
D. 47, XY, +18 B. Replication
E. 47, ХYY C. Transcription
D. Repair
2. The mother’s karyotype has 45 E. Translation
chromosomes. It was determined that
translocation of chromosome 21 to 7. The parents with normal hearing have two
chromosome 14 had occurred. What disorder daughters and a son, who are congenitally
is likely to be observed in the child of this deaf. Their other 5 children are healthy. What
woman if the father’s karyotype is normal? is the pattern of deafness inheritance in this
case?
A. Down syndrome
B. Klinefelter syndrome A. Autosomal recessive
C. Patau syndrome B. Autosomal dominant
D. Edwards syndrome C. X-linked recessive
E. Morris syndrome (androgen insensitivity) D. X-linked dominant
E. Y-linked
3. During regular examination of
schoolchildren, a scrape from the perianal 8. Ingestion of plants and mushrooms that
folds of a 10-year-old girl shows asymmetrical grow along highways is dangerous due to risk
oval eggs with larvae inside. What diagnosis of lead poisoning. What is the main source of
can be made? environmental pollution with this chemical
element?
A. Enterobiasis
B. Ascariasis A. Exhaust fumes
C. Amebiasis B. Sewage
D. Trichuriasis C. Acid rains
E. Ancylostomiasis D. Herbicides
E. Chemical fertilizers
4. A young woman, a foreign student from
Tehran, has made an appointment with the 9. Long-term taking of medicines can
urologist. She complains of the sensation of affect cells of the liver. Particularly, it can
heaviness in her lower abdomen and a small cause marked hypertrophy of agranular
amount of blood being excreted with urine endoplasmic reticulum due to the following
at the end of each urination. Microscopy of function of this organelle:
urine detects the presence of parasite eggs, A. Detoxication of harmful substances
approximately 140х70 micron in size, with a B. Nucleic acid synthesis
terminal spike. What diagnosis can be made C. Protein synthesis
by the infectious diseases specialist? D. Formation of maturation spindle
A. Schistosomiasis E. Intracellular digestion
B. Opisthorchiasis 10. Dwellers of a village located in the taiga
C. Dicroceliasis make a living by harvesting berries. Lately
D. Paragonimiasis the occurrence of alveococcosis in the village
E. Fascioliasis population has increased. What is the source
5. Cells of a person working in the Chornobyl of invasion in this case?
Exclusion Zone have undergone a mutation A. Foxes
in DNA molecule. However, with time the B. Fish
damaged interval of DNA molecule restored C. Sick people
ist initial structure with a specific enzyme. In D. Birds
this case the following occurred: E. Rodents
Нормальна анатомiя 3

1. A patient with severe poisoning was 6. An ovarian tumor was detected in a


brought into the intensive care unit. In the woman. She is prescribed a surgery. What
course of complex treatment the subclavian ligament should be severed by the surgeon to
vein needs to be catheterized for medicine separate the patient’s ovary from the uterus?
administration. This vein is located in the
following topographic structure: A. Proper ovarian ligament
B. Broad ligament of the uterus
A. Spatium antesсalenum C. Lateral umbilical ligament
B. Spatium interscalenum D. Suspensory ligament of the ovary
C. Spatium retrosternocleidomastoideus E. Round ligament of the uterus
D. Spatium interaрoneuroticum suprasternale
E. Trigonum omotrapezoideum 7. A 45-year-old woman presents with breast
cancer. Metastases can spread in this case to
2. A 52-year-old woman came to the the following regional lymph nodes:
neurologist with complaints of loss of skin
sensitivity on the right half of her face in A. Axillary, parasternal
the area of the lower eyelid, nasal arch, and B. Abdominal, cervical
upper lip. What branch of what nerve is C. Cervical, parasternal
damaged in this patient? D. Parasternal, bronchomediastinal
E. Aortic, bronchomediastinal
A. Maxillary branch of the trigeminal nerve
B. Greater petrosal nerve branching from the 8. A 35-year-old man with a hand injury
facial nerve came to the traumatology department.
C. Ophthalmic branch of the trigeminal nerve Examination revealed an incised wound on
D. Mandibular branch of the trigeminal nerve the palmar surface of the left hand; middle
E. Chorda tympani branching from the facial phalanges of digits II-IV cannot be flexed.
nerve What muscles are damaged?
3. A patient with periodontitis of the lower A. Flexor digitorum superficialis
molar came to the doctor. It was determined B. Flexor digitorum profundus
that the inflammatory process spread to C. Lumbrical muscles
the lymph nodes. What lymph nodes were D. Palmar interossei
the first to be affected by the inflammatory E. Dorsal interossei
process?
9. Clinical course of urolithiasis was
A. Submandibular complicated by the passage of a renal
B. Lateral cervical calculus. Where in the ureter is it most likely
C. Anterior cervical to stop?
D. Submental
E. Facial A. At the border between the abdominal and
pelvic segments
4. After an X-ray examination of the B. In the renal pelvis
tuberculosis clinic patient, he was diagnosed C. In the middle of the abdominal segment
with tumor of the right lung. During D. 2 cm above the entrance to the urinary
operation the surgeon removed the middle bladder
lobe of the patient’s right lung. This lobe E. 5 cm above the pelvic segment
includes:
10. The surgeon noticed aggregated
A. Segmentum laterale et segmentum mediale lymphoid nodules (Peyer’s patches) on the
B. Segmentum basale anterius et posterius intestinal mucosa. What portion of the
C. Segmentum anterius et segmentum apicale intestine is it?
D. Segmentum lingualare superius et inferius
E. Segmentum apicale (superius) et segmentum A. Ileum
basale mediale B. Jejunum
C. Cecum
5. A 30-year-old man with an incised wound D. Duodenum
on the plantar surface of the left foot was E. Rectum
brought to the traumatology department.
Lifting of the lateral side of the foot 11. An oncology patient is to undergo a
is limited. What muscle is likely to be surgery on the descending colon. Name the
functionally disturbed? main source of blood supply to this organ:

A. Peroneus longus muscle A. Inferior mesenteric artery


B. Anterior tibial muscle B. Superior mesenteric artery
C. Flexor hallucis longus muscle C. Celiac trunk
D. Triceps muscle of calf D. Middle colic artery
E. Soleus muscle E. Splenic artery
Нормальна анатомiя 4

12. A patient presents with an inner ear A. N. radialis


inflammation. On examination the doctor B. N. ulnaris
states that the 1st neurons of the auditory C. N. cutaneus brachii medialis
analyzer are affected. Where are these D. N. medianus
neurons located? E. N. musculocutaneus
A. G. spirale 14. During an invasive operation the surgeon
B. G. vestibulare needs to access the omental bursa of the
C. G. geniculi peritoneal cavity via the omental foramen
D. G. trigeminale (foramen of Winslow). What anatomical
E. G. ciliare structure makes up the anterior border of
this foramen?
13. A victim has received a deep incised
stab wound to the upper posterior surface A. Hepatoduodenal ligament
of the shoulder. Extension of elbow, hand, B. Hepatorenal ligament
and digits is impaired; skin sensitivity of the C. Visceral surface of liver
posterior surface of the shoulder and forearm D. Superior part of duodenum
is lost. What nerve is damaged in this case? E. Greater omentum
Гiстологiя 5

1. Histone protein synthesis is artificially A. Tissue basophils


blocked in a cell. What cell structure will be B. Fibroblasts
damaged as a result? C. Plasma cells
D. Eosinophils
A. Nuclear chromatin E. Macrophages
B. Nucleolus
C. Golgi apparatus 5. A histological specimen demonstrates
D. Cell membrane a vessel with the wall that consists of
E. Nuclear membrane endothelium, basement membrane, and
loose connective tissue. This vessel belongs
2. Some diseases of large intestine lead to to the following type:
the changes in the quantitative ratio between
mucosal epithelial cells. What cell types A. Non-muscular vein
are normally predominant in the cryptal B. Artery
epithelium of the large intestine? C. Muscular vein
D. Hemocapillary
A. Goblet cells E. Lymph capillary
B. Ciliated columnar epithelial cells
C. Endocrine cells 6. Parenchyma of an organ is composed of
D. Cells with acidophilic granules pseudounipolar neurons localized under the
E. Poorly differentiated cells capsule of connective tissue. Central place
belongs to nerve fibers. Name this organ:
3. Examination of a surgically excised
adrenal gland shows large cells that can be A. Spinal ganglion
impregnated with a potassium dichromate B. Sympathetic ganglion
solution. What hormone is being synthesized C. Intramural ganglion
by these cells? D. Nerve trunk
E. Spinal cord
A. Adrenaline
B. Aldosterone 7. A histological specimen shows significant
C. Secretin amount of mucous connective tissue
D. Thyroxine (Wharton’s jelly), vessels, as well as remnants
E. Cholecystokinin of yolk sac stalk and allantois. Name this
organ:
4. An inflammation can be characterized by
hemocapillary dilation in the affected area, A. Umbilical cord
decreased blood circulation, and increased B. Esophagus
vessel wall permeability. What cells play the C. Ureter
key role in this process? D. Urethra
E. Vermiform appendix
Нормальна фiзiологiя 6

1. A 40-year-old woman on examination A. B


presents with intensified basal metabolic rate. B. A
What hormone present in excess leads to C. A and B
such condition? D. C
E. D and C
A. Triiodothyronine
B. Thyrocalcitonin 7. What changes can be expected to occur
C. Glucagon in the isolated heart of a toad, if excessive
D. Aldosterone amount of calcium chloride is introduced into
E. Somatostatin its perfusate?
2. Toxic damage to hepatic cells resulted in A. Increased cardiac contraction force and
disruption of the patient’s liver function and frequency
the patient developed edemas. What changes B. Decreased cardiac contraction force
of blood plasma are the main cause of edema C. Increased cardiac contraction frequency
development? D. Increased cardiac contraction force
E. Diastolic cardiac arrest
A. Decrease of albumin content
B. Increase of globulin content 8. After hyperventilation an athlete
C. Decrease of fibrinogen content developed a brief respiratory arrest. It
D. Increase of albumin content occurred due to the following changes in the
E. Decrease of globulin content blood:
3. An isolated heart was used to study A. Decrease of CO2 pressure
excitation conduction velocity in different B. Decrease of pH
areas of the heart. What area had the lowest C. Increase of CO2 pressure
velocity of excitation conduction? D. Decrease of O2 pressure
E. Increase of CO2 and O2 pressure
A. Atrioventricular node
B. His bundle 9. A person has increased pulmonary
C. Purkinje fibers ventilation due to physical exertion. What
D. Atrial myocardium indicator of external respiration will be
E. Ventricular myocardium significantly increased compared to the
resting state?
4. A 40-year-old person developed
elevated blood pressure after an emotional A. Respiratory volume
excitement. What is the likely cause of this B. Vital lung capacity
effect? C. Inspiratory reserve volume
D. Expiratory reserve volume
A. Increased sympathetic nervous system E. Total lung capacity
tone
B. Arteriolar dilation 10. A patient developed punctate
C. Decreased cardiac contraction frequency hemorrhages after a tourniquet had been
D. Hyperpolarization of cardiomyocytes applied. It occurred due to functional
E. Increased parasympathetic nervous system disturbance of the following blood
tone corpuscles:
5. A patient has elevated blood pressure A. Platelets
due to increased vascular tone. To lower the B. Eosinophils
blood pressure in this case it is necessary to C. Monocytes
prescribe the blockers of: D. Lymphocytes
E. Neutrophils
A. α-adrenoceptors
B. β -adrenoceptors 11. The dorsal root of the spinal nerve of a
C. α- and β -adrenoceptors test animal was severed. What changes will
D. Muscarinic acetylcholine receptors occur in the innervation area?
E. Histamine H1 receptors
A. Loss of sensitivity
6. ABO blood group is being determined. B. Loss of motor function
Erythrocyte agglutination occurred when C. Decreased muscle tone
standard sera of group I and group II D. Increased muscle tone
were introduced into the blood being E. Loss of sensitivity and motor function
analyzed, while group III serum caused no
agglutination. What agglutinogens do these 12. After a certain CNS structure had been
erythrocytes have? destroyed in a test animal, this animal lost its
orienting reflexes. What structure had been
destroyed?
Нормальна фiзiологiя 7

A. Corpora quadrigemina peptides:


B. Red nuclei
C. Lateral vestibular nuclei A. Endorphins
D. Substantia nigra B. Liberins
E. Medial reticular nuclei C. Vasopressin
D. Oxytocin
13. On examination the patient was E. Statins
determined to have a strong, balanced, inert
type of higher nervous activity according to 19. I.M. Siechenov has proven that a tired
Pavlov’s classification. What temperament limb restores its working capacity faster if
according to Hippocrates is it? during its period of rest another limb works.
It became a basis for the concept of:
A. Phlegmatic
B. Sanguine A. Active rest
C. Choleric B. Parabiosis
D. Melancholic C. Pessimum
E. - D. Optimum
E. Fatigue
14. A 45-year-old woman presents with
insufficient secretion of enterokinase 20. Domestic accident has resulted in a
enzyme. Enterokinase deficiency can cause significant blood loss in the patient, which
disturbance of the following digestive was accompanied by a drop in blood
function: pressure. What hormones ensure quick
restoration of the blood pressure caused by a
A. Protein hydrolysis blood loss?
B. Carbohydrate hydrolysis
C. Lipid hydrolysis A. Adrenaline, vasopressin
D. Vitamin absorption B. Cortisol
E. Lipid absorption C. Reproductive hormones
D. Oxytocin
15. Systemic blood pressure of a person E. Aldosterone
equals 120/65 mm Hg. Blood ejection into
aorta occurs when left ventricular pressure 21. An experiment was conducted to
exceeds: measure the skin sensitivity threshold. What
patches of skin have the highest sensitivity
A. 65 mm Hg threshold?
B. 10 mm Hg
C. 90 mm Hg A. Back
D. 100 mm Hg B. Dorsal surface of the hand
E. 120 mm Hg C. Shoulder
D. Face
16. Prolonged vomiting resulted in E. Shin
dehydration of the patient’s body. Under
these conditons, water retention in the 22. A student, whose educational
body is ensured primarily due to increased achievements throughout the semester were
secretion of the following hormone: poor, feels emotionally tense during the final
test. What is the primary cause that induced
A. Vasopressin the leading mechanism of emotional tension
B. Aldosterone in this case?
C. Natriuretic hormone
D. Adrenaline A. Lack of information
E. Calcitonin B. Lack of time
C. Lack of time and energy
17. KCl concentration in a solution that D. Lack of energy
surrounds an isolated cell was increased. E. Lack of energy and information
How will resting membrane potential (RMP)
and cell excitability change in this case? 23. After a trauma the patient has developed
right-sided paralyses and disturbed pain
A. RMP decreases, excitability increases sensitivity. On the left side no paralyses are
B. RMP increases, excitability increases observed, but pain and thermal sensitivity
C. RMP increases, excitability decreases is disturbed. What is the cause of this
D. RMP decreases, excitability remains condition?
unchanged
E. RMP and excitability remain unchanged A. Unilateral right-side spinal cord injuiry
B. Midbrain injury
18. Human brain produces endogenous C. Brainstem injury
peptides that are similar to morphine and D. Cerebellar injury
can reduce pain perception. Name these E. Motor cortex injury
Бiохiмiя 8

1. A 3-year-old child with elevated body 7. A 27-year-old patient presents with


temperature has taken aspirin and developed pathologic changes in the liver and brain.
increased hemolysis of erythrocytes. In this Blood plasma exhibits acute decrease in
case hemolytic anemia can be caused by copper levels, while urine copper levels
congenital deficiency of the following enzyme: are elevated. The patient is diagnosed with
Wilson disease. To confirm this diagnosis it is
A. Glucose 6-phosphate dehydrogenase necessary to measure activity of the following
B. Glucose 6-phosphatase enzyme in the patient’s blood serum:
C. Glycogen phosphorylase
D. Glycerol-phosphate dehydrogenase A. Ceruloplasmin
E. Gamma-glutamyl transferase B. Carbonic anhydrase
C. Xanthine oxidase
2. Collagenosis patients typically present with D. Leucine aminopeptidase
the processes of connective tissue destruction. E. Alcohol dehydrogenase
The presence of these processes can be
confirmed by the increase in: 8. The Gerontology Institute recommends
older people to take vitamin complexes that
A. Blood oxyproline and oxylysine contain vitamin E. What is the main function
B. Blood creatine and creatinine of this vitamin?
C. LDH-isoenzyme activity in the blood
D. Transaminase activity in the blood A. Antioxidant
E. Blood urates B. Antihemorrhagic
C. Antiscorbutic
3. During diabetes mellitus and starvation, D. Antineuritic
the number of acetone bodies in blood E. Antidermatitic
increases. These bodies are used as a source of
energy and are synthesized from the following 9. Ammonia is extremely toxic for human
substance: CNS. What is the main way of ammonia
neutralization in the nervous tissue?
A. Acetyl-CoA
B. Succinyl-CoA A. Glutamine synthesis
C. Citrate B. Ammonium salts synthesis
D. Malate C. Urea synthesis
E. Ketoglutarate D. Transamination
E. Formation of paired compounds
4. Blood test of the patient revealed albumine
content of 20 g/L and increased activity of 10. People, who for a long time remained in
lactate dehydrogenase isoenzyme 5 (LDH5 ). hypodynamic state, develop intense pain in
These results indicate disorder of the the muscles after a physical exertion. What is
following organ: the most likely cause of this pain?
A. Liver A. Accumulation of lactic acid in muscles
B. Kidneys B. Intensive breakdown of muscle proteins
C. Heart C. Accumulation of creatinine in muscles
D. Lungs D. Decreased content of lipids in muscles
E. Spleen E. Increased content of ADP in muscles
5. A patient presents with an acute attack of 11. Human genetic apparatus consists of
cholelithiasis. Laboratory examination of the approximately 30 thousand of genes, while the
patient’s feces will show the following in this number of antibody variants can be as high as
case: millions. What mechanism leads to formation
of new genes that ensure the synthesis of such
A. Negative reaction to stercobilin a number of antibodies?
B. Positive reaction to stercobilin
C. Connective tissue A. Genetic recombination
D. Partially digested cellulose B. Gene amplification
E. Starch granules C. DNA replication
D. DNA repair
6. A patient with diabetes mellitus after an E. Formation of Okazaki fragments
insulin injection lost his consciousness and
developed convulsions. What will be the result 12. Chronic overdose of glucocorticoids leads
of a biochemical test for blood glucose level to the development of hyperglycemia in a
in this case? patient. Name the process of carbohydrate
metabolism that results in elevated blood
A. 2.5 mmol/L glucose levels:
B. 3.3 mmol/L
C. 8.0 mmol/L
D. 10 mmol/L
E. 5.5 mmol/L
Бiохiмiя 9

A. Gluconeogenesis activator, thromboxane A2. What substance is


B. Glycogenolysis this activator synthesized from?
C. Aerobic glycolysis
D. Pentose-phosphate pathway A. Arachidonic acid
E. Glycogenesis B. Malonic acid
C. Acetic acid
13. A patient for a long time was on an D. Homogentisic acid
imbalanced diet low in proteins, which E. Glutamic acid
resulted in hepatic fatty infiltration. This
condition is likely to develop if a certain 19. A patient with myocardial infarction in
substance is absent in a person’s diet. Name the acute phase has been hospitalized into
this substance: the cardiology unit. To induce platelet lysis in
the patient’s coronary vessels during the early
A. Methionine hours of infarction, the following enzyme
B. Alanine should be used:
C. Cholesterol
D. Acetic acid A. Streptokinase
E. Biotin B. Hyaluronidase
C. Trypsin
14. A patient, who has been subsisting D. Chymotrypsin
exclusively on polished rice, has developed E. Lysozyme
polyneuritis due to thiamine deficiency. What
substance is an indicator of such avitaminosis, 20. In the hematology unit a patient with
when it is excreted with urine? leukemia was prescribed 5-Fluorouracil. This
drug:
A. Pyruvic acid
B. Malate A. Inhibits DNA synthesis
C. Methylmalonic acid B. Stimulates DNase
D. Uric acid C. Inhibits translation
E. Phenyl pyruvate D. Inhibits transcription
E. Catalyzes replication
15. A 40-year-old man with pulmonary
tuberculosis was prescribed isoniazid. 21. A 7-year-old boy is diagnosed with anemia.
Prolonged taking of this drug can result Laboratory analysis detects pyruvate kinase
in development of the following vitamin deficiency in his erythrocytes. What process is
deficiency: disturbed in this boy, playing the main role in
anemia development in this case?
A. Pyridoxine
B. Thiamine A. Anaerobic glycolysis
C. Cobalamin B. Deaminization of amino acids
D. Biotin C. Decarboxylation of amino acids
E. Folic acid D. Gluconeogenesis
E. Anaerobic glycogenolysis
16. Disturbed activity of trypsin and
chymotrypsin leads to disturbed protein 22. Examination of a patient shows decreased
breakup in the small intestine. Activity of leukocyte and erythrocyte count and low
these enzymes depends on the presence of hemoglobin levels in peripheric blood, as well
the following factor: as appearance of large cells (megaloblasts).
What vitamin deficiency can cause these
A. Enterokinase clinical presentations?
B. Pepsin
C. Bile acids A. Folic acid
D. Hydrochloric acid B. Niacin
E. N a+ salts C. Ascorbic acid
D. Riboflavin
17. A newborn presents with weak suckling, E. Biotin
frequent vomiting, and hypotonia. Blood and
urine citrulline are very high. What metabolic 23. Wernicke-Korsakoff syndrome often
process is disturbed? develops in chronic alcoholics, who have a
low-vitamin diet. Decreased transketolase
A. Ornithine cycle activity can be observed in the course of this
B. Tricarboxylic acid cycle disease. What vitamin deficiency causes this
C. Glycolysis development?
D. Gluconeogenesis
E. Cori cycle A. Thiamine
B. Retinol
18. Patients with ischemic heart disease are C. Niacin
usually prescribed small doses of aspirin. This D. Cobalamin
drug inhibits synthesis of platelet aggregation E. Riboflavin
Патологiчна фiзiологiя 10

1. A patient was hospitalized in a A. Cytotoxic


comatose state. The patient has a 5-year- B. Stimulating
long history of diabetes mellitus type 2. C. Anaphylactic
Objectively respiration is noisy, deep, with D. Delayed-type hypersensitivity
acetone breath odor. Blood glucose is 15.2 E. Immune complex-mediated
mmol/L, ketone bodies - 100 micromol/L.
These signs are characteristic of the following 6. A patient with obliterating endarteritis
diabetes complication: has undergone a ganglionic sympathectomy.
Positive therapeutic effect of this surgery
A. Ketoacidotic coma is associated with development of arterial
B. Hepatic coma hyperemia of the lower limbs, which can be
C. Hyperglycemic coma described as:
D. Hypoglycemic coma
E. Hyperosmolar coma A. Neuroparalytic
B. Neurotonic
2. A 63-year-old man suffers from esophageal C. Metabolic
carcinoma, presents with metastases into D. Reactive
the mediastinal lymph nodes and cancerous E. Working
cachexia. What pathogenetic stage of
neoplastic process is observed in the patient? 7. In an experiment a laboratory rat was
subjected to a stress factor (electric current),
A. Progression which resulted in muscular hypotonia,
B. Promotion arterial hypotension, hypothermia, and
C. Transformation hypoglycemia in the animal. What period of
D. Initiation general adaptation syndrome is it?
E. -
A. Shock phase
3. After a prolonged attack of severe B. Antishock phase
headache the patient lost mobility in his left C. Resistance stage
arm and leg. Muscle tone is decreased in D. Exhaustion stage
the affected limbs, the muscles are spasmed, E. -
spinal tendon reflexes are acutely intensified,
reflex zones are increased. What nervous 8. A 14-year-old adolescent has diphtheria.
system disorder can be observed in this During the peak of the disease against
patient? the background of acute drop in body
temperature and tachycardia the blood
A. Central paralysis pressure is 70/50 mm Hg. What type of
B. Peripheral paralysis vascular tone disturbance is it?
C. Extrapyramidal paralysis
D. Flaccid paralysis A. Acute hypotension
E. Reflex paralysis B. -
C. Chronic hypotension
4. Hematologic study shows the following D. Somatoform autonomic dysfunction
pattern: erythrocytes - 2, 8 · 1012 /L, Нb - E. Essential hypotension
80 g/L, color index - 0.85, reticulocytes -
0,1%, platelets - 160 thousand per microliter, 9. 24 hours after an appendectomy the
patient’s blood test shows neutrophilic
leukocytes - 60 · 109 /L. Basocytes - 2%, leukocytosis with a regenerative shift. What
eosinophils - 8%, promyelocytes - 5%, is the most likely mechanism of absolute
myelocytes - 5%, juvenile - 16%, stab leukocytosis development in the patient’s
neutrophils - 20%, segmented neutrophils peripheral blood?
- 34%, lymphocytes - 5%, monocytes -
5%. This clinical presentation indicates the A. Intensification of leukopoiesis
following blood pathology: B. Leukocyte redistribution
C. Decreased leukocyte disintegration
A. Chronic myeloleukemia D. Deceleration of leukocyte migration to the
B. Acute myeloleukemia tissues
C. Hypoplastic anemia E. Immunity activation
D. Undifferentiated leukemia
E. Hemolytic anemia 10. A 59-year-old man, a business manager,
developed intense burning retrosternal pain
5. Antileukocytic antibodies are detected that irradiates to the left arm. The pain
in the blood of a patient with leukopenia. occurred in the evening after the tax audit.
What type of Coombs-Gell hypersensitivity 15 minutes later the patient’s condition
reaction developed in this case? normalized. What mechanism of angina
pectoris development is leading in this
patient?
Патологiчна фiзiологiя 11

A. Increased level of blood catecholamines wife is healthy. What is the probability of


B. Coronary atherosclerosis their child developing insulin-independent
C. Intravascular aggregation of blood cells diabetes mellitus?
D. Coronary artery thrombosis
E. Functional cardiac overload A. Higher than in the population
B. The same as in the population
11. A 30-year-old person has been stung C. Lower than in the population
by a bee. The stung area exhibits edema, D. 100%
hyperemia, and elevated temperature. E. 50%
What is the initial pathogenetic factor of
inflammatory edema in this case? 17. A laboratory rat with chronic kidney
failure presents with osteoporosis, pathologic
A. Increase of microvascular permeability calcification of the internal organs, and
B. Increase of osmotic pressure in the arterial hypertension. These disturbances
inflammation focus are associated with increased activity of the
C. Decrease of oncotic blood pressure following hormone:
D. Increase of capillary blood pressure
E. Disturbed lymphatic efflux A. Parathyroid hormone
B. Thyroxin
12. A 30-year-old woman developed C. Triiodothyronine
facial edemas. Examination detected D. Calcitonin
proteinuria (5.87 g/L), hypoproteinemia, E. Adrenaline
dysproteinemia, and hyperlipidemia. Such
combination of signs is characteristic of: 18. A patient is diagnosed with severe B12 -
deficiency anemia resulting in disturbed
A. Nephrotic syndrome hematopoiesis and appearance of atypical
B. Nephritic syndrome erythrocytes in the blood. The patient has
C. Chronic pyelonephritis a history of total gastric resection. This
D. Acute kidney failure diagnosis can be confirmed if the following
E. Chronic kidney failure cells are present in the peripheral blood:
13. An uconscious young man in the state of A. Megalocytes
morphine intoxication has been brought into B. Microcytes
an admission room. The patient’s respiration C. Elliptocytes
is slow and shallow due to suppression of the D. Normocytes
respiratory center. What kind of respiratory E. Anulocytes
failure occurred in this case?
19. A patient with asphyxia after a
A. Ventilatory disregulation brief respiratory arrest developed single
B. Ventilatory obstruction infrequent respirations with passive
C. Ventilatory restriction expiration, after which he stopped breathing
D. Perfusion completely. What type of respiration was
E. Diffusion observed in this case?
14. A man has been working for a long time A. Gasping respiration
in oil processing. What type of carcinogens B. Apneustic respiration
does he encounter at his workplace? C. Kussmaul respiration
D. Cheyne-Stokes respiration
A. Polycyclic aromatic hydrocarbons E. Biot respiration
B. Amino-azo compounds
C. Nitrosamines 20. A 3-year-old child has been brought by
D. Biological carcinogens ambulance to the intensive care unit of the
E. Amines infectious diseases hospital. On examination
the child is in severe condition, skin and
15. A patient has been suffering from mucosa are dry, tissue turgor is reduced. The
bronchial asthma for 15 years. What changes patient’s history states that profuse diarrhea
in the patient’s leukogram can be expected in and recurrent vomiting were observed
this case? throughout the previous day after the child
A. Eosinophilia had eaten food products of poor quality.
B. Basophilia What type of salt and water imbalance is
C. Leukocytosis likely to have developed in the patient?
D. Leukopenia A. Hypoosmolar dehydration
E. Left shift B. Isoosmolar dehydration
16. A married couple came for a genetic C. Hyperosmolar hyperhydration
counseling. The husband suffers from insulin- D. Isoosmolar hyperhydration
independent diabetes mellitus, while the E. Hypoosmolar hyperhydration
Патологiчна фiзiологiя 12

21. A 48-year-old man is unconscious. He has 22. A 40-year-old man with impaired venous
a history of several syncopal episodes with patency in the lower limbs developed
convulsions. ECG shows deformed QRS edemas. What mechanism plays the main
complexes unconnected with P waves, atrial role in the development of this disturbance?
contractions are approximately 70/min.,
ventricular contractions - 25-30/min. Name A. Elevated filtration pressure
the type of arrhythmia in this case: B. Positive fluid balance
C. Decreased gradient of osmotic pressure
A. Complete atrioventricular block between blood and tissue
B. First-degree atrioventricular block D. Disturbed humoral regulation of water-
C. Second-degree atrioventricular block mineral balance
D. Intraatrial block E. Hypoproteinemia
E. Intraventricular block
Патологiчна анатомiя 13

1. Autopsy of a 60-year-old woman, who A. Skin cancer


for a long time had been suffering from B. Sarcoma
essential hypertension, shows significantly C. Papilloma
diminished kidneys (weight of both kidneys D. Trophic ulcer
is 80 g) with finely granular surface. Uniform E. Fibroma
renal cortical thinning can be observed on 6. Histological analysis of a biopsy skin
section. Name the described changes in the sample obtained from a 24-year-old patient
kidneys: detects caseous necrosis surrounded with
A. Primary contracted kidney cellular infiltrate consisting of lymphocytes,
B. Pyelonephritic contracted kidney among which there are single giant cells;
C. Secondary contracted kidney proliferation of connective tissue and
D. Amyloid contracted kidney endovasculites are observed. Characterize
E. Diabetic nephrosclerosis this pathologic process:

2. Autopsy of a 3-year-old child shows a A. Proliferative granulomatous inflammation


tumor in the cerebellum. The tumor has B. Proliferative interstitial inflammation
no clear margins separating it from the C. Abscess
surrounding tissues. Histologically it is made D. Catarrhal inflammation
of small atypical cells with hyperchromic E. Ichorous inflammation
nuclei. This tumor is most likely a: 7. Autopsy of a man, who had been suffering
A. Medulloblastoma from mitral stenosis, reveals dense brown
B. Medullary sarcoma lungs. What pathologic process had occurred
C. Cancer metastasis in the lungs?
D. Sarcoma metastasis A. Hemosiderosis
E. Glioblastoma B. Hemochromatosis
3. Autopsy revealed a large wedge-shaped C. Jaundice
patch of a dense dark red tissue with clear D. Hemomelanosis
margins in the upper lobe of the right lung. E. Lipofuscinosis
Histological examination detected there 8. A 63-year-old man, who has been suffering
necrosis of the alveolar walls; the alveolar from chronic diffuse obstructive pulmonary
lumen is tightly packed with erythrocytes. emphysema for 15 years, died of progressive
What process occurred in the lungs? heart failure. Autopsy shows nutmeg liver
A. Hemorrhagic infarction cirrhosis, cyanotic induration of kidneys and
B. Carneous degeneration spleen, ascites, and edemas of the lower
C. Gangrene limbs. What type of heart failure can be
D. Hemorrhage characterized by such changes in the internal
E. Atelectasis organs?

4. Regional lymph nodes surrounding an A. Chronic heart failure


infected wound are enlarged. Histological B. Acute right ventricular failure
examination shows increased number of C. Chronic atrial failure
macrophages, lymphocytes, and lymphatic D. Acute left ventricular failure
follicles in the cortical layer of the lymph E. Acute global heart failure
nodes, as well as a large amount of plasma 9. A 6-year-old girl presents with acute
cells. What process in the lymph nodes is onset of a disease. She developed sore
indicated by these histological changes? throat and high temperature that were
A. Antigen stimulation later accompanied by a punctate skin rash.
B. Acquired deficiency of lymphoid tissue Oral examination reveals acute pharyngeal
C. Congenital deficiency of lymphoid tissue hyperemia, raspberry tongue, and enlarged
D. Neoplastic aberration bright red tonsils with dull gray and
E. Transplant rejection yellow foci that spread to the peritonsillar
tissues. The submandibular lymph nodes are
5. A patient has gradually developed a enlarged. What disease are these changes
skin plaque on his face. In the center of characteristic of?
this plaque there are necrotic patch and
an ulcer. Histopathological analysis of the A. Scarlet fever
biopsy material reveals proliferation of B. Measles
atypical epithelial cells with large number of C. Pharyngeal diphtheria
pathologic mitoses. What is the most likely D. Laryngeal diphtheria
diagnosis? E. Meningococcal nasopharyngitis
10. Autopsy of a man, who served
on a nuclear submarine, revealed the
Патологiчна анатомiя 14

following pathologies: bone marrow trip to an African country he developed


atrophy (panmyelophthisis), anemia, periodical jaundice attacks. Autopsy shows
leukopenia, thrombocytopenia, lymphocytic the following: dense enlarged spleen
disintegration in the lymph nodes, spleen, with slate-black pulp; enlarged plethoric
gastrointestinal lymphatic system, and liver, gray-black on section; cerebral gray
hemorrhages into the adrenal glands. What matter is brown-gray; cerebral white matter
disease had developed in this case? contains numerous small hemorrhages. What
infectious disease can be suspected?
A. Acute radiation sickness
B. Decompression sickness A. Malaria
C. Acute leukemia B. Meningococcemia
D. Acute anemia C. Prion infection
E. Vibration disease D. Generalized herpetic infection
E. Generalized cryptococcosis
11. A 9-month-old child presents with
delayed tooth eruption, improper sequence 15. A 27-year-old woman has undergone a
of tooth eruption, and horizontal sector resection of mammary gland tissue.
maxillary configuration (high-arched palate). Macroscopy detects a dense white node, 4 cm
Microscopically enamel mineralization in diameter, with clear margins in the excised
pattern is irregular, enamel columns are tissue. Immediate histological analysis shows
wrinkled, some of them are vacuolated, the tumor to consist of a large amount of
predentin zones are widened, single denticles fibrous stroma with stromal proliferation
can be observed. What disease is it? around the small canaliculi. Canalicular
epithelium overlays the basement membrane
A. Early rickets and retains its polarity. Make the diagnosis:
B. Late rickets
C. Osteomalacia A. Pericanalicular fibroadenoma
D. Gout B. Adenocarcinoma
E. Hypervitaminosis D C. Sarcoma
D. Dyshormonal disorders
12. Autopsy of a 58-year-old man, who for E. Cancer
a long time has been drinking alcohol in
large amounts and died at home, is being 16. In the course of an urgent surgery, the
conducted. Macroscopically the right lung vermiform appendix of the patient was
is dense and enlarged, its tissue is gray excised. The appendix was acutely distended
and homogeneous on section, its pleura is and gray-black throughout its whole length.
covered with grayish membranous deposits. In the distal segment a defect of the
Microscopically the alveolar cavities contain appendix wall was detected, through which
fibrin threads, neutrophils, and hemolysed a foul-smelling gray-brown substance was
erythrocytes. Make the diagnosis: being discharged from the appendix lumen.
Histological analysis shows necrotization of
A. Croupous pneumonia the appendix wall with hemorrhagic foci;
B. Focal pneumonia lumen of the mesenteric artery is filled with
C. Interstitial pneumonia a trombus. What type of appendicitis is it?
D. Primary pulmonary tuberculosis
E. Caseous pneumonia A. Acute gangrenous
B. Acute phlegmonous
13. Autopsy of a 49-year-old woman who C. Acute simple
died of chronic kidney failure shows D. Acute superficial
small dense striated kidneys with areas E. Chronic
of hemorrhages. Microscopically nuclei of
epithelial channels contain hematoxylin 17. A 39-year-old man underwent a surgery
bodies; glomerular capillaries resemble wire for peptic ulcer disease of the stomach. He
loops, have thickened basement membranes, died 7 days after the surgery. On autopsy
and in places contain hyaline thrombi and the peritoneal layers are plethoric, dull,
foci of fibrinoid necrosis. What is the most and covered with massive yellow-green
likely diagnosis? membranous deposits. The peritoneal cavity
contains approximately 300 mL of thick
A. Systemic lupus erythematosus yellow-green fluid. What pathologic process
B. Rheumatism was detected in the peritoneal cavity?
C. Arteriolosclerotic nephrosclerosis
D. Amyloidosis A. Fibrinopurulent peritonitis
E. Atherosclerotic nephrosclerosis B. Serous peritonitis
C. Serofibrinous peritonitis
14. A 34-year-old man died in a comatose D. Peritoneal commissures
state. According to his family after a business E. Fibrinohemorrhagic peritonitis
Мiкробiологiя 15

1. To test donor blood for hepatitis B A. Serological


antigens, it is necessary to use highly sensitive B. Bacteriological
detection methods. What test should be used C. Biological
for this purpose? D. Microscopy
E. Molecular biological
A. Solid-phase enzyme-linked
immunosorbent assay 6. What diagnostic method should be used in
B. Immunoelectrophoresis industry to test the raw leather for presence of
C. Indirect hemagglutination B. antracis?
D. Complement binding
E. Indirect immunofluorescence A. Ascoli’s thermo precipitation test
B. Microscopy with Burry-Gins stain
2. The bacteriological laboratory has received C. Microscopy with Aujeszky stain
for analysis a sample of dried fish from a focus D. Bacteriological analysis
of food poisoning outbreak. The bacteriologist E. Serological test
inoculated the sample into a Kitt-Tarozzi
medium, where growth of tennis racquet- 7. Urinalysis of a patient with acute cystitis
shaped microorganisms could be observed. shows leukocytes and a large number of gram-
These microorganisms are likely to be the negative bacilli. Inoculation has resulted in
causative agents of: the growth of mucous colonies that produce a
green soluble pigment. What microorganism is
A. Botulism the most likely cause of the patient’s disorder?
B. Salmonellosis
C. Dysentery A. Pseudomonas aeruginosa
D. Staphylococcal toxicoinfection B. Escherihia coli
E. Typhoid fever C. Klebsiella pneumoniae
D. Proteus mirabilis
3. To determine toxigenicity of diphtheria E. Salmonella enteritidis
causative agents obtained from patients, the
cultures were inoculated in a Petri dish with 8. Pathologic material (mucosal excretion from
nutrient agar, bilaterally to a strip of filter the nasal passages) obtained from a patient
paper spotted with antidiphtheric antitoxic provisionally diagnosed with influenza was
serum and situated in the center of the sent to the virological laboratory. What quick
Petri dish. After incubation of the inoculated test allows detecting specific viral antigen in
cultures in the agar, strip-like areas of medium the investigated material?
turbidity formed between some of the cultures
and the filter paper. What immunological test A. Direct and indirect immunofluorescence
was conducted? (IF)
B. Direct and indirect enzyme-linked
A. Agar gel precipitation test immunosorbent assay (ELISA)
B. Coombs test C. Hemagglutination inhibition assay (HAI)
C. Agglutination test D. Reverse indirect hemagglutination
D. Ring precipitin test (RIHA)
E. Opsonization test E. Radioimmunoassay (RIA)
4. The bacteriological laboratory needs to 9. First-year schoolchildren have received
prepare for analysis of materials that are tuberculin skin test (Mantoux test) at the
suspected to be contaminated with spores school nurse’s office. The purpose of this test
of anthrax causative agent. What diagnostic was:
preparation allows for quick detection of these
spores? A. To determine the children that need to
receive BCG vaccination
A. Anti-anthrax fluorescent serum B. To preventively vaccinate against
B. Anti-anthrax immunoglobulin tuberculosis
C. Standard anthrax antigen C. To measure immunity stress due to
D. Enzyme-tagged immunoglobulin diphtheria
E. Monoclonal antibodies to anthrax D. To measure allergization rate against
causative agent rickettsia
E. To detect parotitis in the schoolchildren
5. A family has two children. The younger child
is under a year. The child has developed spastic 10. A 12-year-old boy with clinical
cough attacks. Similar clinical presentation presentation of influenza has developed
was observed in the elder preschool child respiratory mycoplasmosis. What type
one month ago. The doctor suspects pertussis of infection has developed under these
infection. What method enables retrospective conditions?
diagnostics of this disease?
A. Mixed infection
B. Iatrogenic infection
C. Superinfection
D. Relapse
E. Autoinfection
Фармакологiя 16

1. A patient with peptic ulcer disease was 6. A patient with inoperable lung cancer
prescribed famotidine. As a result his gastric accompanied by unbearable pain was
juice acidity significantly decreased. What is prescribed an analgesic. Against the
the mechanism of action of this drug? background of analgesic therapy the patient
developed signs of intestinal obstruction.
A. Histamine H2 receptor blockade What analgesic could have caused this
B. Histamine H1 receptor blockade complication?
C. Muscarinic M1 receptor blockade
D. Inhibition of H + , K + -ATPase activity A. Morphine
E. Blockade of histamine receptors in the B. Promedol (Trimeperidine)
sympathetic ganglia C. Omnopon (Papaveretum)
D. Fentanyl
2. A 33-year-old woman, who for a long time E. Analgin (Metamizole)
has been treated for chronic polyarthritis,
complains of elevated blood pressure, 7. A patient has been taking bisacodyl for
changes in adipose tissue distribution, and a long time to treat chronic constipation.
disturbed menstrual cycle. What drug does However, several weeks later the aperient
this patient take? effect of the drug diminished. What is the
possible cause of this?
A. Prednisolone
B. Indometacin A. Acquired tolerance
C. Butadion (Phenylbutazone) B. Drug dependence
D. Synaflan (Fluocinolone acetonide) C. Material cumulation
E. Beclometasone D. Functional cumulation
E. Sensitization
3. The first-aid center has received a victim
of a traffic accident diagnosed with closed 8. A 39-year-old man presents with
displaced fracture of the middle third of hyperkeratosis, disturbed twilight vision,
the thigh. For repositioning of the bone and high risk of infectious processes. What
fragments the patient received 10 mL of vitamin preparation should he be prescribed?
2% dithylinum solution intravenously, which
resulted in the prolonged period of apnoea A. Retinol acetate
and muscle relaxation. What enzyme is B. Pyridoxine hydrochloride
deficient, resulting in such pharmacogenetic C. Riboflavin
enzymopathy? D. Ergocalciferol
E. Tocopherol acetate
A. Pseudocholinesterase
B. Uridine diphosphate glucuronyltransferase 9. The neurological department received a
C. Glucose 6-phosphate dehydrogenase patient complaining of memory deterioration
D. Methemoglobin reductase and loss of mental work capacity that
E. N-acetyltransferase developed after a head trauma. Recommend
him a medicine for improvement in cerebral
4. A patient with diabetes mellitus and metabolism:
allergic dermatitis was prescribed a certain
fluorinated hormone drug in the ointment A. Piracetam (Nootropil)
dosage form. When the patient asked, how B. Meridil (Methylphenidate)
this drug was better than the hydrocortisone C. Sydnocarb (Mesocarb)
ointment, the doctor explained that the D. Caffeine
prescribed medicine: E. Analgin (Metamizole)
A. Had practically no resorptive effect 10. A woman with polyarticular rheumatoid
B. Increased insulin synthesis arthritis was prescribed a non-steroidal anti-
C. Had short-term action inflammatory drug - diclofenac sodium.
D. Was less potent After the patient has been taking it for some
E. Was cheaper time, her concomitant disease exacerbated,
which forced the doctor to cancel the
5. A 45-year-old woman has an attack of prescription of this drug. What concomitant
cardiac fibrillation. She suffers from stage disease could necessitate cancellation of this
II essential hypertension. What is the drug of drug prescription?
choice for stopping this attack?
A. Ulcer disease
A. Anaprilin (Propranolol) B. Ischemic heart disease
B. Sustac forte (Nitroglycerin) C. Diabetes mellitus
C. Potassium chloride D. Essential hypertension
D. Strophanthin E. Bronchial asthma
E. Lidocaine
11. A patient with streptococcal infection
Фармакологiя 17

of the gingiva was prescribed a drug spontaneous respiration the patient received
with β -lactam ring in its structure. What proserin. What pharmacological group does
drug of those listed below belongs to this this drug belong to?
pharmacological group?
A. Cholinesterase inhibitors
A. Benzylpenicillin B. Angiotensin-converting-enzyme inhibitors
B. Rifampicin C. Calcium channel blockers
C. Erythromycin D. Muscarinic antagonists
D. Streptomycin sulfate E. Muscarinic agonists
E. Levomycetin (Chloramphenicol)
16. To treat the burns, a patient was
12. A patient was administered a certain drug prescribed a drug with antiseptic properties
for relief of cardiac rhythm disturbance. This that are based on formation of atomic
drug can be used as a local anesthetic as well. oxygen in the presence of organic substances.
Name this drug: This drug has also an astringent (anti-
inflammatory) effect due to formation of
A. Lidocaine hydrochloride albuminates. Name this drug:
B. Dicain (Tetracaine)
C. Diphenine (Phenytoin) A. Potassium permanganate
D. Cocaine hydrochloride B. Ethyl alcohol
E. Anaesthesin (Benzocaine) C. Sodium bicarbonate
D. Hydrogen peroxide
13. A patient with arrhythmia was E. Chlorhexidine digluconate
hospitalized into the cardiology unit. What
antiarrhythmic drug should be prescribed? 17. Before a surgery the patient was
prescribed a synthetic antiprotozoal drug
A. Amiodarone for prevention of wound infection. The
B. Acetylsalicylic acid prescribed drug is highly effective against
C. Drotaverine hydrochloride Helicobacter pylori. Name this drug:
D. Furacilin (Nitrofural)
E. Diclofenac sodium A. Metronidazole
B. Doxycycline hydrochloride
14. A woman with allergic neurodermatitis C. Chingamin (Chloroquine)
was prescribed a second-generation D. Aciclovir
antihistamine without depressing effect on E. Isoniazid
the CNS. Name this drug:
18. A patient with essential hypertension was
A. Loratadine prescribed a drug that inhibits angiotensin-
B. Diazolin (Mebhydrolin) converting enzyme (ACE). What drug is it?
C. Tavegyl (Clemastine)
D. Dimedrol (Diphenhydramine) A. Lisinopril
E. Ketotifen B. Losartan
C. Nifedipine
15. During gastric resection the patient D. Colestyramine
received mixed anesthesia with tubocurarin E. Carvedilol
chloride muscle relaxant; to restore

You might also like